Π Π°Π·Π½ΠΎΠ΅

Π—Π°Π΄Π°Ρ‡ΠΈ 3 класс ΠΏΠΎ ΠΌΠ°Ρ‚Π΅ΠΌΠ°Ρ‚ΠΈΠΊΠ΅ Π² 4 дСйствия: ΠœΠ°Ρ‚Π΅ΠΌΠ°Ρ‚ΠΈΠΊΠ° 3 класс. Числа ΠΎΡ‚ 1 Π΄ΠΎ 100. Π—Π°Π΄Π°Ρ‡ΠΈ с Ρ€Π΅ΡˆΠ΅Π½ΠΈΡΠΌΠΈ.

Π‘ΠΎΠ΄Π΅Ρ€ΠΆΠ°Π½ΠΈΠ΅

Π—Π°Π΄Π°Ρ‡ΠΈ Π½Π° 3 дСйствия. Π—Π°Π΄Π°Ρ‡ΠΈ Π½Π° Π½Π°Ρ…ΠΎΠΆΠ΄Π΅Π½ΠΈΠ΅ нСизвСстного слагаСмого.

Π—Π°Π΄Π°Ρ‡ΠΈ ΠΏΠΎ ΠΌΠ°Ρ‚Π΅ΠΌΠ°Ρ‚ΠΈΠΊΠ΅ для 3 класса.

ο»Ώο»Ώ

Π—Π°Π΄Π°Ρ‡Π° 1

Π’ Π·ΠΎΠΎΠΌΠ°Π³Π°Π·ΠΈΠ½Π΅ 52 попугая. Π’ 7 ΠΊΠ»Π΅Ρ‚ΠΊΠ°Ρ… ΠΏΠΎ 4 попугая ΠΈ Π² Π½Π΅ΡΠΊΠΎΠ»ΡŒΠΊΠΈΡ… ΠΊΠ»Π΅Ρ‚ΠΊΠ°Ρ… ΠΏΠΎ 3 попугая. Бколько ΠΊΠ»Π΅Ρ‚ΠΎΠΊ с 3 попугаями?

    РСшСниС:
  • 1) 7 * 4 = 28
  • 2) 52 – 28 = 24
  • 3) 24 : 3 = 8
  • Π’Ρ‹Ρ€Π°ΠΆΠ΅Π½ΠΈΠ΅: (52 – 7 * 4) : 3
  • ΠžΡ‚Π²Π΅Ρ‚: 8

Π—Π°Π΄Π°Ρ‡Π° 2

ΠšΡƒΠΏΠΈΠ»ΠΈ 5 ΠΊΠΎΡ€ΠΎΠ±ΠΎΠΊ ΠΏΠΎ 7 ΠΊΠΎΠ½Ρ„Π΅Ρ‚ ΠΈ 3 ΠΊΠΎΡ€ΠΎΠ±ΠΊΠΈ с пряниками. Бколько пряников Π² ΠΊΠΎΡ€ΠΎΠ±ΠΊΠ΅, Ссли ΠΎΠ±Ρ‰Π΅Π΅ количСство ΠΊΠΎΠ½Ρ„Π΅Ρ‚ ΠΈ пряников 62 ΡˆΡ‚.?

    РСшСниС:
  • 1) 5 * 7 = 35
  • 2) 62 – 35 = 27
  • 3) 27 : 3 = 9
  • Π’Ρ‹Ρ€Π°ΠΆΠ΅Π½ΠΈΠ΅: (62 – 5 * 7) : 3
  • ΠžΡ‚Π²Π΅Ρ‚: 9

Π—Π°Π΄Π°Ρ‡Π° 3

Π’ ΠΊΠΎΡ€ΠΎΠ±ΠΊΠ΅ 43 пряника.

Π’ 6 ΠΏΠ°Ρ‡ΠΊΠ°Ρ… ΠΏΠΎ 3 пряника ΠΈ Π² Π½Π΅ΡΠΊΠΎΠ»ΡŒΠΊΠΈΡ… ΠΏΠ°Ρ‡ΠΊΠ°Ρ… ΠΏΠΎ 5 пряников. Бколько ΠΏΠ°Ρ‡Π΅ΠΊ с 5 пряниками?

    РСшСниС:
  • 1) 6 * 3 = 18
  • 2) 43 – 18 = 25
  • 3) 25 : 5 = 5
  • Π’Ρ‹Ρ€Π°ΠΆΠ΅Π½ΠΈΠ΅: (43 – 6 * 3) : 5
  • ΠžΡ‚Π²Π΅Ρ‚: 5

Π—Π°Π΄Π°Ρ‡Π° 4

Π’ Π΄ΠΎΠΌΠ°Ρ… Ρ€Π°ΠΉΠΎΠ½Π° всСго 55 подъСздов. Π’ 7 Π΄ΠΎΠΌΠ°Ρ… ΠΏΠΎ 4 подъСзда ΠΈ Π² Π½Π΅ΡΠΊΠΎΠ»ΡŒΠΊΠΈΡ… Π΄ΠΎΠΌΠ°Ρ… ΠΏΠΎ 3 подъСзда. Бколько Π΄ΠΎΠΌΠΎΠ² с 3 подъСздами?

    РСшСниС:
  • 1) 7 * 4 = 28
  • 2) 55 – 28 = 27
  • 3) 27 : 3 = 9
  • Π’Ρ‹Ρ€Π°ΠΆΠ΅Π½ΠΈΠ΅: (55 – 7 * 4) : 3
  • ΠžΡ‚Π²Π΅Ρ‚: 9
ο»Ώο»Ώ

Π—Π°Π΄Π°Ρ‡Π° 5

ΠšΡƒΠΏΠΈΠ»ΠΈ 3 ΠΏΠ°ΠΊΠ΅Ρ‚Π° ΠΌΠΎΡ€ΠΊΠΎΠ²ΠΈ ΠΏΠΎ 6 ΠΊΠ³ ΠΈ 8 ΠΏΠ°ΠΊΠ΅Ρ‚ΠΎΠ² Π»ΡƒΠΊΠ°. Бколько ΠΊΠΈΠ»ΠΎΠ³Ρ€Π°ΠΌΠΌΠΎΠ² Π»ΡƒΠΊΠ° Π² ΠΏΠ°ΠΊΠ΅Ρ‚Π΅, Ссли всСго ΠΊΡƒΠΏΠΈΠ»ΠΈ 42 ΠΊΠ³?

    РСшСниС:
  • 1) 3 * 6 = 18
  • 2) 42 – 18 = 24
  • 3) 24 : 8 = 3
  • Π’Ρ‹Ρ€Π°ΠΆΠ΅Π½ΠΈΠ΅: (42 – 3 * 6) : 8
  • ΠžΡ‚Π²Π΅Ρ‚: 3

Π—Π°Π΄Π°Ρ‡Π° 6

Π’ Π½Π΅ΡΠΊΠΎΠ»ΡŒΠΊΠΈΡ… ΠΊΠΎΠ½Π²Π΅Ρ€Ρ‚Π°Ρ… Π»Π΅ΠΆΠΈΡ‚ 48 ΠΎΡ‚ΠΊΡ€Ρ‹Ρ‚ΠΎΠΊ. Π’ 8 ΠΊΠΎΠ½Π²Π΅Ρ€Ρ‚Π°Ρ… ΠΏΠΎ 3 ΠΎΡ‚ΠΊΡ€Ρ‹Ρ‚ΠΊΠΈ ΠΈ Π² Π½Π΅ΡΠΊΠΎΠ»ΡŒΠΊΠΈΡ… ΠΊΠΎΠ½Π²Π΅Ρ€Ρ‚Π°Ρ… ΠΏΠΎ 4 ΠΎΡ‚ΠΊΡ€Ρ‹Ρ‚ΠΊΠΈ. Бколько ΠΊΠΎΠ½Π²Π΅Ρ€Ρ‚ΠΎΠ² с 4 ΠΎΡ‚ΠΊΡ€Ρ‹Ρ‚ΠΊΠ°ΠΌΠΈ?

    РСшСниС:
  • 1) 8 * 3 = 24
  • 2) 48 – 24 = 24
  • 3) 24 : 4 = 6
  • Π’Ρ‹Ρ€Π°ΠΆΠ΅Π½ΠΈΠ΅: (48 – 8 * 3) : 4
  • ΠžΡ‚Π²Π΅Ρ‚: 6
ο»Ώο»Ώ

Π—Π°Π΄Π°Ρ‡Π° 7

ΠŸΠΎΡΡ‚Π°Π²ΠΈΠ»ΠΈ 5 рядов ΠΏΠΎ 7 ΡΡ‚ΡƒΠ»ΡŒΠ΅Π² ΠΈ 3 ряда крСсСл. Бколько крСсСл Π² ряду, Ссли всСго поставили 59 ΡΡ‚ΡƒΠ»ΡŒΠ΅Π² ΠΈ крСсСл?

    РСшСниС:
  • 1) 5 * 7 = 35
  • 2) 59 – 35 = 24
  • 3) 24 : 3 = 8
  • Π’Ρ‹Ρ€Π°ΠΆΠ΅Π½ΠΈΠ΅: (59 – 5 * 7) : 3
  • ΠžΡ‚Π²Π΅Ρ‚: 8

Π—Π°Π΄Π°Ρ‡Π° 8

Π’ ΠΊΠΎΡ€ΠΎΠ±ΠΊΠ΅ Π»Π΅ΠΆΠ°Π»ΠΎ 64 Ρ€ΡƒΡ‡ΠΊΠΈ. Π’ 4 Π½Π°Π±ΠΎΡ€Π°Ρ… ΠΏΠΎ 7 Ρ€ΡƒΡ‡Π΅ΠΊ ΠΈ Π² Π½Π΅ΡΠΊΠΎΠ»ΡŒΠΊΠΈΡ… Π½Π°Π±ΠΎΡ€Π°Ρ… ΠΏΠΎ 9 Ρ€ΡƒΡ‡Π΅ΠΊ. Бколько Π±Ρ‹Π»ΠΎ Π½Π°Π±ΠΎΡ€ΠΎΠ² с 9 Ρ€ΡƒΡ‡ΠΊΠ°ΠΌΠΈ?

    РСшСниС:
  • 1) 4 * 7 = 28
  • 2) 64 – 28 = 36
  • 3) 36 : 9 = 4
  • Π’Ρ‹Ρ€Π°ΠΆΠ΅Π½ΠΈΠ΅: (64 – 4 * 7) : 9
  • ΠžΡ‚Π²Π΅Ρ‚: 4

Π—Π°Π΄Π°Ρ‡Π° 9

ΠšΡƒΠΏΠΈΠ»ΠΈ 9 ΠΌΠ°Π»Π΅Π½ΡŒΠΊΠΈΡ… канистр с ΠΌΠΈΠ½Π΅Ρ€Π°Π»ΡŒΠ½ΠΎΠΉ Π²ΠΎΠ΄ΠΎΠΉ ΠΏΠΎ 5 Π» ΠΈ Π΅Ρ‰Ρ‘ 4 большиС канистры. Бколько Π»ΠΈΡ‚Ρ€ΠΎΠ² Π² большой канистрС, Ссли всСго ΠΊΡƒΠΏΠΈΠ»ΠΈ 77 Π»ΠΈΡ‚Ρ€ΠΎΠ² ΠΌΠΈΠ½Π΅Ρ€Π°Π»ΡŒΠ½ΠΎΠΉ Π²ΠΎΠ΄Ρ‹?

    РСшСниС:
  • 1) 9 * 5 = 45
  • 2) 77 – 45 = 32
  • 3) 32 : 4 = 8
  • Π’Ρ‹Ρ€Π°ΠΆΠ΅Π½ΠΈΠ΅: (77 – 9 * 5) : 4
  • ΠžΡ‚Π²Π΅Ρ‚:
    8

Π—Π°Π΄Π°Ρ‡Π° 10

Π’ Π΄ΠΎΠΌΠ΅ 53 стула. Π’ 7 ΠΊΠΎΠΌΠ½Π°Ρ‚Π°Ρ… ΠΏΠΎ 5 ΡΡ‚ΡƒΠ»ΡŒΠ΅Π² ΠΈ Π² Π½Π΅ΡΠΊΠΎΠ»ΡŒΠΊΠΈΡ… ΠΊΠΎΠΌΠ½Π°Ρ‚Π°Ρ… ΠΏΠΎ 3 стула. Бколько ΠΊΠΎΠΌΠ½Π°Ρ‚ с 3 ΡΡ‚ΡƒΠ»ΡŒΡΠΌΠΈ?

    РСшСниС:
  • 1) 7 * 5 = 35
  • 2) 53 – 35 = 18
  • 3) 18 : 3 = 6
  • Π’Ρ‹Ρ€Π°ΠΆΠ΅Π½ΠΈΠ΅: (53 – 7 * 5) :3
  • ΠžΡ‚Π²Π΅Ρ‚: 6

Π—Π°Π΄Π°Ρ‡Π° 11

ΠšΡƒΠΏΠΈΠ»ΠΈ 7 ΠΏΠ°ΠΊΠ΅Ρ‚ΠΎΠ² сахара ΠΏΠΎ 4 ΠΊΠ³ ΠΈ 9 ΠΏΠ°ΠΊΠ΅Ρ‚ΠΎΠ² ΠΊΡ€ΡƒΠΏΡ‹. Бколько ΠΊΠΈΠ»ΠΎΠ³Ρ€Π°ΠΌΠΌΠΎΠ² ΠΊΡ€ΡƒΠΏΡ‹ Π² ΠΎΠ΄Π½ΠΎΠΌ ΠΏΠ°ΠΊΠ΅Ρ‚Π΅, Ссли всСго ΠΊΡƒΠΏΠΈΠ»ΠΈ 73 ΠΊΠ³?

    РСшСниС:
  • 1) 7 * 4 = 28
  • 2) 73 – 28 = 45
  • 3) 45 : 9 = 5
  • Π’Ρ‹Ρ€Π°ΠΆΠ΅Π½ΠΈΠ΅: (73 – 7 * 4) : 9
  • ΠžΡ‚Π²Π΅Ρ‚: 5

Π—Π°Π΄Π°Ρ‡Π° 12

Π’ посёлкС 77 Π΄ΠΎΠΌΠΎΠ². На 4 ΡƒΠ»ΠΈΡ†Π°Ρ… ΠΏΠΎ 8 Π΄ΠΎΠΌΠΎΠ² ΠΈ Π½Π° Π½Π΅ΡΠΊΠΎΠ»ΡŒΠΊΠΈΡ… ΡƒΠ»ΠΈΡ†Π°Ρ… ΠΏΠΎ 9 Π΄ΠΎΠΌΠΎΠ². Бколько ΡƒΠ»ΠΈΡ† с 9 Π΄ΠΎΠΌΠ°ΠΌΠΈ?

    РСшСниС:
  • 1) 4 * 8 = 32
  • 2) 77 – 32 = 45
  • 3) 45 : 9 = 5
  • Π’Ρ‹Ρ€Π°ΠΆΠ΅Π½ΠΈΠ΅: (77 – 4 *8) : 9
  • ΠžΡ‚Π²Π΅Ρ‚: 5

Π—Π°Π΄Π°Ρ‡Π° 13

Посадили 3 ряда астр ΠΏΠΎ 6 Ρ†Π²Π΅Ρ‚ΠΊΠΎΠ² Π² ΠΊΠ°ΠΆΠ΄ΠΎΠΌ ряду ΠΈ 4 ряда нарциссов. Бколько нарциссов Π² ΠΎΠ΄Π½ΠΎΠΌ ряду, Ссли всСго посадили 50 Ρ†Π²Π΅Ρ‚ΠΊΠΎΠ²?

    РСшСниС:
  • 1) 3 * 6 = 18
  • 2) 50 – 18 = 32
  • 3) 32 : 4 = 8
  • Π’Ρ‹Ρ€Π°ΠΆΠ΅Π½ΠΈΠ΅: (50 – 3 * 6) : 4
  • ΠžΡ‚Π²Π΅Ρ‚: 8

Π—Π°Π΄Π°Ρ‡Π° 14

На Π½Π΅ΡΠΊΠΎΠ»ΡŒΠΊΠΈΡ… Ρ‚ΠΎΡ€Ρ‚Π°Ρ… 46 ΠΌΠ°Ρ€Ρ†ΠΈΠΏΠ°Π½ΠΎΠ²Ρ‹Ρ… яблок. На 7 Ρ‚ΠΎΡ€Ρ‚Π°Ρ… ΠΏΠΎ 3 яблока ΠΈ Π½Π° Π½Π΅ΡΠΊΠΎΠ»ΡŒΠΊΠΈΡ… Ρ‚ΠΎΡ€Ρ‚Π°Ρ… ΠΏΠΎ 5 яблок. Бколько Ρ‚ΠΎΡ€Ρ‚ΠΎΠ² с 5 яблоками?

    РСшСниС:
  • 1) 7 * 3 = 21
  • 2) 46 – 21 = 25
  • 3) 25 : 5 = 5
  • Π’Ρ‹Ρ€Π°ΠΆΠ΅Π½ΠΈΠ΅: (46 – 7 * 3) : 5
  • ΠžΡ‚Π²Π΅Ρ‚: 5

Π—Π°Π΄Π°Ρ‡Π° 15

Посадили 2 ряда Π³Ρ€ΡƒΡˆ ΠΏΠΎ 8 Π΄Π΅Ρ€Π΅Π²ΡŒΠ΅Π² ΠΈ 4 ряда вишСн. Бколько вишСн Π² ряду, Ссли всСго посадили 44 Π΄Π΅Ρ€Π΅Π²Π°?

    РСшСниС:
  • 1) 2 * 8 = 16
  • 2) 44 – 16 = 28
  • 3) 28 : 4 = 7
  • Π’Ρ‹Ρ€Π°ΠΆΠ΅Π½ΠΈΠ΅: (44 – 2 * 8) : 4
  • ΠžΡ‚Π²Π΅Ρ‚: 7
ο»Ώ

На страницС ΠΈΡΠΏΠΎΠ»ΡŒΠ·ΠΎΠ²Π°Π½Ρ‹ ΠΌΠ°Ρ‚Π΅Ρ€ΠΈΠ°Π»Ρ‹ ΠΈΠ· ΠΊΠ½ΠΈΠ³ΠΈ О. Π’. Π£Π·ΠΎΡ€ΠΎΠ²ΠΎΠΉ, Π•. А. НСфСдовой Β«300 Π·Π°Π΄Π°Ρ‡ ΠΏΠΎ ΠΌΠ°Ρ‚Π΅ΠΌΠ°Ρ‚ΠΈΠΊΠ΅. 3 класс Β» 2007Π³.

ο»Ώ

Π‘Π±ΠΎΡ€Π½ΠΈΠΊ Ρ€Π°Π·Π½ΠΎΡƒΡ€ΠΎΠ²Π½Π΅Π²Ρ‹Ρ… Π·Π°Π΄Π°Π½ΠΈΠΉ ΠΏΠΎ ΠΌΠ°Ρ‚Π΅ΠΌΠ°Ρ‚ΠΈΠΊΠ΅ (3-4 классы)

Β  Β 

Β 

ΠŸΠΎΡΡΠ½ΠΈΡ‚Π΅Π»ΡŒΠ½Π°Ρ записка

ΠΠ°Ρ‡Π°Π»ΡŒΠ½Π°Ρ школа являСтся Π²Π°ΠΆΠ½Ρ‹ΠΌ этапом возрастного развития ΠΈ становлСния личности Π΄Π΅Ρ‚Π΅ΠΉ, ΠΎΠ½Π° Π΄ΠΎΠ»ΠΆΠ½Π° ΠΈ Π½Π΅ΠΏΡ€Π΅ΠΌΠ΅Π½Π½ΠΎ обязана Π³Π°Ρ€Π°Π½Ρ‚ΠΈΡ€ΠΎΠ²Π°Ρ‚ΡŒ высокий ΡƒΡ€ΠΎΠ²Π΅Π½ΡŒ образования. Π’ настоящСС врСмя ΠΎΠ΄Π½Π° ΠΈΠ· Π²Π΅Π΄ΡƒΡ‰ΠΈΡ… Ρ‚Π΅Π½Π΄Π΅Π½Ρ†ΠΈΠΉ Π² Ρ€Π°Π·Π²ΠΈΡ‚ΠΈΠΈ нашСй Π½Π°Ρ‡Π°Π»ΡŒΠ½ΠΎΠΉ ΡˆΠΊΠΎΠ»Ρ‹ – Π΅Ρ‘ Π΄ΠΈΡ„Ρ„Π΅Ρ€Π΅Π½Ρ†ΠΈΡ€ΠΎΠ²Π°Π½Π½ΠΎΠ΅ ΠΎΠ±ΡƒΡ‡Π΅Π½ΠΈΠ΅. ΠΠΊΡ‚ΡƒΠ°Π»ΡŒΠ½ΠΎΡΡ‚ΡŒ Π΄ΠΈΡ„Ρ„Π΅Ρ€Π΅Π½Ρ†ΠΈΡ€ΠΎΠ²Π°Π½Π½ΠΎΠ³ΠΎ ΠΏΠΎΠ΄Ρ…ΠΎΠ΄Π° в совокупности Π΅Π³ΠΎ ΠΊΠΎΠΌΠΏΠΎΠ½Π΅Π½Ρ‚ΠΎΠ² Π²Ρ‹Π΄Π²ΠΈΠ³Π°Π΅Ρ‚ Π½Π°Β ΠΏΠ΅Ρ€Π²Ρ‹ΠΉ ΠΏΠ»Π°Π½ ΠΎΡ€Π³Π°Π½ΠΈΠ·Π°Ρ†ΠΈΡŽ ΠΈΒ ΡƒΠΏΡ€Π°Π²Π»Π΅Π½ΠΈΠ΅ Ρ†Π΅Π»Π΅Π½Π°ΠΏΡ€Π°Π²Π»Π΅Π½Π½ΠΎΠΉ ΡƒΡ‡Π΅Π±Π½ΠΎΠΉ Π΄Π΅ΡΡ‚Π΅Π»ΡŒΠ½ΠΎΡΡ‚ΠΈ Π²Β ΠΎΠ±Ρ‰Π΅ΠΌ контСкстС Π΅Π³ΠΎ ΠΆΠΈΠ·Π½Π΅Π΄Π΅ΡΡ‚Π΅Π»ΡŒΠ½ΠΎΡΡ‚ΠΈΒ β€” цСлСнаправлСнности интСрСсов, ΠΆΠΈΠ·Π½Π΅Π½Π½Ρ‹Ρ… ΠΏΠ»Π°Π½ΠΎΠ², цСнностных ΠΎΡ€ΠΈΠ΅Π½Ρ‚Π°Ρ†ΠΈΠ΅ΠΉ, понимания смысла ΠΆΠΈΠ·Π½ΠΈ для развития творчСского ΠΏΠΎΡ‚Π΅Π½Ρ†ΠΈΠ°Π»Π° личности.

ΠŸΠΎΠ½ΡΡ‚ΠΈΠ΅ β€œΠ”ΠΈΡ„Ρ„Π΅Ρ€Π΅Π½Ρ†ΠΈΡ€ΠΎΠ²Π°Π½Π½ΠΎΠ΅ ( Ρ€Π°Π·Π½ΠΎΡƒΡ€ΠΎΠ²Π½Π΅Π²ΠΎΠ΅ ) обучСниС” Π² ΠΏΠ΅Ρ€Π΅Π²ΠΎΠ΄Π΅ с латинского β€œdifferent” ΠΎΠ·Π½Π°Ρ‡Π°Π΅Ρ‚ Ρ€Π°Π·Π΄Π΅Π»Π΅Π½ΠΈΠ΅, Ρ€Π°Π·Π»ΠΎΠΆΠ΅Π½ΠΈΠ΅ Ρ†Π΅Π»ΠΎΠ³ΠΎ Π½Π° Ρ€Π°Π·Π»ΠΈΡ‡Π½Ρ‹Π΅ части, Ρ„ΠΎΡ€ΠΌΡ‹, ступСни, ΡƒΡ€ΠΎΠ²Π½ΠΈ.

Π¦Π΅Π»ΠΈ Π΄ΠΈΡ„Ρ„Π΅Ρ€Π΅Π½Ρ†ΠΈΡ€ΠΎΠ²Π°Π½Π½ΠΎΠ³ΠΎ обучСния: организация ΡƒΡ‡Π΅Π±Π½ΠΎΠ³ΠΎ процСсса Π½Π° основС ΡƒΡ‡Π΅Ρ‚Π° ΠΈΠ½Π΄ΠΈΠ²ΠΈΠ΄ΡƒΠ°Π»ΡŒΠ½Ρ‹Ρ… особСнностСй личности, Ρ‚.Π΅. Π½Π° ΡƒΡ€ΠΎΠ²Π½Π΅ Π΅Π³ΠΎ возмоТностСй ΠΈ способностСй.

Основная Π·Π°Π΄Π°Ρ‡Π°: ΠΏΠΎΠ²Ρ‹ΡΠΈΡ‚ΡŒ Ρ€Π΅Π·ΡƒΠ»ΡŒΡ‚Π°Ρ‚Ρ‹ обучСния ΠΈΒ Ρ€Π°Π·Π²ΠΈΠ²Π°Ρ‚ΡŒ ΠΏΠΎΠ·Π½Π°Π²Π°Ρ‚Π΅Π»ΡŒΠ½Ρ‹Π΅ процСссы, поэтому здСсь прСдставлСны упраТнСния для учащихся с Π±Π°Π·ΠΎΠ²Ρ‹ΠΌΠΈ знаниями ΠΏΡ€Π΅Π΄ΠΌΠ΅Ρ‚Π° Π½Π° срСднСм ΠΈ Π½ΠΈΠ·ΠΊΠΎΠΌ ΡƒΡ€ΠΎΠ²Π½Π΅, Π° Ρ‚Π°ΠΊΠΆΠ΅ задания ΠΏΠΎΠ²Ρ‹ΡˆΠ΅Π½Π½ΠΎΠΉ слоТности.

Π˜Π½ΡΡ‚Ρ€ΡƒΠΌΠ΅Π½Ρ‚Π°Ρ€ΠΈΠΉ сборника – это ΠΊΠ°Ρ€Ρ‚ΠΎΡ‡ΠΊΠΈ ΠΈ задания для Π΄ΠΎΠ±Ρ€ΠΎΠ²ΠΎΠ»ΡŒΠ½ΠΎΠ³ΠΎ выполнСния ΠΏΠΎ курсу ΠΌΠ°Ρ‚Π΅ΠΌΠ°Ρ‚ΠΈΠΊΠΈ Π² 3-4 классах с инструкциСй ΠΏΠΎ Π²Ρ‹ΠΏΠΎΠ»Π½Π΅Π½ΠΈΡŽ ΠΈ шкалой оцСнивания. Данная Ρ€Π°Π·Ρ€Π°Π±ΠΎΡ‚ΠΊΠ° Π²ΠΊΠ»ΡŽΡ‡Π°Π΅Ρ‚ Ρ‚Ρ€Π΅Ρ…ΡΡ‚ΠΎΡ€ΠΎΠ½Π½ΡŽΡŽ Π½Π°ΠΏΡ€Π°Π²Π»Π΅Π½Π½ΠΎΡΡ‚ΡŒ:

1) ΠΏΠΎ этапам ΡƒΡ‡Π΅Π±Π½ΠΎΠ³ΠΎ процСсса;

2) ΠΏΠΎ Ρ‚Π΅ΠΌΠ°ΠΌ;

3) ΠΏΠΎ уровням успСваСмости учащихся.

1)Π§Ρ‚ΠΎΠ±Ρ‹ Π² процСссС обучСния большС Π²Π»ΠΈΡΡ‚ΡŒ Π½Π° ΠΏΡ€ΠΎΠ΄Π²ΠΈΠΆΠ΅Π½ΠΈΠ΅ ΡƒΡ‡Π΅Π½ΠΈΠΊΠΎΠ² Π² Ρ€Π°Π·Π²ΠΈΡ‚ΠΈΠΈ ΠΌΡ‹ΡˆΠ»Π΅Π½ΠΈΡ, цСлСсообразно Π²Π²ΠΎΠ΄ΠΈΡ‚ΡŒ Π±ΠΎΠ»Π΅Π΅ Ρ‚Ρ€ΡƒΠ΄Π½Ρ‹Π΅ вопросы ΠΈ задания Π½Π° всСх этапах обучСния. РазноуровнСвая диффСрСнциация обучСния ΡˆΠΈΡ€ΠΎΠΊΠΎ примСняСтся ΠΏΡ€ΠΈ ΠΈΠ·ΡƒΡ‡Π΅Π½ΠΈΠΈ Π½ΠΎΠ²ΠΎΠ³ΠΎ ΠΌΠ°Ρ‚Π΅Ρ€ΠΈΠ°Π»Π°; Π² Π΄ΠΈΡ„Ρ„Π΅Ρ€Π΅Π½Ρ†ΠΈΡ€ΠΎΠ²Π°Π½Π½ΠΎΠΉ домашнСй Ρ€Π°Π±ΠΎΡ‚Π΅; ΡƒΡ‡Π΅Ρ‚ Π·Π½Π°Π½ΠΈΠΉ Π½Π° ΡƒΡ€ΠΎΠΊΠ΅; Π² Ρ‚Π΅ΠΊΡƒΡ‰Π΅ΠΉ ΠΏΡ€ΠΎΠ²Π΅Ρ€ΠΊΠ΅ усвоСния ΠΏΡ€ΠΎΠΉΠ΄Π΅Π½Π½ΠΎΠ³ΠΎ ΠΌΠ°Ρ‚Π΅Ρ€ΠΈΠ°Π»Π°; ΡΠ°ΠΌΠΎΡΡ‚ΠΎΡΡ‚Π΅Π»ΡŒΠ½Ρ‹Ρ… Ρ€Π°Π±ΠΎΡ‚Π°Ρ…; Π½Π° ΡƒΡ€ΠΎΠΊΠ°Ρ… закрСплСния.

2) Π‘Π±ΠΎΡ€Π½ΠΈΠΊ содСрТит Ρ‡Π°ΡΡ‚ΡŒ Ρ‚Π΅ΠΌ ΠΏΠΎ курсу ΠΌΠ°Ρ‚Π΅ΠΌΠ°Ρ‚ΠΈΠΊΠΈ Π² 3-4 классах ΠΈ являСтся Π΄ΠΎΠΏΠΎΠ»Π½Π΅Π½ΠΈΠ΅ΠΌ ΠΊ ΡƒΡ‡Π΅Π±Π½ΠΈΠΊΡƒ ΠΈ Π΄Ρ€ΡƒΠ³ΠΈΠΌ дидактичСским ΠΌΠ°Ρ‚Π΅Ρ€ΠΈΠ°Π»Π°ΠΌ.

3) По Ρ€Π΅Π·ΡƒΠ»ΡŒΡ‚Π°Ρ‚Π°ΠΌ диагностирования класс Π΄Π΅Π»ΠΈΠΌ ΠΏΠΎ уровням (Π½ΠΈΠ·ΠΊΠΈΠ΅, срСдниС, высокиС ΡƒΡ‡Π΅Π±Π½Ρ‹Π΅ способности) ΠΈ ΠΏΠ°Ρ€Π°Π»Π»Π΅Π»ΡŒΠ½ΠΎ ΠΏΡ€Π΅Π΄Π»Π°Π³Π°Π΅ΠΌ ΡƒΡ‡Π΅Π½ΠΈΠΊΠ°ΠΌ самим ΠΎΠΏΡ€Π΅Π΄Π΅Π»ΡΡ‚ΡŒ свои возмоТности – Π²Ρ‹Π±ΠΎΡ€ уровня задания (ΠΏΠ΅Ρ€Π΅Ρ…ΠΎΠ΄ Π½Π° ΡΠ»Π΅Π΄ΡƒΡŽΡ‰ΠΈΠΉ)+ самооцСнка ( ΠΎΠ½Π° Π΄ΠΎΠ»ΠΆΠ½Π° Π±Ρ‹Ρ‚ΡŒ Π°Π΄Π΅ΠΊΠ²Π°Ρ‚Π½ΠΎΠΉ). Π’ случаС Ссли ΡƒΡ‡Π΅Π½ΠΈΠΊ Π½Π΅ выполняСт Π·Π°Π΄Π°Π½ΠΈΠ΅ высокого уровня слоТности, Ρ‚ΠΎ дСйствуСт дальнСйшСС выявлСниС Π³Ρ€Π°Π½ΠΈΡ†Ρ‹, ΠΎΡ‚ ΠΊΠΎΡ‚ΠΎΡ€ΠΎΠΉ ΠΎΠ½ Π½Π°Ρ‡ΠΈΠ½Π°Π΅Ρ‚ ΡΠ°ΠΌΠΎΡΡ‚ΠΎΡΡ‚Π΅Π»ΡŒΠ½ΠΎ Ρ€Π°Π±ΠΎΡ‚Π°Ρ‚ΡŒ.

ΠŸΡ€Π΅Π΄ΡΡ‚Π°Π²Π»Π΅Π½Π½Ρ‹ΠΉ сборник ΠΏΠΎΠ·Π²ΠΎΠ»ΠΈΡ‚ ΠΏΠ΅Π΄Π°Π³ΠΎΠ³Π°ΠΌ Π½Π°Ρ‡Π°Π»ΡŒΠ½ΠΎΠΉ ΡˆΠΊΠΎΠ»Ρ‹, рСгулярно занимаясь с Π΄Π΅Ρ‚ΡŒΠΌΠΈ, ΠΏΠΎΠ²Ρ‹ΡΠΈΡ‚ΡŒ ΡƒΡ€ΠΎΠ²Π΅Π½ΡŒ успСваСмости ΠΈ Ρ€Π°Π·Π²ΠΈΠ²Π°Ρ‚ΡŒ:

– логичСскоС ΠΌΡ‹ΡˆΠ»Π΅Π½ΠΈΠ΅;

-ΠΌΡ‹ΡΠ»ΠΈΡ‚Π΅Π»ΡŒΠ½Ρ‹Π΅ ΠΎΠΏΠ΅Ρ€Π°Ρ†ΠΈΠΈ Π°Π½Π°Π»ΠΈΠ·Π° ΠΈ синтСза;

-ΠΏΠΎΠ·Π½Π°Π²Π°Ρ‚Π΅Π»ΡŒΠ½Ρ‹ΠΉ интСрСс;

-ΡΠ°ΠΌΠΎΡΡ‚ΠΎΡΡ‚Π΅Π»ΡŒΠ½ΠΎΡΡ‚ΡŒ;

-Π°Π΄Π΅ΠΊΠ²Π°Ρ‚Π½ΡƒΡŽ самооцСнку.

Π’Π°Ρ€ΠΈΠ°Ρ‚ΠΈΠ²Π½ΠΎΡΡ‚ΡŒ сборника ΠΏΠΎΠ·Π²ΠΎΠ»ΠΈΡ‚ ΡƒΡ‡Π΅Π½ΠΈΠΊΠ°ΠΌ Π±ΠΎΠ»Π΅Π΅ осознанно ΡƒΡΠ²Π°ΠΈΠ²Π°Ρ‚ΡŒ ΠΈΠ·ΡƒΡ‡Π°Π΅ΠΌΡ‹ΠΉ ΠΌΠ°Ρ‚Π΅Ρ€ΠΈΠ°Π», Π½Π΅ Π±Ρ‹Ρ‚ΡŒ пассивными ΡΠ»ΡƒΡˆΠ°Ρ‚Π΅Π»ΡΠΌΠΈ, Π° ΡΡ‚Π°Π½ΠΎΠ²ΠΈΡ‚ΡŒΡΡ Π°ΠΊΡ‚ΠΈΠ²Π½Ρ‹ΠΌΠΈ участниками ΡƒΡ€ΠΎΠΊΠ°, Π½ΠΎ ΠΏΡ€ΠΈ условии ΡΠ°ΠΌΠΎΡΡ‚ΠΎΡΡ‚Π΅Π»ΡŒΠ½ΠΎΠ³ΠΎ выполнСния Π·Π°Π΄Π°Π½ΠΈΠΉ.

ΠšΡ€ΠΈΡ‚Π΅Ρ€ΠΈΠΈ оцСнивания:


Β 

Β«5Β» Ρ€Π°Π±ΠΎΡ‚Π° Π²Ρ‹ΠΏΠΎΠ»Π½Π΅Π½Π° Π²Π΅Ρ€Π½ΠΎ.

Β«4Β» Π΄ΠΎΠΏΡƒΡ‰Π΅Π½Ρ‹ 1-2 ошибки.

Β«3Β» 1 Π·Π°Π΄Π°Π½ΠΈΠ΅ Π²Ρ‹ΠΏΠΎΠ»Π½Π΅Π½ΠΎ Π½Π΅Π²Π΅Ρ€Π½ΠΎ, Π»ΠΈΠ±ΠΎ Π΅ΡΡ‚ΡŒ 3-4 ошибки.

«2» болСС 5 ошибок.

Π˜Π·ΡƒΡ‡Π΅Π½ΠΈΠ΅ Π½ΠΎΠ²ΠΎΠ³ΠΎ ΠΌΠ°Ρ‚Π΅Ρ€ΠΈΠ°Π»Π°.

Π’Π•ΠœΠ: Β«Π’Π Π•ΠœΠ―Β». 3 класс.

ЦСль: актуализация ΠΎΠΏΠΎΡ€Π½Ρ‹Ρ… Π·Π½Π°Π½ΠΈΠΉ.

Π˜Π½ΡΡ‚Ρ€ΡƒΠΊΡ†ΠΈΡ: Вспомни Π΅Π΄ΠΈΠ½ΠΈΡ†Ρ‹ Π²Ρ€Π΅ΠΌΠ΅Π½ΠΈ. Π’Ρ‹Π±Π΅Ρ€ΠΈ ΠΊΠ°Ρ€Ρ‚ΠΎΡ‡ΠΊΡƒ ΠΈ Ρ€Π΅ΡˆΠΈ. Какой ΡƒΡ€ΠΎΠ²Π΅Π½ΡŒ Ρ‚Ρ‹ Π²Ρ‹Π±Ρ€Π°Π»?

1 Π²Π°Ρ€ΠΈΠ°Π½Ρ‚ 2 Π²Π°Ρ€ΠΈΠ°Π½Ρ‚

Β 

Π—Π°ΠΏΠΈΡˆΠΈ Π²Π΅Π»ΠΈΡ‡ΠΈΠ½Ρ‹ Π² порядкС возрастания:

30 с, 32 Ρ‡, 7 Π»Π΅Ρ‚, 34 ΠΌΠΈΠ½, 1 Π²Π΅ΠΊ, 10 с

Β 

Π Π°Π·Π΄Π΅Π»ΠΈ Π½Π° Π³Ρ€ΡƒΠΏΠΏΡ‹ Π²Π΅Π»ΠΈΡ‡ΠΈΠ½Ρ‹.

1 Ρ‡,1 Ρ‚, 1 ΠΌΠΈΠ½,1 с, 1 Ρ†, 1 Π³ΠΎΠ΄,1 ΠΊΠ³,

1 см, 1 Π³Ρ€Π°ΠΌΠΌ.

Β 

Π—Π°ΠΏΠΎΠ»Π½ΠΈ пропуски

1) 2 Ρ‡ 10 ΠΌΠΈΠ½ =…мин

2) 1 Ρ‡ 45 ΠΌΠΈΠ½ =…мин

3) 5Ρ‡ =…мин

4) 3 Ρ‡ =…мин

Β 

ΠŸΡ€ΠΎΠ²Π΅Ρ€ΡŒ, Π²Π΅Ρ€Π½Ρ‹ Π»ΠΈ нСравСнства?

3 Ρ‡ 10 ΠΌΠΈΠ½ > 310 ΠΌΠΈΠ½

5 мин 30 с < 330 с

52 Ρ‡ > 2 сут

1 Ρ‡ = 60 с

Β 

Найди Π·Π½Π°Ρ‡Π΅Π½ΠΈΠ΅ Π²Ρ‹Ρ€Π°ΠΆΠ΅Π½ΠΈΠΉ:

1) 2 Ρ‡ 45 ΠΌΠΈΠ½ + 15 ΠΌΠΈΠ½

2) 3 Ρ‡ 55 ΠΌΠΈΠ½ + 2 Ρ‡ 20 ΠΌΠΈΠ½

3) 4 Ρ‡ 10 ΠΌΠΈΠ½ – 35 ΠΌΠΈΠ½

4) 3 Ρ‡ 50 ΠΌΠΈΠ½ + 5 ΠΌΠΈΠ½

Β 

Найди Π·Π½Π°Ρ‡Π΅Π½ΠΈΠ΅ Π²Ρ‹Ρ€Π°ΠΆΠ΅Π½ΠΈΠΉ:

1Ρ‡:15ΠΌΠΈΠ½

30ΠΌΠΈΠ½:6

2сут:4

2мин 10 с:10 с

    Β 

    Β 

    Β 

    Β 

    Β 

    Β 

    Β 

    Β 

    Β 

    Β 

    Β 

    Β 

    Β 

    Β 

    Β 

    Β 

    Β 

    Β 

    Β 

    Π’Π•ΠœΠ: Β«Π£ΠœΠΠžΠ–Π•ΠΠ˜Π• ΠœΠΠžΠ“ΠžΠ—ΠΠΠ§ΠΠžΠ“Πž Π§Π˜Π‘Π›Π НА ΠžΠ”ΠΠžΠ—ΠΠΠ§ΠΠžΠ• Β». 4 класс.

    ЦСль: актуализация ΠΎΠΏΠΎΡ€Π½Ρ‹Ρ… Π·Π½Π°Π½ΠΈΠΉ.

    Π˜Π½ΡΡ‚Ρ€ΡƒΠΊΡ†ΠΈΡ: Π˜ΡΠΏΠΎΠ»ΡŒΠ·ΡƒΡ Π°Π»Π³ΠΎΡ€ΠΈΡ‚ΠΌ, вычисли Π·Π½Π°Ρ‡Π΅Π½ΠΈΠ΅. Π’Ρ‹Π±Π΅Ρ€ΠΈ ΠΏΡ€ΠΈΠΌΠ΅Ρ€ ΠΈ Ρ€Π΅ΡˆΠΈ. Какой ΡƒΡ€ΠΎΠ²Π΅Π½ΡŒ Ρ‚Ρ‹ Π²Ρ‹Π±Ρ€Π°Π»?

    1 Π²Π°Ρ€ΠΈΠ°Π½Ρ‚Β  Β  Β  Β  Β  Β  Β  Β  Β  Β  Β  Β  Β  Β  Β  Β  Β  Β  Β  Β  Β  Β  Β  Β  2 Π²Π°Ρ€ΠΈΠ°Π½Ρ‚

    1 1234 x 1Β  Β  Β  Β  Β  Β  Β  Β  Β  Β  Β  Β  Β  Β  Β  Β  Β  Β  Β  Β  Β  Β  Β  Β  Β  Β  Β  Β  Β  Β  1 2143 x 1

    12134 x 2Β  Β  Β  Β  Β  Β  Β  Β  Β  Β  Β  Β  Β  Β  Β  Β  Β  Β  Β  Β  Β  Β  Β  Β  Β  Β  Β  Β  Β  Β  12234 x 2

    2 2314 x 4Β  Β  Β  Β  Β  Β  Β  Β  Β  Β  Β  Β  Β  Β  Β  Β  Β  Β  Β  Β  Β  Β  Β  Β  Β  Β  Β  Β  Β  Β  2 2314 x 4

    25134 x 4Β  Β  Β  Β  Β  Β  Β  Β  Β  Β  Β  Β  Β  Β  Β  Β  Β  Β  Β  Β  Β  Β  Β  Β  Β  Β  Β  Β  Β  Β  Β 24531 x 4

    3 6891 x 7Β  Β  Β  Β  Β  Β  Β  Β  Β  Β  Β  Β  Β  Β  Β  Β  Β  Β  Β  Β  Β  Β  Β  Β  Β  Β  Β  Β  Β  Β  3 7586 x 5

    6891 x 70Β  Β  Β  Β  Β  Β  Β  Β  Β  Β  Β  Β  Β  Β  Β  Β  Β  Β  Β  Β  Β  Β  Β  Β  Β  Β  Β  Β  Β  Β  Β  Β 7586 x 50

    Β 

    Β 

    ДиффСрСнцированная домашняя Ρ€Π°Π±ΠΎΡ‚Π°.

    Π’Π•ΠœΠ: Β« Π—ΠΠ”ΠΠ§Π˜: ЦЕНА. ΠšΠžΠ›Π˜Π§Π•Π‘Π’Π’Πž. БВОИМОБВЬ». 3 класс.

    ЦСль: Ρ€Π°Π·Π²ΠΈΡ‚ΠΈΠ΅ умСния Ρ€Π΅ΡˆΠ°Ρ‚ΡŒ Π·Π°Π΄Π°Ρ‡ΠΈ Π½Π° Π½Π°Ρ…ΠΎΠΆΠ΄Π΅Π½ΠΈΠ΅ Ρ†Π΅Π½Ρ‹, количСства, стоимости.

    1 Π²Π°Ρ€ΠΈΠ°Π½Ρ‚

    Π—Π°Π΄Π°Ρ‡Π°. ΠœΠ΅Ρ‚Ρ€ шСлка стоит 20 Ρ€., Π° ΠΌΠ΅Ρ‚Ρ€ ситца -5 Ρ€. На ΠΏΠ»Π°Ρ‚ΡŒΠ΅ Π½ΡƒΠΆΠ½ΠΎ 3 ΠΌ шСлка ΠΈΠ»ΠΈ 2 ΠΌ 50 см ситца. Бколько шСлка ΠΈ ситца Π½ΡƒΠΆΠ½ΠΎ Π½Π° 1 ΠΏΠ»Π°Ρ‚ΡŒΠ΅?

    1 ΡƒΡ€ΠΎΠ²Π΅Π½ΡŒ. РСши Π·Π°Π΄Π°Ρ‡Ρƒ ΠΏΠΎ дСйствиям.

    2 ΡƒΡ€ΠΎΠ²Π΅Π½ΡŒ. Π—Π°ΠΏΠΈΡˆΠΈ Ρ€Π΅ΡˆΠ΅Π½ΠΈΠ΅ Π·Π°Π΄Π°Ρ‡ΠΈ Π²Ρ‹Ρ€Π°ΠΆΠ΅Π½ΠΈΠ΅ΠΌ.

    3 ΡƒΡ€ΠΎΠ²Π΅Π½ΡŒ. Из ΠΊΠ°ΠΊΠΎΠΉ Ρ‚ΠΊΠ°Π½ΠΈ ΠΏΠ»Π°Ρ‚ΡŒΠ΅ Π±ΡƒΠ΄Π΅Ρ‚ Π΄ΠΎΡ€ΠΎΠΆΠ΅ ΠΈ Π½Π° сколько?

    2 Π²Π°Ρ€ΠΈΠ°Π½Ρ‚

    Π—Π°Π΄Π°Ρ‡Π° Π£ΠΏΠ°ΠΊΠΎΠ²ΠΊΠ° ΠΈΠ· 3 Π±ΡƒΠ»ΠΎΡ‡Π΅ΠΊ стоит 6 Ρ€. Бколько Π±ΡƒΠ»ΠΎΡ‡Π΅ΠΊ ΠΌΠΎΠΆΠ½ΠΎ ΠΊΡƒΠΏΠΈΡ‚ΡŒ Π½Π° 24 Ρ€.?

    Π£Ρ€ΠΎΠ²Π΅Π½ΡŒ 1)Π—Π°ΠΏΠΈΡˆΠΈ условиС Π·Π°Π΄Π°Ρ‡ΠΈ Π² Ρ‚Π°Π±Π»ΠΈΡ†Π΅, Ρ€Π΅ΡˆΠΈ Π·Π°Π΄Π°Ρ‡Ρƒ.

    ЦЕНА

    ΠšΠžΠ›Π˜Π§Π•Π‘Π’Π’Πž

    БВОИМОБВЬ

    Β 

    Β 

    Β 

    Π£Ρ€ΠΎΠ²Π΅Π½ΡŒ 2)РСши Π·Π°Π΄Π°Ρ‡Ρƒ Π²Ρ‹Ρ€Π°ΠΆΠ΅Π½ΠΈΠ΅ΠΌ.

    Π£Ρ€ΠΎΠ²Π΅Π½ΡŒ 3)РСши Π·Π°Π΄Π°Ρ‡Ρƒ устно. Π‘ΠΎΡΡ‚Π°Π²ΡŒ ΠΎΠ±Ρ€Π°Ρ‚Π½ΡƒΡŽ Π·Π°Π΄Π°Ρ‡Ρƒ Π½Π° Π½Π°Ρ…ΠΎΠΆΠ΄Π΅Π½ΠΈΠ΅ стоимости ΠΈ Ρ€Π΅ΡˆΠΈ Π΅Ρ‘.

    Β 

    Π’Π•ΠœΠ: Β«Π—ΠΠ”ΠΠ§Π˜ НА Π”Π’Π˜Π–Π•ΠΠ˜Π•Β». 4 класс.

    ЦСль: Ρ€Π°Π·Π²ΠΈΠ²Π°Ρ‚ΡŒ ΡƒΠΌΠ΅Π½ΠΈΠ΅ Ρ€Π΅ΡˆΠ°Ρ‚ΡŒ Π·Π°Π΄Π°Ρ‡ΠΈ Π½Π° Π΄Π²ΠΈΠΆΠ΅Π½ΠΈΠ΅.

    1 Π²Π°Ρ€ΠΈΠ°Π½Ρ‚

    Π—Π°Π΄Π°Ρ‡Π°. ПоСзд ΠΏΡ€ΠΎΡˆΠ΅Π» Π±Π΅Π· остановок 420 ΠΊΠΌ со ΡΠΊΠΎΡ€ΠΎΡΡ‚ΡŒΡŽ 70 ΠΊΠΌ/Ρ‡, послС остановки Π½Π° 14 ΠΌΠΈΠ½ΡƒΡ‚ ΠΏΡ€ΠΎΡˆΠ΅Π» Π΅Ρ‰Π΅ 300 ΠΊΠΌ со ΡΠΊΠΎΡ€ΠΎΡΡ‚ΡŒΡŽ 75 ΠΊΠΌ/Ρ‡. КакоС расстояниС ΠΎΠ½ ΠΏΡ€Π΅ΠΎΠ΄ΠΎΠ»Π΅Π»?

    1 ΡƒΡ€ΠΎΠ²Π΅Π½ΡŒ РСши Π·Π°Π΄Π°Ρ‡Ρƒ ΠΏΠΎ дСйствиям. Какая Π²Π΅Π»ΠΈΡ‡ΠΈΠ½Π° Π² условии лишняя?

    2 ΡƒΡ€ΠΎΠ²Π΅Π½ΡŒ Π—Π°ΠΏΠΈΡˆΠΈ Ρ€Π΅ΡˆΠ΅Π½ΠΈΠ΅ Π·Π°Π΄Π°Ρ‡ΠΈ Π²Ρ‹Ρ€Π°ΠΆΠ΅Π½ΠΈΠ΅ΠΌ.

    3 ΡƒΡ€ΠΎΠ²Π΅Π½ΡŒ КакоС расстояниС ΠΏΡ€ΠΎΡˆΠ΅Π» ΠΏΠΎΠ΅Π·Π΄ ΠΎΠ±Ρ€Π°Ρ‚Π½ΠΎ?

    2 Π²Π°Ρ€ΠΈΠ°Π½Ρ‚

    Π—Π°Π΄Π°Ρ‡Π° ΠΠ²Ρ‚ΠΎΠΌΠΎΠ±ΠΈΠ»ΡŒ ΠΏΡ€ΠΎΠ΅Ρ…Π°Π» 180 ΠΊΠΌ Π·Π° Π΄Π²Π° с ΠΏΠΎΠ»ΠΎΠ²ΠΈΠ½ΠΎΠΉ часа. Вычисли Π΅Π³ΠΎ ΡΠΊΠΎΡ€ΠΎΡΡ‚ΡŒ.

    Π£Ρ€ΠΎΠ²Π΅Π½ΡŒ 1)Π—Π°ΠΏΠΈΡˆΠΈ условиС Π·Π°Π΄Π°Ρ‡ΠΈ Π² Ρ‚Π°Π±Π»ΠΈΡ†Π΅, Ρ€Π΅ΡˆΠΈ Π·Π°Π΄Π°Ρ‡Ρƒ.

    Π‘ΠΊΠΎΡ€ΠΎΡΡ‚ΡŒ

    ВрСмя

    РасстояниС

    Β 

    Β 

    Β 

    Π£Ρ€ΠΎΠ²Π΅Π½ΡŒ 2)Π‘Π΄Π΅Π»Π°ΠΉ ΠΊ Π·Π°Π΄Π°Ρ‡Π΅ Ρ‡Π΅Ρ€Ρ‚Ρ‘ΠΆ, Ρ€Π΅ΡˆΠΈ Π·Π°Π΄Π°Ρ‡Ρƒ.

    Β 

    Π£Ρ€ΠΎΠ²Π΅Π½ΡŒ 3)РСши Π·Π°Π΄Π°Ρ‡Ρƒ устно. Π‘ΠΎΡΡ‚Π°Π²ΡŒ ΠΎΠ±Ρ€Π°Ρ‚Π½ΡƒΡŽ Π·Π°Π΄Π°Ρ‡Ρƒ Π½Π° Π½Π°Ρ…ΠΎΠΆΠ΄Π΅Π½ΠΈΠ΅ Π²Ρ€Π΅ΠΌΠ΅Π½ΠΈ ΠΈ Ρ€Π΅ΡˆΠΈ Π΅Ρ‘.

    Β 

    Π£Ρ‡Π΅Ρ‚ Π·Π½Π°Π½ΠΈΠΉ Π½Π° ΡƒΡ€ΠΎΠΊΠ΅

    Π’Π•ΠœΠ: Π‘ΠœΠ«Π‘Π› Π”Π•Π›Π•ΠΠ˜Π―.3 класс

    ЦСль: установлСниС взаимосвязи ΠΌΠ΅ΠΆΠ΄Ρƒ ΠΊΠΎΠΌΠΏΠΎΠ½Π΅Π½Ρ‚Π°ΠΌΠΈ умноТСния ΠΈ дСлСния, Ρ€Π°Ρ†ΠΈΠΎΠ½Π°Π»ΡŒΠ½Ρ‹Π΅ способы вычислСний.

    Π˜Π½ΡΡ‚Ρ€ΡƒΠΊΡ†ΠΈΡ: ΠŸΡ€ΠΎΡ‡ΠΈΡ‚Π°ΠΉ Π²Ρ‹Ρ€Π°ΠΆΠ΅Π½ΠΈΠ΅. Назови ΠΊΠΎΠΌΠΏΠΎΠ½Π΅Π½Ρ‚Ρ‹ дСйствия. Π’Ρ‹Π±Π΅Ρ€ΠΈ ΡƒΡ€ΠΎΠ²Π΅Π½ΡŒ слоТности. ΠŸΠΎΡΡ‚Π°Π²ΡŒ ΠΏΡ€ΠΎΠ³Π½ΠΎΡΡ‚ΠΈΡ‡Π΅ΡΠΊΡƒΡŽ, Π° Π·Π°Ρ‚Π΅ΠΌ ΠΈΡ‚ΠΎΠ³ΠΎΠ²ΡƒΡŽ самооцСнку.

    1 ΡƒΡ€ΠΎΠ²Π΅Π½ΡŒ. Найди Π·Π½Π°Ρ‡Π΅Π½ΠΈΠ΅, ΠΈΡΠΏΠΎΠ»ΡŒΠ·ΡƒΡ Π΄Π°Π½Π½Ρ‹Π΅ равСнства:

    24-8-8-8=0—–24:8=* 10-2-2-2-2-2=0—–10:2=*

    24-6-6-6-6=0β€”24:6=* 10-5-5=0————-10:5=*

    2 ΡƒΡ€ΠΎΠ²Π΅Π½ΡŒ. Найди Π·Π½Π°Ρ‡Π΅Π½ΠΈΠ΅, ΠΈΡΠΏΠΎΠ»ΡŒΠ·ΡƒΡ Π΄Π°Π½Π½Ρ‹Π΅ равСнства:

    5Π₯9=45Β  Β  Β  Β  Β  Β  Β  Β  Β  Β 6Π₯9=54Β  Β  Β  Β  Β  Β  Β  Β  Β  7Π₯9=63

    45:5=Β  Β  Β  Β  Β  Β  Β  Β  Β  Β  Β  Β 54:6=Β  Β  Β  Β  Β  Β  Β  Β  Β  Β 63:7=

    45:9=Β  Β  Β  Β  Β  Β  Β  Β  Β  Β  Β  Β 54:9=Β  Β  Β  Β  Β  Β  Β  Β  Β  Β  63:9=

    3 ΡƒΡ€ΠΎΠ²Π΅Π½ΡŒ. Π”ΠΎΠΊΠ°ΠΆΠΈ, Π½Π΅ выполняя вычислСний, Ρ‡Ρ‚ΠΎ значСния Π²Ρ‹Ρ€Π°ΠΆΠ΅Π½ΠΈΠΉ Π² ΠΊΠ°ΠΆΠ΄ΠΎΠΌ столбикС ΠΎΠ΄ΠΈΠ½Π°ΠΊΠΎΠ²Ρ‹.

    9Ρ…7+9+5=Β  Β  Β  Β  Β  Β  Β  Β  Β  Β  Β  8Ρ…6+8+3=

    7Π₯9+9+5=Β  Β  Β  Β  Β  Β  Β  Β  Β  Β  Β  8Π₯7+3=

    9Π₯8+5=Β  Β  Β  Β  Β  Β  Β  Β  Β  Β  Β  Β  Β 7Π₯8+3=

    Π’Π•ΠœΠ: ΠŸΠžΠ Π―Π”ΠžΠš Π’Π«ΠŸΠžΠ›ΠΠ•ΠΠ˜Π― Π”Π•Π™Π‘Π’Π’Π˜Π™ Π’ Π’Π«Π ΠΠ–Π•ΠΠ˜Π―Π₯.3 класс.

    ЦСль: знакомство с ΠΏΡ€Π°Π²ΠΈΠ»Π°ΠΌΠΈ выполнСния дСйствий Π² выраТСниях.

    1 ΡƒΡ€ΠΎΠ²Π΅Π½ΡŒ. Π§Π΅ΠΌ ΠΏΠΎΡ…ΠΎΠΆΠΈ ΠΏΡ€ΠΈΠΌΠ΅Ρ€Ρ‹? Π Π°ΡΡΡ‚Π°Π²ΡŒ порядок дСйствий. Объясни.

    72-9-3+6Β  Β  Β  Β  Β  Β  Β  Β  Β  Β  Β  Β  Β  Β  27-3+2-6Β  Β  Β  Β  Β  Β  Β  Β  Β  Β 48-6+7+8

    72:9Ρ…3:6Β  Β  Β  Β  Β  Β  Β  Β  Β  Β  Β  Β  Β  Β  Β 27:3Ρ…2:6Β  Β  Β  Β  Β  Β  Β  Β  Β  Β 48:6Ρ…7:8

    2 ΡƒΡ€ΠΎΠ²Π΅Π½ΡŒ. Π Π°ΡΡΡ‚Π°Π²ΡŒ порядок дСйствий. Π‘ΠΎΡΡ‚Π°Π²ΡŒ 1 ΠΏΡ€ΠΈΠΌΠ΅Ρ€ ΠΈ вычисли Π΅Π³ΠΎ.

    β–²+β–²:β–²+β–²Ρ…β–²-β–²Β  Β  Β  Β  Β  Β  Β  Β  Β  Β  Β  Β  Β  Β  Β  Β  β–²Ρ…β–²(β–²+β–²)-β–²

    3 ΡƒΡ€ΠΎΠ²Π΅Π½ΡŒ. Π Π°ΡΡΡ‚Π°Π²ΡŒ порядок дСйствий. Вычисли.

    18+360:90+6Ρ…8-50Β  Β  Β  Β  Β  Β  Β  Β  Β 630:70+(20-5)-(9+6)Β  Β  Β  Β  Β  Β  Β  Β  Β  Β  5Ρ…4+(3+19)-10

    2 Π²Π°Ρ€ΠΈΠ°Π½Ρ‚

    1 ΡƒΡ€ΠΎΠ²Π΅Π½ΡŒ. ΠŸΡ€ΠΎΠ²Π΅Ρ€ΡŒ, ΠΎΠ΄ΠΈΠ½Π°ΠΊΠΎΠ²ΠΎ Π»ΠΈ Π·Π½Π°Ρ‡Π΅Π½ΠΈΠ΅ Π²Ρ‹Ρ€Π°ΠΆΠ΅Π½ΠΈΠΉ Π² ΠΊΠ°ΠΆΠ΄ΠΎΠΉ ΠΏΠ°Ρ€Π΅?

    17+(4Ρ…3)Ρ…2-8Β  Β  Β  Β  Β  Β  Β  Β  Β  Β  Β  Β  8Ρ…(4+3)+6-4

    17+4Ρ…(3Ρ…2)-8Β  Β  Β  Β  Β  Β  Β  Β  Β  Β  Β  Β  Β 8Ρ…4+(3+6)-4

    2 ΡƒΡ€ΠΎΠ²Π΅Π½ΡŒ. Π Π°ΡΡΡ‚Π°Π²ΡŒ арифмСтичСскиС дСйствия (+ – :Ρ…). Π‘ΠΎΡΡ‚Π°Π²ΡŒ 1 ΠΏΡ€ΠΈΠΌΠ΅Ρ€.

    β– 3β– 1β– 2β– Β  Β  Β  Β  Β  Β β– 2β– 3β– 1β– Β  Β  Β  Β  Β  β– 3β– 2(β– 1β– )Β  Β  Β  Β  Β  Β  Β β– 2(β– 1β– )3β– 

    3 ΡƒΡ€ΠΎΠ²Π΅Π½ΡŒ. Π Π°Π·Π³Π°Π΄Π°ΠΉ ΠΏΡ€Π°Π²ΠΈΠ»ΠΎ, ΠΏΠΎ ΠΊΠΎΡ‚ΠΎΡ€ΠΎΠΌΡƒ составлСны ΠΏΡ€ΠΈΠΌΠ΅Ρ€Ρ‹. Вычисли.

    9Ρ…5-6Ρ…4:8 β†’ 45-6Ρ…4:8 β†’ 45-24:8 β†’ 9+18-8

    ВСкущая ΠΏΡ€ΠΎΠ²Π΅Ρ€ΠΊΠ° усвоСния ΠΏΡ€ΠΎΠΉΠ΄Π΅Π½Π½ΠΎΠ³ΠΎ ΠΌΠ°Ρ‚Π΅Ρ€ΠΈΠ°Π»Π°

    Β 

    Π’Π•ΠœΠ: Β«Π Π•Π¨Π•ΠΠ˜Π• ЗАДАЧ НА НАΠ₯ΠžΠ–Π”Π•ΠΠ˜Π• ΠŸΠ›ΠžΠ©ΠΠ”Π˜ И ΠŸΠ•Π Π˜ΠœΠ•Π’Π ΠΒ» 4 класс

    ЦСль: Ρ€Π°Π·Π²ΠΈΡ‚ΠΈΠ΅ умСния Ρ€Π΅ΡˆΠ°Ρ‚ΡŒ Π·Π°Π΄Π°Ρ‡ΠΈ Π½Π° Π½Π°Ρ…ΠΎΠΆΠ΄Π΅Π½ΠΈΠ΅ ΠΏΠ»ΠΎΡ‰Π°Π΄ΠΈ ΠΈ ΠΏΠ΅Ρ€ΠΈΠΌΠ΅Ρ‚Ρ€Π°, ΡΠΎΠΎΡ‚Π½ΠΎΡΠΈΡ‚ΡŒ Π΅Π΄ΠΈΠ½ΠΈΡ†Ρ‹ измСрСния Π΄Π»ΠΈΠ½Ρ‹ ΠΈ ΠΏΠ»ΠΎΡ‰Π°Π΄ΠΈ.

    1 ΡƒΡ€ΠΎΠ²Π΅Π½ΡŒ. РСши Π·Π°Π΄Π°Ρ‡Ρƒ: β€œΠŸΠ»ΠΎΡ‰Π°Π΄ΡŒ ΠΏΡ€ΡΠΌΠΎΡƒΠ³ΠΎΠ»ΡŒΠ½ΠΈΠΊΠ° Ρ€Π°Π²Π½Π° 36см2. Π¨ΠΈΡ€ΠΈΠ½Π° ΠΏΡ€ΡΠΌΠΎΡƒΠ³ΠΎΠ»ΡŒΠ½ΠΈΠΊΠ° 4см. Π§Π΅ΠΌΡƒ Ρ€Π°Π²Π΅Π½ ΠΏΠ΅Ρ€ΠΈΠΌΠ΅Ρ‚Ρ€ ΠΏΡ€ΡΠΌΠΎΡƒΠ³ΠΎΠ»ΡŒΠ½ΠΈΠΊΠ°?”

    2 ΡƒΡ€ΠΎΠ²Π΅Π½ΡŒ. РСши Π·Π°Π΄Π°Ρ‡Ρƒ: β€œΠŸΠ»ΠΎΡ‰Π°Π΄ΡŒ ΠΏΡ€ΡΠΌΠΎΡƒΠ³ΠΎΠ»ΡŒΠ½ΠΈΠΊΠ° 32см2. Какова Π΄Π»ΠΈΠ½Π° ΠΈ ΡˆΠΈΡ€ΠΈΠ½Π° ΠΏΡ€ΡΠΌΠΎΡƒΠ³ΠΎΠ»ΡŒΠ½ΠΈΠΊΠ°, Ссли ΡˆΠΈΡ€ΠΈΠ½Π° Π² 2 Ρ€Π°Π·Π° ΠΊΠΎΡ€ΠΎΡ‡Π΅, Ρ‡Π΅ΠΌ Π΅Π³ΠΎ Π΄Π»ΠΈΠ½Π°?”

    3 ΡƒΡ€ΠΎΠ²Π΅Π½ΡŒ. РСши Π·Π°Π΄Π°Ρ‡Ρƒ: β€œΠŸΠ΅Ρ€ΠΈΠΌΠ΅Ρ‚Ρ€ ΠΏΡ€ΡΠΌΠΎΡƒΠ³ΠΎΠ»ΡŒΠ½ΠΈΠΊΠ° Ρ€Π°Π²Π΅Π½ 26 см, ΠΏΠ»ΠΎΡ‰Π°Π΄ΡŒ – 42см2. ΠžΠΏΡ€Π΅Π΄Π΅Π»ΠΈ Π΅Π³ΠΎ Π΄Π»ΠΈΠ½Ρƒ ΠΈ ΡˆΠΈΡ€ΠΈΠ½Ρƒβ€

    Β 

    Π’Π•ΠœΠ: Β«ΠŸΠ›ΠžΠ©ΠΠ”Π¬ И ΠŸΠ•Π Π˜ΠœΠ•Π’Π Β» 4 класс

    ЦСль: ΠΏΡ€ΠΎΠ²Π΅Ρ€ΠΈΡ‚ΡŒ ΡƒΠΌΠ΅Π½ΠΈΠ΅ ΡΠΎΠΎΡ‚Π½ΠΎΡΠΈΡ‚ΡŒ Π΅Π΄ΠΈΠ½ΠΈΡ†Ρ‹ измСрСния Π΄Π»ΠΈΠ½Ρ‹ ΠΈ Π΅Π΄ΠΈΠ½ΠΈΡ†Ρ‹ измСрСния ΠΏΠ»ΠΎΡ‰Π°Π΄ΠΈ, сравнСниС Π²Π΅Π»ΠΈΡ‡ΠΈΠ½.

    1 ΡƒΡ€ΠΎΠ²Π΅Π½ΡŒ. Π‘Ρ€Π°Π²Π½ΠΈ Π²Π΅Π»ΠΈΡ‡ΠΈΠ½Ρ‹. Π—Π°Ρ‡Π΅Ρ€ΠΊΠ½ΠΈ ΠΏΠ°Ρ€Ρ‹, ΠΊΠΎΡ‚ΠΎΡ€Ρ‹Π΅ нСльзя ΡΡ€Π°Π²Π½ΠΈΡ‚ΡŒ.

    360см__26см 6дм 

    Β 

    Π‘Π°ΠΌΠΎΡΡ‚ΠΎΡΡ‚Π΅Π»ΡŒΠ½Π°Ρ Ρ€Π°Π±ΠΎΡ‚Π°.

    Π’Π•ΠœΠ: БКОРОБВЬ. Π’Π Π•ΠœΠ―. Π ΠΠ‘Π‘Π’ΠžΠ―ΠΠ˜Π•.4 класс

    ЦСль: Ρ€Π°Π·Π²ΠΈΠ²Π°Ρ‚ΡŒ ΡƒΠΌΠ΅Π½ΠΈΠ΅ Ρ€Π΅ΡˆΠ°Ρ‚ΡŒ Π·Π°Π΄Π°Ρ‡ΠΈ Π½Π° Π΄Π²ΠΈΠΆΠ΅Π½ΠΈΠ΅; Π·Π°ΠΊΡ€Π΅ΠΏΠ»ΡΡ‚ΡŒ знания взаимосвязи ΠΌΠ΅ΠΆΠ΄Ρƒ ΡΠΊΠΎΡ€ΠΎΡΡ‚ΡŒΡŽ, Π²Ρ€Π΅ΠΌΠ΅Π½Π΅ΠΌ ΠΈ расстояниСм.

    Π Π°Π±ΠΎΡ‚Π° β„–1

    Π—Π°Π΄Π°Ρ‡Π°: Π‘ΠΊΠΎΡ€ΠΎΡΡ‚ΡŒ автомобиля 90ΠΊΠΌ/Ρ‡. КакоС расстояниС ΠΎΠ½ ΠΏΡ€ΠΎΠ΅Π΄Π΅Ρ‚ Π·Π° 4 часов?

    1 ΡƒΡ€ΠΎΠ²Π΅Π½ΡŒ. РСши Π·Π°Π΄Π°Ρ‡Ρƒ, с ΠΏΠΎΠΌΠΎΡ‰ΡŒΡŽ Ρ‚Π°Π±Π»ΠΈΡ†Ρ‹.

    2 ΡƒΡ€ΠΎΠ²Π΅Π½ΡŒ. РСши Π·Π°Π΄Π°Ρ‡Ρƒ устно, ΡΠΎΡΡ‚Π°Π²ΡŒ ΠΎΠ±Ρ€Π°Ρ‚Π½ΡƒΡŽ Π·Π°Π΄Π°Ρ‡Ρƒ Π½Π° Π½Π°Ρ…ΠΎΠΆΠ΄Π΅Π½ΠΈΠ΅ Π²Ρ€Π΅ΠΌΠ΅Π½ΠΈ ΠΈ Ρ€Π΅ΡˆΠΈ Π΅Ρ‘.

    3 ΡƒΡ€ΠΎΠ²Π΅Π½ΡŒ. Π—Π° сколько Π²Ρ€Π΅ΠΌΠ΅Π½ΠΈ ΠΎΠ½ ΠΏΡ€ΠΎΠ΅Π΄Π΅Ρ‚ это ΠΆΠ΅ расстояниС, Ссли Π΅Π³ΠΎ ΡΠΊΠΎΡ€ΠΎΡΡ‚ΡŒ Π±ΡƒΠ΄Π΅Ρ‚ Π½Π° 30ΠΊΠΌ/Ρ‡ мСньшС?

    Π Π°Π±ΠΎΡ‚Π° β„–2

    1 ΡƒΡ€ΠΎΠ²Π΅Π½ΡŒ. Π›Ρ‹ΠΆΠ½ΠΈΠΊ ΡˆΡ‘Π» 3 часа со ΡΠΊΠΎΡ€ΠΎΡΡ‚ΡŒΡŽ 12 ΠΊΠΌ/Ρ‡. Бколько Π²Ρ€Π΅ΠΌΠ΅Π½ΠΈ понадобится, Ρ‡Ρ‚ΠΎΠ±Ρ‹ ΠΏΡ€ΠΎΠΉΡ‚ΠΈ Ρ‚Π°ΠΊΠΎΠ΅ ΠΆΠ΅ расстояниС ΠΏΠ΅ΡˆΠ΅Ρ…ΠΎΠ΄Ρƒ, ΡΠΊΠΎΡ€ΠΎΡΡ‚ΡŒ ΠΊΠΎΡ‚ΠΎΡ€ΠΎΠ³ΠΎ 6 ΠΊΠΌ/Ρ‡?

    2 ΡƒΡ€ΠΎΠ²Π΅Π½ΡŒ. Π›Ρ‹ΠΆΠ½ΠΈΠΊ ΡˆΡ‘Π» 3 часа со ΡΠΊΠΎΡ€ΠΎΡΡ‚ΡŒΡŽ 12 ΠΊΠΌ/Ρ‡. Бколько Π²Ρ€Π΅ΠΌΠ΅Π½ΠΈ понадобится, Ρ‡Ρ‚ΠΎΠ±Ρ‹ ΠΏΡ€ΠΎΠΉΡ‚ΠΈ Ρ‚Π°ΠΊΠΎΠ΅ ΠΆΠ΅ расстояниС ΠΏΠ΅ΡˆΠ΅Ρ…ΠΎΠ΄Ρƒ, ΡΠΊΠΎΡ€ΠΎΡΡ‚ΡŒ ΠΊΠΎΡ‚ΠΎΡ€ΠΎΠ³ΠΎ Π² 2 Ρ€Π°Π·Π° мСньшС скорости Π»Ρ‹ΠΆΠ½ΠΈΠΊΠ°?

    3 ΡƒΡ€ΠΎΠ²Π΅Π½ΡŒ. Π›Ρ‹ΠΆΠ½ΠΈΠΊ Π·Π° 3 часа ΠΏΡ€ΠΎΡˆΡ‘Π» 36 ΠΊΠΌ. Бколько Π²Ρ€Π΅ΠΌΠ΅Π½ΠΈ понадобится, Ρ‡Ρ‚ΠΎΠ±Ρ‹ ΠΏΡ€ΠΎΠΉΡ‚ΠΈ Ρ‚Π°ΠΊΠΎΠ΅ ΠΆΠ΅ расстояниС ΠΏΠ΅ΡˆΠ΅Ρ…ΠΎΠ΄Ρƒ, Ссли Π΅Π³ΠΎ ΡΠΊΠΎΡ€ΠΎΡΡ‚ΡŒ Π² 2 Ρ€Π°Π·Π° мСньшС скорости Π»Ρ‹ΠΆΠ½ΠΈΠΊΠ°?

    Β 

    Π Π°Π±ΠΎΡ‚Π° β„–3(встрСчноС Π΄Π²ΠΈΠΆΠ΅Π½ΠΈΠ΅)

    1 ΡƒΡ€ΠΎΠ²Π΅Π½ΡŒ. Из ΠΎΠ΄Π½ΠΎΠ³ΠΎ Π³ΠΎΡ€ΠΎΠ΄Π° Π² Π΄Ρ€ΡƒΠ³ΠΎΠΉ ΠΎΠ΄Π½ΠΎΠ²Ρ€Π΅ΠΌΠ΅Π½Π½ΠΎ навстрСчу Π΄Ρ€ΡƒΠ³ Π΄Ρ€ΡƒΠ³Ρƒ Π²Ρ‹ΡˆΠ»ΠΈ Π΄Π²Π° ΠΏΠΎΠ΅Π·Π΄Π° ΠΈ Π²ΡΡ‚Ρ€Π΅Ρ‚ΠΈΠ»ΠΈΡΡŒ Ρ‡Π΅Ρ€Π΅Π· 9 часов. Π‘ΠΊΠΎΡ€ΠΎΡΡ‚ΡŒ ΠΎΠ΄Π½ΠΎΠ³ΠΎ ΠΏΠΎΠ΅Π·Π΄Π° 35 ΠΊΠΌ/Ρ‡, ΡΠΊΠΎΡ€ΠΎΡΡ‚ΡŒ Π΄Ρ€ΡƒΠ³ΠΎΠ³ΠΎ – 70 ΠΊΠΌ/Ρ‡. НайдитС расстояниС ΠΌΠ΅ΠΆΠ΄Ρƒ Π³ΠΎΡ€ΠΎΠ΄Π°ΠΌΠΈ.

    2 ΡƒΡ€ΠΎΠ²Π΅Π½ΡŒ. Из ΠΎΠ΄Π½ΠΎΠ³ΠΎ Π³ΠΎΡ€ΠΎΠ΄Π° Π² Π΄Ρ€ΡƒΠ³ΠΎΠΉ ΠΎΠ΄Π½ΠΎΠ²Ρ€Π΅ΠΌΠ΅Π½Π½ΠΎ навстрСчу Π΄Ρ€ΡƒΠ³ Π΄Ρ€ΡƒΠ³Ρƒ Π²Ρ‹ΡˆΠ»ΠΈ Π΄Π²Π° ΠΏΠΎΠ΅Π·Π΄Π° ΠΈ Π²ΡΡ‚Ρ€Π΅Ρ‚ΠΈΠ»ΠΈΡΡŒ Ρ‡Π΅Ρ€Π΅Π· 9 часов. Π‘ΠΊΠΎΡ€ΠΎΡΡ‚ΡŒ ΠΎΠ΄Π½ΠΎΠ³ΠΎ ΠΏΠΎΠ΅Π·Π΄Π° – 30 ΠΊΠΌ/Ρ‡, Π° Π΄Ρ€ΡƒΠ³ΠΎΠ³ΠΎ Π½Π° 35 ΠΊΠΌ/Ρ‡ большС. НайдитС расстояниС ΠΌΠ΅ΠΆΠ΄Ρƒ Π³ΠΎΡ€ΠΎΠ΄Π°ΠΌΠΈ.

    3 ΡƒΡ€ΠΎΠ²Π΅Π½ΡŒ. Из ΠΎΠ΄Π½ΠΎΠ³ΠΎ Π³ΠΎΡ€ΠΎΠ΄Π° Π² Π΄Ρ€ΡƒΠ³ΠΎΠΉ ΠΎΠ΄Π½ΠΎΠ²Ρ€Π΅ΠΌΠ΅Π½Π½ΠΎ навстрСчу Π΄Ρ€ΡƒΠ³ Π΄Ρ€ΡƒΠ³Ρƒ Π²Ρ‹ΡˆΠ»ΠΈ Π΄Π²Π° ΠΏΠΎΠ΅Π·Π΄Π° ΠΈ Π²ΡΡ‚Ρ€Π΅Ρ‚ΠΈΠ»ΠΈΡΡŒ Ρ‡Π΅Ρ€Π΅Π· 9 часов. Π‘ΠΊΠΎΡ€ΠΎΡΡ‚ΡŒ ΠΎΠ΄Π½ΠΎΠ³ΠΎ ΠΏΠΎΠ΅Π·Π΄Π° 35 ΠΊΠΌ/Ρ‡, Π° Π΄Ρ€ΡƒΠ³ΠΎΠ³ΠΎ Π² 2 Ρ€Π°Π·Π° большС. НайдитС расстояниС ΠΌΠ΅ΠΆΠ΄Ρƒ Π³ΠΎΡ€ΠΎΠ΄Π°ΠΌΠΈ.

    Π Π°Π±ΠΎΡ‚Π° β„–4 (ΠΏΡ€ΠΎΡ‚ΠΈΠ²ΠΎΠΏΠΎΠ»ΠΎΠΆΠ½Ρ‹Π΅ направлСния)

    1 ΡƒΡ€ΠΎΠ²Π΅Π½ΡŒ. Из ΠΎΠ΄Π½ΠΎΠ³ΠΎ сСла ΠΎΠ΄Π½ΠΎΠ²Ρ€Π΅ΠΌΠ΅Π½Π½ΠΎ Π² ΠΏΡ€ΠΎΡ‚ΠΈΠ²ΠΎΠΏΠΎΠ»ΠΎΠΆΠ½Ρ‹Ρ… направлСниях Π²Ρ‹ΡˆΠ»ΠΈ Π΄Π²Π° автобуса. Один автобус ΡˆΡ‘Π» со ΡΠΊΠΎΡ€ΠΎΡΡ‚ΡŒΡŽ 60 ΠΊΠΌ/Ρ‡, Π° Π΄Ρ€ΡƒΠ³ΠΎΠΉ – со ΡΠΊΠΎΡ€ΠΎΡΡ‚ΡŒΡŽ 80 ΠΊΠΌ/Ρ‡. На ΠΊΠ°ΠΊΠΎΠΌ расстоянии Π΄Ρ€ΡƒΠ³ ΠΎΡ‚ Π΄Ρ€ΡƒΠ³Π° Π±ΡƒΠ΄ΡƒΡ‚ Π½Π°Ρ…ΠΎΠ΄ΠΈΡ‚ΡŒΡΡ автобусы Ρ‡Π΅Ρ€Π΅Π· 3 часа?

    2 ΡƒΡ€ΠΎΠ²Π΅Π½ΡŒ. Из ΠΎΠ΄Π½ΠΎΠ³ΠΎ сСла ΠΎΠ΄Π½ΠΎΠ²Ρ€Π΅ΠΌΠ΅Π½Π½ΠΎ Π² ΠΏΡ€ΠΎΡ‚ΠΈΠ²ΠΎΠΏΠΎΠ»ΠΎΠΆΠ½Ρ‹Ρ… направлСниях Π²Ρ‹ΡˆΠ»ΠΈ Π΄Π²Π° автобуса. Один ΠΈΠ· Π½ΠΈΡ… ΡˆΡ‘Π» со ΡΠΊΠΎΡ€ΠΎΡΡ‚ΡŒΡŽ 60 ΠΊΠΌ/Ρ‡, Π° Π΄Ρ€ΡƒΠ³ΠΎΠΉ Π½Π° 20 ΠΊΠΌ/Ρ‡ большС. На ΠΊΠ°ΠΊΠΎΠΌ расстоянии Π΄Ρ€ΡƒΠ³ ΠΎΡ‚ Π΄Ρ€ΡƒΠ³Π° Π±ΡƒΠ΄ΡƒΡ‚ Π½Π°Ρ…ΠΎΠ΄ΠΈΡ‚ΡŒΡΡ автобусы Ρ‡Π΅Ρ€Π΅Π· 3 часа?

    3 ΡƒΡ€ΠΎΠ²Π΅Π½ΡŒ. Из ΠΎΠ΄Π½ΠΎΠ³ΠΎ сСла ΠΎΠ΄Π½ΠΎΠ²Ρ€Π΅ΠΌΠ΅Π½Π½ΠΎ Π² ΠΏΡ€ΠΎΡ‚ΠΈΠ²ΠΎΠΏΠΎΠ»ΠΎΠΆΠ½Ρ‹Ρ… направлСниях Π²Ρ‹ΡˆΠ»ΠΈ Π΄Π²Π° автобуса. КакоС расстояниС Π±ΡƒΠ΄Π΅Ρ‚ ΠΌΠ΅ΠΆΠ΄Ρƒ Π½ΠΈΠΌΠΈ Ρ‡Π΅Ρ€Π΅Π· 3 часа, Ссли ΠΊΠ°ΠΆΠ΄Ρ‹ΠΉ час ΠΎΠ½ΠΈ ΡƒΠ΄Π°Π»ΡΠ»ΠΈΡΡŒ Π΄Ρ€ΡƒΠ³ ΠΎΡ‚ Π΄Ρ€ΡƒΠ³Π° Π½Π° 140 ΠΊΠΌ?

    Π Π°Π±ΠΎΡ‚Π° β„–5(Π΄Π²ΠΈΠΆΠ΅Π½ΠΈΠ΅ Π² ΠΎΠ±Ρ€Π°Ρ‚Π½ΠΎΠΌ Π½Π°ΠΏΡ€Π°Π²Π»Π΅Π½ΠΈΠΈ)

    1 ΡƒΡ€ΠΎΠ²Π΅Π½ΡŒ. РасстояниС Π΄ΠΎ стадиона 1200ΠΌ ΠΌΠ°Π»ΡŒΡ‡ΠΈΠΊ ΠΏΡ€ΠΎΡˆΠ΅Π» Π·Π° 15 ΠΌΠΈΠ½ΡƒΡ‚. На ΠΎΠ±Ρ€Π°Ρ‚Π½Ρ‹ΠΉ ΠΏΡƒΡ‚ΡŒ ΠΎΠ½ ΠΏΠΎΡ‚Ρ€Π°Ρ‚ΠΈΠ» Π½Π° 5 ΠΌΠΈΠ½ΡƒΡ‚ большС. Π‘ ΠΊΠ°ΠΊΠΎΠΉ ΡΠΊΠΎΡ€ΠΎΡΡ‚ΡŒΡŽ шСл ΠΌΠ°Π»ΡŒΡ‡ΠΈΠΊ ΠΎΠ±Ρ€Π°Ρ‚Π½ΠΎ?

    2 ΡƒΡ€ΠΎΠ²Π΅Π½ΡŒ. ВСлосипСдист ΠΎΡ‚ Π³ΠΎΡ€ΠΎΠ΄Π° Π΄ΠΎ Π΄Π°Ρ‡ΠΈ Π΅Ρ…Π°Π» 3 часа со ΡΠΊΠΎΡ€ΠΎΡΡ‚ΡŒΡŽ 12ΠΊΠΌ/Ρ‡, Π° Π½Π° ΠΎΠ±Ρ€Π°Ρ‚Π½Ρ‹ΠΉ ΠΏΡƒΡ‚ΡŒ ΠΎΠ½ Π·Π°Ρ‚Ρ€Π°Ρ‚ΠΈΠ» 4 часа. На сколько ΠΊΠΌ/Ρ‡ вСлосипСдист ΠΈΠ·ΠΌΠ΅Π½ΠΈΠ» свою ΡΠΊΠΎΡ€ΠΎΡΡ‚ΡŒ?

    3 ΡƒΡ€ΠΎΠ²Π΅Π½ΡŒ. РасстояниС ΠΌΠ΅ΠΆΠ΄Ρƒ пристанями 150ΠΊΠΌ Ρ‚Π΅ΠΏΠ»ΠΎΡ…ΠΎΠ΄ ΠΏΡ€ΠΎΡˆΠ΅Π» Π·Π° 6 часов. На ΠΎΠ±Ρ€Π°Ρ‚Π½ΠΎΠΌ ΠΏΡƒΡ‚ΠΈ Π΅Π³ΠΎ ΡΠΊΠΎΡ€ΠΎΡΡ‚ΡŒ ΡƒΠ²Π΅Π»ΠΈΡ‡ΠΈΠ»Π°ΡΡŒ Π½Π° 5 ΠΊΠΌ/Ρ‡ .Π—Π° ΠΊΠ°ΠΊΠΎΠ΅ врСмя Ρ‚Π΅ΠΏΠ»ΠΎΡ…ΠΎΠ΄ ΠΏΡ€ΠΎΠΉΠ΄Π΅Ρ‚ вСсь ΠΏΡƒΡ‚ΡŒ Ρ‚ΡƒΠ΄Π° ΠΈ ΠΎΠ±Ρ€Π°Ρ‚Π½ΠΎ?

    Π Π°Π±ΠΎΡ‚Π° β„–6

    1 ΡƒΡ€ΠΎΠ²Π΅Π½ΡŒ. РСши Π·Π°Π΄Π°Ρ‡Ρƒ: β€œΠ”Π²Π° ΠΏΠΎΠ΅Π·Π΄Π° ΠΈΠ΄ΡƒΡ‚ навстрСчу Π΄Ρ€ΡƒΠ³ Π΄Ρ€ΡƒΠ³Ρƒ со станций, расстояниС ΠΌΠ΅ΠΆΠ΄Ρƒ ΠΊΠΎΡ‚ΠΎΡ€Ρ‹ΠΌΠΈ 485 ΠΊΠΌ. ΠŸΠ΅Ρ€Π²Ρ‹ΠΉ Π²Ρ‹ΡˆΠ΅Π» Ρ€Π°Π½ΡŒΡˆΠ΅ Π½Π° 2 Ρ‡ ΠΈ двиТСтся со ΡΠΊΠΎΡ€ΠΎΡΡ‚ΡŒΡŽ 53ΠΊΠΌ/Ρ‡. Π§Π΅Ρ€Π΅Π· 3Ρ‡ послС Π²Ρ‹Ρ…ΠΎΠ΄Π° Π²Ρ‚ΠΎΡ€ΠΎΠ³ΠΎ ΠΏΠΎΠ΅Π·Π΄Π° ΠΎΠ½ΠΈ Π²ΡΡ‚Ρ€Π΅Ρ‚ΠΈΠ»ΠΈΡΡŒ. Какова ΡΠΊΠΎΡ€ΠΎΡΡ‚ΡŒ Π²Ρ‚ΠΎΡ€ΠΎΠ³ΠΎ ΠΏΠΎΠ΅Π·Π΄Π°?

    2 ΡƒΡ€ΠΎΠ²Π΅Π½ΡŒ. Π‘ΠΎΡΡ‚Π°Π²ΡŒ ΠΎΠ±Ρ€Π°Ρ‚Π½ΡƒΡŽ Π·Π°Π΄Π°Ρ‡Ρƒ.

    3 ΡƒΡ€ΠΎΠ²Π΅Π½ΡŒ. ИзмСни условиС Π·Π°Π΄Π°Ρ‡ΠΈ Ρ‚Π°ΠΊ, Ρ‡Ρ‚ΠΎΠ±Ρ‹ ΠΎΠ½Π° Ρ€Π΅ΡˆΠ°Π»Π°ΡΡŒ мСньшим количСством дСйствий.

    Β 

    Π£Ρ€ΠΎΠΊΠΈ закрСплСния

    Π’Π•ΠœΠ: Π•Π”Π˜ΠΠ˜Π¦Π« Π’Π Π•ΠœΠ•ΠΠ˜. 3 класс.

    Π’Π°Ρ€ΠΈΠ°Π½Ρ‚: 1

    ЦСль: установлСниС ΡΠΎΠΎΡ‚Π½ΠΎΡˆΠ΅Π½ΠΈΠΉ ΠΌΠ΅ΠΆΠ΄Ρƒ Π΅Π΄ΠΈΠ½ΠΈΡ†Π°ΠΌΠΈ Π²Ρ€Π΅ΠΌΠ΅Π½ΠΈ, Π·Π°ΠΊΡ€Π΅ΠΏΠΈΡ‚ΡŒ умСния ΡΡ€Π°Π²Π½ΠΈΠ²Π°Ρ‚ΡŒ, ΡΠΊΠ»Π°Π΄Ρ‹Π²Π°Ρ‚ΡŒ ΠΈ Π²Ρ‹Ρ‡ΠΈΡ‚Π°Ρ‚ΡŒ ΠΈΠΌΠ΅Π½ΠΎΠ²Π°Π½Π½Ρ‹Π΅ числа.

    Π˜Π½ΡΡ‚Ρ€ΡƒΠΊΡ†ΠΈΡ: Π’Ρ‹Π±Π΅Ρ€ΠΈ ΡƒΡ€ΠΎΠ²Π΅Π½ΡŒ слоТности ΠΈ Ρ€Π΅ΡˆΠΈ.

    Π£Ρ€ΠΎΠ²Π΅Π½ΡŒ 1

    1) Π—Π°ΠΏΠΈΡˆΠΈ Π² порядкС убывания: Π‘ΡƒΡ‚ΠΊΠΈ, Π²Π΅ΠΊ, сСкунда, нСдСля, мСсяц, ΠΌΠΈΠ½ΡƒΡ‚Π°, час.

    2) Π—Π°ΠΏΠΎΠ»Π½ΠΈ пропуски: 3 Ρ‡ =…мин  Β  Β  3 Π³ΠΎΠ΄Π° = …мСс  Β  Β  2 Ρ‡ =…мин  Β  Β  2 Π³ΠΎΠ΄Π° =…мСс.

    4 Ρ‡ =…мин  Β  Β  Β  Β 7 сут. = Π½Π΅Π΄.

    3) ΠŸΡ€ΠΎΠ²Π΅Ρ€ΡŒ, Π²Π΅Ρ€Π½Ρ‹ Π»ΠΈ нСравСнства?

    3 Ρ‡ 10 ΠΌΠΈΠ½ > 310 ΠΌΠΈΠ½Β  Β  Β  Β  5 ΠΌΠΈΠ½ 30 с < 330 с

    52 Ρ‡ > 2 сут.Β  Β  Β  Β  Β  Β  Β  Β  Β  Β  Β 1 Ρ‡ = 60 с

    Π£Ρ€ΠΎΠ²Π΅Π½ΡŒ 2

    1) Рассмотри запись, ΠΌΠΎΠΆΠ½ΠΎ Π»ΠΈ Ρ€Π°Π·Π±ΠΈΡ‚ΡŒ Π½Π° Π³Ρ€ΡƒΠΏΠΏΡ‹ Π²Π΅Π»ΠΈΡ‡ΠΈΠ½Ρ‹? Π§Ρ‚ΠΎ лишнСС?

    1 Ρ‡,1 Ρ‚, 1 ΠΌΠΈΠ½,1 с, 1 Ρ†, 1 Π³ΠΎΠ΄,1 ΠΊΠ³, 1 см, 1 Π³Ρ€Π°ΠΌΠΌ.

    2) ΠŸΠ΅Ρ€Π΅Π²Π΅Π΄ΠΈ Π² часы 7200 с =…ч 180 ΠΌΠΈΠ½ =…ч

    10 сут =…ч 3 сут 2 Ρ‡ =…ч

    420 ΠΌΠΈΠ½ =…ч 3600 с =…ч

    3) Π‘Ρ€Π°Π²Π½ΠΈ ΠΈΠΌΠ΅Π½ΠΎΠ²Π°Π½Π½Ρ‹Π΅ числа (< > = )

    7 ΠΌΠΈΠ½ 15 с…445 с 8 Ρ‡ 18 мин…7 Ρ‡ 78 ΠΌΠΈΠ½

    9 Ρ‡ 12 мин…563 ΠΌΠΈΠ½ 2 Ρ‡ 12 мин…7200с

    Π£Ρ€ΠΎΠ²Π΅Π½ΡŒ 3

    1) Π—Π°ΠΏΠΈΡˆΠΈ Π²Π΅Π»ΠΈΡ‡ΠΈΠ½Ρ‹ Π² порядкС возрастания:

    30 с, 32 Ρ‡, 7 Π»Π΅Ρ‚, 34 ΠΌΠΈΠ½, 1 Π²Π΅ΠΊ, 10 с

    2) По ΠΊΠ°ΠΊΠΎΠΌΡƒ ΠΏΡ€ΠΈΠ·Π½Π°ΠΊΡƒ записаны Π²Π΅Π»ΠΈΡ‡ΠΈΠ½Ρ‹? ΠŸΡ€ΠΎΠ΄ΠΎΠ»ΠΆΠΈ ΠΏΡ€Π°Π²ΠΈΠ»ΠΎ.

    Β 3 Ρ‡Β  Β 4 Ρ‡Β  Β 5 Ρ‡Β  Β  180 ΠΌΠΈΠ½Β  Β  Β 10800 с

    3) Найди Π·Π½Π°Ρ‡Π΅Π½ΠΈΠ΅ Π²Ρ‹Ρ€Π°ΠΆΠ΅Π½ΠΈΠΉ:

    2 Ρ‡ 45 ΠΌΠΈΠ½ + 15 ΠΌΠΈΠ½ 3 Ρ‡ 50 ΠΌΠΈΠ½ + 5 ΠΌΠΈΠ½

    3 Ρ‡ 55 ΠΌΠΈΠ½ + 2 Ρ‡ 20 ΠΌΠΈΠ½ 2 Ρ‡ 15 ΠΌΠΈΠ½ – 45 ΠΌΠΈΠ½

    4 Ρ‡ 10 ΠΌΠΈΠ½ – 35 ΠΌΠΈΠ½ 1 Ρ‡ 30 ΠΌΠΈΠ½ + 1 Ρ‡ 30 ΠΌΠΈΠ½

    Β 

    Π’Π°Ρ€ΠΈΠ°Π½Ρ‚: 2

    ЦСль: установлСниС ΡΠΎΠΎΡ‚Π½ΠΎΡˆΠ΅Π½ΠΈΠΉ ΠΌΠ΅ΠΆΠ΄Ρƒ Π΅Π΄ΠΈΠ½ΠΈΡ†Π°ΠΌΠΈ Π²Ρ€Π΅ΠΌΠ΅Π½ΠΈ, ΠΎΡ‚Ρ€Π°Π±ΠΎΡ‚ΠΊΠ° умСния ΡΡ€Π°Π²Π½ΠΈΠ²Π°Ρ‚ΡŒ, ΡΠΊΠ»Π°Π΄Ρ‹Π²Π°Ρ‚ΡŒ ΠΈ Π²Ρ‹Ρ‡ΠΈΡ‚Π°Ρ‚ΡŒ ΠΈΠΌΠ΅Π½ΠΎΠ²Π°Π½Π½Ρ‹Π΅ числа.

    Π˜Π½ΡΡ‚Ρ€ΡƒΠΊΡ†ΠΈΡ: Π’Ρ‹Π±Π΅Ρ€ΠΈ ΡƒΡ€ΠΎΠ²Π΅Π½ΡŒ слоТности ΠΈ Ρ€Π΅ΡˆΠΈ.

    Π£Ρ€ΠΎΠ²Π΅Π½ΡŒ 1

    1) Π—Π°ΠΏΠΈΡˆΠΈ Π² порядкС убывания:

    Π‘ΡƒΡ‚ΠΊΠΈ, Π²Π΅ΠΊ, сСкунда, нСдСля, мСсяц, ΠΌΠΈΠ½ΡƒΡ‚Π°, час.

    2)Π—Π°ΠΏΠΎΠ»Π½ΠΈ пропуски: 1 Ρ‡ =…мин 2 Π³ΠΎΠ΄Π° = …мСс

    2 Ρ‡ =…мин 5 Π»Π΅Ρ‚ =…мСс.

    8 Ρ‡ =…мин 72 Ρ‡ =…сут.

    3)ΠŸΡ€ΠΎΠ²Π΅Ρ€ΡŒ, Π²Π΅Ρ€Π½Ρ‹ Π»ΠΈ нСравСнства?

    2 Ρ‡ 10 ΠΌΠΈΠ½ >210 ΠΌΠΈΠ½Β  Β  Β  Β  Β  Β  Β  Β  Β  Β  4 ΠΌΠΈΠ½ 30 с <300 с

    70 Ρ‡ >3 сут.Β  Β  Β  Β  Β  Β  Β  Β  Β  Β  Β  Β  Β  Β  Β  Β  Β 1 ΠΌΠΈΠ½ = 60 с

    Π£Ρ€ΠΎΠ²Π΅Π½ΡŒ 2

    1) Рассмотри запись, ΠΌΠΎΠΆΠ½ΠΎ Π»ΠΈ Ρ€Π°Π·Π±ΠΈΡ‚ΡŒ Π½Π° Π³Ρ€ΡƒΠΏΠΏΡ‹ Π²Π΅Π»ΠΈΡ‡ΠΈΠ½Ρ‹? Π§Ρ‚ΠΎ лишнСС?

    1 ΠΌΠΈΠ½, 1 Ρ†, 1 Ρ‡, 1 мСс, 1 Ρ‚ 1 Π²Π΅ΠΊ, 1 Π³Ρ€Π°ΠΌΠΌ, 1 ΠΌ, 1 ΠΊΠ³.

    2)ΠŸΠ΅Ρ€Π΅Π²Π΅Π΄ΠΈ Π² часы: 10800 с =…ч  Β  Β  240 ΠΌΠΈΠ½ =…ч

    7 сут =…ч  Β  Β  Β  Β  Β  Β  Β  Β  Β  Β  Β  Β  1 сут 7 Ρ‡ =…ч

    120 ΠΌΠΈΠ½ =…ч  Β  Β  Β  Β  Β  Β  Β  Β  Β  Β 3600 с =…ч

    3) Π‘Ρ€Π°Π²Π½ΠΈ ΠΈΠΌΠ΅Π½ΠΎΠ²Π°Π½Π½Ρ‹Π΅ числа (< > = )

    5 ΠΌΠΈΠ½ 15 с…445 с  Β  Β  Β  Β  Β  Β  Β  Β 4 Ρ‡ 22 мин…4 Ρ‡ 20 ΠΌΠΈΠ½

    6 Ρ‡ 40 мин…400 ΠΌΠΈΠ½Β  Β  Β  Β  Β  Β  Β 1 Ρ‡ 52 мин…3660с

    Π£Ρ€ΠΎΠ²Π΅Π½ΡŒ 3

    1) Π—Π°ΠΏΠΈΡˆΠΈ Π²Π΅Π»ΠΈΡ‡ΠΈΠ½Ρ‹ Π² порядкС возрастания:

    80 с, 24 Ρ‡, 6 Π»Π΅Ρ‚, 1 ΠΌΠΈΠ½, 2 Π²Π΅ΠΊ, 2 с

    2) По ΠΊΠ°ΠΊΠΎΠΌΡƒ ΠΏΡ€ΠΈΠ·Π½Π°ΠΊΡƒ записаны Π²Π΅Π»ΠΈΡ‡ΠΈΠ½Ρ‹? ΠŸΡ€ΠΎΠ΄ΠΎΠ»ΠΆΠΈ ΠΏΡ€Π°Π²ΠΈΠ»ΠΎ.

    Β 2 Ρ‡ 4 Ρ‡ 5 Ρ‡Β  120 ΠΌΠΈΠ½Β  Β 7200 с

    3) Найди Π·Π½Π°Ρ‡Π΅Π½ΠΈΠ΅ Π²Ρ‹Ρ€Π°ΠΆΠ΅Π½ΠΈΠΉ:

    1 Ρ‡ 35 ΠΌΠΈΠ½ + 15 ΠΌΠΈΠ½ 5 Ρ‡ 55 ΠΌΠΈΠ½ + 5 ΠΌΠΈΠ½

    4 Ρ‡ 50 ΠΌΠΈΠ½ + 2 Ρ‡ 20 ΠΌΠΈΠ½ 3 Ρ‡ 35 ΠΌΠΈΠ½ – 40 ΠΌΠΈΠ½

    2 Ρ‡ 20 ΠΌΠΈΠ½ – 35 ΠΌΠΈΠ½ 2 Ρ‡ 30 ΠΌΠΈΠ½ + 2 Ρ‡ 30 ΠΌΠΈΠ½

    АрифмСтичСскиС дСйствия. 3-ΠΉ класс

    Π’ΠΈΠΏ ΡƒΡ€ΠΎΠΊΠ°: Π·Π°ΠΊΡ€Π΅ΠΏΠ»Π΅Π½ΠΈΠ΅ ΠΏΡ€ΠΎΠΉΠ΄Π΅Π½Π½ΠΎΠ³ΠΎ ΠΌΠ°Ρ‚Π΅Ρ€ΠΈΠ°Π»Π°.

    ЦСль ΡƒΡ€ΠΎΠΊΠ°:

    • Π·Π°ΠΊΡ€Π΅ΠΏΠΈΡ‚ΡŒ ΡƒΠΌΠ΅Π½ΠΈΠ΅ ΠΈΡΠΏΠΎΠ»ΡŒΠ·ΠΎΠ²Π°Ρ‚ΡŒ свойства арифмСтичСских дСйствий для ΡƒΡΠΏΠ΅ΡˆΠ½ΠΎΠ³ΠΎ Ρ€Π΅ΡˆΠ΅Π½ΠΈΡ Π²Ρ‹Ρ€Π°ΠΆΠ΅Π½ΠΈΠΉ ΠΈ Π·Π°Π΄Π°Ρ‡ Π² нСсколько дСйствий;
    • умСния ΠΏΠΎΠ»ΡŒΠ·ΠΎΠ²Π°Ρ‚ΡŒΡΡ Π°Π»Π³ΠΎΡ€ΠΈΡ‚ΠΌΠΎΠΌ порядка дСйствий ΠΏΡ€ΠΈ Π²Ρ‹ΠΏΠΎΠ»Π½Π΅Π½ΠΈΠΈ вычислСнии ΠΏΡ€ΠΈ Ρ€Π΅ΡˆΠ΅Π½ΠΈΠΈ Π²Ρ‹Ρ€Π°ΠΆΠ΅Π½ΠΈΠΉ, ΡƒΡ€Π°Π²Π½Π΅Π½ΠΈΠΉ ΠΈ Π·Π°Π΄Π°Ρ‡;
    • ΡΠΏΠΎΡΠΎΠ±ΡΡ‚Π²ΠΎΠ²Π°Ρ‚ΡŒ Ρ€Π°Π·Π²ΠΈΡ‚ΠΈΡŽ логичСского ΠΌΡ‹ΡˆΠ»Π΅Π½ΠΈΡ, ΡƒΠΌΠ΅Π½ΠΈΡŽ ΠΎΠ±ΠΎΠ±Ρ‰Π°Ρ‚ΡŒ ΠΈ Π΄Π΅Π»Π°Ρ‚ΡŒ обоснованныС Π²Ρ‹Π²ΠΎΠ΄Ρ‹;
    • ΡΠΏΠΎΡΠΎΠ±ΡΡ‚Π²ΠΎΠ²Π°Ρ‚ΡŒ Ρ€Π°Π·Π²ΠΈΡ‚ΠΈΡŽ ΠΏΠΎΠ·Π½Π°Π²Π°Ρ‚Π΅Π»ΡŒΠ½ΠΎΠΉ активности ΠΈ матСматичСской Ρ€Π΅Ρ‡ΠΈ учащихся;Β 
    • Π‘ΠΏΠΎΡΠΎΠ±ΡΡ‚Π²ΠΎΠ²Π°Ρ‚ΡŒ Ρ„ΠΎΡ€ΠΌΠΈΡ€ΠΎΠ²Π°Π½ΠΈΡŽ:
      • умСния Ρ€Π°Π±ΠΎΡ‚Π°Ρ‚ΡŒ Π² Π±Ρ€ΠΈΠ³Π°Π΄Π°Ρ…, ΡΠ»ΡƒΡˆΠ°Ρ‚ΡŒ Ρ‚ΠΎΠ²Π°Ρ€ΠΈΡˆΠ°;
      • Π²ΠΎΡΠΏΠΈΡ‚Π°Π½ΠΈΡŽ самокритичности, самооцСнки своСй Ρ€Π°Π±ΠΎΡ‚Ρ‹;
      • Π²ΠΎΡΠΏΠΈΡ‚Π°Π½ΠΈΡŽ Π°ΠΊΡ‚ΠΈΠ²Π½ΠΎΠΉ личности.

    ΠŸΠΎΠ½ΡΡ‚ΠΈΡ: арифмСтичСскоС дСйствиС, Π²Ρ‹Ρ€Π°ΠΆΠ΅Π½ΠΈΠ΅, слоТСниС, Π²Ρ‹Ρ‡ΠΈΡ‚Π°Π½ΠΈΠ΅, ΡƒΠΌΠ½ΠΎΠΆΠ΅Π½ΠΈΠ΅, ΠΌΠ½ΠΎΠΆΠΈΡ‚Π΅Π»ΡŒ, Β ΠΏΡ€ΠΎΠΈΠ·Π²Π΅Π΄Π΅Π½ΠΈΠ΅,Β  Π΄Π΅Π»Π΅Π½ΠΈΠ΅, Π΄Π΅Π»ΠΈΠΌΠΎΠ΅, Π΄Π΅Π»ΠΈΡ‚Π΅Π»ΡŒ, частноС, ΡƒΡ€Π°Π²Π½Π΅Π½ΠΈΠ΅, ΠΏΠ΅Ρ€ΠΈΠΌΠ΅Ρ‚Ρ€, ΠΏΠ»ΠΎΡ‰Π°Π΄ΡŒ.Β Β Β Β Β Β Β Β Β Β Β Β Β Β Β Β Β Β 

    ΠžΠ±ΠΎΡ€ΡƒΠ΄ΠΎΠ²Π°Π½ΠΈΠ΅: гСомСтричСскиС Ρ„ΠΈΠ³ΡƒΡ€Ρ‹ Ρ€Π°Π·Π½ΠΎΠΉ Ρ„ΠΎΡ€ΠΌΡ‹ Ρ‚Ρ€Π΅Ρ… Ρ†Π²Π΅Ρ‚ΠΎΠ², ΠΏΠΈΡ€Π°ΠΌΠΈΠ΄Ρ‹ ΠΈ ΠΏΠ°Π»ΠΈΡ‚Ρ€Π° Π½Π° ΡƒΠΌΠ½ΠΎΠΆΠ΅Π½ΠΈΠ΅ ΠΈ Π΄Π΅Π»Π΅Π½ΠΈΠ΅ ΠΈΠ· пособия Β«Π‘ΠΏΠ΅ΠΊΡ‚Ρ€ ΠΌΠ°Ρ‚Π΅ΠΌΠ°Ρ‚ΠΈΠΊΠ°Β», листы с заданиям для Π±Ρ€ΠΈΠ³Π°Π΄ (ΠŸΡ€ΠΈΠ»ΠΎΠΆΠ΅Π½ΠΈΠ΅ 1), ΡƒΡ‡Π΅Π±Π½ΠΈΠΊ ΠΌΠ°Ρ‚Π΅ΠΌΠ°Ρ‚ΠΈΠΊΠΈ 3 класс 1 Ρ‡Π°ΡΡ‚ΡŒ авторского ΠΊΠΎΠ»Π»Π΅ΠΊΡ‚ΠΈΠ²Π° Π’.Π•. Π”Π΅ΠΌΠΈΠ΄ΠΎΠ²ΠΎΠΉ, Π‘.А. Козловой, А.П. Π’ΠΎΠ½ΠΊΠΈΡ….

    Π₯ΠΎΠ΄ ΡƒΡ€ΠΎΠΊΠ°

    1. ΠžΡ€Π³Π°Π½ΠΈΠ·Π°Ρ†ΠΈΠΎΠ½Π½Ρ‹ΠΉ ΠΌΠΎΠΌΠ΅Π½Ρ‚.

    Π‘ ΠΏΠΎΠΌΠΎΡ‰ΡŒΡŽ «волшСбной ΡˆΠ»ΡΠΏΡ‹Β» Π΄Π΅Ρ‚ΠΈ дСлятся Π½Π° 4 ΠΊΠΎΠΌΠ°Π½Π΄Ρ‹. (ΠŸΡ€ΠΈ Π²Ρ…ΠΎΠ΄Π΅ Π² класс дСтям прСдлагаСтся гСомСтричСскиС Ρ„ΠΈΠ³ΡƒΡ€ΠΊΠΈ: Ρ‚Ρ€Π΅ΡƒΠ³ΠΎΠ»ΡŒΠ½ΠΈΠΊΠΈ, ΠΊΡ€ΡƒΠ³ΠΈ, ΠΊΠ²Π°Π΄Ρ€Π°Ρ‚Ρ‹, ΠΈ ΠΏΡ€ΡΠΌΠΎΡƒΠ³ΠΎΠ»ΡŒΠ½ΠΈΠΊΠΈ Ρ€Π°Π·Π½ΠΎΠ³ΠΎ Ρ†Π²Π΅Ρ‚Π°.)

    РассаТиваСмся Π·Π° столы. (Π”Π΅Ρ‚ΠΈ с Ρ‚Ρ€Π΅ΡƒΠ³ΠΎΠ»ΡŒΠ½ΠΈΠΊΠ°ΠΌΠΈ садятся Π·Π° ΠΎΠ΄ΠΈΠ½ стол, с ΠΊΠ²Π°Π΄Ρ€Π°Ρ‚Π°ΠΌΠΈ – Π·Π° Π΄Ρ€ΡƒΠ³ΠΎΠΉ, с ΠΏΡ€ΡΠΌΠΎΡƒΠ³ΠΎΠ»ΡŒΠ½ΠΈΠΊΠ°ΠΌΠΈ – Π·Π° Ρ‚Ρ€Π΅Ρ‚ΠΈΠΉ ΠΈ с ΠΊΡ€ΡƒΠΆΠΊΠ°ΠΌΠΈ Π·Π° Ρ‡Π΅Ρ‚Π²Π΅Ρ€Ρ‚Ρ‹ΠΉ), Ρ‚.ΠΎ. ΠΌΡ‹ всС Ρ€Π°Π·Π±ΠΈΠ»ΠΈΡΡŒ Π½Π° Ρ‡Π΅Ρ‚Ρ‹Ρ€Π΅ ΠΊΠΎΠΌΠ°Π½Π΄Ρ‹ – Π±Ρ€ΠΈΠ³Π°Π΄Ρ‹. (Π’ ΠΊΠΎΠΌΠ°Π½Π΄Π°Ρ… силы ΠΌΠΎΠ³ΡƒΡ‚ Π±Ρ‹Ρ‚ΡŒ Π½Π΅Ρ€Π°Π²Π½Ρ‹ΠΌΠΈ, Π° Ρ‚Π°ΠΊΠΆΠ΅ Π² ΠΎΠ΄Π½Ρƒ ΠΊΠΎΠΌΠ°Π½Π΄Ρƒ ΠΌΠΎΠ³ΡƒΡ‚ ΠΏΠΎΠΏΠ°ΡΡ‚ΡŒ Π΄Π΅Ρ‚ΠΈ ΠΊΠΎΡ‚ΠΎΡ€Ρ‹Π΅ Π½Π΅ ΠΎΡ‡Π΅Π½ΡŒ Ρ…ΠΎΡ€ΠΎΡˆΠΎ ΠΎΠ±Ρ‰Π°ΡŽΡ‚ΡΡ Π΄Ρ€ΡƒΠ³ с Π΄Ρ€ΡƒΠ³ΠΎΠΌ, Π½ΠΎ ΠΈΠΌ придСтся Ρ€Π°Π±ΠΎΡ‚Π°Ρ‚ΡŒ вмСстС, Ρ‡Ρ‚ΠΎ ΠΏΠΎΠΌΠΎΠΆΠ΅Ρ‚ Π½Π°ΡƒΡ‡ΠΈΡ‚ΡŒΡΡΒ  ΠΎΠ±Ρ‰Π°Ρ‚ΡŒΡΡ ΠΈ Π½Π°ΡƒΡ‡ΠΈΡ‚ΡŒΡΡ Ρ‚Π΅Ρ€ΠΏΠΈΠΌΠΎ ΠΎΡ‚Π½ΠΎΡΠΈΡ‚ΡŒΡΡ ΠΊ ΠΏΡ€ΠΎΠ±Π»Π΅ΠΌΠ°ΠΌ своим ΠΈ Ρ‚ΠΎΠ²Π°Ρ€ΠΈΡ‰Π΅ΠΉ ΠΏΠΎ ΠΊΠΎΠΌΠ°Π½Π΄Π΅).

    2. Π Π°Π·ΠΌΠΈΠ½ΠΊΠ°.

    1) Π˜Π³Ρ€Π° Β«ΠŸΠ°Π»ΠΈΡ‚Ρ€Π°Β».

    Π£ ΠΊΠ°ΠΆΠ΄ΠΎΠ³ΠΎ ΠΈΠ³Ρ€ΠΎΠΊΠ° ΠΊΠΎΠΌΠ°Π½Π΄Ρ‹ ΠΏΠΎ Π΄Π²Π° ΠΊΡ€ΡƒΠΆΠΊΠ° ΠΈ Π΄Π΅Ρ‚ΠΈ ΠΈΡ‰ΡƒΡ‚ свои рисунки Π² Ρ†Π΅Π½Ρ‚Ρ€Π΅ ΠΊΡ€ΡƒΠ³Π°, Π·Π°Ρ‚Π΅ΠΌ находят Ρ‚ΠΎΡ‡Π½ΠΎ Ρ‚Π°ΠΊΠΎΠΉ ΠΆΠ΅ рисунок Π½Π° большСй окруТности.Β  ПослС Ρ‚ΠΎΠ³ΠΎ, ΠΊΠ°ΠΊ всС Ρ‡Π»Π΅Π½Ρ‹ ΠΊΠΎΠΌΠ°Π½Π΄Ρ‹ Π²Ρ‹ΠΏΠΎΠ»Π½ΠΈΠ»ΠΈ Π·Π°Π΄Π°Π½ΠΈΠ΅, ΡƒΡ‡ΠΈΡ‚Π΅Π»ΡŒ провСряСт  ΠΏΡ€Π°Π²ΠΈΠ»ΡŒΠ½ΠΎΡΡ‚ΡŒ выполнСния. (Π˜Π³Ρ€Π° способствуСт Ρ€Π°Π·Π²ΠΈΡ‚ΠΈΡŽ ΠΈ ΠΊΠΎΠ½Ρ†Π΅Π½Ρ‚Ρ€Π°Ρ†ΠΈΠΈ внимания, умСния ΡΡ€Π°Π²Π½ΠΈΠ²Π°Ρ‚ΡŒ ΠΈ ΠΎΠ±ΠΎΠ±Ρ‰Π°Ρ‚ΡŒ). (Π€ΠΎΡ‚ΠΎ β„– 1, 2, 3, 4)

    Π€ΠΎΡ‚ΠΎ β„– 1.Β  Начало Ρ€Π°Π±ΠΎΡ‚Ρ‹ с  ΠΈΠ³Ρ€ΠΎΠΉ Β«ΠŸΠ°Π»ΠΈΡ‚Ρ€Π°Β» , пособиС ΠΈΠ· сСрии Β«Π‘ΠΏΠ΅ΠΊΡ‚Ρ€ – ΠΌΠ°Ρ‚Π΅ΠΌΠ°Ρ‚ΠΈΠΊΠ°Β»

    Π€ΠΎΡ‚ΠΎ β„– 2. НайдСна ошибка.

    Π€ΠΎΡ‚ΠΎ β„– 3.ΠžΡˆΠΈΠ±ΠΊΡƒ исправляСм.

    Π€ΠΎΡ‚ΠΎ β„– 4. Π˜ΡΠΏΡ€Π°Π²Π»Π΅Π½Π° ошибка.

    На фотографиях прСдставлСны этапы Ρ€Π°Π±ΠΎΡ‚Ρ‹ с Β«ΠŸΠ°Π»ΠΈΡ‚Ρ€ΠΎΠΉΒ» 1 Π·Π°Π΄Π°Π½ΠΈΠ΅ Π½Π°Β  внимания, ΠΈ умСния ΡΡ€Π°Π²Π½ΠΈΠ²Π°Ρ‚ΡŒ.

    2) РСшСниС Π²Ρ‹Ρ€Π°ΠΆΠ΅Π½ΠΈΠΉ Π² ΠΏΠΈΡ€Π°ΠΌΠΈΠ΄Π°Ρ… Β«Π’Π°Π±Π»ΠΈΡ‡Π½ΠΎΠ΅ ΡƒΠΌΠ½ΠΎΠΆΠ΅Π½ΠΈΠ΅ ΠΈ Π΄Π΅Π»Π΅Π½ΠΈΠ΅Β».

    На это Π·Π°Π΄Π°Π½ΠΈΠ΅ даСтся Ρ€ΠΎΠ²Π½ΠΎ Ρ‚Ρ€ΠΈ ΠΌΠΈΠ½ΡƒΡ‚Ρ‹. ПослС Ρ‡Π΅Π³ΠΎ ΡƒΡ‡ΠΈΡ‚Π΅Π»ΡŒ просматриваСт количСство ΠΈ ΠΏΡ€Π°Π²ΠΈΠ»ΡŒΠ½ΠΎΡΡ‚ΡŒ Ρ€Π΅ΡˆΡ‘Π½Π½Ρ‹Ρ… ΠΏΡ€ΠΈΠΌΠ΅Ρ€ΠΎΠ².

    3. ЭстафСта.

    ΠšΠΎΠΌΠ°Π½Π΄Ρ‹ ΠΏΠΎΠ»ΡƒΡ‡Π°ΡŽΡ‚ Π·Π°Π΄Π°Π½ΠΈΠ΅ Π½Π° листочках ΠΏΠΎ ΠΎΠ΄Π½ΠΎΠΉ Ρ†Π΅ΠΏΠΎΡ‡ΠΊΠ΅ Π²Ρ‹Ρ€Π°ΠΆΠ΅Π½ΠΈΠΉ. ΠšΠ°ΠΆΠ΄Ρ‹ΠΉ ΠΈΠ³Ρ€ΠΎΠΊ ΠΊΠΎΠΌΠ°Π½Π΄Ρ‹ Π΄ΠΎΠ»ΠΆΠ΅Π½ Π·Π°ΠΏΠΈΡΠ°Ρ‚ΡŒ Ρ‚ΠΎΠ»ΡŒΠΊΠΎ ΠΎΠ΄ΠΈΠ½ ΠΎΡ‚Π²Π΅Ρ‚ ΠΈ ΠΏΡ€Π΅Π΄Π°Ρ‚ΡŒ ΡΠ»Π΅Π΄ΡƒΡŽΡ‰Π΅ΠΌΡƒ.

    β„–1.

    β„– 2. Π Π°ΡΡˆΠΈΡ„Ρ€ΠΎΠ²Ρ‹Π²Π°Π΅ΠΌ Π·Π°ΠΊΠΎΠ΄ΠΈΡ€ΠΎΠ²Π°Π½Π½ΠΎΠ΅ слово.

    ΠšΠ»ΡŽΡ‡ ΠΊ ΡˆΠΈΡ„Ρ€Ρƒ:

    10

    6

    24

    15

    16

    20

    с

    Ρ‚

    Π°

    Π²

    ΠΎ

    ΠΉ

    Π—Π°Π΄Π°Π½ΠΈΠ΅ для Π±Ρ€ΠΈΠ³Π°Π΄.

    ( 40 – 4 ) : 9 + 42 : 7 =

    10

    с

    80 : 8 βˆ™ 2 – 5 =

    15

    Π²

    96 – 4 βˆ™ 20 =

    16

    ΠΎ

    0 βˆ™ (31 – 25 ) + 4 βˆ™ 5 =

    20

    ΠΉ

    9 βˆ™Β  2 : 3 + 24 : 6 =

    10

    с

    8Β  βˆ™ 3 : ( 28 ; 7 ) =

    6

    Ρ‚

    100 : 10Β  + 35 : 7 =

    15

    Π²

    100 – (60 : 3 + 7 βˆ™ 8)

    24

    Π°

    Π£Ρ‡ΠΈΡ‚Π΅Π»ΡŒ: Π£ нас ΠΏΠΎΠ»ΡƒΡ‡ΠΈΠ»ΠΎΡΡŒ слово «свойства». ΠŸΡ€Π΅Π΄ΠΏΠΎΠ»ΠΎΠΆΠΈΡ‚Π΅, какая Ρ‚Π΅ΠΌΠ° нашСго ΡƒΡ€ΠΎΠΊΠ°. Π§Ρ‚ΠΎ Π²Ρ‹ ΠΌΠΎΠΆΠ΅Ρ‚Π΅ Ρ€Π°ΡΡΠΊΠ°Π·Π°Ρ‚ΡŒ ΠΎ свойствах?

    Π”Π΅Ρ‚ΠΈ ΠΎΡ‚Π²Π΅Ρ‡Π°ΡŽΡ‚, Ρ‡Ρ‚ΠΎ это ΠΌΠΎΠ³ΡƒΡ‚ Π±Ρ‹Ρ‚ΡŒ свойства арифмСтичСских дСйствий.

    Π£Ρ‡ΠΈΡ‚Π΅Π»ΡŒ: Какими свойствами ΠΎΠ±Π»Π°Π΄Π°Π΅Ρ‚ дСйствиС ΡƒΠΌΠ½ΠΎΠΆΠ΅Π½ΠΈΠ΅? (ΠŸΠ΅Ρ€Π΅ΠΌΠ΅ΡΡ‚ΠΈΡ‚Π΅Π»ΡŒΠ½ΠΎΠ΅, ΡΠΎΡ‡Π΅Ρ‚Π°Ρ‚Π΅Π»ΡŒΠ½ΠΎΠ΅ ΠΈ Ρ€Π°ΡΠΏΡ€Π΅Π΄Π΅Π»ΠΈΡ‚Π΅Π»ΡŒΠ½ΠΎΠ΅)

    – Для Ρ‡Π΅Π³ΠΎ Π½Π°Π΄ΠΎ Π²Π»Π°Π΄Π΅Ρ‚ΡŒ этими свойствами?

    Π£Ρ‡ΠΈΡ‚Π΅Π»ΡŒ Π΄Π°Π΅Ρ‚ Π·Π°Π΄Π°Π½ΠΈΠ΅ дСтям ΠΈΠ· ΡƒΡ‡Π΅Π±Π½ΠΈΠΊΠ° стр. 50 β„– 4.

    Π‘Ρ€Π°Π²Π½ΠΈΡ‚ΡŒ алгСбраичСскиС выраТСния ΠΈ Π²Ρ‹ΠΏΠΎΠ»Π½ΠΈΡ‚ΡŒ β„– 6 «лист – задания» Π±Ρ€ΠΈΠ³Π°Π΄Π°ΠΌ. (Задания выполняСт ΠΊΠ°ΠΆΠ΄Ρ‹ΠΉ ΡƒΡ‡Π΅Π½ΠΈΠΊ Π² своСй Ρ‚Π΅Ρ‚Ρ€Π°Π΄ΠΈ).

    β„– 4. Π‘Ρ‚Ρ€. 50.

    Π‘Ρ€Π°Π²Π½ΠΈ:

    a + (b + c) * (a + b) + cΒ Β Β Β Β Β  a – (d + f) * a – d – fΒ 

    (c + d) – y * ( c – y) = dΒ Β Β Β Β  (c + d) – y * (d – y) + c

    Π’ΠΎΡ‡Π½ΠΎ Ρ‚Π°ΠΊ ΠΆΠ΅ рассуТдая, Π²Ρ‹ΠΏΠΎΠ»Π½ΠΈΡ‚Π΅ сравнСниС Π²Ρ‹Ρ€Π°ΠΆΠ΅Π½ΠΈΠΉ:

    ( Π° + 7 ) βˆ™Β  5 *Β  Π°Β Β  βˆ™Β  5 + 7Β  βˆ™Β Β  5

    ( Π° + 7 ) βˆ™Β  5 *Β  Π°Β Β  βˆ™Β  5 + 3 5

    ( Π° + 7 ) βˆ™Β  5 *Β  Π°Β  +Β  3 5

    Π£Ρ‡ΠΈΡ‚Π΅Π»ΡŒ: ΠšΡ€ΠΎΠΌΠ΅ свойств арифмСтичСских дСйствий ΠΎΠ±ΡΠ·Π°Ρ‚Π΅Π»ΡŒΠ½ΠΎ Π½Π°Π΄ΠΎ Π²Π»Π°Π΄Π΅Ρ‚ΡŒ ΠΏΡ€Π°Π²ΠΈΠ»Π°ΠΌΠΈ порядка дСйствий.

    Π£Ρ‡ΠΈΡ‚Π΅Π»ΡŒ: вспомнитС, Π² ΠΊΠ°ΠΊΠΎΠΌ порядкС Π²Ρ‹ΠΏΠΎΠ»Π½ΡΡŽΡ‚ΡΡ дСйствия Π² выраТСниях, Π² ΠΊΠΎΡ‚ΠΎΡ€Ρ‹Ρ…:

    • ΠΏΡ€ΠΈΡΡƒΡ‚ΡΡ‚Π²ΡƒΡŽΡ‚ Ρ‚ΠΎΠ»ΡŒΠΊΠΎ дСйствия слоТСния ΠΈ вычитания.
    • ΠΏΡ€ΠΈΡΡƒΡ‚ΡΡ‚Π²ΡƒΡŽΡ‚ Ρ‚ΠΎΠ»ΡŒΠΊΠΎ дСйствия умноТСния ΠΈ дСлСния.
    • ΠΏΡ€ΠΈΡΡƒΡ‚ΡΡ‚Π²ΡƒΡŽΡ‚ дСйствия слоТСния ΠΈ вычитания, умноТСния ΠΈ дСлСния.
    • ΠΏΡ€ΠΈΡΡƒΡ‚ΡΡ‚Π²ΡƒΡŽΡ‚ дСйствия слоТСния ΠΈ вычитания, умноТСния ΠΈ дСлСния.
    • Π½Π°Π»ΠΈΡ‡ΠΈΠ΅ скобок Π² Π²Ρ‹Ρ€Π°ΠΆΠ΅Π½ΠΈΠΈ ΡƒΠΊΠ°Π·Ρ‹Π²Π°Π΅Ρ‚ ΠΎ соблюдСнии Π΅Ρ‰Ρ‘ ΠΎΠ΄Π½ΠΎΠ³ΠΎ ΠΏΡ€Π°Π²ΠΈΠ»Π°. Какого?

    Π£ΠΊΠ°ΠΆΠΈΡ‚Π΅ порядок дСйствий Π² выраТСниях, Π² числовых выраТСниях Π½Π°ΠΉΠ΄ΠΈΡ‚Π΅ Π·Π½Π°Ρ‡Π΅Π½ΠΈΠ΅.Β 

    Π”Π΅Ρ‚ΠΈ ΠΊΠΎΠ»Π»Π΅ΠΊΡ‚ΠΈΠ²Π½ΠΎ ΠΏΠΎ Π±Ρ€ΠΈΠ³Π°Π΄Π°ΠΌ Π²Ρ‹ΠΏΠΎΠ»Π½ΡΡŽΡ‚ Π·Π°Π΄Π°Π½ΠΈΠ΅ β„– 3.

    n – a + d – c
    53 – 11 + 7 – 9
    n : a βˆ™ d : c
    (3 + 4) βˆ™ (40 : 8) – 11
    (n + b) : c βˆ™ d – m
    n + b : (c βˆ™ d) – m

    n – ( a + d – c )
    53 – (11 + 7 – 9)
    n βˆ™Β  ( aΒ  βˆ™ d : c )
    47 + 6 βˆ™7 – 36 : 4
    n + b : (c βˆ™ d – m)
    (n + b) : (c βˆ™ d – m)

    n – (a + d ) – c
    53 – (11 + 7) – 9
    n : (a : d ) βˆ™ c
    4 βˆ™ (7 βˆ™ 8 –Β  50) : 3 + 6 βˆ™ 0
    (n + b : c )βˆ™ d – m
    (n + b : c )βˆ™ (d – m)

    Π£Ρ‡ΠΈΡ‚Π΅Π»ΡŒ: ΠœΡ‹ ΠΏΠΎΠ²Ρ‚ΠΎΡ€ΠΈΠ»ΠΈ ΠΏΡ€Π°Π²ΠΈΠ»Π° порядка дСйствий, ΠΈ ΠΏΠΎΠ»ΡŒΠ·ΡƒΡΡΡŒ этими ΠΏΡ€Π°Π²ΠΈΠ»Π°ΠΌΠΈ нашли значСния числовых Π²Ρ‹Ρ€Π°ΠΆΠ΅Π½ΠΈΠΉ. ΠŸΠΎΠ»ΡƒΡ‡ΠΈΠ»ΠΈ ΡΠ»Π΅Π΄ΡƒΡŽΡ‰ΠΈΠ΅ числа: 40, 44, 26, 24, 80, 8.

    – Π§Ρ‚ΠΎ интСрСсного Π²Ρ‹ Π·Π°ΠΌΠ΅Ρ‚ΠΈΠ»ΠΈ Π² этом ряду чисСл?

    – МоТно Π»ΠΈ Π½Π°ΠΉΡ‚ΠΈ срСди Π½ΠΈΡ… лишнСС число?

    Π”Π΅Ρ‚ΠΈ ΠΏΡ€Π΅Π΄Π»Π°Π³Π°ΡŽΡ‚ лишнСС число – 8 (ΠΎΠ΄Π½ΠΎΠ·Π½Π°Ρ‡Π½ΠΎΠ΅) ΠΈΠ»ΠΈ 26 Ρ‚.ΠΊ. Π½Π΅ дСлится Π½Π° 4, 20 – ΠΊΡ€ΡƒΠ³Π»ΠΎΠ΅ число.

    – КакиС числа дСлятся Π½Π° 8, 10, 5. ΠŸΠΎΠ΄Ρ‡Π΅Ρ€ΠΊΠ½ΠΈΡ‚Π΅Β  эти числа. ( Π˜ΡΠΏΠΎΠ»ΡŒΠ·ΡƒΠ΅ΠΌ Ρ†Π²Π΅Ρ‚Π½Ρ‹Π΅ ΠΊΠ°Ρ€Π°Π½Π΄Π°ΡˆΠΈ)

    4. Π€ΠΈΠ·ΠΊΡƒΠ»ΡŒΡ‚ΠΌΠΈΠ½ΡƒΡ‚ΠΊΠ°.

    «Выросли Π΄Π΅Ρ€Π΅Π²ΡŒΡ Π² ΠΏΠΎΠ»Π΅. Π₯ΠΎΡ€ΠΎΡˆΠΎ расти Π½Π° Π²ΠΎΠ»Π΅!
    КаТдоС стараСтся, ΠΊ Π½Π΅Π±Ρƒ ΠΊ солнцу тянСтся,
    (ΠŸΠΎΡ‚ΡΠ³ΠΈΠ²Π°Π½ΠΈΡ – Ρ€ΡƒΠΊΠΈ Π²Π²Π΅Ρ€Ρ… ΠΈ Π² стороны.)
    Π’ΠΎΡ‚ ΠΏΠΎΠ΄ΡƒΠ» вСсёлый Π²Π΅Ρ‚Π΅Ρ€, Π·Π°ΠΊΠ°Ρ‡Π°Π»ΠΈΡΡŒ Ρ‚ΡƒΡ‚ ΠΆΠ΅ Π²Π΅Ρ‚ΠΊΠΈ,
    (ΠœΠ°Ρ…ΠΈ Ρ€ΡƒΠΊΠ°ΠΌΠΈ Π²Π²Π΅Ρ€Ρ…, Π² стороны.)
    Π”Π°ΠΆΠ΅ толстыС стволы наклонились Π΄ΠΎ Π·Π΅ΠΌΠ»ΠΈ.
    Π’ΠΏΡ€Π°Π²ΠΎ – Π²Π»Π΅Π²ΠΎ, Π²Π·Π°Π΄ – Π²ΠΏΠ΅Ρ€Ρ‘Π΄ Ρ‚Π°ΠΊ Π΄Π΅Ρ€Π΅Π²ΡŒΡ Π²Π΅Ρ‚Π΅Ρ€ Π³Π½Ρ‘Ρ‚.
    (Наклоны Π²ΠΏΠ΅Ρ€Ρ‘Π΄, Π½Π°Π·Π°Π΄ ΠΈ Π² стороны.)
    Он ΠΈΡ… Π²Π΅Ρ€Ρ‚ΠΈΡ‚, ΠΎΠ½ ΠΈΡ… ΠΊΡ€ΡƒΡ‚ΠΈΡ‚. Π”Π° ΠΊΠΎΠ³Π΄Π° ΠΆΠ΅ ΠΎΡ‚Π΄Ρ‹Ρ… Π±ΡƒΠ΄Π΅Ρ‚.
    (Π’Ρ€Π°Ρ‰Π΅Π½ΠΈΠ΅ Ρ‚ΡƒΠ»ΠΎΠ²ΠΈΡ‰Π΅ΠΌ).
    Π’Π΅Ρ‚Π΅Ρ€ стих. Π’Π·ΠΎΡˆΠ»Π° Π»ΡƒΠ½Π°. Наступила Ρ‚ΠΈΡˆΠΈΠ½Π°.
    (Π”Π΅Ρ‚ΠΈ садятся Π·Π° ΠΏΠ°Ρ€Ρ‚Ρ‹).

    Π£Ρ‡ΠΈΡ‚Π΅Π»ΡŒ ΠΏΡ€Π΅Π΄Π»Π°Π³Π°Π΅Ρ‚ дСтям Ρ€Π΅ΡˆΠΈΡ‚ΡŒ Π·Π°Π΄Π°Ρ‡ΠΈ, ΠΊΠΎΡ‚ΠΎΡ€Ρ‹Π΅ написаны Π½Π° листах-заданиях.

    ΠšΠΎΠΌΠ°Π½Π΄Ρ‹ совмСстно Ρ€Π΅ΡˆΠ°ΡŽΡ‚,Β  β„– 1, 2, 3 ΠΈ Π·Π°ΠΏΠΈΡΡ‹Π²Π°ΡŽΡ‚ Π² Ρ‚Π΅Ρ‚Ρ€Π°Π΄ΠΈ Ρ€Π΅ΡˆΠ΅Π½ΠΈΡ ΠΏΠΎ дСйствиям ΠΈΠ»ΠΈ Π²Ρ‹Ρ€Π°ΠΆΠ΅Π½ΠΈΠ΅ΠΌ.

    Π—Π°Π΄Π°Ρ‡Π° β„–1.

    Какой Ρ‚ΠΎΠ»Ρ‰ΠΈΠ½Ρ‹ Π±ΡƒΠ΄Π΅Ρ‚ альбом Π² 20 страниц, Ссли ΠΊΠ°ΠΆΠ΄Ρ‹Π΅ 5 листов ΠΈΠΌΠ΅ΡŽΡ‚ Ρ‚ΠΎΠ»Ρ‰ΠΈΠ½Ρƒ 2 см?

    Π—Π°Π΄Π°Ρ‡Π° β„–2.

    Найди ΠΏΠ΅Ρ€ΠΈΠΌΠ΅Ρ‚Ρ€ ΠΏΡ€ΡΠΌΠΎΡƒΠ³ΠΎΠ»ΡŒΠ½ΠΈΠΊΠ° ΠΈ Π΅Π³ΠΎ ΠΏΠ»ΠΎΡ‰Π°Π΄ΡŒ, Ссли ΠΎΠ΄Π½Π° ΠΈΠ· Π΅Π³ΠΎ сторон Ρ€Π°Π²Π½Π° 6 см, это Π² 2 Ρ€Π°Π·Π° мСньшС, Ρ‡Π΅ΠΌΒ  другая сторона. Найди Π²Π΅Π»ΠΈΡ‡ΠΈΠ½Ρƒ стороны ΠΊΠ²Π°Π΄Ρ€Π°Ρ‚Π° с Ρ‚Π°ΠΊΠΈΠΌ ΠΆΠ΅ ΠΏΠ΅Ρ€ΠΈΠΌΠ΅Ρ‚Ρ€ΠΎΠΌ.

    Π—Π°Π΄Π°Ρ‡Π° β„– 4 (ΡƒΡ‡Π΅Π±Π½ΠΈΠΊ, с. 51 β„– 7)

    Π£Ρ‡ΠΈΡ‚Π΅Π»ΡŒ провСряСт Ρ€Π°Π±ΠΎΡ‚Ρƒ. Один ΡƒΡ‡Π΅Π½ΠΈΠΊ ΠΈΠ· ΠΊΠΎΠΌΠ°Π½Π΄Ρ‹ (Π±Ρ€ΠΈΠ³Π°Π΄Ρ‹) записываСт Ρ€Π΅ΡˆΠ΅Π½ΠΈΠ΅ Π½Π° доскС ΠΎΠ΄Π½ΠΎΠΉ ΠΈΠ· Π·Π°Π΄Π°Ρ‡ ΠΈ ΠΎΠ±ΡŠΡΡΠ½ΡΠ΅Ρ‚.

    β„– 1. 2см βˆ™ (20 : 2 : 5) = 2 βˆ™ (20 : (2 βˆ™ 5)) = 4 (см)

    β„– 2. 6Β  βˆ™ 2 βˆ™ 6Β  = 6 βˆ™ 6 βˆ™ 2 = 72 (ΠΊΠ². см)

    ( 6 + 6 βˆ™ 2) βˆ™ 2 : 4 = 9 (см)

    (6 βˆ™ 2 + 6 βˆ™ 2 βˆ™ 2) : 4 = 9 (см)

    β„– 3. (a + b) βˆ™ 2 – (a – 2 + b) βˆ™ 2 = a βˆ™ 2

    Π£Ρ‡ΠΈΡ‚Π΅Π»ΡŒ: ΠŸΡ€ΠΈ записи Ρ€Π΅ΡˆΠ΅Π½ΠΈΠΉ Π²Ρ‹Ρ€Π°ΠΆΠ΅Π½ΠΈΠ΅ΠΌ ΠΌΡ‹ соблюдали ΠΏΡ€Π°Π²ΠΈΠ»Π° порядка дСйствий ΠΈ воспользовались свойствами дСйствий. ΠŸΡ€Π΅Π΄Π»Π°Π³Π°ΡŽ Ρ€Π°ΡΡΠΌΠΎΡ‚Ρ€Π΅Ρ‚ΡŒ тСкст Π·Π°Π΄Π°Ρ‡ΠΈ ΠΈ Π΅Ρ‘ Ρ€Π΅ΡˆΠ΅Π½ΠΈΡΒ  β„–5 стр. 51. ΠžΠ±ΡŠΡΡΠ½ΠΈΡ‚Π΅, Ρ‡Ρ‚ΠΎ ΠΎΠ·Π½Π°Ρ‡Π°Π΅Ρ‚ ΠΊΠ°ΠΆΠ΄ΠΎΠ΅Β  Π²Ρ‹Ρ€Π°ΠΆΠ΅Π½ΠΈΠ΅.

    2 βˆ™ 15;Β Β Β Β Β Β Β Β Β Β Β Β  3 βˆ™ 15;Β Β Β Β Β Β Β Β Β Β Β  2 βˆ™ 15 + 3 βˆ™ 15;Β Β Β Β Β Β Β Β Β Β  3 βˆ™ 15 – 2 βˆ™ 15;

    – Π’ΠΎΠ·ΠΌΠΎΠΆΠ½Ρ‹ Π»ΠΈ Π΄Ρ€ΡƒΠ³ΠΈΠ΅ выраТСния?

    Π”Π΅Ρ‚ΠΈ, вспомнив Ρ€Π°ΡΠΏΡ€Π΅Π΄Π΅Π»ΠΈΡ‚Π΅Π»ΡŒΠ½ΠΎΠ΅ свойство умноТСния, ΠΏΡ€Π΅Π΄Π»Π°Π³Π°ΡŽΡ‚ ΡΠ»Π΅Π΄ΡƒΡŽΡ‰ΠΈΠ΅ выраТСния: (2 + 3)Β  βˆ™ 15;Β Β Β Β  (2 –Β  3) βˆ™ 15;

    Π”Π°Π»Π΅Π΅ ΡƒΡ‡ΠΈΡ‚Π΅Π»ΡŒ Π΄Π°Ρ‘Ρ‚ Π·Π°Π΄Π°Π½ΠΈΠ΅ ΠΏΠΎ Π²Π°Ρ€ΠΈΠ°Π½Ρ‚Π°ΠΌ β„– 6. Π—Π°ΠΏΠΈΡΠ°Ρ‚ΡŒ Ρ€Π΅ΡˆΠ΅Π½ΠΈΠ΅ ΠΏΠΎ дСйствиям с пояснСниСм ΠΈ Π²Ρ‹Ρ€Π°ΠΆΠ΅Π½ΠΈΠ΅ΠΌ, β„– 3 стр. 50 – Ρ€Π΅ΡˆΠ΅Π½ΠΈΠ΅ ΠΎΠ΄Π½ΠΎΠ³ΠΎ уравнСния Π½Π° Π²Ρ‹Π±ΠΎΡ€, ΠΊΠ°ΠΊΠΎΠ΅ большС нравится.

    Π±). – 1 Π²Π°Ρ€ΠΈΠ°Π½Ρ‚;Β Β Β  Π²). – 2 Π²Π°Ρ€ΠΈΠ°Π½Ρ‚;

    Π­Ρ‚ΠΎ Π·Π°Π΄Π°Π½ΠΈΠ΅ ΡƒΡ‡ΠΈΡ‚Π΅Π»ΡŒ провСряСт ΠΏΡ€ΠΈ ΠΏΡ€ΠΎΠ²Π΅Ρ€ΠΊΠ΅ Ρ‚Π΅Ρ‚Ρ€Π°Π΄Π΅ΠΉ.

    5. Π˜Ρ‚ΠΎΠ³ ΡƒΡ€ΠΎΠΊΠ°: ΠΊΠ°ΠΊ Π²Ρ‹ Π΄ΡƒΠΌΠ°Π΅Ρ‚Π΅, Ρ‡Ρ‚ΠΎ ΠΏΠΎΠ»Π΅Π·Π½ΠΎΠ³ΠΎ сСгодня для вас Π±Ρ‹Π»ΠΎ Π½Π° ΡƒΡ€ΠΎΠΊΠ΅.

    Π”Π΅Ρ‚ΠΈ Π²Ρ‹ΡΠΊΠ°Π·Ρ‹Π²Π°ΡŽΡ‚ своё ΠΌΠ½Π΅Π½ΠΈΠ΅.

    Π£Ρ‡ΠΈΡ‚Π΅Π»ΡŒ просит ΠΏΠΎΠΊΠ°Π·Π°Ρ‚ΡŒ Π΄Π΅Ρ‚Π΅ΠΉ своё настроСниС Π½Π° ΡƒΡ€ΠΎΠΊΠ΅.

    – А Ρ‚Π΅ΠΏΠ΅Ρ€ΡŒ я ΠΏΠΎΠΏΡ€ΠΎΡˆΡƒ вас Π½Π°Ρ€ΠΈΡΠΎΠ²Π°Ρ‚ΡŒ Β«ΠΌΠΎΡ€Π΄Π°ΡˆΠΊΡƒΒ» вашСго настроСния Π½Π° ΡƒΡ€ΠΎΠΊΠ΅.

    УчащиСся Ρ€ΠΈΡΡƒΡŽΡ‚ Π½Π° полях Π² Ρ‚Π΅Ρ‚Ρ€Π°Π΄ΠΈ послС классной Ρ€Π°Π±ΠΎΡ‚Ρ‹ условныС Π·Π½Π°ΠΊΠΈ, оцСнивая свою ΡƒΡΠΏΠ΅ΡˆΠ½ΠΎΡΡ‚ΡŒ Ρ€Π°Π±ΠΎΡ‚Ρ‹ Π½Π° ΡƒΡ€ΠΎΠΊΠ΅.Β 

    ☼ – радостноС, Ρƒ мСня всё Π»Π΅Π³ΠΊΠΎ ΠΏΠΎΠ»ΡƒΡ‡Π°Π»ΠΎΡΡŒ,

    β˜Ίβ€“ ΠΎΠΆΠΈΠ΄Π°Π½ΠΈΠ΅ Π½ΠΎΠ²ΠΎΠ³ΠΎ, Β ΠΌΠ½Π΅ Π±Ρ‹Π»ΠΎ интСрСсно,

    Ồ – Ρ‚Ρ€Π΅Π²ΠΎΠΆΠ½ΠΎΠ΅, Ρƒ мСня Π½Π΅ всё ΠΏΠΎΠ»ΡƒΡ‡ΠΈΠ»ΠΎΡΡŒ, Π½ΠΎ я разобрался, ΠΌΠ½Π΅ ΠΏΠΎΠΌΠΎΠ³Π»ΠΈ, ΠΎΠ±ΡŠΡΡΠ½ΠΈΠ»ΠΈβ€¦, Β 

    α»• – ΠΌΠ½Π΅ Π½ΡƒΠΆΠ½Π° ΠΏΠΎΠΌΠΎΡ‰ΡŒ, Ρ…ΠΎΡ€ΠΎΡˆΠΎ, Ρ‡Ρ‚ΠΎΒ  ΠΌΠΎΠΆΠ½ΠΎ Π±Ρ‹Π»ΠΎ ΡΠΏΡ€ΠΎΡΠΈΡ‚ΡŒ Ρƒ …

    Π£Ρ‡ΠΈΡ‚Π΅Π»ΡŒ ΠΎΠ±ΡŠΡΡΠ½ΡΠ΅Ρ‚ домашнСС Π·Π°Π΄Π°Π½ΠΈΠ΅. Π‘Ρ‚Ρ€Π°Π½ΠΈΡ†Π° 50 β„–5 – ΠΏΠΎΠΊΠ°Π·Π°Ρ‚ΡŒ способ вычислСний, страница 51 β„– 6Β  Π³). – Π·Π°ΠΏΠΈΡΠ°Ρ‚ΡŒ Ρ€Π΅ΡˆΠ΅Π½ΠΈΠ΅ Π·Π°Π΄Π°Ρ‡ΠΈ ΠΏΠΎ дСйствиям с пояснСниСм, Π° Π²Ρ‚ΠΎΡ€ΠΎΠΉ способ Ρ€Π΅ΡˆΠ΅Π½ΠΈΡ Π·Π°Π΄Π°Ρ‡ΠΈ – Π·Π°ΠΏΠΈΡΠ°Ρ‚ΡŒ Π²Ρ‹Ρ€Π°ΠΆΠ΅Π½ΠΈΠ΅ΠΌ.

    Π›ΠΈΡ‚Π΅Ρ€Π°Ρ‚ΡƒΡ€Π°:

    1. «Моя ΠΌΠ°Ρ‚Π΅ΠΌΠ°Ρ‚ΠΈΠΊΠ°Β» 3 класс 1 Ρ‡Π°ΡΡ‚ΡŒ. Π’.Π•. Π”Π΅ΠΌΠΈΠ΄ΠΎΠ²Π°, Π‘.А. Козлова, А.П. Π’ΠΎΠ½ΠΊΠΈΡ…;
    2. ДидактичСский ΠΌΠ°Ρ‚Π΅Ρ€ΠΈΠ°Π» ΠΊ ΡƒΡ‡Π΅Π±Π½ΠΈΠΊΡƒ Β« ΠœΠ°Ρ‚Π΅ΠΌΠ°Ρ‚ΠΈΠΊΠ°Β» 3 класс Π‘.А. Козлова, Π’.Н. Π“Π΅Ρ€Π°ΡΡŒΠΊΠΈΠ½Π°, Β Π›.А. Π’ΠΎΠ»ΠΊΠΎΠ²Π°;
    3. Β«ΠœΠ΅Ρ‚ΠΎΠ΄ΠΈΡ‡Π΅ΡΠΊΠΈΠ΅ Ρ€Π΅ΠΊΠΎΠΌΠ΅Π½Π΄Π°Ρ†ΠΈΠΈ для учитСля» Β«ΠœΠ°Ρ‚Π΅ΠΌΠ°Ρ‚ΠΈΠΊΠ΅Β» 3 класс, Π‘.А. Козлова, А. Π“. Π ΡƒΠ±ΠΈΠ½;
    4. Β«Π£Ρ‡ΠΈΠΌ ΠΌΠ°Ρ‚Π΅ΠΌΠ°Ρ‚ΠΈΠΊΡƒ с ΡƒΠ²Π»Π΅Ρ‡Π΅Π½ΠΈΠ΅ΠΌΒ» А.Π’. ΠšΠΎΡ‡Π΅Ρ€Π³ΠΈΠ½Π°, Π›.И. Π“Π°ΠΉΠ΄ΠΈΠ½Π°;
    5. Β«Π˜Π³Ρ€Π°Ρ, учимся матСматикС»  Π§ΠΈΠ»ΠΈΠ½Π³ΠΈΡ€ΠΎΠ²Π°Β  Π›. К. 1993 Π³;
    6. Β«Π€ΠΈΠ·ΠΊΡƒΠ»ΡŒΡ‚ΡƒΡ€Π½Ρ‹Π΅ ΠΌΠΈΠ½ΡƒΡ‚ΠΊΠΈΒ» О.Π’. Π£Π·ΠΎΡ€ΠΎΠ²Π°, Π•.А. НСфСдова.

    ΠŸΡ€ΠΎΠ³Ρ€Π°ΠΌΠΌΠ° элСктивного курса ΠΏΠΎ ΠΌΠ°Ρ‚Π΅ΠΌΠ°Ρ‚ΠΈΠΊΠ΅ Учимся Ρ€Π΅ΡˆΠ°Ρ‚ΡŒ Π·Π°Π΄Π°Ρ‡ΠΈ 3 класс

    ΠœΡƒΠ½ΠΈΡ†ΠΈΠΏΠ°Π»ΡŒΠ½ΠΎΠ΅ ΠΊΠ°Π·Ρ‘Π½Π½ΠΎΠ΅ ΠΎΠ±Ρ‰Π΅ΠΎΠ±Ρ€Π°Π·ΠΎΠ²Π°Ρ‚Π΅Π»ΡŒΠ½ΠΎΠ΅ ΡƒΡ‡Ρ€Π΅ΠΆΠ΄Π΅Π½ΠΈΠ΅ «БрСдняя ΠΎΠ±Ρ‰Π΅ΠΎΠ±Ρ€Π°Π·ΠΎΠ²Π°Ρ‚Π΅Π»ΡŒΠ½Π°Ρ школа β„–4Β»
    УВВЕРЖДЕНА ΠŸΠ Π˜ΠšΠΠ—ΠžΠœ МКОУБОШ β„–4 ΠΎΡ‚ Β«___Β»________2014 Π³. β„–___

    Π”ΠΈΡ€Π΅ΠΊΡ‚ΠΎΡ€ МКОУБОШ β„–4

    _____________ М.Π’. ΠšΡ€Π°Π²Ρ†ΠΎΠ²Π°

    Π ΠΠ‘ΠžΠ§ΠΠ― ΠŸΠ ΠžΠ“Π ΠΠœΠœΠ Π­Π›Π•ΠšΠ’Π˜Π’ΠΠžΠ“Πž КУРБА ПО ΠœΠΠ’Π•ΠœΠΠ’Π˜ΠšΠ•

    «УЧИМБЯ РЕШАВЬ Π—ΠΠ”ΠΠ§Π˜Β»

    3 ΠšΠ›ΠΠ‘Π‘

    Ломонос Ольга Π‘Π΅Ρ€Π³Π΅Π΅Π²Π½Π°

    ΡƒΡ‡ΠΈΡ‚Π΅Π»ΡŒ Π½Π°Ρ‡Π°Π»ΡŒΠ½Ρ‹Ρ… классов

    МКОУ БОШ β„–4

    с.ΠœΠ°Π»Ρ‹Π΅ Π―Π³ΡƒΡ€Ρ‹

    ΠŸΠΎΡΡΠ½ΠΈΡ‚Π΅Π»ΡŒΠ½Π°Ρ записка

    Данная ΠΏΡ€ΠΎΠ³Ρ€Π°ΠΌΠΌΠ°Β Β Ρ€Π°Π·Ρ€Π°Π±ΠΎΡ‚Π°Π½Π° Π² соотвСтствии с Π½ΠΎΡ€ΠΌΠ°Ρ‚ΠΈΠ²Π½Ρ‹ΠΌΠΈ Π΄ΠΎΠΊΡƒΠΌΠ΅Π½Ρ‚Π°ΠΌΠΈ Ρ„Π΅Π΄Π΅Ρ€Π°Π»ΡŒΠ½ΠΎΠ³ΠΎ ΠΈ Ρ€Π΅Π³ΠΈΠΎΠ½Π°Π»ΡŒΠ½ΠΎΠ³ΠΎ уровня.

    Β ΠŸΡ€ΠΈΠΊΠ°Π· ΠœΠΈΠ½ΠΈΡΡ‚Π΅Ρ€ΡΡ‚Π²Π° образования ΠΈ Π½Π°ΡƒΠΊΠΈ Российской Π€Π΅Π΄Π΅Ρ€Π°Ρ†ΠΈΠΈ “Об ΡƒΡ‚Π²Π΅Ρ€ΠΆΠ΄Π΅Π½ΠΈΠΈ ΠΈ Π²Π²Π΅Π΄Π΅Π½ΠΈΠΈ Π² дСйствиС Ρ„Π΅Π΄Π΅Ρ€Π°Π»ΡŒΠ½ΠΎΠ³ΠΎ государствСнного ΠΎΠ±Ρ€Π°Π·ΠΎΠ²Π°Ρ‚Π΅Π»ΡŒΠ½ΠΎΠ³ΠΎ стандарта Π½Π°Ρ‡Π°Π»ΡŒΠ½ΠΎΠ³ΠΎ ΠΎΠ±Ρ‰Π΅Π³ΠΎ образования” ΠΎΡ‚ 06.10.2009, β„– 373.

    НовыС ΠΎΠ±Ρ€Π°Π·ΠΎΠ²Π°Ρ‚Π΅Π»ΡŒΠ½Ρ‹Π΅ стандарты поставили ΠΏΠ΅Ρ€Π΅Π΄ школой Π·Π°Π΄Π°Ρ‡Ρƒ ΠΎΠ±Ρ‰Π΅ΠΊΡƒΠ»ΡŒΡ‚ΡƒΡ€Π½ΠΎΠ³ΠΎ, личностного ΠΈ ΠΏΠΎΠ·Π½Π°Π²Π°Ρ‚Π΅Π»ΡŒΠ½ΠΎΠ³ΠΎ развития учащихся, ΠΎΠ±Π΅ΡΠΏΠ΅Ρ‡ΠΈΠ²Π°ΡŽΡ‰Π΅Π³ΠΎ Ρ‚Π°ΠΊΡƒΡŽ ΠΊΠ»ΡŽΡ‡Π΅Π²ΡƒΡŽ ΠΊΠΎΠΌΠΏΠ΅Ρ‚Π΅Π½Ρ†ΠΈΡŽ, ΠΊΠ°ΠΊ ΡƒΠΌΠ΅Π½ΠΈΠ΅ ΡƒΡ‡ΠΈΡ‚ΡŒΡΡ. РСшСниС поставлСнной Π·Π°Π΄Π°Ρ‡ΠΈ прСдполагаСтся ΠΎΡΡƒΡ‰Π΅ΡΡ‚Π²ΠΈΡ‚ΡŒ Ρ‡Π΅Ρ€Π΅Π· Ρ„ΠΎΡ€ΠΌΠΈΡ€ΠΎΠ²Π°Π½ΠΈΠ΅ ΡƒΠ½ΠΈΠ²Π΅Ρ€ΡΠ°Π»ΡŒΠ½Ρ‹Ρ… ΡƒΡ‡Π΅Π±Π½Ρ‹Ρ… дСйствий (Π£Π£Π”), ΠΎΠ±Π΅ΡΠΏΠ΅Ρ‡ΠΈΠ²Π°ΡŽΡ‰ΠΈΡ… ΡΠΏΠΎΡΠΎΠ±Π½ΠΎΡΡ‚ΡŒ учащихся ΠΊ ΡΠ°ΠΌΠΎΡ€Π°Π·Π²ΠΈΡ‚ΠΈΡŽ ΠΈ ΡΠ°ΠΌΠΎΡΠΎΠ²Π΅Ρ€ΡˆΠ΅Π½ΡΡ‚Π²ΠΎΠ²Π°Π½ΠΈΡŽ.

    ΠŸΠ΅Π΄Π°Π³ΠΎΠ³ΠΈΡ‡Π΅ΡΠΊΠ°Ρ Ρ†Π΅Π»Π΅ΡΠΎΠΎΠ±Ρ€Π°Π·Π½ΠΎΡΡ‚ΡŒ ΠΏΡ€ΠΎΠ³Ρ€Π°ΠΌΠΌΡ‹ ΠΎΠ±ΡŠΡΡΠ½ΡΠ΅Ρ‚ΡΡ Ρ„ΠΎΡ€ΠΌΠΈΡ€ΠΎΠ²Π°Π½ΠΈΠ΅ΠΌ ΠΏΡ€ΠΈΠ΅ΠΌΠΎΠ² умствСнной Π΄Π΅ΡΡ‚Π΅Π»ΡŒΠ½ΠΎΡΡ‚ΠΈ: Π°Π½Π°Π»ΠΈΠ·Π°, синтСза, сравнСния, классификации, Π°Π½Π°Π»ΠΎΠ³ΠΈΠΈ ΠΈ обобщСния, поиска ΠΈΠ½Ρ„ΠΎΡ€ΠΌΠ°Ρ†ΠΈΠΈ Π΅Π΅ ΠΎΠ±Ρ€Π°Π±ΠΎΡ‚ΠΊΠ° ΠΈ ΠΎΡ†Π΅Π½ΠΊΠ°

    Данная ΠΏΡ€ΠΎΠ³Ρ€Π°ΠΌΠΌΠ° Ρ„ΠΎΡ€ΠΌΠΈΡ€ΡƒΠ΅Ρ‚ ΠΏΠ΅Ρ€Π²ΠΎΠ½Π°Ρ‡Π°Π»ΡŒΠ½Ρ‹Π΅ ΠΈΡΡΠ»Π΅Π΄ΠΎΠ²Π°Ρ‚Π΅Π»ΡŒΡΠΊΠΈΠ΅ умСния учащихся Π½Π°Ρ‡Π°Π»ΡŒΠ½Ρ‹Ρ… классов, Π²ΠΊΠ»ΡŽΡ‡Π°Π΅Ρ‚ ΠΌΠ»Π°Π΄ΡˆΠΈΡ… школьников Π² Π°ΠΊΡ‚ΠΈΠ²Π½ΡƒΡŽ ΠΏΠΎΠ·Π½Π°Π²Π°Ρ‚Π΅Π»ΡŒΠ½ΡƒΡŽ Π΄Π΅ΡΡ‚Π΅Π»ΡŒΠ½ΠΎΡΡ‚ΡŒ, Π² частности, ΡƒΡ‡Π΅Π±Π½ΠΎ-ΠΈΡΡΠ»Π΅Π΄ΠΎΠ²Π°Ρ‚Π΅Π»ΡŒΡΠΊΡƒΡŽ. ΠΠΊΡ‚ΡƒΠ°Π»ΡŒΠ½ΠΎΡΡ‚ΡŒ ΠΏΡ€ΠΎΠ³Ρ€Π°ΠΌΠΌΡ‹ обусловлСна Ρ‚Π΅ΠΌ, Ρ‡Ρ‚ΠΎ Π² настоящСС врСмя Π±Π΅Π· основных ΠΌΡ‹ΡΠ»ΠΈΡ‚Π΅Π»ΡŒΠ½Ρ‹Ρ… ΠΎΠΏΠ΅Ρ€Π°Ρ†ΠΈΠΉ, ΠΊΠΎΡ‚ΠΎΡ€Ρ‹Π΅ ΠΏΠΎΠ·Π²ΠΎΠ»ΡΡŽΡ‚ Π²ΠΊΠ»ΡŽΡ‡ΠΈΡ‚ΡŒ ΠΈΠ½Ρ‚Π΅Π»Π»Π΅ΠΊΡ‚ΡƒΠ°Π»ΡŒΠ½ΡƒΡŽ Π΄Π΅ΡΡ‚Π΅Π»ΡŒΠ½ΠΎΡΡ‚ΡŒ младшСго школьника Π² Ρ€Π°Π·Π»ΠΈΡ‡Π½Ρ‹Π΅ ΡΠΎΠΎΡ‚Π½ΠΎΡˆΠ΅Π½ΠΈΡ с Π΄Ρ€ΡƒΠ³ΠΈΠΌΠΈ сторонами Π΅Π³ΠΎ личности, ΠΏΡ€Π΅ΠΆΠ΄Π΅ всСго с ΠΌΠΎΡ‚ΠΈΠ²Π°Ρ†ΠΈΠ΅ΠΉ ΠΈ интСрСсами, Π½Π΅ Π±ΡƒΠ΄Π΅Ρ‚ ΠΎΠΊΠ°Π·Ρ‹Π²Π°Ρ‚ΡŒ ΠΏΠΎΠ»ΠΎΠΆΠΈΡ‚Π΅Π»ΡŒΠ½ΠΎΠ΅ влияниС Π½Π° Ρ€Π°Π·Π²ΠΈΡ‚ΠΈΠ΅ внимания, памяти, эмоции ΠΈ Ρ€Π΅Ρ‡ΠΈ Ρ€Π΅Π±Π΅Π½ΠΊΠ°. Π˜ΡΡΠ»Π΅Π΄ΠΎΠ²Π°Ρ‚Π΅Π»ΡŒΡΠΊΠ°Ρ Ρ€Π°Π±ΠΎΡ‚Π° Π² Π½Π°Ρ‡Π°Π»ΡŒΠ½ΠΎΠΉ школС сСйчас особСнно Π°ΠΊΡ‚ΡƒΠ°Π»ΡŒΠ½Π°, ΠΏΠΎΡΠΊΠΎΠ»ΡŒΠΊΡƒ ΠΈΠΌΠ΅Π½Π½ΠΎ Π½Π° этом этапС учСбная Π΄Π΅ΡΡ‚Π΅Π»ΡŒΠ½ΠΎΡΡ‚ΡŒ являСтся Π²Π΅Π΄ΡƒΡ‰Π΅ΠΉ ΠΈ опрСдСляСт Ρ€Π°Π·Π²ΠΈΡ‚ΠΈΠ΅ Π³Π»Π°Π²Π½Ρ‹Ρ… ΠΏΠΎΠ·Π½Π°Π²Π°Ρ‚Π΅Π»ΡŒΠ½Ρ‹Ρ… особСнностСй Ρ€Π°Π·Π²ΠΈΠ²Π°ΡŽΡ‰Π΅ΠΉΡΡ личности. Π’ этот ΠΏΠ΅Ρ€ΠΈΠΎΠ΄ Ρ€Π°Π·Π²ΠΈΠ²Π°ΡŽΡ‚ΡΡ Ρ„ΠΎΡ€ΠΌΡ‹ ΠΌΡ‹ΡˆΠ»Π΅Π½ΠΈΡ, ΠΎΠ±Π΅ΡΠΏΠ΅Ρ‡ΠΈΠ²Π°ΡŽΡ‰ΠΈΠ΅ Π² дальнСйшСм усвоСниС систСмы Π½Π°ΡƒΡ‡Π½Ρ‹Ρ… Π·Π½Π°Π½ΠΈΠΉ, Ρ€Π°Π·Π²ΠΈΡ‚ΠΈΠ΅ Π½Π°ΡƒΡ‡Π½ΠΎΠ³ΠΎ, тСорСтичСского ΠΌΡ‹ΡˆΠ»Π΅Π½ΠΈΡ. Π’ ΠΌΠ»Π°Π΄ΡˆΠΈΡ… классах Π·Π°ΠΊΠ»Π°Π΄Ρ‹Π²Π°ΡŽΡ‚ΡΡ прСдпосылки ΡΠ°ΠΌΠΎΡΡ‚ΠΎΡΡ‚Π΅Π»ΡŒΠ½ΠΎΠΉ ΠΎΡ€ΠΈΠ΅Π½Ρ‚Π°Ρ†ΠΈΠΈ Π½Π΅ Ρ‚ΠΎΠ»ΡŒΠΊΠΎ Π² ΡƒΡ‡Π΅Π±Π΅, Π½ΠΎ ΠΈ Π² ΠΆΠΈΠ·Π½ΠΈ.

    ΠŸΠ»ΠΎΠ΄ΠΎΡ‚Π²ΠΎΡ€Π½Ρ‹ΠΌ ΠΌΠ°Ρ‚Π΅Ρ€ΠΈΠ°Π»ΠΎΠΌ для развития Π£Π£Π” Π² курсС ΠΌΠ°Ρ‚Π΅ΠΌΠ°Ρ‚ΠΈΠΊΠΈ Π½Π°Ρ‡Π°Π»ΡŒΠ½Ρ‹Ρ… классов ΡΠ²Π»ΡΡŽΡ‚ΡΡ тСкстовыС Π·Π°Π΄Π°Ρ‡ΠΈ. Π’Ρ€Π°Π΄ΠΈΡ†ΠΈΠΎΠ½Π½ΠΎ ΠΊ Π½ΠΈΠΌ относят Π·Π°Π΄Π°Ρ‡ΠΈ, ΠΊΠΎΡ‚ΠΎΡ€Ρ‹Π΅ Ρ‚Ρ€Π΅Π±ΡƒΡŽΡ‚ Π²Ρ‹Π±ΠΎΡ€Π° арифмСтичСских дСйствий ΠΈ выполнСния вычислСний для ΠΎΡ‚Π²Π΅Ρ‚Π° Π½Π° поставлСнный вопрос. Однако новая ΠΏΠ°Ρ€Π°Π΄ΠΈΠ³ΠΌΠ° Π½Π°Ρ‡Π°Π»ΡŒΠ½ΠΎΠ³ΠΎ образования, направлСнная Π½Π° ΡΠΎΡ†ΠΈΠ°Π»ΡŒΠ½ΠΎΠ΅, ΠΏΠΎΠ·Π½Π°Π²Π°Ρ‚Π΅Π»ΡŒΠ½ΠΎΠ΅, ΠΊΠΎΠΌΠΌΡƒΠ½ΠΈΠΊΠ°Ρ‚ΠΈΠ²Π½ΠΎΠ΅ ΠΈ ΠΈΠ½Ρ„ΠΎΡ€ΠΌΠ°Ρ†ΠΈΠΎΠ½Π½ΠΎΠ΅ Ρ€Π°Π·Π²ΠΈΡ‚ΠΈΠ΅ ΠΌΠ»Π°Π΄ΡˆΠΈΡ… школьников, Π½Π΅ Ρ‚ΠΎΠ»ΡŒΠΊΠΎ Ρ‚Ρ€Π΅Π±ΡƒΠ΅Ρ‚ овладСния ΠΎΠ±Ρ‰ΠΈΠΌ ΡƒΠΌΠ΅Π½ΠΈΠ΅ΠΌ Ρ€Π΅ΡˆΠ°Ρ‚ΡŒ арифмСтичСскиС Π·Π°Π΄Π°Ρ‡ΠΈ, Π½ΠΎ ΠΈ Π·Π½Π°Ρ‡ΠΈΡ‚Π΅Π»ΡŒΠ½ΠΎ Ρ€Π°ΡΡˆΠΈΡ€ΡΠ΅Ρ‚ содСрТаниС самого понятия тСкстовая Π·Π°Π΄Π°Ρ‡Π°. Анализ соврСмСнных ΡƒΡ‡Π΅Π±Π½ΠΈΠΊΠΎΠ² ΠΏΠΎ ΠΌΠ°Ρ‚Π΅ΠΌΠ°Ρ‚ΠΈΠΊΠ΅ для Π½Π°Ρ‡Π°Π»ΡŒΠ½Ρ‹Ρ… классов позволяСт ΠΊΠΎΠ½ΡΡ‚Π°Ρ‚ΠΈΡ€ΠΎΠ²Π°Ρ‚ΡŒ, Ρ‡Ρ‚ΠΎ наряду с арифмСтичСскими (тСкстовыми) Π·Π°Π΄Π°Ρ‡Π°ΠΌΠΈ Π² Π½ΠΈΡ… Π²ΠΊΠ»ΡŽΡ‡Π΅Π½Ρ‹ логичСскиС, ΠΊΠΎΠΌΠ±ΠΈΠ½Π°Ρ‚ΠΎΡ€Π½Ρ‹Π΅, гСомСтричСскиС, ситуационныС Π·Π°Π΄Π°Ρ‡ΠΈ, Ρ‚Ρ€Π΅Π±ΡƒΡŽΡ‰ΠΈΠ΅ ΠΎΡ‚ ΡƒΡ‡Π΅Π½ΠΈΠΊΠ° умСния ΠΈΠ½Ρ‚Π΅Π³Ρ€ΠΈΡ€ΠΎΠ²Π°Ρ‚ΡŒ знания Π½Π΅ Ρ‚ΠΎΠ»ΡŒΠΊΠΎ ΠΈΠ· Ρ€Π°Π·Π½Ρ‹Ρ… Ρ€Π°Π·Π΄Π΅Π»ΠΎΠ² Π½Π°Ρ‡Π°Π»ΡŒΠ½ΠΎΠ³ΠΎ курса ΠΌΠ°Ρ‚Π΅ΠΌΠ°Ρ‚ΠΈΠΊΠΈ, Π½ΠΎ ΠΈ ΠΈΠ· Ρ€Π°Π·Π½Ρ‹Ρ… ΡƒΡ‡Π΅Π±Π½Ρ‹Ρ… ΠΏΡ€Π΅Π΄ΠΌΠ΅Ρ‚ΠΎΠ².

    ΠŸΡ€ΠΈ Π°Π½Π°Π»ΠΈΠ·Π΅ ситуаций, описанных Π² Π·Π°Π΄Π°Ρ‡Π°Ρ…, младшиС школьники ΠΎΠ²Π»Π°Π΄Π΅Π²Π°ΡŽΡ‚ ΡƒΠΌΠ΅Π½ΠΈΠ΅ΠΌ ΠΈΡΠΊΠ°Ρ‚ΡŒ ΠΈ Π²Ρ‹Π΄Π΅Π»ΡΡ‚ΡŒ Π½Π΅ΠΎΠ±Ρ…ΠΎΠ΄ΠΈΠΌΡƒΡŽ ΠΈΠ½Ρ„ΠΎΡ€ΠΌΠ°Ρ†ΠΈΡŽ, ΠΏΡ€ΠΈΠΎΠ±Ρ€Π΅Ρ‚Π°ΡŽΡ‚ ΠΎΠΏΡ‹Ρ‚ смыслового чтСния ΠΈ Π°Π½Π°Π»ΠΈΠ·Π° ΠΎΠ±ΡŠΠ΅ΠΊΡ‚ΠΎΠ² с Ρ†Π΅Π»ΡŒΡŽ выдСлСния сущСствСнных ΠΈ нСсущСствСнных ΠΏΡ€ΠΈΠ·Π½Π°ΠΊΠΎΠ². На этапС поиска Ρ€Π΅ΡˆΠ΅Π½ΠΈΡ Π·Π°Π΄Π°Ρ‡ΠΈ Ρ€Π°Π·Π²ΠΈΠ²Π°ΡŽΡ‚ΡΡ Ρ‚Π°ΠΊΠΈΠ΅ Π£Π£Π”, ΠΊΠ°ΠΊ установлСниС ΠΏΡ€ΠΈΡ‡ΠΈΠ½Π½ΠΎ-слСдствСнных связСй, построСниС логичСской Ρ†Π΅ΠΏΠΎΡ‡ΠΊΠΈ рассуТдСний, Π²Ρ‹Π±ΠΎΡ€ Π½Π°ΠΈΠ±ΠΎΠ»Π΅Π΅ эффСктивных способов Ρ€Π΅ΡˆΠ΅Π½ΠΈΡ Π·Π°Π΄Π°Ρ‡ΠΈ Π² зависимости ΠΎΡ‚ ΠΊΠΎΠ½ΠΊΡ€Π΅Ρ‚Π½Ρ‹Ρ… условий, постановка ΠΈ Ρ„ΠΎΡ€ΠΌΡƒΠ»ΠΈΡ€ΠΎΠ²Π°Π½ΠΈΠ΅ ΠΏΡ€ΠΎΠ±Π»Π΅ΠΌΡ‹, ΡΠ°ΠΌΠΎΡΡ‚ΠΎΡΡ‚Π΅Π»ΡŒΠ½ΠΎΠ΅ созданиС Π°Π»Π³ΠΎΡ€ΠΈΡ‚ΠΌΠΎΠ² Π΄Π΅ΡΡ‚Π΅Π»ΡŒΠ½ΠΎΡΡ‚ΠΈ. ПослСднСС особСнно Π°ΠΊΡ‚ΡƒΠ°Π»ΡŒΠ½ΠΎ, Ρ‚Π°ΠΊ ΠΊΠ°ΠΊ Π²ΠΎ ΠΌΠ½ΠΎΠ³ΠΈΡ… Π·Π°Π΄Π°Ρ‡Π°Ρ… Ρ€Π°Π·Ρ€Π°Π±ΠΎΡ‚ΠΊΠ° способа дСйствия, ΠΏΠ»Π°Π½Π° ΠΈΠ»ΠΈ Π°Π»Π³ΠΎΡ€ΠΈΡ‚ΠΌΠ° Ρ€Π΅ΡˆΠ΅Π½ΠΈΡ являСтся основной Ρ†Π΅Π»ΡŒΡŽ. Π­Ρ‚ΠΎΡ‚ аспСкт Π²Π°ΠΆΠ΅Π½ ΠΈ для Π²ΠΊΠ»ΡŽΡ‡Π΅Π½ΠΈΡ ΠΈΠ½Ρ„ΠΎΡ€ΠΌΠ°Ρ†ΠΈΠΎΠ½Π½ΠΎΠ³ΠΎ направлСния Π² Π½Π°Ρ‡Π°Π»ΡŒΠ½Ρ‹ΠΉ курс ΠΌΠ°Ρ‚Π΅ΠΌΠ°Ρ‚ΠΈΠΊΠΈ. ИмСнно Ρ‡Π΅Ρ€Π΅Π· Ρ€Π΅ΡˆΠ΅Π½ΠΈΠ΅ Π·Π°Π΄Π°Ρ‡ ΠΌΠΎΠΆΠ½ΠΎ СстСствСнным ΠΎΠ±Ρ€Π°Π·ΠΎΠΌ Ρ„ΠΎΡ€ΠΌΠΈΡ€ΠΎΠ²Π°Ρ‚ΡŒ элСмСнты ΠΈΠ½Ρ„ΠΎΡ€ΠΌΠ°Ρ†ΠΈΠΎΠ½Π½ΠΎΠΉ ΠΊΡƒΠ»ΡŒΡ‚ΡƒΡ€Ρ‹: ΠΏΠΎΠ·Π½Π°ΠΊΠΎΠΌΠΈΡ‚ΡŒ учащихся со способами ΠΎΠ±Ρ€Π°Π±ΠΎΡ‚ΠΊΠΈ ΠΈΠ½Ρ„ΠΎΡ€ΠΌΠ°Ρ†ΠΈΠΈ ΠΈ наглядными Ρ„ΠΎΡ€ΠΌΠ°ΠΌΠΈ Π΅Π΅ прСдставлСния Π² Π²ΠΈΠ΄Π΅ Ρ‚Π°Π±Π»ΠΈΡ†, Π³Ρ€Π°Ρ„ΠΎΠ², схСм, Π±Π»ΠΎΠΊ-схСм ΠΈ Π΄Ρ€ΡƒΠ³ΠΈΡ… ΠΌΠΎΠ΄Π΅Π»Π΅ΠΉ.

    ΠžΠ±Ρ€Π°Π·ΠΎΠ²Π°Ρ‚Π΅Π»ΡŒΠ½Π°Ρ ΠΏΡ€ΠΎΠ³Ρ€Π°ΠΌΠΌΠ° ΠΏΡ€Π΅Π΄Π½Π°Π·Π½Π°Ρ‡Π΅Π½Π° для учащихся 3 класса. Π“Π»Π°Π²Π½ΠΎΠ΅ Π½Π°ΠΏΡ€Π°Π²Π»Π΅Π½ΠΈΠ΅ – раскрытиС ΠΈ Ρ€Π°Π·Π²ΠΈΡ‚ΠΈΠ΅ особСнностСй ΠΏΠΎΠ·Π½Π°Π²Π°Ρ‚Π΅Π»ΡŒΠ½Ρ‹Ρ… способностСй учащихся, ощущСния, восприятия, памяти, прСдставлСния, вообраТСния, ΠΌΡ‹ΡˆΠ»Π΅Π½ΠΈΡ, внимания, ΠΏΡ€Π΅Π΄ΠΏΠΎΠ»Π°Π³Π°Π΅Ρ‚ Π»ΠΈΡ‡Π½ΠΎΡΡ‚Π½ΡƒΡŽ ΠΎΡ€ΠΈΠ΅Π½Ρ‚Π°Ρ†ΠΈΡŽ, Π΄Π΅ΡΡ‚Π΅Π»ΡŒΠ½ΠΎΡΡ‚Π½Ρ‹ΠΉ ΠΈ Ρ€Π°Π·Π²ΠΈΠ²Π°ΡŽΡ‰ΠΈΠΉ Ρ…Π°Ρ€Π°ΠΊΡ‚Π΅Ρ€ содСрТания обучСния, способствуСт Ρ€Π°Π·Π²ΠΈΡ‚ΠΈΡŽ стрСмлСния ΠΈ способности ΠΊ ΡΠ°ΠΌΠΎΡΡ‚ΠΎΡΡ‚Π΅Π»ΡŒΠ½ΠΎΠΌΡƒ ΠΏΡ€ΠΈΠΎΠ±Ρ€Π΅Ρ‚Π΅Π½ΠΈΡŽ Π½ΠΎΠ²Ρ‹Ρ… Π·Π½Π°Π½ΠΈΠΉ.

    ЦСль Π΄Π°Π½Π½ΠΎΠ³ΠΎ курса – Π²ΠΎΠ²Π»Π΅Ρ‡Π΅Π½ΠΈΠ΅ учащихся Π² процСсс приобрСтСния ΠΈΠΌΠΈ матСматичСских Π·Π½Π°Π½ΠΈΠΉ, ΡƒΠΌΠ΅Π½ΠΈΠΉ ΠΈ матСматичСской ΠΊΡƒΠ»ΡŒΡ‚ΡƒΡ€Ρ‹.

    ΠŸΡ€ΠΎΠ³Ρ€Π°ΠΌΠΌΠ° Π΄Π°Π΅Ρ‚ Π²ΠΎΠ·ΠΌΠΎΠΆΠ½ΠΎΡΡ‚ΡŒ Π² соотвСтствии с ΡƒΡ‡Π΅Π±Π½Ρ‹ΠΌ ΠΏΠ»Π°Π½ΠΎΠΌ ΡƒΠ²Π΅Π»ΠΈΡ‡ΠΈΡ‚ΡŒ врСмя Π½Π° ΠΈΠ·ΡƒΡ‡Π΅Π½ΠΈΠ΅ ΠΎΡ‚Π΄Π΅Π»ΡŒΠ½Ρ‹Ρ… Ρ‚Π΅ΠΌ курса, позволяСт ΡƒΡ‚ΠΎΡ‡Π½ΠΈΡ‚ΡŒ ΡΠΏΠΎΡΠΎΠ±Π½ΠΎΡΡ‚ΡŒ ΠΈ Π³ΠΎΡ‚ΠΎΠ²Π½ΠΎΡΡ‚ΡŒ ΡƒΡ‡Π΅Π½ΠΈΠΊΠΎΠ² ΠΊ Π΄Π°Π»ΡŒΠ½Π΅ΠΉΡˆΠ΅ΠΌΡƒ ΠΏΠΎΠ²Ρ‹ΡˆΠ΅Π½ΠΈΡŽ своСго уровня развития ΠΈ Ρ€Π΅ΡˆΠ°Π΅Ρ‚ ΡΠ»Π΅Π΄ΡƒΡŽΡ‰ΠΈΠ΅ Π·Π°Π΄Π°Ρ‡ΠΈ:

    • Ρ€Π°Π·Π½ΠΎΠΎΠ±Ρ€Π°Π·ΠΈΡ‚ΡŒ процСсс обучСния;

    • ΡΡ„ΠΎΡ€ΠΌΠΈΡ€ΠΎΠ²Π°Ρ‚ΡŒ устойчивыС знания ΠΏΠΎ ΠΏΡ€Π΅Π΄ΠΌΠ΅Ρ‚Ρƒ;

    • Π²ΠΎΡΠΏΠΈΡ‚Ρ‹Π²Π°Ρ‚ΡŒ ΠΎΠ±Ρ‰ΡƒΡŽ ΠΌΠ°Ρ‚Π΅ΠΌΠ°Ρ‚ΠΈΡ‡Π΅ΡΠΊΡƒΡŽ ΠΊΡƒΠ»ΡŒΡ‚ΡƒΡ€Ρƒ;

    • Ρ€Π°Π·Π²ΠΈΠ²Π°Ρ‚ΡŒ матСматичСскоС (логичСскоС) ΠΌΡ‹ΡˆΠ»Π΅Π½ΠΈΠ΅;

    • Ρ€Π°ΡΡˆΠΈΡ€ΡΡ‚ΡŒ матСматичСский ΠΊΡ€ΡƒΠ³ΠΎΠ·ΠΎΡ€;

    • Ρ„ΠΎΡ€ΠΌΠΈΡ€ΠΎΠ²Π°Ρ‚ΡŒ ΡƒΠΌΠ΅Π½ΠΈΠ΅ Ρ€Π΅ΡˆΠ°Ρ‚ΡŒ ΠΊΠΎΠΌΠ±ΠΈΠ½Π°Ρ‚ΠΎΡ€Π½Ρ‹Π΅ ΠΈ логичСскиС Π·Π°Π΄Π°Ρ‡ΠΈ;

    • ΠΏΠΎΠ²Ρ‹ΡˆΠ°Ρ‚ΡŒ интСрСс ΠΊ ΠΏΡ€Π΅Π΄ΠΌΠ΅Ρ‚Ρƒ ΠΈ Π΅Π³ΠΎ ΠΈΠ·ΡƒΡ‡Π΅Π½ΠΈΡŽ;

    • Π²Ρ‹Ρ€Π°Π±ΠΎΡ‚Π°Ρ‚ΡŒ ΡΠ°ΠΌΠΎΡΡ‚ΠΎΡΡ‚Π΅Π»ΡŒΠ½Ρ‹ΠΉ ΠΈ творчСский ΠΏΠΎΠ΄Ρ…ΠΎΠ΄Ρ‹ ΠΊ ΠΈΠ·ΡƒΡ‡Π΅Π½ΠΈΡŽ ΠΌΠ°Ρ‚Π΅ΠΌΠ°Ρ‚ΠΈΠΊΠΈ.

    ΠžΠΆΠΈΠ΄Π°Π΅ΠΌΡ‹Π΅ Ρ€Π΅Π·ΡƒΠ»ΡŒΡ‚Π°Ρ‚Ρ‹:

    ЛичностныС Ρ€Π΅Π·ΡƒΠ»ΡŒΡ‚Π°Ρ‚Ρ‹ изучСния курса «Учимся Ρ€Π΅ΡˆΠ°Ρ‚ΡŒ Π·Π°Π΄Π°Ρ‡ΠΈΒ»

    Π£ ΡƒΡ‡Π΅Π½ΠΈΠΊΠ° Π±ΡƒΠ΄ΡƒΡ‚ сформированы:

    • внутрСнняя позиция школьника Π½Π° ΡƒΡ€ΠΎΠ²Π½Π΅ ΠΏΠΎΠ»ΠΎΠΆΠΈΡ‚Π΅Π»ΡŒΠ½ΠΎΠ³ΠΎ ΠΎΡ‚Π½ΠΎΡˆΠ΅Π½ΠΈΡ ΠΊ школС;

    • ΡƒΡ‡Π΅Π±Π½ΠΎ-ΠΏΠΎΠ·Π½Π°Π²Π°Ρ‚Π΅Π»ΡŒΠ½Ρ‹ΠΉ интСрСс ΠΊ Π½ΠΎΠ²ΠΎΠΌΡƒ ΠΌΠ°Ρ‚Π΅Ρ€ΠΈΠ°Π»Ρƒ ΠΈ способам Ρ€Π΅ΡˆΠ΅Π½ΠΈΡ Π½ΠΎΠ²ΠΎΠΉ ΡƒΡ‡Π΅Π±Π½ΠΎΠΉ Π·Π°Π΄Π°Ρ‡ΠΈ;

    • Π³ΠΎΡ‚ΠΎΠ²Π½ΠΎΡΡ‚ΡŒ Ρ†Π΅Π»Π΅Π½Π°ΠΏΡ€Π°Π²Π»Π΅Π½Π½ΠΎ ΠΈΡΠΏΠΎΠ»ΡŒΠ·ΠΎΠ²Π°Ρ‚ΡŒ матСматичСскиС знания, умСния ΠΈ Π½Π°Π²Ρ‹ΠΊΠΈ Π² ΡƒΡ‡Π΅Π±Π½ΠΎΠΉ Π΄Π΅ΡΡ‚Π΅Π»ΡŒΠ½ΠΎΡΡ‚ΠΈ ΠΈ Π² повсСднСвной ΠΆΠΈΠ·Π½ΠΈ;

    • ΡΠΏΠΎΡΠΎΠ±Π½ΠΎΡΡ‚ΡŒ ΠΎΡΠΎΠ·Π½Π°Π²Π°Ρ‚ΡŒ ΠΈ ΠΎΡ†Π΅Π½ΠΈΠ²Π°Ρ‚ΡŒ свои мысли, дСйствия ΠΈ Π²Ρ‹Ρ€Π°ΠΆΠ°Ρ‚ΡŒ ΠΈΡ… Π² Ρ€Π΅Ρ‡ΠΈ, ΡΠΎΠΎΡ‚Π½ΠΎΡΠΈΡ‚ΡŒ Ρ€Π΅Π·ΡƒΠ»ΡŒΡ‚Π°Ρ‚ дСйствия с поставлСнной Ρ†Π΅Π»ΡŒΡŽ;

    • ΡΠΏΠΎΡΠΎΠ±Π½ΠΎΡΡ‚ΡŒ ΠΊ ΠΎΡ€Π³Π°Π½ΠΈΠ·Π°Ρ†ΠΈΠΈ ΡΠ°ΠΌΠΎΡΡ‚ΠΎΡΡ‚Π΅Π»ΡŒΠ½ΠΎΠΉ ΡƒΡ‡Π΅Π±Π½ΠΎΠΉ Π΄Π΅ΡΡ‚Π΅Π»ΡŒΠ½ΠΎΡΡ‚ΠΈ.

    Π£ ΡƒΡ‡Π΅Π½ΠΈΠΊΠ° ΠΌΠΎΠ³ΡƒΡ‚ Π±Ρ‹Ρ‚ΡŒ сформированы:

    – Π²Π½ΡƒΡ‚Ρ€Π΅Π½Π½Π΅ΠΉ ΠΏΠΎΠ·ΠΈΡ†ΠΈΠΈ школьника Π½Π° ΡƒΡ€ΠΎΠ²Π½Π΅ понимания нСобходимости учСния, Π²Ρ‹Ρ€Π°ΠΆΠ΅Π½Π½ΠΎΠ³ΠΎ Π² ΠΏΡ€Π΅ΠΎΠ±Π»Π°Π΄Π°Π½ΠΈΠΈ ΡƒΡ‡Π΅Π±Π½ΠΎ-ΠΏΠΎΠ·Π½Π°Π²Π°Ρ‚Π΅Π»ΡŒΠ½Ρ‹Ρ… ΠΌΠΎΡ‚ΠΈΠ²ΠΎΠ²;

    – устойчивого ΠΏΠΎΠ·Π½Π°Π²Π°Ρ‚Π΅Π»ΡŒΠ½ΠΎΠ³ΠΎ интСрСса ΠΊ Π½ΠΎΠ²Ρ‹ΠΌ ΠΎΠ±Ρ‰ΠΈΠΌ способам Ρ€Π΅ΡˆΠ΅Π½ΠΈΡ Π·Π°Π΄Π°Ρ‡

    – Π°Π΄Π΅ΠΊΠ²Π°Ρ‚Π½ΠΎΠ³ΠΎ понимания ΠΏΡ€ΠΈΡ‡ΠΈΠ½ ΡƒΡΠΏΠ΅ΡˆΠ½ΠΎΡΡ‚ΠΈ ΠΈΠ»ΠΈ Π½Π΅ΡƒΡΠΏΠ΅ΡˆΠ½ΠΎΡΡ‚ΠΈ ΡƒΡ‡Π΅Π±Π½ΠΎΠΉ Π΄Π΅ΡΡ‚Π΅Π»ΡŒΠ½ΠΎΡΡ‚ΠΈ.

    ΠœΠ΅Ρ‚Π°ΠΏΡ€Π΅Π΄ΠΌΠ΅Ρ‚Π½Ρ‹Π΅ Ρ€Π΅Π·ΡƒΠ»ΡŒΡ‚Π°Ρ‚Ρ‹

    РСгулятивныС ΡƒΠ½ΠΈΠ²Π΅Ρ€ΡΠ°Π»ΡŒΠ½Ρ‹Π΅ ΡƒΡ‡Π΅Π±Π½Ρ‹Π΅ дСйствия

    Π£Ρ‡Π΅Π½ΠΈΠΊ научится:

    ΠΏΡ€ΠΈΠ½ΠΈΠΌΠ°Ρ‚ΡŒ ΠΈ ΡΠΎΡ…Ρ€Π°Π½ΡΡ‚ΡŒ ΡƒΡ‡Π΅Π±Π½ΡƒΡŽ Π·Π°Π΄Π°Ρ‡Ρƒ ΠΈ Π°ΠΊΡ‚ΠΈΠ²Π½ΠΎ Π²ΠΊΠ»ΡŽΡ‡Π°Ρ‚ΡŒΡΡ Π² Π΄Π΅ΡΡ‚Π΅Π»ΡŒΠ½ΠΎΡΡ‚ΡŒ, Π½Π°ΠΏΡ€Π°Π²Π»Π΅Π½Π½ΡƒΡŽ Π½Π° Π΅Ρ‘ Ρ€Π΅ΡˆΠ΅Π½ΠΈΠ΅ Π² сотрудничСствС с ΡƒΡ‡ΠΈΡ‚Π΅Π»Π΅ΠΌ ΠΈ одноклассниками;

    ΠΏΠ»Π°Π½ΠΈΡ€ΠΎΠ²Π°Ρ‚ΡŒ своС дСйствиС Π² соотвСтствии с поставлСнной Π·Π°Π΄Π°Ρ‡Π΅ΠΉ ΠΈ условиями Π΅Π΅ Ρ€Π΅Π°Π»ΠΈΠ·Π°Ρ†ΠΈΠΈ, Π² Ρ‚ΠΎΠΌ числС Π²ΠΎ Π²Π½ΡƒΡ‚Ρ€Π΅Π½Π½Π΅ΠΌ ΠΏΠ»Π°Π½Π΅;

    Ρ€Π°Π·Π»ΠΈΡ‡Π°Ρ‚ΡŒ способ ΠΈ Ρ€Π΅Π·ΡƒΠ»ΡŒΡ‚Π°Ρ‚ дСйствия; ΠΊΠΎΠ½Ρ‚Ρ€ΠΎΠ»ΠΈΡ€ΠΎΠ²Π°Ρ‚ΡŒ процСсс ΠΈ Ρ€Π΅Π·ΡƒΠ»ΡŒΡ‚Π°Ρ‚Ρ‹ Π΄Π΅ΡΡ‚Π΅Π»ΡŒΠ½ΠΎΡΡ‚ΠΈ;

    Π²Π½ΠΎΡΠΈΡ‚ΡŒ Π½Π΅ΠΎΠ±Ρ…ΠΎΠ΄ΠΈΠΌΡ‹Π΅ ΠΊΠΎΡ€Ρ€Π΅ΠΊΡ‚ΠΈΠ²Ρ‹ Π² дСйствиС послС Π΅Π³ΠΎ Π·Π°Π²Π΅Ρ€ΡˆΠ΅Π½ΠΈΡ, Π½Π° основС Π΅Π³ΠΎ ΠΎΡ†Π΅Π½ΠΊΠΈ ΠΈ ΡƒΡ‡Π΅Ρ‚Π° Ρ…Π°Ρ€Π°ΠΊΡ‚Π΅Ρ€Π° сдСланных ошибок;

    Π²Ρ‹ΠΏΠΎΠ»Π½ΡΡ‚ΡŒ ΡƒΡ‡Π΅Π±Π½Ρ‹Π΅ дСйствия Π² ΠΌΠ°Ρ‚Π΅Ρ€ΠΈΠ°Π»ΠΈΠ·ΠΎΠ²Π°Π½Π½ΠΎΠΉ, Π³Ρ€ΠΎΠΌΠΊΠΎΡ€Π΅Ρ‡Π΅Π²ΠΎΠΉ ΠΈ умствСнной Ρ„ΠΎΡ€ΠΌΠ΅;

    Π°Π΄Π΅ΠΊΠ²Π°Ρ‚Π½ΠΎ ΠΎΡ†Π΅Π½ΠΈΠ²Π°Ρ‚ΡŒ свои достиТСния, ΠΎΡΠΎΠ·Π½Π°Π²Π°Ρ‚ΡŒ Π²ΠΎΠ·Π½ΠΈΠΊΠ°ΡŽΡ‰ΠΈΠ΅ трудности ΠΈ ΠΈΡΠΊΠ°Ρ‚ΡŒ способы ΠΈΡ… прСодолСния

    Π£Ρ‡Π΅Π½ΠΈΠΊ ΠΏΠΎΠ»ΡƒΡ‡ΠΈΡ‚ Π²ΠΎΠ·ΠΌΠΎΠΆΠ½ΠΎΡΡ‚ΡŒ Π½Π°ΡƒΡ‡ΠΈΡ‚ΡŒΡΡ:

    β€’ Π² сотрудничСствС с ΡƒΡ‡ΠΈΡ‚Π΅Π»Π΅ΠΌ ΡΡ‚Π°Π²ΠΈΡ‚ΡŒ Π½ΠΎΠ²Ρ‹Π΅ ΡƒΡ‡Π΅Π±Π½Ρ‹Π΅ Π·Π°Π΄Π°Ρ‡ΠΈ;

    β€’ ΠΏΡ€ΠΎΡΠ²Π»ΡΡ‚ΡŒ ΠΏΠΎΠ·Π½Π°Π²Π°Ρ‚Π΅Π»ΡŒΠ½ΡƒΡŽ ΠΈΠ½ΠΈΡ†ΠΈΠ°Ρ‚ΠΈΠ²Ρƒ Π² ΡƒΡ‡Π΅Π±Π½ΠΎΠΌ сотрудничСствС;

    β€’ ΡΠ°ΠΌΠΎΡΡ‚ΠΎΡΡ‚Π΅Π»ΡŒΠ½ΠΎ ΡƒΡ‡ΠΈΡ‚Ρ‹Π²Π°Ρ‚ΡŒ Π²Ρ‹Π΄Π΅Π»Π΅Π½Π½Ρ‹Π΅ ΡƒΡ‡ΠΈΡ‚Π΅Π»Π΅ΠΌ ΠΎΡ€ΠΈΠ΅Π½Ρ‚ΠΈΡ€Ρ‹ дСйствия Π² Π½ΠΎΠ²ΠΎΠΌ ΡƒΡ‡Π΅Π±Π½ΠΎΠΌ ΠΌΠ°Ρ‚Π΅Ρ€ΠΈΠ°Π»Π΅;

    β€’ ΠΎΡΡƒΡ‰Π΅ΡΡ‚Π²Π»ΡΡ‚ΡŒ ΠΊΠΎΠ½ΡΡ‚Π°Ρ‚ΠΈΡ€ΡƒΡŽΡ‰ΠΈΠΉ ΠΈ ΠΏΡ€Π΅Π΄Π²ΠΎΡΡ…ΠΈΡ‰Π°ΡŽΡ‰ΠΈΠΉ ΠΊΠΎΠ½Ρ‚Ρ€ΠΎΠ»ΡŒ ΠΏΠΎ Ρ€Π΅Π·ΡƒΠ»ΡŒΡ‚Π°Ρ‚Ρƒ ΠΈ ΠΏΠΎ способу дСйствия, Π°ΠΊΡ‚ΡƒΠ°Π»ΡŒΠ½Ρ‹ΠΉ ΠΊΠΎΠ½Ρ‚Ρ€ΠΎΠ»ΡŒ Π½Π° ΡƒΡ€ΠΎΠ²Π½Π΅ ΠΏΡ€ΠΎΠΈΠ·Π²ΠΎΠ»ΡŒΠ½ΠΎΠ³ΠΎ внимания;

    β€’ ΡΠ°ΠΌΠΎΡΡ‚ΠΎΡΡ‚Π΅Π»ΡŒΠ½ΠΎ Π°Π΄Π΅ΠΊΠ²Π°Ρ‚Π½ΠΎ ΠΎΡ†Π΅Π½ΠΈΠ²Π°Ρ‚ΡŒ ΠΏΡ€Π°Π²ΠΈΠ»ΡŒΠ½ΠΎΡΡ‚ΡŒ выполнСния дСйствия ΠΈ Π²Π½ΠΎΡΠΈΡ‚ΡŒ Π½Π΅ΠΎΠ±Ρ…ΠΎΠ΄ΠΈΠΌΡ‹Π΅ ΠΊΠΎΡ€Ρ€Π΅ΠΊΡ‚ΠΈΠ²Ρ‹ Π² исполнСниС ΠΊΠ°ΠΊ ΠΏΠΎ Ρ…ΠΎΠ΄Ρƒ Π΅Π³ΠΎ Ρ€Π΅Π°Π»ΠΈΠ·Π°Ρ†ΠΈΠΈ, Ρ‚Π°ΠΊ ΠΈ Π² ΠΊΠΎΠ½Ρ†Π΅ дСйствия.

    ΠŸΠΎΠ·Π½Π°Π²Π°Ρ‚Π΅Π»ΡŒΠ½Ρ‹Π΅ ΡƒΠ½ΠΈΠ²Π΅Ρ€ΡΠ°Π»ΡŒΠ½Ρ‹Π΅ ΡƒΡ‡Π΅Π±Π½Ρ‹Π΅ дСйствия

    Π£Ρ‡Π΅Π½ΠΈΠΊ научится:

    – ΠΎΡΡƒΡ‰Π΅ΡΡ‚Π²Π»ΡΡ‚ΡŒ поиск Π½Π΅ΠΎΠ±Ρ…ΠΎΠ΄ΠΈΠΌΠΎΠΉ ΠΈΠ½Ρ„ΠΎΡ€ΠΌΠ°Ρ†ΠΈΠΈ для выполнСния ΡƒΡ‡Π΅Π±Π½Ρ‹Ρ… Π·Π°Π΄Π°Π½ΠΈΠΉ с использованиСм ΡƒΡ‡Π΅Π±Π½ΠΎΠΉ Π»ΠΈΡ‚Π΅Ρ€Π°Ρ‚ΡƒΡ€Ρ‹;

    – ΠΈΡΠΏΠΎΠ»ΡŒΠ·ΠΎΠ²Π°Ρ‚ΡŒ Π·Π½Π°ΠΊΠΎΠ²ΠΎ-символичСскиС срСдства, Π² Ρ‚ΠΎΠΌ числС ΠΌΠΎΠ΄Π΅Π»ΠΈ ΠΈ схСмы для Ρ€Π΅ΡˆΠ΅Π½ΠΈΡ Π·Π°Π΄Π°Ρ‡;

    – ΠΎΡΡƒΡ‰Π΅ΡΡ‚Π²Π»ΡΡ‚ΡŒ синтСз ΠΊΠ°ΠΊ составлСниС Ρ†Π΅Π»ΠΎΠ³ΠΎ ΠΈΠ· частСй;

    – ΠΏΡ€ΠΎΠ²ΠΎΠ΄ΠΈΡ‚ΡŒ сравнСниС ΠΈ ΠΊΠ»Π°ΡΡΠΈΡ„ΠΈΠΊΠ°Ρ†ΠΈΡŽ ΠΏΠΎ Π·Π°Π΄Π°Π½Π½Ρ‹ΠΌ критСриям;

    – ΡƒΡΡ‚Π°Π½Π°Π²Π»ΠΈΠ²Π°Ρ‚ΡŒ ΠΏΡ€ΠΈΡ‡ΠΈΠ½Π½ΠΎ-слСдствСнныС связи;

    – ΡΡ‚Ρ€ΠΎΠΈΡ‚ΡŒ рассуТдСния Π² Ρ„ΠΎΡ€ΠΌΠ΅ связи простых суТдСний ΠΎΠ± ΠΎΠ±ΡŠΠ΅ΠΊΡ‚Π΅, Π΅Π³ΠΎ строСнии, свойствах ΠΈ связях;

    – ΠΎΠ±ΠΎΠ±Ρ‰Π°Ρ‚ΡŒ, Ρ‚.Π΅. ΠΎΡΡƒΡ‰Π΅ΡΡ‚Π²Π»ΡΡ‚ΡŒ Π³Π΅Π½Π΅Ρ€Π°Π»ΠΈΠ·Π°Ρ†ΠΈΡŽ ΠΈ Π²Ρ‹Π²Π΅Π΄Π΅Π½ΠΈΠ΅ общности для Ρ†Π΅Π»ΠΎΠ³ΠΎ ряда ΠΈΠ»ΠΈ класса Π΅Π΄ΠΈΠ½ΠΈΡ‡Π½Ρ‹Ρ… ΠΎΠ±ΡŠΠ΅ΠΊΡ‚ΠΎΠ² Π½Π° основС выдСлСния сущностной связи;

    – ΠΎΡΡƒΡ‰Π΅ΡΡ‚Π²Π»ΡΡ‚ΡŒ ΠΏΠΎΠ΄Π²Π΅Π΄Π΅Π½ΠΈΠ΅ ΠΏΠΎΠ΄ понятиС Π½Π° основС распознавания ΠΎΠ±ΡŠΠ΅ΠΊΡ‚ΠΎΠ², выдСлСния сущСствСнных ΠΏΡ€ΠΈΠ·Π½Π°ΠΊΠΎΠ² ΠΈ ΠΈΡ… синтСза;

    – ΡƒΡΡ‚Π°Π½Π°Π²Π»ΠΈΠ²Π°Ρ‚ΡŒ Π°Π½Π°Π»ΠΎΠ³ΠΈΠΈ;

    Π£Ρ‡Π΅Π½ΠΈΠΊ ΠΏΠΎΠ»ΡƒΡ‡ΠΈΡ‚ Π²ΠΎΠ·ΠΌΠΎΠΆΠ½ΠΎΡΡ‚ΡŒ Π½Π°ΡƒΡ‡ΠΈΡ‚ΡŒΡΡ:

    – ΠΎΡΡƒΡ‰Π΅ΡΡ‚Π²Π»ΡΡ‚ΡŒ синтСз ΠΊΠ°ΠΊ составлСниС Ρ†Π΅Π»ΠΎΠ³ΠΎ ΠΈΠ· частСй, ΡΠ°ΠΌΠΎΡΡ‚ΠΎΡΡ‚Π΅Π»ΡŒΠ½ΠΎ достраивая ΠΈ восполняя Π½Π΅Π΄ΠΎΡΡ‚Π°ΡŽΡ‰ΠΈΠ΅ ΠΊΠΎΠΌΠΏΠΎΠ½Π΅Π½Ρ‚Ρ‹

    – ΠΎΡΡƒΡ‰Π΅ΡΡ‚Π²Π»ΡΡ‚ΡŒ сравнСниС ΠΈ ΠΊΠ»Π°ΡΡΠΈΡ„ΠΈΠΊΠ°Ρ†ΠΈΡŽ, ΡΠ°ΠΌΠΎΡΡ‚ΠΎΡΡ‚Π΅Π»ΡŒΠ½ΠΎ выбирая основания ΠΈ ΠΊΡ€ΠΈΡ‚Π΅Ρ€ΠΈΠΈ для ΡƒΠΊΠ°Π·Π°Π½Π½Ρ‹Ρ… логичСских ΠΎΠΏΠ΅Ρ€Π°Ρ†ΠΈΠΉ;

    – ΡΡ‚Ρ€ΠΎΠΈΡ‚ΡŒ логичСскоС рассуТдСниС, Π²ΠΊΠ»ΡŽΡ‡Π°ΡŽΡ‰Π΅Π΅ установлСниС ΠΏΡ€ΠΈΡ‡ΠΈΠ½Π½ΠΎ-слСдствСнных связСй.

    ΠšΠΎΠΌΠΌΡƒΠ½ΠΈΠΊΠ°Ρ‚ΠΈΠ²Π½Ρ‹Π΅ ΡƒΠ½ΠΈΠ²Π΅Ρ€ΡΠ°Π»ΡŒΠ½Ρ‹Π΅ ΡƒΡ‡Π΅Π±Π½Ρ‹Π΅ дСйствия

    Π£Ρ‡Π΅Π½ΠΈΠΊ научится:

    – Π²Ρ‹Ρ€Π°ΠΆΠ°Ρ‚ΡŒ Π² Ρ€Π΅Ρ‡ΠΈ свои мысли ΠΈ дСйствия;

    – ΡΡ‚Ρ€ΠΎΠΈΡ‚ΡŒ понятныС для ΠΏΠ°Ρ€Ρ‚Π½Π΅Ρ€Π° высказывания, ΡƒΡ‡ΠΈΡ‚Ρ‹Π²Π°ΡŽΡ‰ΠΈΠ΅, Ρ‡Ρ‚ΠΎ ΠΏΠ°Ρ€Ρ‚Π½Π΅Ρ€ Π²ΠΈΠ΄ΠΈΡ‚ ΠΈ Π·Π½Π°Π΅Ρ‚, Π° Ρ‡Ρ‚ΠΎ Π½Π΅Ρ‚;

    – Π·Π°Π΄Π°Π²Π°Ρ‚ΡŒ вопросы;

    – ΠΈΡΠΏΠΎΠ»ΡŒΠ·ΠΎΠ²Π°Ρ‚ΡŒ Ρ€Π΅Ρ‡ΡŒ для рСгуляции своСго дСйствия.

    Π£Ρ‡Π΅Π½ΠΈΠΊ ΠΏΠΎΠ»ΡƒΡ‡ΠΈΡ‚ Π²ΠΎΠ·ΠΌΠΎΠΆΠ½ΠΎΡΡ‚ΡŒ Π½Π°ΡƒΡ‡ΠΈΡ‚ΡŒΡΡ:- Π°Π΄Π΅ΠΊΠ²Π°Ρ‚Π½ΠΎ ΠΈΡΠΏΠΎΠ»ΡŒΠ·ΠΎΠ²Π°Ρ‚ΡŒ Ρ€Π΅Ρ‡ΡŒ для планирования ΠΈ рСгуляции своСго дСйствия;- Π°Ρ€Π³ΡƒΠΌΠ΅Π½Ρ‚ΠΈΡ€ΠΎΠ²Π°Ρ‚ΡŒ свою ΠΏΠΎΠ·ΠΈΡ†ΠΈΡŽ ΠΈ ΠΊΠΎΠΎΡ€Π΄ΠΈΠ½ΠΈΡ€ΠΎΠ²Π°Ρ‚ΡŒ Π΅Ρ‘ с позициями ΠΏΠ°Ρ€Ρ‚Π½Π΅Ρ€ΠΎΠ² Π² совмСстной Π΄Π΅ΡΡ‚Π΅Π»ΡŒΠ½ΠΎΡΡ‚ΠΈ;

    – ΠΎΡΡƒΡ‰Π΅ΡΡ‚Π²Π»ΡΡ‚ΡŒ Π²Π·Π°ΠΈΠΌΠ½Ρ‹ΠΉ ΠΊΠΎΠ½Ρ‚Ρ€ΠΎΠ»ΡŒ ΠΈ ΠΎΠΊΠ°Π·Ρ‹Π²Π°Ρ‚ΡŒ Π² сотрудничСствС Π½Π΅ΠΎΠ±Ρ…ΠΎΠ΄ΠΈΠΌΡƒΡŽ ΠΏΠΎΠΌΠΎΡ‰ΡŒ.

    Π ΠΠ‘Π‘ΠœΠžΠ’Π Π•ΠΠ Π½Π° засСдании МО

    ΡƒΡ‡ΠΈΡ‚Π΅Π»Π΅ΠΉ Π½Π°Ρ‡Π°Π»ΡŒΠ½Ρ‹Ρ… классов

    И Π Π•ΠšΠžΠœΠ•ΠΠ”ΠžΠ’ΠΠΠ

    ΠΊ ΡƒΡ‚Π²Π΅Ρ€ΠΆΠ΄Π΅Π½ΠΈΡŽ

    ΠΏΡ€ΠΎΡ‚ΠΎΠΊΠΎΠ» β„–1

    ΠΎΡ‚ Β«___Β» _____________2014 Π³. Π ΡƒΠΊΠΎΠ²ΠΎΠ΄ΠΈΡ‚Π΅Π»ΡŒ МО _________Π›.И. Π›ΠΈΡ‚ΠΎΠ²Ρ‡Π΅Π½ΠΊΠΎ
    Π‘ΠžΠ“Π›ΠΠ‘ΠžΠ’ΠΠΠ Π—Π°ΠΌ.Π΄ΠΈΡ€Π΅ΠΊΡ‚ΠΎΡ€Π° ΠΏΠΎ Π£Π’Π  _____________ О.Π’. Π”ΡƒΠ±ΠΎΠ²ΠΈΠΊ Β«___»сСнтября 2014 Π³.

    ΠšΠ°Π»Π΅Π½Π΄Π°Ρ€Π½ΠΎ-тСматичСскоС ΠΏΠ»Π°Π½ΠΈΡ€ΠΎΠ²Π°Π½ΠΈΠ΅ ΠΏΠΎ элСктивному курсу Β« Учимся Ρ€Π΅ΡˆΠ°Ρ‚ΡŒ Π·Π°Π΄Π°Ρ‡ΠΈΒ» 3 класс

    β„–

    ΠΏ/ΠΏ

    НаимСнованиС Ρ€Π°Π·Π΄Π΅Π»ΠΎΠ²

    ΠΈ Ρ‚Π΅ΠΌ ΠΏΡ€ΠΎΠ³Ρ€Π°ΠΌΠΌΡ‹

    Кол-во

    часов

    Π’ΠΈΠ΄Ρ‹ Π΄Π΅ΡΡ‚Π΅Π»ΡŒΠ½ΠΎΡΡ‚ΠΈ

    учащихся

    ΠžΠ±ΠΎΡ€ΡƒΠ΄ΠΎΠ²Π°Π½ΠΈΠ΅, ΠΊΠΎΠ½Ρ‚Ρ€ΠΎΠ»ΡŒΠ½ΠΎ-ΠΈΠ·ΠΌΠ΅Ρ€ΠΈΡ‚Π΅Π»ΡŒΠ½Ρ‹Π΅ ΠΌΠ°Ρ‚Π΅Ρ€ΠΈΠ°Π»Ρ‹

    ΠŸΡ€ΠΈΠΌΠ΅Ρ‡Π°Π½ΠΈΡ

    ΠšΠ°Π»Π΅Π½Π΄Π°Ρ€Π½Ρ‹Π΅ сроки

    ΠΏΠ»Π°Π½ΠΈΡ€ΡƒΠ΅ΠΌΡ‹Π΅

    фактичСски

    НахоТдСниС суммы.

    1

    РСшСниС Π·Π°Π΄Π°Ρ‡.

    Π’Π΅Ρ‚Ρ€Π°Π΄ΡŒ, ΠΈΠ½Ρ‚Π΅Ρ€Π½Π΅Ρ‚-рСсурсы

    6.09

    НахоТдСниС разности

    1

    РСшСниС Π·Π°Π΄Π°Ρ‡.

    Инд.ΠΊΠ°Ρ€Ρ‚ΠΎΡ‡ΠΊΠΈ, Ρ‚Π΅Ρ‚Ρ€Π°Π΄ΡŒ

    13.09

    Π£Π²Π΅Π»ΠΈΡ‡Π΅Π½ΠΈΠ΅ Π½Π° нСсколько Π΅Π΄ΠΈΠ½ΠΈΡ†

    1

    РСшСниС Π·Π°Π΄Π°Ρ‡.

    Инд.ΠΊΠ°Ρ€Ρ‚ΠΎΡ‡ΠΊΠΈ, Ρ‚Π΅Ρ‚Ρ€Π°Π΄ΡŒ

    20.09

    УмСньшСниС Π½Π° нСсколько Π΅Π΄ΠΈΠ½ΠΈΡ†

    1

    РСшСниС Π·Π°Π΄Π°Ρ‡.

    Инд.ΠΊΠ°Ρ€Ρ‚ΠΎΡ‡ΠΊΠΈ, Ρ‚Π΅Ρ‚Ρ€Π°Π΄ΡŒ

    27.09

    БоставныС Π·Π°Π΄Π°Ρ‡ΠΈ.

    1

    РСшСниС Π·Π°Π΄Π°Ρ‡.

    Инд.ΠΊΠ°Ρ€Ρ‚ΠΎΡ‡ΠΊΠΈ, Ρ‚Π΅Ρ‚Ρ€Π°Π΄ΡŒ

    4.10

    НахоТдСниС нСизвСстного слагаСмого

    1

    РСшСниС Π·Π°Π΄Π°Ρ‡.

    Инд.ΠΊΠ°Ρ€Ρ‚ΠΎΡ‡ΠΊΠΈ, Ρ‚Π΅Ρ‚Ρ€Π°Π΄ΡŒ

    11.10

    На разностноС сравнСниС

    1

    РСшСниС Π·Π°Π΄Π°Ρ‡.

    Π’Π΅Ρ‚Ρ€Π°Π΄ΡŒ, ΠΈΠ½Ρ‚Π΅Ρ€Π½Π΅Ρ‚-рСсурсы

    18.10

    На разностноС сравнСниС

    1

    РСшСниС Π·Π°Π΄Π°Ρ‡.

    Инд.ΠΊΠ°Ρ€Ρ‚ΠΎΡ‡ΠΊΠΈ, Ρ‚Π΅Ρ‚Ρ€Π°Π΄ΡŒ

    25.10

    НахоТдСниС нСизвСстного ΡƒΠΌΠ΅Π½ΡŒΡˆΠ°Π΅ΠΌΠΎΠ³ΠΎ

    1

    РСшСниС Π·Π°Π΄Π°Ρ‡.

    Инд.ΠΊΠ°Ρ€Ρ‚ΠΎΡ‡ΠΊΠΈ, Ρ‚Π΅Ρ‚Ρ€Π°Π΄ΡŒ

    1.11

    НахоТдСниС нСизвСстного ΡƒΠΌΠ΅Π½ΡŒΡˆΠ°Π΅ΠΌΠΎΠ³ΠΎ

    1

    РСшСниС Π·Π°Π΄Π°Ρ‡.

    Π£Ρ‡Π΅Π±Π½ΠΈΠΊ, рабочая Ρ‚Π΅Ρ‚Ρ€Π°Π΄ΡŒ, Π˜Π½Ρ‚Π΅Ρ€Π½Π΅Ρ‚-рСсурсы

    15.11

    НахоТдСниС нСизвСстного Π²Ρ‹Ρ‡ΠΈΡ‚Π°Π΅ΠΌΠΎΠ³ΠΎ

    1

    РСшСниС Π·Π°Π΄Π°Ρ‡.

    Π£Ρ‡Π΅Π±Π½ΠΈΠΊ, рабочая Ρ‚Π΅Ρ‚Ρ€Π°Π΄ΡŒ, Π˜Π½Ρ‚Π΅Ρ€Π½Π΅Ρ‚-рСсурсы

    22.11

    НахоТдСниС нСизвСстного Π²Ρ‹Ρ‡ΠΈΡ‚Π°Π΅ΠΌΠΎΠ³ΠΎ

    1

    РСшСниС Π·Π°Π΄Π°Ρ‡.

    Инд.ΠΊΠ°Ρ€Ρ‚ΠΎΡ‡ΠΊΠΈ, Ρ‚Π΅Ρ‚Ρ€Π°Π΄ΡŒ

    29.11

    БоставныС Π·Π°Π΄Π°Ρ‡ΠΈ.

    1

    РСшСниС Π·Π°Π΄Π°Ρ‡.

    Инд.ΠΊΠ°Ρ€Ρ‚ΠΎΡ‡ΠΊΠΈ, Ρ‚Π΅Ρ‚Ρ€Π°Π΄ΡŒ

    6.12

    БоставныС Π·Π°Π΄Π°Ρ‡ΠΈ. ВСматичСский Π·Π°Ρ‡Ρ‘Ρ‚β„–1

    1

    Π‘Π°ΠΌΠΎΡΡ‚ΠΎΡΡ‚Π΅Π»ΡŒΠ½Π°Ρ Ρ€Π°Π±ΠΎΡ‚Π° ΠΏΠΎ ΠΈΠ½Π΄ΠΈΠ²ΠΈΠ΄ΡƒΠ°Π»ΡŒΠ½Ρ‹ΠΌ ΠΊΠ°Ρ€Ρ‚ΠΎΡ‡ΠΊΠ°ΠΌ

    Инд.ΠΊΠ°Ρ€Ρ‚ΠΎΡ‡ΠΊΠΈ, Ρ‚Π΅Ρ‚Ρ€Π°Π΄ΡŒ

    13.12

    Π£Π²Π΅Π»ΠΈΡ‡Π΅Π½ΠΈΠ΅ Π² нСсколько Ρ€Π°Π·

    1

    РСшСниС Π·Π°Π΄Π°Ρ‡.

    Π’Π΅Ρ‚Ρ€Π°Π΄ΡŒ, ΠΈΠ½Ρ‚Π΅Ρ€Π½Π΅Ρ‚-рСсурсы

    20.12

    Π£Π²Π΅Π»ΠΈΡ‡Π΅Π½ΠΈΠ΅ Π² нСсколько Ρ€Π°Π·

    1

    РСшСниС Π·Π°Π΄Π°Ρ‡.

    27.12

    УмСньшСниС Π² нСсколько Ρ€Π°Π·

    1

    РСшСниС Π·Π°Π΄Π°Ρ‡.

    Π’Π΅Ρ‚Ρ€Π°Π΄ΡŒ, ΠΈΠ½Ρ‚Π΅Ρ€Π½Π΅Ρ‚-рСсурсы

    17.01

    УмСньшСниС Π² нСсколько Ρ€Π°Π·

    1

    РСшСниС Π·Π°Π΄Π°Ρ‡.

    Инд.ΠΊΠ°Ρ€Ρ‚ΠΎΡ‡ΠΊΠΈ, Ρ‚Π΅Ρ‚Ρ€Π°Π΄ΡŒ

    24.01

    НахоТдСниС произвСдСния

    1

    РСшСниС Π·Π°Π΄Π°Ρ‡.

    Π’Π΅Ρ‚Ρ€Π°Π΄ΡŒ, ΠΈΠ½Ρ‚Π΅Ρ€Π½Π΅Ρ‚-рСсурсы

    31.01

    НахоТдСниС произвСдСния

    1

    РСшСниС Π·Π°Π΄Π°Ρ‡.

    Инд.ΠΊΠ°Ρ€Ρ‚ΠΎΡ‡ΠΊΠΈ, Ρ‚Π΅Ρ‚Ρ€Π°Π΄ΡŒ

    7.02

    НахоТдСниС частного

    1

    РСшСниС Π·Π°Π΄Π°Ρ‡.

    Инд.ΠΊΠ°Ρ€Ρ‚ΠΎΡ‡ΠΊΠΈ, Ρ‚Π΅Ρ‚Ρ€Π°Π΄ΡŒ

    21.02

    НахоТдСниС частного

    1

    РСшСниС Π·Π°Π΄Π°Ρ‡.

    Инд.ΠΊΠ°Ρ€Ρ‚ΠΎΡ‡ΠΊΠΈ, Ρ‚Π΅Ρ‚Ρ€Π°Π΄ΡŒ

    28.02

    НахоТдСниС нСизвСстного мноТитСля

    1

    РСшСниС Π·Π°Π΄Π°Ρ‡.

    Π’Π΅Ρ‚Ρ€Π°Π΄ΡŒ, ΠΈΠ½Ρ‚Π΅Ρ€Π½Π΅Ρ‚-рСсурсы

    7.03

    НахоТдСниС нСизвСстного мноТитСля

    1

    РСшСниС Π·Π°Π΄Π°Ρ‡.

    Инд.ΠΊΠ°Ρ€Ρ‚ΠΎΡ‡ΠΊΠΈ, Ρ‚Π΅Ρ‚Ρ€Π°Π΄ΡŒ

    14.03

    БоставныС Π·Π°Π΄Π°Ρ‡ΠΈ.

    1

    РСшСниС Π·Π°Π΄Π°Ρ‡.

    Π’Π΅Ρ‚Ρ€Π°Π΄ΡŒ, ΠΈΠ½Ρ‚Π΅Ρ€Π½Π΅Ρ‚-рСсурсы

    21.03

    БоставныС Π·Π°Π΄Π°Ρ‡ΠΈ. ВСматичСский Π·Π°Ρ‡Ρ‘Ρ‚β„–2

    1

    Π‘Π°ΠΌΠΎΡΡ‚ΠΎΡΡ‚Π΅Π»ΡŒΠ½Π°Ρ Ρ€Π°Π±ΠΎΡ‚Π° ΠΏΠΎ ΠΈΠ½Π΄ΠΈΠ²ΠΈΠ΄ΡƒΠ°Π»ΡŒΠ½Ρ‹ΠΌ ΠΊΠ°Ρ€Ρ‚ΠΎΡ‡ΠΊΠ°ΠΌ

    Инд.ΠΊΠ°Ρ€Ρ‚ΠΎΡ‡ΠΊΠΈ, Ρ‚Π΅Ρ‚Ρ€Π°Π΄ΡŒ

    4.04

    НахоТдСниС нСизвСстного Π΄Π΅Π»ΠΈΠΌΠΎΠ³ΠΎ

    РСшСниС Π·Π°Π΄Π°Ρ‡.

    Π’Π΅Ρ‚Ρ€Π°Π΄ΡŒ, ΠΈΠ½Ρ‚Π΅Ρ€Π½Π΅Ρ‚-рСсурсы

    11.04

    НахоТдСниС нСизвСстного Π΄Π΅Π»ΠΈΠΌΠΎΠ³ΠΎ

    1

    РСшСниС Π·Π°Π΄Π°Ρ‡.

    Π˜Π½Ρ‚Π΅Ρ€Π½Π΅Ρ‚-рСсурсы, Ρ‚Π΅Ρ‚Ρ€Π°Π΄ΡŒ

    18.04

    . НахоТдСниС нСизвСстного дСлитСля

    1

    РСшСниС Π·Π°Π΄Π°Ρ‡.

    Индивид.ΠΊΠ°Ρ€Ρ‚ΠΎΡ‡ΠΊΠΈ, Ρ‚Π΅Ρ‚Ρ€Π°Π΄ΡŒ

    25.04

    НахоТдСниС нСизвСстного дСлитСля

    1

    РСшСниС Π·Π°Π΄Π°Ρ‡.

    Индивид.ΠΊΠ°Ρ€Ρ‚ΠΎΡ‡ΠΊΠΈ, Ρ‚Π΅Ρ‚Ρ€Π°Π΄ΡŒ

    2.05

    БоставныС Π·Π°Π΄Π°Ρ‡ΠΈ.

    1

    РСшСниС Π·Π°Π΄Π°Ρ‡.

    Π˜Π½Ρ‚Π΅Ρ€Π½Π΅Ρ‚-рСсурсы,

    ΠΏΡ€Π΅Π·Π΅Π½Ρ‚Π°Ρ†ΠΈΠΈ учащихся

    16.05

    БоставныС Π·Π°Π΄Π°Ρ‡ΠΈ.

    1

    РСшСниС Π·Π°Π΄Π°Ρ‡.

    Π˜Π½Ρ‚Π΅Ρ€Π½Π΅Ρ‚-рСсурсы,

    23.05

    БоставныС Π·Π°Π΄Π°Ρ‡ΠΈ.

    1

    РСшСниС Π·Π°Π΄Π°Ρ‡.

    Π’Π΅Ρ‚Ρ€Π°Π΄ΡŒ, ΠΈΠ½Ρ‚Π΅Ρ€Π½Π΅Ρ‚-рСсурсы

    29.05

    Π˜Ρ‚ΠΎΠ³ΠΎΠ²ΠΎΠ΅ занятиС. ВСматичСский Π·Π°Ρ‡Ρ‘Ρ‚β„–3

    1

    Π‘Π°ΠΌΠΎΡΡ‚ΠΎΡΡ‚Π΅Π»ΡŒΠ½Π°Ρ Ρ€Π°Π±ΠΎΡ‚Π° ΠΏΠΎ ΠΈΠ½Π΄ΠΈΠ²ΠΈΠ΄ΡƒΠ°Π»ΡŒΠ½Ρ‹ΠΌ ΠΊΠ°Ρ€Ρ‚ΠΎΡ‡ΠΊΠ°ΠΌ

    Индивид.ΠΊΠ°Ρ€Ρ‚ΠΎΡ‡ΠΊΠΈ.

    30.05

    Π˜Ρ‚ΠΎΠ³ΠΎ: 34 часа

    Π Π•Π¦Π•ΠΠ—Π˜Π―

    Π½Π° ΠΏΡ€ΠΎΠ³Ρ€Π°ΠΌΠΌΡƒ элСктивного курса

    «Учимся Ρ€Π΅ΡˆΠ°Ρ‚ΡŒ Π·Π°Π΄Π°Ρ‡ΠΈΒ»

    учитСля Π½Π°Ρ‡Π°Π»ΡŒΠ½Ρ‹Ρ… классов

    МКОУ «БрСдняя ΠΎΠ±Ρ‰Π΅ΠΎΠ±Ρ€Π°Π·ΠΎΠ²Π°Ρ‚Π΅Π»ΡŒΠ½Π°Ρ школа β„– 4Β» с. ΠœΠ°Π»Ρ‹Π΅ Π―Π³ΡƒΡ€Ρ‹

    Ломонос Ольги Π‘Π΅Ρ€Π³Π΅Π΅Π²Π½Ρ‹

    Данная ΠΏΡ€ΠΎΠ³Ρ€Π°ΠΌΠΌΠ°Β Β Ρ€Π°Π·Ρ€Π°Π±ΠΎΡ‚Π°Π½Π° Π² соотвСтствии с Π½ΠΎΡ€ΠΌΠ°Ρ‚ΠΈΠ²Π½Ρ‹ΠΌΠΈ Π΄ΠΎΠΊΡƒΠΌΠ΅Π½Ρ‚Π°ΠΌΠΈ Ρ„Π΅Π΄Π΅Ρ€Π°Π»ΡŒΠ½ΠΎΠ³ΠΎ ΠΈ Ρ€Π΅Π³ΠΈΠΎΠ½Π°Π»ΡŒΠ½ΠΎΠ³ΠΎ уровня.

    Β ΠŸΡ€ΠΈΠΊΠ°Π· ΠœΠΈΠ½ΠΈΡΡ‚Π΅Ρ€ΡΡ‚Π²Π° образования ΠΈ Π½Π°ΡƒΠΊΠΈ Российской Π€Π΅Π΄Π΅Ρ€Π°Ρ†ΠΈΠΈ “Об ΡƒΡ‚Π²Π΅Ρ€ΠΆΠ΄Π΅Π½ΠΈΠΈ ΠΈ Π²Π²Π΅Π΄Π΅Π½ΠΈΠΈ Π² дСйствиС Ρ„Π΅Π΄Π΅Ρ€Π°Π»ΡŒΠ½ΠΎΠ³ΠΎ государствСнного ΠΎΠ±Ρ€Π°Π·ΠΎΠ²Π°Ρ‚Π΅Π»ΡŒΠ½ΠΎΠ³ΠΎ стандарта Π½Π°Ρ‡Π°Π»ΡŒΠ½ΠΎΠ³ΠΎ ΠΎΠ±Ρ‰Π΅Π³ΠΎ образования” ΠΎΡ‚ 06.10.2009, β„– 373.

    На основС Β«ΠšΠΎΠ½Ρ†Π΅ΠΏΡ†ΠΈΠΈ Π΄ΡƒΡ…ΠΎΠ²Π½ΠΎ-нравствСнного развития ΠΈ воспитания личности Π³Ρ€Π°ΠΆΠ΄Π°Π½ΠΈΠ½Π° России: Москва Β«ΠŸΡ€ΠΎΡΠ²Π΅Ρ‰Π΅Π½ΠΈΠ΅Β»2009Π³ΠΎΠ΄Π°, Π² объСмС 34 ΡƒΡ‡Π΅Π±Π½Ρ‹Ρ… занятия.

    ΠΠΊΡ‚ΡƒΠ°Π»ΡŒΠ½ΠΎΡΡ‚ΡŒ Ρ€Π°Π±ΠΎΡ‚Ρ‹ обусловлСна Ρ€Π΅Ρ„ΠΎΡ€ΠΌΠ°ΠΌΠΈ Π² соврСмСнном ΠΎΠ±Ρ€Π°Π·ΠΎΠ²Π°Π½ΠΈΠΈ, Π½Π°ΠΏΡ€Π°Π²Π»Π΅Π½Π½ΠΎΡΡ‚ΡŒΡŽ ΡˆΠΊΠΎΠ»Ρ‹ Π½Π° Ρ€Π°Π·Π²ΠΈΡ‚ΠΈΠ΅ способностСй личности Ρ€Π΅Π±Π΅Π½ΠΊΠ°.

    Автором ΡΠ΅Ρ€ΡŒΠ΅Π·Π½ΠΎ ΠΏΡ€ΠΎΠ΄ΡƒΠΌΠ°Π½Ρ‹ Π»ΠΎΠ³ΠΈΠΊΠ° ΠΈ структура ΠΏΡ€ΠΎΠ³Ρ€Π°ΠΌΠΌΡ‹:
    ΠΏΠΎΡΡΠ½ΠΈΡ‚Π΅Π»ΡŒΠ½Π°Ρ записка с постановкой Ρ†Π΅Π»Π΅ΠΉ ΠΈ раскрытиСм Ρ„ΠΎΡ€ΠΌ ΠΈ ΠΌΠ΅Ρ‚ΠΎΠ΄ΠΎΠ² Ρ€Π°Π±ΠΎΡ‚Ρ‹ Π½Π° ΡƒΡ€ΠΎΠΊΠ°Ρ…, условия Ρ€Π΅Π°Π»ΠΈΠ·Π°Ρ†ΠΈΠΈ ΠΏΡ€ΠΎΠ³Ρ€Π°ΠΌΠΌΡ‹ ΠΈ модСль выпуска, тСматичСскоС     содСрТаниС.Β 

    ΠŸΠΎΡΠ»Π΅Π΄Π½Π΅Π΅Β ΡΠ²ΠΈΠ΄Π΅Ρ‚Π΅Π»ΡŒΡΡ‚Π²ΡƒΠ΅Ρ‚Β ΠΎ Π²Π΅Ρ€Π½ΠΎΠΌ мСтодичСском ΠΏΠΎΠ΄Ρ…ΠΎΠ΄Π΅, Π½Π°ΡƒΡ‡Π½ΠΎΠΌ Ρ…Π°Ρ€Π°ΠΊΡ‚Π΅Ρ€Π΅ Ρ€Π°Π±ΠΎΡ‚Ρ‹. ΠŸΡ€ΠΎΠ³Ρ€Π°ΠΌΠΌΠ° построСна с учСтом ΠΏΡ€ΠΈΠ½Ρ†ΠΈΠΏΠΎΠ² систСмности, научности, доступности, пСрспСктивности и прССмствСнности ΠΌΠ΅ΠΆΠ΄Ρƒ Ρ€Π°Π·Π΄Π΅Π»Π°ΠΌΠΈ курса.

    РСализация Π΄Π°Π½Π½ΠΎΠΉ ΠΏΡ€ΠΎΠ³Ρ€Π°ΠΌΠΌΡ‹ ΠΏΠΎΠ·Π²ΠΎΠ»ΠΈΡ‚ учащимся Π½Π°Ρ‡Π°Π»ΡŒΠ½Ρ‹Ρ… классов ΠΎΡ€ΠΈΠ΅Π½Ρ‚ΠΈΡ€ΠΎΠ²Π°Ρ‚ΡŒΡΡ Π² основных Ρ€Π°Π·Π΄Π΅Π»Π°Ρ… ΠΌΠ°Ρ‚Π΅ΠΌΠ°Ρ‚ΠΈΠΊΠΈ, Π±ΡƒΠ΄Π΅Ρ‚ ΡΠΏΠΎΡΠΎΠ±ΡΡ‚Π²ΠΎΠ²Π°Ρ‚ΡŒ Π²ΠΎΠ²Π»Π΅Ρ‡Π΅Π½ΠΈΡŽ учащихся Π² процСсс приобрСтСния ΠΈΠΌΠΈ матСматичСских Π·Π½Π°Π½ΠΈΠΉ, ΡƒΠΌΠ΅Π½ΠΈΠΉ ΠΈ матСматичСской ΠΊΡƒΠ»ΡŒΡ‚ΡƒΡ€Ρ‹.

    ΠŸΡ€ΠΎΠ³Ρ€Π°ΠΌΠΌΠ° Π΄Π°Π΅Ρ‚ Π²ΠΎΠ·ΠΌΠΎΠΆΠ½ΠΎΡΡ‚ΡŒ Π² соотвСтствии с ΡƒΡ‡Π΅Π±Π½Ρ‹ΠΌ ΠΏΠ»Π°Π½ΠΎΠΌ ΡƒΠ²Π΅Π»ΠΈΡ‡ΠΈΡ‚ΡŒ врСмя Π½Π° ΠΈΠ·ΡƒΡ‡Π΅Π½ΠΈΠ΅ ΠΎΡ‚Π΄Π΅Π»ΡŒΠ½Ρ‹Ρ… Ρ‚Π΅ΠΌ курса, позволяСт ΡƒΡ‚ΠΎΡ‡Π½ΠΈΡ‚ΡŒ ΡΠΏΠΎΡΠΎΠ±Π½ΠΎΡΡ‚ΡŒ ΠΈ Π³ΠΎΡ‚ΠΎΠ²Π½ΠΎΡΡ‚ΡŒ ΡƒΡ‡Π΅Π½ΠΈΠΊΠΎΠ² ΠΊ Π΄Π°Π»ΡŒΠ½Π΅ΠΉΡˆΠ΅ΠΌΡƒ ΠΏΠΎΠ²Ρ‹ΡˆΠ΅Π½ΠΈΡŽ своСго уровня развития ΠΈ Ρ€Π΅ΡˆΠ°Π΅Ρ‚ ΡΠ»Π΅Π΄ΡƒΡŽΡ‰ΠΈΠ΅ Π·Π°Π΄Π°Ρ‡ΠΈ:

    • Ρ€Π°Π·Π½ΠΎΠΎΠ±Ρ€Π°Π·ΠΈΡ‚ΡŒ процСсс обучСния;

    • ΡΡ„ΠΎΡ€ΠΌΠΈΡ€ΠΎΠ²Π°Ρ‚ΡŒ устойчивыС знания ΠΏΠΎ ΠΏΡ€Π΅Π΄ΠΌΠ΅Ρ‚Ρƒ;

    • Π²ΠΎΡΠΏΠΈΡ‚Ρ‹Π²Π°Ρ‚ΡŒ ΠΎΠ±Ρ‰ΡƒΡŽ ΠΌΠ°Ρ‚Π΅ΠΌΠ°Ρ‚ΠΈΡ‡Π΅ΡΠΊΡƒΡŽ ΠΊΡƒΠ»ΡŒΡ‚ΡƒΡ€Ρƒ;

    • Ρ€Π°Π·Π²ΠΈΠ²Π°Ρ‚ΡŒ матСматичСскоС (логичСскоС) ΠΌΡ‹ΡˆΠ»Π΅Π½ΠΈΠ΅;

    • Ρ€Π°ΡΡˆΠΈΡ€ΡΡ‚ΡŒ матСматичСский ΠΊΡ€ΡƒΠ³ΠΎΠ·ΠΎΡ€;

    • Ρ„ΠΎΡ€ΠΌΠΈΡ€ΠΎΠ²Π°Ρ‚ΡŒ ΡƒΠΌΠ΅Π½ΠΈΠ΅ Ρ€Π΅ΡˆΠ°Ρ‚ΡŒ ΠΊΠΎΠΌΠ±ΠΈΠ½Π°Ρ‚ΠΎΡ€Π½Ρ‹Π΅ ΠΈ логичСскиС Π·Π°Π΄Π°Ρ‡ΠΈ;

    • ΠΏΠΎΠ²Ρ‹ΡˆΠ°Ρ‚ΡŒ интСрСс ΠΊ ΠΏΡ€Π΅Π΄ΠΌΠ΅Ρ‚Ρƒ ΠΈ Π΅Π³ΠΎ ΠΈΠ·ΡƒΡ‡Π΅Π½ΠΈΡŽ;

    • Π²Ρ‹Ρ€Π°Π±ΠΎΡ‚Π°Ρ‚ΡŒ ΡΠ°ΠΌΠΎΡΡ‚ΠΎΡΡ‚Π΅Π»ΡŒΠ½Ρ‹ΠΉ ΠΈ творчСский ΠΏΠΎΠ΄Ρ…ΠΎΠ΄Ρ‹ ΠΊ ΠΈΠ·ΡƒΡ‡Π΅Π½ΠΈΡŽ ΠΌΠ°Ρ‚Π΅ΠΌΠ°Ρ‚ΠΈΠΊΠΈ.

    ΠŸΡ€ΠΎΠ³Ρ€Π°ΠΌΠΌΠ° элСктивного курса Π½Π°ΠΏΡ€Π°Π²Π»Π΅Π½Π° Π½Π° ΠΈΠ½Ρ‚Π΅Π»Π»Π΅ΠΊΡ‚ΡƒΠ°Π»ΡŒΠ½ΠΎΠ΅ ΠΈ Ρ€Π΅Ρ‡Π΅Π²ΠΎΠ΅ Ρ€Π°Π·Π²ΠΈΡ‚ΠΈΠ΅ учащСгося, ΡΠΎΠ²Π΅Ρ€ΡˆΠ΅Π½ΡΡ‚Π²ΠΎΠ²Π°Π½ΠΈΠ΅ языкового ΠΌΡ‹ΡˆΠ»Π΅Π½ΠΈΡ, Ρ„ΠΎΡ€ΠΌΠΈΡ€ΠΎΠ²Π°Π½ΠΈΠ΅ ΠΌΠΎΡ‚ΠΈΠ²Π°Ρ†ΠΈΠΈ ΠΊ ΠΎΠ±ΡƒΡ‡Π΅Π½ΠΈΡŽ, осознанной практичСской грамотности учащихся. Она способствуСт Ρ€Π°ΡΡˆΠΈΡ€Π΅Π½ΠΈΡŽ ΠΊΡ€ΡƒΠ³ΠΎΠ·ΠΎΡ€Π° учащСгося, Π΅Π³ΠΎ ΠΏΠΎΠ΄Π³ΠΎΡ‚ΠΎΠ²ΠΊΠ΅ ΠΊ ΡƒΡ‡Π°ΡΡ‚ΠΈΡŽ Π² ΠΎΠ»ΠΈΠΌΠΏΠΈΠ°Π΄Π°Ρ… ΠΈ ΠΈΠ½Ρ‚Π΅Π»Π»Π΅ΠΊΡ‚ΡƒΠ°Π»ΡŒΠ½Ρ‹Ρ… ΠΈΠ³Ρ€Π°Ρ… ΠΏΠΎ ΠΌΠ°Ρ‚Π΅ΠΌΠ°Ρ‚ΠΈΠΊΠ΅.

    Основная Π·Π°Π΄Π°Ρ‡Π° обучСния ΠΌΠ°Ρ‚Π΅ΠΌΠ°Ρ‚ΠΈΠΊΠΈ Π² школС – ΠΎΠ±Π΅ΡΠΏΠ΅Ρ‡ΠΈΡ‚ΡŒ ΠΏΡ€ΠΎΡ‡Π½ΠΎΠ΅ ΠΈ ΡΠΎΠ·Π½Π°Ρ‚Π΅Π»ΡŒΠ½ΠΎΠ΅ ΠΎΠ²Π»Π°Π΄Π΅Π½ΠΈΠ΅ учащимися систСмой матСматичСских Π·Π½Π°Π½ΠΈΠΉ ΠΈ ΡƒΠΌΠ΅Π½ΠΈΠΉ, Π½Π΅ΠΎΠ±Ρ…ΠΎΠ΄ΠΈΠΌΡ‹Ρ… Π² повсСднСвной ΠΆΠΈΠ·Π½ΠΈ ΠΈ Ρ‚Ρ€ΡƒΠ΄ΠΎΠ²ΠΎΠΉ Π΄Π΅ΡΡ‚Π΅Π»ΡŒΠ½ΠΎΡΡ‚ΠΈ ΠΊΠ°ΠΆΠ΄ΠΎΠΌΡƒ Ρ‡Π»Π΅Π½Ρƒ общСства, достаточных для изучСния смСТных дисциплин ΠΈ продолТСния образования.

    Как извСстно, нСспособных Π΄Π΅Ρ‚Π΅ΠΉ Π½Π΅Ρ‚, Π½ΡƒΠΆΠ½ΠΎ просто ΠΏΠΎΠΌΠΎΡ‡ΡŒ Ρ€Π΅Π±Π΅Π½ΠΊΡƒ Ρ€Π°Π·Π²ΠΈΡ‚ΡŒ Π΅Π³ΠΎ способности, ΡΠ΄Π΅Π»Π°Ρ‚ΡŒ процСсс обучСния ΡƒΠ²Π»Π΅ΠΊΠ°Ρ‚Π΅Π»ΡŒΠ½Ρ‹ΠΌ ΠΈ интСрСсным. Π’ этом ΠΌΠΎΠ³ΡƒΡ‚ ΠΏΠΎΠΌΠΎΡ‡ΡŒ внСклассныС занятия ΠΏΠΎ ΠΌΠ°Ρ‚Π΅ΠΌΠ°Ρ‚ΠΈΠΊΠ΅ Π² Ρ„ΠΎΡ€ΠΌΠ΅ Ρ„Π°ΠΊΡƒΠ»ΡŒΡ‚Π°Ρ‚ΠΈΠ²Π°.

    Π‘Ρ‡ΠΈΡ‚Π°ΡŽ Π°ΠΊΡ‚ΡƒΠ°Π»ΡŒΠ½Ρ‹ΠΌ Π²Π²Π΅Π΄Π΅Π½ΠΈΠ΅ ΠΏΡ€ΠΎΠ³Ρ€Π°ΠΌΠΌΡ‹ элСктивного курса «Учимся Ρ€Π΅ΡˆΠ°Ρ‚ΡŒ Π·Π°Π΄Π°Ρ‡ΠΈΒ».

    Π­Ρ‚ΠΎ ΠΏΠ΅Ρ€Π²Ρ‹ΠΉ шаг ΠΊ дальнСйшСй ΠΏΡ€Π΅Π΄ΠΏΡ€ΠΎΡ„ΠΈΠ»ΡŒΠ½ΠΎΠΉ ΠΈ ΠΏΡ€ΠΎΡ„ΠΈΠ»ΡŒΠ½ΠΎΠΉ ΠΏΠΎΠ΄Π³ΠΎΡ‚ΠΎΠ²ΠΊΠ΅ учащихся Π² Π½Π°ΠΏΡ€Π°Π²Π»Π΅Π½ΠΈΠΈ Ρ‚ΠΎΡ‡Π½Ρ‹Ρ… Π½Π°ΡƒΠΊ.

    ΠŸΡ€ΠΎΠ³Ρ€Π°ΠΌΠΌΠ° элСктивного курса «Учимся Ρ€Π΅ΡˆΠ°Ρ‚ΡŒ Π·Π°Π΄Π°Ρ‡ΠΈΒ» ΠΏΠΎΠ΄Ρ‚Π²Π΅Ρ€ΠΆΠ΄Π΅Π½Π° ΠΊΠΎΠ½ΠΊΡ€Π΅Ρ‚Π½ΠΎΠΉ школьной ΠΏΡ€Π°ΠΊΡ‚ΠΈΠΊΠΎΠΉ (Π°ΠΏΡ€ΠΎΠ±ΠΈΡ€ΠΎΠ²Π°Π»Π°ΡΡŒ с 2013 Π³ΠΎΠ΄Π°), ΠΎΡ†Π΅Π½Π΅Π½Π° ΠΏΠΎΠ»ΠΎΠΆΠΈΡ‚Π΅Π»ΡŒΠ½ΠΎ, ΠΎΠ΄ΠΎΠ±Ρ€Π΅Π½Π° ΠΈ Ρ€Π΅ΠΊΠΎΠΌΠ΅Π½Π΄ΠΎΠ²Π°Π½Π° ΠΊ использованию.

    Β 

    Π ΡƒΠΊΠΎΠ²ΠΎΠ΄ΠΈΡ‚Π΅Π»ΡŒ мСтодичСского объСдинСния

    ΡƒΡ‡ΠΈΡ‚Π΅Π»Π΅ΠΉ Π½Π°Ρ‡Π°Π»ΡŒΠ½Ρ‹Ρ… классов

    МКОУ БОШ β„–4 с. ΠœΠ°Π»Ρ‹Π΅ Π―Π³ΡƒΡ€Ρ‹

    ВуркмСнского Ρ€Π°ΠΉΠΎΠ½Π°

    Π‘Ρ‚Π°Π²Ρ€ΠΎΠΏΠΎΠ»ΡŒΡΠΊΠΎΠ³ΠΎ края Π›.И. Π›ΠΈΡ‚ΠΎΠ²Ρ‡Π΅Π½ΠΊΠΎΒ Β Β Β Β Β Β Β Β Β Β Β 

    ΠšΠΎΠ½Ρ‚Ρ€ΠΎΠ»ΡŒΠ½Ρ‹Π΅ Ρ€Π°Π±ΠΎΡ‚Ρ‹ ΠΏΠΎ ΠΌΠ°Ρ‚Π΅ΠΌΠ°Ρ‚ΠΈΠΊΠ΅ для 3 класса

    ΠŸΡ€ΠΎΡΠΌΠΎΡ‚Ρ€ содСрТимого Π΄ΠΎΠΊΡƒΠΌΠ΅Π½Ρ‚Π°
    Β«itogovaya_kontrolnaya_rabota_3_1Β»

    ΠŸΡ€ΠΎΡΠΌΠΎΡ‚Ρ€ содСрТимого Π΄ΠΎΠΊΡƒΠΌΠ΅Π½Ρ‚Π°
    Β«kon.rab_._numeratsiya_do_1000._mat.3kl._shkola_rossiiΒ»

    ΠŸΡ€ΠΎΡΠΌΠΎΡ‚Ρ€ содСрТимого Π΄ΠΎΠΊΡƒΠΌΠ΅Π½Ρ‚Π°
    Β«kon.rab_._slozh._i_vych._do_1000._mat.3kl._shkola_rossiiΒ»

    ΠšΠΎΠ½Ρ‚Ρ€ΠΎΠ»ΡŒΠ½Π°Ρ Ρ€Π°Π±ΠΎΡ‚Π° ΠΏΠΎ Ρ‚Π΅ΠΌΠ΅ Β«Π‘Π»ΠΎΠΆΠ΅Π½ΠΈΠ΅ ΠΈ Π²Ρ‹Ρ‡ΠΈΡ‚Π°Π½ΠΈΠ΅

    Π² ΠΏΡ€Π΅Π΄Π΅Π»Π°Ρ… 1000Β».

    3класс (4 Ρ‡Π΅Ρ‚Π²Π΅Ρ€Ρ‚ΡŒ)

    Π’Π°Ρ€ΠΈΠ°Π½Ρ‚ 1.

    1.РСши Π·Π°Π΄Π°Ρ‡Ρƒ.

    Π£ ΠΏΡ€ΠΎΠ΄Π°Π²Ρ†Π° Π±Ρ‹Π»ΠΎ 230 Π³Π°Π·Π΅Ρ‚. Π”ΠΎ ΠΎΠ±Π΅Π΄Π° ΠΎΠ½ ΠΏΡ€ΠΎΠ΄Π°Π» 110 Π³Π°Π·Π΅Ρ‚, Π° послС ΠΎΠ±Π΅Π΄Π°

    Π΅Ρ‰Ρ‘ 70. Бколько Π³Π°Π·Π΅Ρ‚ ΠΎΡΡ‚Π°Π»ΠΎΡΡŒ Ρƒ ΠΏΡ€ΠΎΠ΄Π°Π²Ρ†Π°?

    2. Π’Ρ‹ΠΏΠΎΠ»Π½ΠΈ вычислСния.

    754 583 643 356 606

    + 263 67 +239238 565

    3. Вычисли значСния Π²Ρ‹Ρ€Π°ΠΆΠ΅Π½ΠΈΠΉ.

    60 : 15 + 92 : 4 910 – 400 Ρ… 2

    27 + 91 : 7 600 : 3 + 90

    29 + 31 Ρ… 2 300 Ρ… 3 + 40

    4. Π—Π°ΠΏΠΎΠ»Π½ΠΈ пропуски.

    408 см = … ΠΌ … см 3ΠΌ 50см = … см

    750 см = … ΠΌ … см 16 Π΄ΠΌ 8см = … см

    5. РСши уравнСния.

    700 – Π₯ = 200 Π₯ – 400 = 500

    Π’Π°Ρ€ΠΈΠ°Π½Ρ‚ 2.

    1.РСши Π·Π°Π΄Π°Ρ‡Ρƒ.

    Π’ кассС Ρ†ΠΈΡ€ΠΊΠ° Π±Ρ‹Π»ΠΎ 460 Π±ΠΈΠ»Π΅Ρ‚ΠΎΠ². Π’ субботу ΠΏΡ€ΠΎΠ΄Π°Π»ΠΈ 140 Π±ΠΈΠ»Π΅Ρ‚ΠΎΠ², Π° Π² Π²ΠΎΡΠΊΡ€Π΅ΡΠ΅Π½ΡŒΠ΅ Π΅Ρ‰Ρ‘ 200. Бколько Π±ΠΈΠ»Π΅Ρ‚ΠΎΠ² ΠΎΡΡ‚Π°Π»ΠΎΡΡŒ Π² кассС Ρ†ΠΈΡ€ΠΊΠ°?

    2. Π’Ρ‹ΠΏΠΎΠ»Π½ΠΈ вычислСния.

    318 453 426 537 326

    + 451 76 +379 + 173180

    3. Вычисли значСния Π²Ρ‹Ρ€Π°ΠΆΠ΅Π½ΠΈΠΉ.

    5 Ρ… 18 – 4 Ρ… 15 800 : 2 + 50

    (57 + 35) : 23 470 – 200 Ρ… 2

    36 : 3 + 3 200 Ρ… 3 + 70

    4. Π—Π°ΠΏΠΎΠ»Π½ΠΈ пропуски.

    350 см = … ΠΌ … см 9ΠΌ 20см = … см

    603 см = … ΠΌ … см 26 Π΄ΠΌ 7см = … см

    5. РСши уравнСния.

    400 – Π₯ = 100 800 – Π₯ = 200

    ΠŸΡ€ΠΎΡΠΌΠΎΡ‚Ρ€ содСрТимого Π΄ΠΎΠΊΡƒΠΌΠ΅Π½Ρ‚Π°
    Β«kon.rab_._tabl._umnozh._i_delen._na_6_-_9._mat.3kl._shkola_rossiiΒ»

    ΠŸΡ€ΠΎΡΠΌΠΎΡ‚Ρ€ содСрТимого Π΄ΠΎΠΊΡƒΠΌΠ΅Π½Ρ‚Π°
    Β«kon.rab_._tablichnoe_umnozh._i_del._na_2_-_5_mat.3kl._shkola_rossiiΒ»

    ΠšΠΎΠ½Ρ‚Ρ€ΠΎΠ»ΡŒΠ½Π°Ρ Ρ€Π°Π±ΠΎΡ‚Π° ΠΏΠΎ Ρ‚Π΅ΠΌΠ΅ Β«Π’Π°Π±Π»ΠΈΡ‡Π½ΠΎΠ΅ ΡƒΠΌΠ½ΠΎΠΆΠ΅Π½ΠΈΠ΅ ΠΈ Π΄Π΅Π»Π΅Π½ΠΈΠ΅ Π½Π° числа 2 – 5Β». 3класс (2 Ρ‡Π΅Ρ‚Π²Π΅Ρ€Ρ‚ΡŒ)

    Π’Π°Ρ€ΠΈΠ°Π½Ρ‚ 1.

    1.РСши Π·Π°Π΄Π°Ρ‡Ρƒ:

    Π’ ΠΊΠΎΡ€Π·ΠΈΠ½Π΅ Π±Ρ‹Π»ΠΎ 12 яблок, Π° Π³Ρ€ΡƒΡˆ Π² 2 Ρ€Π°Π·Π° мСньшС, Ρ‡Π΅ΠΌ яблок. На сколько яблок Π² ΠΊΠΎΡ€Π·ΠΈΠ½Π΅ большС, Ρ‡Π΅ΠΌ Π³Ρ€ΡƒΡˆ?

    2. Найди значСния Π²Ρ‹Ρ€Π°ΠΆΠ΅Π½ΠΈΠΉ:

    24 : (42 – 34) Ρ… 2 18 : (3 Ρ… 3) + 98

    7 Ρ… 3 + 9 Ρ… 4 16 + (84 – 3 Ρ… 6)

    90 – 3 Ρ… 9 + 18 56 + 24 : 4 Ρ… 3

    3. Π—Π°ΠΏΠΈΡˆΠΈ числа ΠΎΡ‚ 3 Π΄ΠΎ 40, ΠΊΠΎΡ‚ΠΎΡ€Ρ‹Π΅ дСлятся 4 Π±Π΅Π· остатка.

    4.РСши уравнСния:

    Π₯ : 4 = 7 56 – Ρ… = 34

    5. НачСрти ΠΊΠ²Π°Π΄Ρ€Π°Ρ‚ АВБD со стороной 3 см. Найди Π΅Π³ΠΎ ΠΏΠ΅Ρ€ΠΈΠΌΠ΅Ρ‚Ρ€.

    Π’Π°Ρ€ΠΈΠ°Π½Ρ‚ 2.

    1.РСши Π·Π°Π΄Π°Ρ‡Ρƒ:

    Π’ Π³ΠΎΠ»ΠΎΠ²ΠΎΠ»ΠΎΠΌΠΊΠ΅ Π·Π°ΡˆΠΈΡ„Ρ€ΠΎΠ²Π°Π½ΠΎ 9 Π½Π°Π·Π²Π°Π½ΠΈΠΉ Ρ€Π΅ΠΊ, Π° Π½Π°Π·Π²Π°Π½ΠΈΠΉ Π³ΠΎΡ€ΠΎΠ΄ΠΎΠ² Π² 3 Ρ€Π°Π·Π° большС. На сколько Π½Π°Π·Π²Π°Π½ΠΈΠΉ Ρ€Π΅ΠΊ Π·Π°ΡˆΠΈΡ„Ρ€ΠΎΠ²Π°Π½ΠΎ мСньшС, Ρ‡Π΅ΠΌ Π½Π°Π·Π²Π°Π½ΠΈΠΉ Π³ΠΎΡ€ΠΎΠ΄ΠΎΠ²?

    2. Найди значСния Π²Ρ‹Ρ€Π°ΠΆΠ΅Π½ΠΈΠΉ:

    49 + 3 Ρ… 7 – 16 37 + 6 Ρ… (34 – 29)

    4 Ρ… 8 – 3 Ρ… 7 27 : (3 Ρ… 3) + 97

    24 : (42 – 36) + 18 100 – 18 : 3 Ρ… 4

    3. Π—Π°ΠΏΠΈΡˆΠΈ числа ΠΎΡ‚ 2 Π΄ΠΎ 30, ΠΊΠΎΡ‚ΠΎΡ€Ρ‹Π΅ дСлятся 3 Π±Π΅Π· остатка.

    4.РСши уравнСния:

    4 Ρ… Π‘ = 32 Π₯ – 26 = 42

    5. НачСрти ΠΊΠ²Π°Π΄Ρ€Π°Ρ‚ АВБD со стороной 4 см. Найди Π΅Π³ΠΎ ΠΏΠ΅Ρ€ΠΈΠΌΠ΅Ρ‚Ρ€.

    ΠŸΡ€ΠΎΡΠΌΠΎΡ‚Ρ€ содСрТимого Π΄ΠΎΠΊΡƒΠΌΠ΅Π½Ρ‚Π°
    Β«kon.rab_._umn._i_delen._ustn._vychisl._mat.3kl._shkola_rossiiΒ»

    ΠŸΡ€ΠΎΡΠΌΠΎΡ‚Ρ€ содСрТимого Π΄ΠΎΠΊΡƒΠΌΠ΅Π½Ρ‚Π°
    Β«kon.rab_._umnozh._trekhzn._na_odnozn._pismen._priyomy_mat.3kl._shkola_rossiiΒ»

    ΠšΠΎΠ½Ρ‚Ρ€ΠΎΠ»ΡŒΠ½Π°Ρ Ρ€Π°Π±ΠΎΡ‚Π° ΠΏΠΎ Ρ‚Π΅ΠΌΠ΅ Β«Π£ΠΌΠ½ΠΎΠΆΠ΅Π½ΠΈΠ΅ Ρ‚Ρ€Ρ‘Ρ…Π·Π½Π°Ρ‡Π½ΠΎΠ³ΠΎ числа Π½Π° ΠΎΠ΄Π½ΠΎΠ·Π½Π°Ρ‡Π½ΠΎΠ΅ (ΠΏΠΈΡΡŒΠΌΠ΅Π½Π½Ρ‹Π΅ ΠΏΡ€ΠΈΡ‘ΠΌΡ‹)Β». 3класс (4 Ρ‡Π΅Ρ‚Π²Π΅Ρ€Ρ‚ΡŒ)

    Π’Π°Ρ€ΠΈΠ°Π½Ρ‚ 1.

    1.РСши Π·Π°Π΄Π°Ρ‡Ρƒ.

    Π’ Π±ΠΈΠ±Π»ΠΈΠΎΡ‚Π΅ΠΊΡƒ ΠΏΡ€ΠΈΠ²Π΅Π·Π»ΠΈ ΡƒΡ‡Π΅Π±Π½ΠΈΠΊΠΈ ΠΏΠΎ ΠΌΠ°Ρ‚Π΅ΠΌΠ°Ρ‚ΠΈΠΊΠ΅ ΠΈ Ρ‡Ρ‚Π΅Π½ΠΈΡŽ, всСго 147 ΡƒΡ‡Π΅Π±Π½ΠΈΠΊΠΎΠ². Π£Ρ‡Π΅Π±Π½ΠΈΠΊΠΈ ΠΏΠΎ Ρ‡Ρ‚Π΅Π½ΠΈΡŽ поставили Π½Π° 3 ΠΏΠΎΠ»ΠΊΠΈ, ΠΏΠΎ 25 ΡƒΡ‡Π΅Π±Π½ΠΈΠΊΠΎΠ² Π½Π° ΠΊΠ°ΠΆΠ΄ΡƒΡŽ. Π£Ρ‡Π΅Π±Π½ΠΈΠΊΠΈ ΠΏΠΎ ΠΌΠ°Ρ‚Π΅ΠΌΠ°Ρ‚ΠΈΠΊΠ΅ поставили ΠΏΠΎΡ€ΠΎΠ²Π½Ρƒ Π½Π° 2 ΠΏΠΎΠ»ΠΊΠΈ. Бколько ΡƒΡ‡Π΅Π±Π½ΠΈΠΊΠΎΠ² ΠΏΠΎ ΠΌΠ°Ρ‚Π΅ΠΌΠ°Ρ‚ΠΈΠΊΠ΅ Π½Π° ΠΊΠ°ΠΆΠ΄ΠΎΠΉ ΠΏΠΎΠ»ΠΊΠ΅?

    2. Π’Ρ‹ΠΏΠΎΠ»Π½ΠΈ ΡƒΠΌΠ½ΠΎΠΆΠ΅Π½ΠΈΠ΅ Π² столбик.

    123 Ρ… 3 94 Ρ… 6 232 Ρ… 4 141 Ρ… 6

    3. Вычисли значСния Π²Ρ‹Ρ€Π°ΠΆΠ΅Π½ΠΈΠΉ.

    46 + 39 : 3 410 – 270 : 9

    (15 + 49) : 8 180 Ρ… 4 – 320 : 4

    49 + 360 : 6 76 : 2 + 76 – 38

    4. Π‘Ρ€Π°Π²Π½ΠΈ.

    5Π΄ΠΌ 7см … 5ΠΌ 80 см 40 см …. 40 Π΄ΠΌ

    7 ΠΌ 1Π΄ΠΌ … 7ΠΌ 11см 4ΠΌ … 40Π΄ΠΌ

    5. НачСрти равносторонний Ρ‚Ρ€Π΅ΡƒΠ³ΠΎΠ»ΡŒΠ½ΠΈΠΊ АВБ со стороной 3 см. Найди Π΅Π³ΠΎ ΠΏΠ΅Ρ€ΠΈΠΌΠ΅Ρ‚Ρ€.

    Π’Π°Ρ€ΠΈΠ°Π½Ρ‚ 2.

    1.РСши Π·Π°Π΄Π°Ρ‡Ρƒ.

    Π’ ΠΌΠ°Π³Π°Π·ΠΈΠ½ ΠΏΡ€ΠΈΠ²Π΅Π·Π»ΠΈ Ρ‡Π°ΠΉ Π² Π±ΠΎΠ»ΡŒΡˆΠΈΡ… ΠΈ ΠΌΠ°Π»Π΅Π½ΡŒΠΊΠΈΡ… ΠΏΠ°Ρ‡ΠΊΠ°Ρ…, всСго 160 ΠΏΠ°Ρ‡Π΅ΠΊ. МалСнькиС ΠΏΠ°Ρ‡ΠΊΠΈ поставили Π½Π° 2 ΠΏΠΎΠ»ΠΊΠΈ, ΠΏΠΎ 44 ΠΏΠ°Ρ‡ΠΊΠΈ Π½Π° ΠΊΠ°ΠΆΠ΄ΡƒΡŽ. Π‘ΠΎΠ»ΡŒΡˆΠΈΠ΅ ΠΏΠ°Ρ‡ΠΊΠΈ поставили ΠΏΠΎΡ€ΠΎΠ²Π½Ρƒ Π½Π° 3 ΠΏΠΎΠ»ΠΊΠΈ. Бколько Π±ΠΎΠ»ΡŒΡˆΠΈΡ… ΠΏΠ°Ρ‡Π΅ΠΊ чая Π½Π° ΠΎΠ΄Π½ΠΎΠΉ ΠΏΠΎΠ»ΠΊΠ΅?

    2. Π’Ρ‹ΠΏΠΎΠ»Π½ΠΈ ΡƒΠΌΠ½ΠΎΠΆΠ΅Π½ΠΈΠ΅ Π² столбик.

    121 Ρ… 4 79 Ρ… 5 224 Ρ… 4 131 Ρ… 7

    3. Вычисли значСния Π²Ρ‹Ρ€Π°ΠΆΠ΅Π½ΠΈΠΉ.

    47 + 56 : 14 790 – 400 : 2

    16 Ρ… (60 : 10) 980 : 2 + 720 : 8

    78 + 600 : 3 640 : (8 – 4) Ρ… 5

    4. Π‘Ρ€Π°Π²Π½ΠΈ.

    4Π΄ΠΌ 7см … 3ΠΌ 70 см 305 см …. 3 ΠΌ 5Π΄ΠΌ

    5 ΠΌ 3Π΄ΠΌ … 5ΠΌ 30см 6ΠΌ … 60Π΄ΠΌ

    5. НачСрти равносторонний Ρ‚Ρ€Π΅ΡƒΠ³ΠΎΠ»ΡŒΠ½ΠΈΠΊ KLM со стороной 4 см. Найди Π΅Π³ΠΎ ΠΏΠ΅Ρ€ΠΈΠΌΠ΅Ρ‚Ρ€.

    ΠŸΡ€ΠΎΡΠΌΠΎΡ‚Ρ€ содСрТимого Π΄ΠΎΠΊΡƒΠΌΠ΅Π½Ρ‚Π°
    Β«kon.rab_._vkhodnaya_3kl._shkola_rossiiΒ»

    ΠŸΡ€ΠΎΡΠΌΠΎΡ‚Ρ€ содСрТимого Π΄ΠΎΠΊΡƒΠΌΠ΅Π½Ρ‚Π°
    Β«kon.rab_._vnetabl._umnozh._i_delen._mat.3kl._shkola_rossiiΒ»

    ΠšΠΎΠ½Ρ‚Ρ€ΠΎΠ»ΡŒΠ½Ρ‹Π΅ Ρ€Π°Π±ΠΎΡ‚Ρ‹ ΠΏΠΎ ΠΌΠ°Ρ‚Π΅ΠΌΠ°Ρ‚ΠΈΠΊΠ΅ 3 класс

    ΠŸΡ€ΠΈΠ»ΠΎΠΆΠ΅Π½ΠΈΠ΅

    Π£Ρ€ΠΎΠΊ 27

    ΠšΠžΠΠ’Π ΠžΠ›Π¬ΠΠΠ― Π ΠΠ‘ΠžΠ’Π

    Π¦Π΅Π»ΠΈ: ΠΏΡ€ΠΎΠ²Π΅Ρ€ΠΈΡ‚ΡŒ ΡƒΠΌΠ΅Π½ΠΈΠ΅ учащихся Ρ€Π΅ΡˆΠ°Ρ‚ΡŒ простыС Π·Π°Π΄Π°Ρ‡ΠΈ Π½Π° ΡƒΠΌΠ½ΠΎΠΆΠ΅Π½ΠΈΠ΅ ΠΈ Π΄Π΅Π»Π΅Π½ΠΈΠ΅, уравнСния Π½Π° слоТСниС ΠΈ Π²Ρ‹Ρ‡ΠΈΡ‚Π°Π½ΠΈΠ΅; усвоС­ниС Ρ‚Π°Π±Π»ΠΈΡ‡Π½Ρ‹Ρ… случаСв умноТСния ΠΈ дСлСния Π½Π° 2 ΠΈ 3, Π° Ρ‚Π°ΠΊΠΆΠ΅ Π·Π½Π°Β­Π½ΠΈΠ΅ порядка дСйствий Π² выраТСниях со скобками ΠΈ Π±Π΅Π·.

    Π’Π°Ρ€ΠΈΠ°Π½Ρ‚ I

    1. Мама ΠΊΡƒΠΏΠΈΠ»Π° 3 ΠΏΠ°ΠΊΠ΅Ρ‚Π° картофСля ΠΏΠΎ 5 ΠΊΠ³ Π² ΠΊΠ°ΠΆΠ΄ΠΎΠΌ. Бколько ΠΊΠΈΠ»ΠΎΠ³Ρ€Π°ΠΌΠΌΠΎΠ² картофСля ΠΊΡƒΠΏΠΈΠ»Π° ΠΌΠ°ΠΌΠ°?

    2. 3 ΠΌΠ°Π»ΡŒΡ‡ΠΈΠΊΠ° Ρ€Π°Π·Π΄Π΅Π»ΠΈΠ»ΠΈ ΠΏΠΎΡ€ΠΎΠ²Π½Ρƒ ΠΌΠ΅ΠΆΠ΄Ρƒ собой 18 ΠΎΡ€Π΅Ρ…ΠΎΠ². Бколько ΠΎΡ€Π΅Ρ…ΠΎΠ² ΠΏΠΎΠ»ΡƒΡ‡ΠΈΠ» ΠΊΠ°ΠΆΠ΄Ρ‹ΠΉ?

    3. Π Π΅ΡˆΠΈΡ‚Π΅ уравнСния:

    Ρ… + 36 = 56 Π₯ – 13 = 20

    4. Π Π΅ΡˆΠΈΡ‚Π΅ ΠΏΡ€ΠΈΠΌΠ΅Ρ€Ρ‹:

    8-2 6βˆ™3 15 :5

    21:3 3βˆ™8 12:6

    18:2 4βˆ™3 2βˆ™9

    5. НайдитС Π·Π½Π°Ρ‡Π΅Π½ΠΈΠ΅ Π²Ρ‹Ρ€Π°ΠΆΠ΅Π½ΠΈΠΉ: 60 – (24 + 3) : 3 = 24 : 3 + 9βˆ™2=

    6.* Π― ΠΆΠΈΠ²Ρƒ Π² сСмьС с ΠΌΠ°ΠΌΠΎΠΉ, ΠΏΠ°ΠΏΠΎΠΉ, Π±Π°Π±ΡƒΡˆΠΊΠΎΠΉ ΠΈ Π΄Π΅Π΄ΡƒΡˆΠΊΠΎΠΉ. Бколько Ρ‚Π°ΠΏΠΎΡ‡Π΅ΠΊ для всСх Ρ‡Π»Π΅Π½ΠΎΠ² ΠΌΠΎΠ΅ΠΉ сСмьи Π΄ΠΎΠ»ΠΆΠ½ΠΎ Π±Ρ‹Ρ‚ΡŒ Ρƒ нас Π΄ΠΎΠΌΠ°, Ссли Ρƒ ΠΊΠ°ΠΆΠ΄ΠΎΠ³ΠΎ Ρ‡Π»Π΅Π½Π° сСмьи Π±ΡƒΠ΄Π΅Ρ‚ ΠΏΠΎ ΠΎΠ΄Π½ΠΎΠΉ ΠΏΠ°Ρ€Π΅ Ρ‚Π°ΠΏΠΎΡ‡Π΅ΠΊ?

    Π’Π°Ρ€ΠΈΠ°Π½Ρ‚ II

    1. Π’ Ρ‚Π΅Π»Π΅Π²ΠΈΠΊΡ‚ΠΎΡ€ΠΈΠ½Π΅ участвовали 3 ΠΊΠΎΠΌΠ°Π½Π΄Ρ‹ ΠΏΠΎ 6 Ρ‡Π΅Π»ΠΎΠ²Π΅ΠΊ Π² ΠΊΠ°ΠΆΒ­Π΄ΠΎΠΉ. Бколько всСго Ρ‡Π΅Π»ΠΎΠ²Π΅ΠΊ участвовало Π² этой Ρ‚Π΅Π»Π΅Π²ΠΈΠΊΡ‚ΠΎΡ€ΠΈΠ½Π΅?

    2. Из 24 ΠΊΡƒΠ±ΠΈΠΊΠΎΠ² Π›Π΅Π½Π° построила 3 ΠΎΠ΄ΠΈΠ½Π°ΠΊΠΎΠ²Ρ‹Π΅ башни. Бколько ΠΊΡƒΠ±ΠΈΠΊΠΎΠ² Π² ΠΊΠ°ΠΆΠ΄ΠΎΠΉ башнС?

    3. Π Π΅ΡˆΠΈΡ‚Π΅ уравнСния: Ρ… + 48 = 96 Ρ… – 23 = 8

    4. Π Π΅ΡˆΠΈΡ‚Π΅ ΠΏΡ€ΠΈΠΌΠ΅Ρ€Ρ‹:

    12:3 8 βˆ™ 2 24 : 8

    5 βˆ™3 18: 3 3 βˆ™ 7

    14 : 2 2 βˆ™ 5 27 : 9

    5. НайдитС Π·Π½Π°Ρ‡Π΅Π½ΠΈΠ΅ Π²Ρ‹Ρ€Π°ΠΆΠ΅Π½ΠΈΠΉ:

    (71 – 65) βˆ™3 18:3 + 8βˆ™2

    6.* Π—Π° столом сидСла вся наша сСмья: я, ΠΌΠ°ΠΌΠ°, ΠΏΠ°ΠΏΠ°, Π±Ρ€Π°Ρ‚ ΠΈ Π±Π°Β­Π±ΡƒΡˆΠΊΠ°. ΠšΠ°ΠΆΠ΄Ρ‹ΠΉ ΠΈΠ· нас съСл ΠΏΠΎ 2 ΠΏΠΈΡ€ΠΎΠΆΠΊΠ°. На сколько мСньшС стало ΠΏΠΈΡ€ΠΎΠΆΠΊΠΎΠ²?

    * Π—Π°Π΄Π°Ρ‡ΠΈ Π½Π° смСкалку.

    Π£Ρ€ΠΎΠΊ 42

    ΠšΠžΠΠ’Π ΠžΠ›Π¬ΠΠΠ― Π ΠΠ‘ΠžΠ’Π ЗА I Π§Π•Π’Π’Π•Π Π’Π¬

    Π’Π°Ρ€ΠΈΠ°Π½Ρ‚ I

    1. Π’ саду росло 4 гладиолуса, Π° Π»ΠΈΠ»ΠΈΠΉ – Π½Π° 8 большС. Π’ΠΎ сколь­ко Ρ€Π°Π· мСньшС росло Π² саду гладиолусов, Ρ‡Π΅ΠΌ Π»ΠΈΠ»ΠΈΠΉ?

    2. НайдитС Π·Π½Π°Ρ‡Π΅Π½ΠΈΠ΅ Π²Ρ‹Ρ€Π°ΠΆΠ΅Π½ΠΈΠΉ:

    1. 5βˆ™3 – 18 = (36 : 6 + 3) βˆ™ 2 =

    3.НайдитС ΠΏΠ»ΠΎΡ‰Π°Π΄ΡŒ ΠΈ ΠΏΠ΅Ρ€ΠΈΠΌΠ΅Ρ‚Ρ€ ΠΏΡ€ΡΠΌΠΎΡƒΠ³ΠΎΠ»ΡŒΠ½ΠΈΠΊΠ° со сторонами 7 см ΠΈ 5 см.

    4. Π Π΅ΡˆΠΈΡ‚Π΅ уравнСния:

    Ρ… βˆ™ 4 = 36 Ρ… : 6 = 9

    5. Π‘Ρ€Π°Π²Π½ΠΈΡ‚Π΅:

    3 Π΄ΠΌ 4 см … 4 Π΄ΠΌ 3 см 40 см … 4 Π΄ΠΌ

    63 см … 7 Π΄ΠΌ 2 см … 9 ΠΌΠΌ

    6. ΠšΠ°ΠΆΠ΄Ρ‹Π΅ понСдСльник, срСду ΠΈ Ρ‡Π΅Ρ‚Π²Π΅Ρ€Π³ ΠΏΠ°ΠΏΠ° ΠΏΠΎΠΊΡƒΠΏΠ°Π΅Ρ‚ ΠΏΠΎ ΠΎΠ΄Β­Π½ΠΎΠΉ Π³Π°Π·Π΅Ρ‚Π΅. Бколько Π³Π°Π·Π΅Ρ‚ ΠΏΠ°ΠΏΠ° ΠΊΡƒΠΏΠΈΡ‚ Π·Π° 8 нСдСль?

    Π’Π°Ρ€ΠΈΠ°Π½Ρ‚ II

    1. Π₯озяйка ΠΊΡƒΠΏΠΈΠ»Π° 3 ΠΊΠ³ яблок, Π° Π³Ρ€ΡƒΡˆ – Π½Π° 6 ΠΊΠ³ большС. Π’ΠΎ сколько Ρ€Π°Π· большС ΠΊΡƒΠΏΠΈΠ»Π° хозяйка Π³Ρ€ΡƒΡˆ, Ρ‡Π΅ΠΌ яблок?

    2. НайдитС Π·Π½Π°Ρ‡Π΅Π½ΠΈΠ΅ Π²Ρ‹Ρ€Π°ΠΆΠ΅Π½ΠΈΠΉ:

    1. βˆ™ 8 – 20) βˆ™ 6= 38 + 3 βˆ™ 4 – 17 =

    3. Π”Π»ΠΈΠ½Π° ΠΏΡ€ΡΠΌΠΎΡƒΠ³ΠΎΠ»ΡŒΠ½ΠΈΠΊΠ° 8 Π΄ΠΌ, Π° ΡˆΠΈΡ€ΠΈΠ½Π° 6 Π΄ΠΌ. НайдитС Π΅Π³ΠΎ ΠΏΠ»ΠΎΡ‰Π°Π΄ΡŒ ΠΈ ΠΏΠ΅Ρ€ΠΈΠΌΠ΅Ρ‚Ρ€.

    4. Π Π΅ΡˆΠΈΡ‚Π΅ уравнСния:

    7 βˆ™ Ρ… = 28 48 : Ρ… = 6

    5. Π‘Ρ€Π°Π²Π½ΠΈΡ‚Π΅

    6 ΠΌΠΌ … 4 см 70 см … 7 Π΄ΠΌ

    8 Π΄ΠΌ 2 см … 2 Π΄ΠΌ 8 см 34 см … 4 Π΄ΠΌ

    6. ΠšΠ°ΠΆΠ΄Ρ‹Π΅ Π²Ρ‚ΠΎΡ€Π½ΠΈΠΊ, Ρ‡Π΅Ρ‚Π²Π΅Ρ€Π³ ΠΈ субботу ΠΠ»Ρ‘ΡˆΠ° смотрит ΠΏΠΎ ΠΎΠ΄Π½ΠΎΒ­ΠΌΡƒ Π½ΠΎΠ²ΠΎΠΌΡƒ ΠΌΡƒΠ»ΡŒΡ‚Ρ„ΠΈΠ»ΡŒΠΌΡƒ. Бколько Π½ΠΎΠ²Ρ‹Ρ… ΠΌΡƒΠ»ΡŒΡ‚Ρ„ΠΈΠ»ΡŒΠΌΠΎΠ² ΠΠ»Ρ‘ΡˆΠ° по­смотрит Π·Π° 6 нСдСль?

    Π£Ρ€ΠΎΠΊ 62

    ΠšΠžΠΠ’Π ΠžΠ›Π¬ΠΠΠ― Π ΠΠ‘ΠžΠ’Π

    Π¦Π΅Π»ΠΈ: ΠΏΡ€ΠΎΠ²Π΅Ρ€ΠΈΡ‚ΡŒ ΡƒΠΌΠ΅Π½ΠΈΠ΅ учащихся Ρ€Π΅ΡˆΠ°Ρ‚ΡŒ Π·Π°Π΄Π°Ρ‡ΠΈ, уравнСния Π½Π° слоТСниС ΠΈ Π²Ρ‹Ρ‡ΠΈΡ‚Π°Π½ΠΈΠ΅, Π½Π°Ρ…ΠΎΠ΄ΠΈΡ‚ΡŒ ΠΏΠ»ΠΎΡ‰Π°Π΄ΡŒ ΠΏΡ€ΡΠΌΠΎΡƒΠ³ΠΎΠ»ΡŒΠ½ΠΈΠΊΠ°; Π·Π½Π°Β­Π½ΠΈΠ΅ порядка дСйствий ΠΈ ΠΏΠ΅Ρ€Π΅Π²ΠΎΠ΄ Π΅Π΄ΠΈΠ½ΠΈΡ† Π΄Π»ΠΈΠ½Ρ‹.

    Π’Π°Ρ€ΠΈΠ°Π½Ρ‚ I

    1. Π’ 5 ΠΊΠ»Π΅Ρ‚ΠΊΠ°Ρ… ΠΏΠΎ 2 ΠΊΠ°Π½Π°Ρ€Π΅ΠΉΠΊΠΈ. ΠŸΠΎΠ΄Π°Ρ€ΠΈΠ»ΠΈ 4 ΠΏΡ‚ΠΈΡ†Ρ‹. Бколько ΠΊΠ°Β­Π½Π°Ρ€Π΅Π΅ΠΊ ΠΎΡΡ‚Π°Π»ΠΎΡΡŒ?

    2. НайдитС значСния Π²Ρ‹Ρ€Π°ΠΆΠ΅Π½ΠΈΠΉ.

    90 – 4 βˆ™ 2 βˆ™ 9

    18 + (12 + 25) : 4

    27 : 3 βˆ™ (35 : 5)

    3. ΠŸΠ΅Ρ€Π΅Π²Π΅Π΄ΠΈΡ‚Π΅.

    5 ΠΌ 4 Π΄ΠΌ = … Π΄ΠΌ 78 см = … Π΄ΠΌ … см

    3 Π΄ΠΌ 7 см = … см 60 Π΄ΠΌ = … ΠΌ

    4. Π”Π»ΠΈΠ½Π° ΠΏΡ€ΡΠΌΠΎΡƒΠ³ΠΎΠ»ΡŒΠ½ΠΈΠΊΠ° 6 см, Π° ΡˆΠΈΡ€ΠΈΠ½Π° Π½Π° 4 см ΠΊΠΎΡ€ΠΎΡ‡Π΅. Най­дитС ΠΏΠ»ΠΎΡ‰Π°Π΄ΡŒ ΠΏΡ€ΡΠΌΠΎΡƒΠ³ΠΎΠ»ΡŒΠ½ΠΈΠΊΠ°.

    5. Π Π΅ΡˆΠΈΡ‚Π΅ уравнСния.

    Π₯ + 20 = 48 Ρ… – 44 = 18

    Π’Π°Ρ€ΠΈΠ°Π½Ρ‚ II

    1. На 3 Π²Π΅Ρ‚ΠΊΠ°Ρ… ΠΏΠΎ 9 шишСк. Π‘Π΅Π»ΠΊΠ° ΡƒΡ‚Π°Ρ‰ΠΈΠ»Π° 3 шишки. Бколько шишСк ΠΎΡΡ‚Π°Π»ΠΎΡΡŒ?

    2. НайдитС значСния Π²Ρ‹Ρ€Π°ΠΆΠ΅Π½ΠΈΠΉ.

    49 : (16 – 9) βˆ™ 9

    58 + 56 : (14 – 7)

    40 – 5 βˆ™ 8 + 19

    3. ΠŸΠ΅Ρ€Π΅Π²Π΅Π΄ΠΈΡ‚Π΅.

    8 ΠΌ 3 Π΄ΠΌ = … Π΄ΠΌ

    1 Π΄ΠΌ 9 см = … см

    25 см = … Π΄ΠΌ … см

    70 Π΄ΠΌ = … ΠΌ

    4. Π¨ΠΈΡ€ΠΈΠ½Π° ΠΏΡ€ΡΠΌΠΎΡƒΠ³ΠΎΠ»ΡŒΠ½ΠΈΠΊΠ° 7 см, Π° Π΄Π»ΠΈΠ½Π° Π½Π° 2 см большС. Най­дитС ΠΏΠ»ΠΎΡ‰Π°Π΄ΡŒ ΠΏΡ€ΡΠΌΠΎΡƒΠ³ΠΎΠ»ΡŒΠ½ΠΈΠΊΠ°.

    5. Π Π΅ΡˆΠΈΡ‚Π΅ уравнСния

    75 + Ρ… = 90 Ρ… – 17 = 33

    Π£Ρ€ΠΎΠΊ 75

    ΠšΠžΠΠ’Π ΠžΠ›Π¬ΠΠΠ― Π ΠΠ‘ΠžΠ’Π

    Π’Π°Ρ€ΠΈΠ°Π½Ρ‚ I

    1. Π’ 4 Π½Π°Π±ΠΎΡ€Π°Ρ… 32 листа Ρ†Π²Π΅Ρ‚Π½ΠΎΠΉ Π±ΡƒΠΌΠ°Π³ΠΈ. Π’ ΡΠΊΠΎΠ»ΡŒΠΊΠΈΡ… Π½Π°Π±ΠΎΡ€Π°Ρ… находятся 72 листа Π±ΡƒΠΌΠ°Π³ΠΈ?

    2. Π’ΡΡ‚Π°Π²ΡŒΡ‚Π΅ Π² окошки Π·Π½Π°ΠΊΠΈ βˆ™ ΠΈΠ»ΠΈ : ΠΈ ΠΏΡ€ΠΎΠΏΡƒΡ‰Π΅Π½Π½Ρ‹Π΅ числа Ρ‚Π°ΠΊ, Ρ‡Ρ‚ΠΎΠ±Ρ‹ ΠΏΡ€ΠΈΠΌΠ΅Ρ€Ρ‹ стали Π²Π΅Ρ€Π½Ρ‹ΠΌΠΈ:

    βͺβͺ8 = 9 5βͺβͺ = 0

    3βͺβͺ = 24 βͺβͺ8 = 1

    36 βͺβͺ= 6 βͺβͺ6 = 0

    βͺβͺ5 = 9 4βͺβͺ= 4

    3.НайдитС Π·Π½Π°Ρ‡Π΅Π½ΠΈΠ΅ Π²Ρ‹Ρ€Π°ΠΆΠ΅Π½ΠΈΠΉ:

    45 : (33 – 24) βˆ™ 6 63 +27 : (30 : 10)

    4. ΠŸΠ»ΠΎΡ‰Π°Π΄ΡŒ ΠΏΡ€ΡΠΌΠΎΡƒΠ³ΠΎΠ»ΡŒΠ½ΠΈΠΊΠ° 42 см2, Π΅Π³ΠΎ ΡˆΠΈΡ€ΠΈΠ½Π° 6 см. НайдитС Π΄Π»ΠΈΠ½Ρƒ ΠΈ ΠΏΠ΅Ρ€ΠΈΠΌΠ΅Ρ‚Ρ€ этого ΠΏΡ€ΡΠΌΠΎΡƒΠ³ΠΎΠ»ΡŒΠ½ΠΈΠΊΠ°.

    5. НайдитС Π·Π½Π°Ρ‡Π΅Π½ΠΈΠ΅ выраТСния, Π° + 27, Ссли, Π° = 8, Π° = 12, Π° = 16, Π° = 4.

    6. Π’ нашСм Π΄ΠΎΠΌΠ΅ ΠΆΠΈΠ²ΡƒΡ‚ ΠšΠ°Ρ‚Ρ, Маша ΠΈ Π›Π΅Π½Π°. Π’Ρ‡Π΅Ρ€Π° я Π²ΠΈΠ΄Π΅Π» ΠšΠ°Ρ‚ΡŽ ΠΈ ΠœΠ°ΡˆΡƒ. Одной ΠΈΠ· Π½ΠΈΡ… 9 Π»Π΅Ρ‚, Π΄Ρ€ΡƒΠ³ΠΎΠΉ – 8. БСгодня я Π²ΠΈΠ΄Π΅Π» ΠœΠ°ΡˆΡƒ ΠΈ Π›Π΅Π½Ρƒ. Одной ΠΈΠ· Π½ΠΈΡ…, 10 Π»Π΅Ρ‚, Π΄Ρ€ΡƒΠ³ΠΎΠΉ – 9. ΠšΠΎΠΌΡƒ сколько Π»Π΅Ρ‚?

    Π’Π°Ρ€ΠΈΠ°Π½Ρ‚ II

    1. На 3 ΠΊΠΎΡΡ‚ΡŽΠΌΠ° ΠΈΠ΄Ρ‘Ρ‚ 9 ΠΌ Ρ‚ΠΊΠ°Π½ΠΈ. Бколько ΠΌΠ΅Ρ‚Ρ€ΠΎΠ² Ρ‚ΠΊΠ°Π½ΠΈ ΠΏΠΎΠΉΠ΄Ρ‘Ρ‚ Π½Π° 6 Ρ‚Π°ΠΊΠΈΡ… ΠΊΠΎΡΡ‚ΡŽΠΌΠΎΠ²?

    2. Π’ΡΡ‚Π°Π²ΡŒΡ‚Π΅ Π² окошки Π·Π½Π°ΠΊΠΈ β€’ ΠΈΠ»ΠΈ : ΠΈ ΠΏΡ€ΠΎΠΏΡƒΡ‰Π΅Π½Π½Ρ‹Π΅ числа Ρ‚Π°ΠΊ, Ρ‡Ρ‚ΠΎΠ±Ρ‹ ΠΏΡ€ΠΈΠΌΠ΅Ρ€Ρ‹ стали Π²Π΅Ρ€Π½Ρ‹ΠΌΠΈ:

    βͺβͺ7 = 6 9βͺβͺ = 0

    5βͺβͺ = 40 βͺβͺ15 = 1

    56βͺβͺ = 7 βͺβͺ7 = 0

    βͺβͺ3 21 3βͺβͺ = 3

    3.НайдитС Π·Π½Π°Ρ‡Π΅Π½ΠΈΠ΅ Π²Ρ‹Ρ€Π°ΠΆΠ΅Π½ΠΈΠΉ:

    60 – 18 : 2 βˆ™ 3 96 – (35 – 5) : 6

    4. ΠŸΠ»ΠΎΡ‰Π°Π΄ΡŒ ΠΏΡ€ΡΠΌΠΎΡƒΠ³ΠΎΠ»ΡŒΠ½ΠΈΠΊΠ° 72 см2, Π΅Π³ΠΎ Π΄Π»ΠΈΠ½Π° 8 см. НайдитС ΡˆΠΈΡ€ΠΈΠ½Ρƒ ΠΈ ΠΏΠ΅Ρ€ΠΈΠΌΠ΅Ρ‚Ρ€ этого ΠΏΡ€ΡΠΌΠΎΡƒΠ³ΠΎΠ»ΡŒΠ½ΠΈΠΊΠ°.

    5. НайдитС Π·Π½Π°Ρ‡Π΅Π½ΠΈΠ΅ выраТСния 36 + Π², Ссли Π² = 9, Π² = 5, Π² = 29, Π² =17.

    6. Π¦Π²Π΅Ρ‚ΠΊΠΈ картофСля Π±Ρ‹Π²Π°ΡŽΡ‚ ΠΎΡ‚ΠΊΡ€Ρ‹Ρ‚Ρ‹ ΠΎΡ‚ 6 Ρ‡ ΡƒΡ‚Ρ€Π° Π΄ΠΎ 14 Ρ‡ дня, Ρ†Π²Π΅Ρ‚ΠΊΠΈ льна – ΠΎΡ‚ 6 Ρ‡ Π΄ΠΎ 16 Ρ‡, Π° Ρ†Π²Π΅Ρ‚ΠΊΠΈ ΠΊΠ°Π»Π΅Π½Π΄ΡƒΠ»Ρ‹ ΠΎΡ‚ 9 Ρ‡ Π΄ΠΎ 15 Ρ‡. Π’ ΠΊΠ°ΠΊΠΈΠ΅ часы эти Ρ†Π²Π΅Ρ‚ΠΊΠΈ раскрыты ΠΎΠ΄Π½ΠΎΠ²Ρ€Π΅ΠΌΠ΅Π½Π½ΠΎ?

    Π£Ρ€ΠΎΠΊ 98

    ΠšΠžΠΠ’Π ΠžΠ›Π¬ΠΠΠ― Π ΠΠ‘ΠžΠ’Π

    ЦСль: ΠΏΡ€ΠΎΠ²Π΅Ρ€ΠΈΡ‚ΡŒ усвоСниС Π²Π½Π΅Ρ‚Π°Π±Π»ΠΈΡ‡Π½ΠΎΠ³ΠΎ ΠΏΡ€ΠΈΡ‘ΠΌΠ° умноТСния ΠΈ дСлСния чисСл Π² ΠΏΡ€Π΅Π΄Π΅Π»Π°Ρ… 100.

    Π’Π°Ρ€ΠΈΠ°Π½Ρ‚ I

    1.Π Π΅ΡˆΠΈΡ‚Π΅ ΠΏΡ€ΠΈΠΌΠ΅Ρ€Ρ‹

    7 βˆ™ 12 = 96 : 3 =

    25 βˆ™ 3 = 76 : 2 =

    18 βˆ™ 5 = 70 : 14 =

    4 βˆ™ 21 = 84 : 28 =

    2. Π Π΅ΡˆΠΈΡ‚Π΅ Π·Π°Π΄Π°Ρ‡Ρƒ.

    Школьники посадили 4 ряда яблонь ΠΏΠΎ 15 Π΄Π΅Ρ€Π΅Π²ΡŒΠ΅Π² Π² ΠΊΠ°ΠΆΠ΄ΠΎΠΌ ряду ΠΈ 3 ряда слив ΠΏΠΎ 10 Π΄Π΅Ρ€Π΅Π²ΡŒΠ΅Π² Π² ΠΊΠ°ΠΆΠ΄ΠΎΠΌ ряду. На сколько боль­шС посадили яблонь, Ρ‡Π΅ΠΌ слив?

    3. Π Π΅ΡˆΠΈΡ‚Π΅ Π·Π°Π΄Π°Ρ‡Ρƒ.

    Π”Π»ΠΈΠ½Π° ΠΏΡ€ΡΠΌΠΎΡƒΠ³ΠΎΠ»ΡŒΠ½ΠΈΠΊΠ° 15 см, ΡˆΠΈΡ€ΠΈΠ½Π° 7 см. НайдитС ΠΏΠ»ΠΎΡ‰Π°Π΄ΡŒ ΠΈ ΠΏΠ΅Ρ€ΠΈΠΌΠ΅Ρ‚Ρ€ этого ΠΏΡ€ΡΠΌΠΎΡƒΠ³ΠΎΠ»ΡŒΠ½ΠΈΠΊΠ°.

    4. Π‘Ρ€Π°Π²Π½ΠΈΡ‚Π΅.

    8 Π΄ΠΌ 3 см … 3 Π΄ΠΌ 8 см 61 см … 7 Π΄ΠΌ

    1 ΠΌ … 6Π΄ΠΌ 4 ΠΌ 5 Π΄ΠΌ … 45 Π΄ΠΌ

    5. Π Π΅ΡˆΠΈΡ‚Π΅ уравнСния.

    Π₯ βˆ™ 14 = 84 96 : Π₯ = 24

    Π’Π°Ρ€ΠΈΠ°Π½Ρ‚ II

    1. Π Π΅ΡˆΠΈΡ‚Π΅ ΠΏΡ€ΠΈΠΌΠ΅Ρ€Ρ‹.

    14 βˆ™ 7= 90:15 =

    3 βˆ™ 26 = 46 : 2 =

    19 βˆ™ 5= 92:4 =

    48 βˆ™ 2 = 72 : 24 =

    2. Π Π΅ΡˆΠΈΡ‚Π΅ Π·Π°Π΄Π°Ρ‡Ρƒ.

    Π“Ρ€ΡƒΠΏΠΏΠ° экскурсантов Ρ€Π°Π·ΠΌΠ΅ΡΡ‚ΠΈΠ»Π°ΡΡŒ Π² 2 ΠΊΠ°Ρ‚Π΅Ρ€Π°Ρ… ΠΏΠΎ 16 Ρ‡Π΅Π»ΠΎΠ²Π΅ΠΊ Π² ΠΊΠ°ΠΆΠ΄ΠΎΠΌ ΠΈ Π² 3 Π»ΠΎΠ΄ΠΊΠ°Ρ… ΠΏΠΎ 8 Ρ‡Π΅Π»ΠΎΠ²Π΅ΠΊ Π² ΠΊΠ°ΠΆΠ΄ΠΎΠΉ. На сколько большС Ρ‡Π΅Β­Π»ΠΎΠ²Π΅ΠΊ Π±Ρ‹Π»ΠΎ Π² ΠΊΠ°Ρ‚Π΅Ρ€Π°Ρ…, Ρ‡Π΅ΠΌ Π² Π»ΠΎΠ΄ΠΊΠ°Ρ…?

    3. Π Π΅ΡˆΠΈΡ‚Π΅ Π·Π°Π΄Π°Ρ‡Ρƒ.

    Π”Π»ΠΈΠ½Π° ΠΏΡ€ΡΠΌΠΎΡƒΠ³ΠΎΠ»ΡŒΠ½ΠΈΠΊΠ° 18 см, ΡˆΠΈΡ€ΠΈΠ½Π° 5 см. НайдитС ΠΏΠ»ΠΎΡ‰Π°Π΄ΡŒ ΠΈ (Π΅Ρ€ΠΈΠΌΠ΅Ρ‚Ρ€ этого ΠΏΡ€ΡΠΌΠΎΡƒΠ³ΠΎΠ»ΡŒΠ½ΠΈΠΊΠ°.

    4. Π‘Ρ€Π°Π²Π½ΠΈΡ‚Π΅.

    53 см … 5 Π΄ΠΌ 9 ΠΌ 4 Π΄ΠΌ … 94 Π΄ΠΌ

    7 Π΄ΠΌ 2 см … 2 Π΄ΠΌ 7 см 8 Π΄ΠΌ … 1 ΠΌ

    5. Π Π΅ΡˆΠΈΡ‚Π΅ уравнСния.

    Π₯ : 23 = 4 16 βˆ™ Π₯ = 64

    Π£Ρ€ΠΎΠΊ 109

    ΠšΠžΠΠ’Π ΠžΠ›Π¬ΠΠΠ― Π ΠΠ‘ΠžΠ’Π

    ЦСль: ΠΏΡ€ΠΎΠ²Π΅Ρ€ΠΈΡ‚ΡŒ усвоСниС ΠΏΡ€ΠΈΡ‘ΠΌΠ° дСлСния с остатком ΠΈ Π΅Π³ΠΎ ΠΏΡ€ΠΎΠ²Π΅Ρ€ΠΊΡƒ, Π·Π½Π°Π½ΠΈΠ΅ порядка дСйствий Π² выраТСниях, ΡƒΠΌΠ΅Π½ΠΈΠ΅ Ρ€Π΅ΡˆΠ°Ρ‚ΡŒ Π·Π°Π΄Π°Ρ‡ΠΈ.

    Π’Π°Ρ€ΠΈΠ°Π½Ρ‚ I

    1. Π’Ρ‹ΠΏΠΎΠ»Π½ΠΈ Π΄Π΅Π»Π΅Π½ΠΈΠ΅ с остатком ΠΈ ΠΏΡ€ΠΎΠ²Π΅Ρ€ΠΊΡƒ ΠΊ Π½Π΅ΠΌΡƒ

    75:8 85:20 51:7

    2. Π’ΡΡ‚Π°Π²ΡŒ числа Π² окошки, Ρ‡Ρ‚ΠΎΠ±Ρ‹ ΠΏΠΎΠ»ΡƒΡ‡ΠΈΠ»ΠΈΡΡŒ Π²Π΅Ρ€Π½Ρ‹Π΅ равСнства.

    βͺ:9 = 9(ост.βͺ) βͺ:3 = 7(остβͺ)

    βͺ:7 = 7(ост.βͺ) βͺ:7=10(ост.βͺ)

    3. Найди значСния Π²Ρ‹Ρ€Π°ΠΆΠ΅Π½ΠΈΠΉ.

    56:20–6:12 (39 + 33) : 24βˆ™9

    48 + 32: (64: 8) 93–3βˆ™8:6

    4. РСши Π·Π°Π΄Π°Ρ‡Ρƒ.

    Π‘Ρ‚Π΅ΠΊΠΎΠ»ΡŒΡ‰ΠΈΠΊΡƒ Π½ΡƒΠΆΠ½ΠΎ Π±Ρ‹Π»ΠΎ Π²ΡΡ‚Π°Π²ΠΈΡ‚ΡŒ 96 стёкол. Он ΡƒΠΆΠ΅ вставил Π² 14 ΠΎΠΊΠΎΠ½ ΠΏΠΎ 3 стСкла Π² ΠΊΠ°ΠΆΠ΄ΠΎΠ΅. Бколько стёкол ΠΎΡΡ‚Π°Π»ΠΎΡΡŒ Π²ΡΡ‚Π°Π²ΠΈΡ‚ΡŒ ΡΡ‚Π΅ΠΊΠΎΠ»ΡŒΡ‰ΠΈΠΊΡƒ?

    5. РСши Π·Π°Π΄Π°Ρ‡Ρƒ.

    На Ρ‚Π°Ρ€Π΅Π»ΠΊΠ΅ Π±Ρ‹Π»ΠΎ 48 Π±Π»ΠΈΠ½ΠΎΠ². Бколько Π±Π»ΠΈΠ½ΠΎΠ² съСли, Ссли Π½Π° Ρ‚Π°Β­Ρ€Π΅Π»ΠΊΠ΅ ΠΎΡΡ‚Π°Π»ΠΎΡΡŒ Π² 3 Ρ€Π°Π·Π° мСньшС Π±Π»ΠΈΠ½ΠΎΠ², Ρ‡Π΅ΠΌ Π±Ρ‹Π»ΠΎ?

    Π’Π°Ρ€ΠΈΠ°Π½Ρ‚ II

    1. Π’Ρ‹ΠΏΠΎΠ»Π½ΠΈ Π΄Π΅Π»Π΅Π½ΠΈΠ΅ с остатком ΠΈ ΠΏΡ€ΠΎΠ²Π΅Ρ€ΠΊΡƒ ΠΊ Π½Π΅ΠΌΡƒ.

    57 : 8 74 : 7 69 : 20

    2. Π’ΡΡ‚Π°Π²ΡŒ числа Π² окошки, Ρ‡Ρ‚ΠΎΠ±Ρ‹ ΠΏΠΎΠ»ΡƒΡ‡ΠΈΠ»ΠΈΡΡŒ Π²Π΅Ρ€Π½Ρ‹Π΅ равСнства.

    βͺ : 6 = 5(ост.βͺ) βͺ: 38 = 7(остβͺ)

    βͺ : 4 = 8(ост.βͺ) βͺ : 5=9(ост.βͺ)

    3. Найди значСния Π²Ρ‹Ρ€Π°ΠΆΠ΅Π½ΠΈΠΉ.

    80 + (24 – 4 βˆ™5) 42 : 3 + 72 : 24

    78 – 8 βˆ™ 8 : 2 24 + 36 : (54 : 9)

    4. РСши Π·Π°Π΄Π°Ρ‡Ρƒ.

    Π’ большой ΠΊΠΎΡ€Π·ΠΈΠ½Π΅ Π±Ρ‹Π»ΠΎ 95 Ρ‚ΡŽΠ»ΡŒΠΏΠ°Π½ΠΎΠ². ΠŸΡ€ΠΎΠ΄Π°Π²Π΅Ρ† сдСлал ΠΈΠ· Π½ΠΈΡ… 12 Π±ΡƒΠΊΠ΅Ρ‚ΠΎΠ², ΠΏΠΎ 7 Ρ‚ΡŽΠ»ΡŒΠΏΠ°Π½ΠΎΠ² Π² ΠΊΠ°ΠΆΠ΄ΠΎΠΌ Π±ΡƒΠΊΠ΅Ρ‚Π΅. Бколько ΠΎΡΡ‚Π°Π»ΠΎΡΡŒ Ρ‚ΡŽΠ»ΡŒΠΏΠ°Π½ΠΎΠ²?

    5. РСши Π·Π°Π΄Π°Ρ‡Ρƒ.

    На столС стояло 52 стакана с соком. Бколько стаканов с соком Π²Ρ‹ΠΏΠΈΠ»ΠΈ, Ссли Π½Π° столС ΠΎΡΡ‚Π°Π»ΠΎΡΡŒ Π² 4 Ρ€Π°Π·Π° мСньшС стаканов с соком, Ρ‰Π΅ΠΌ Π±Ρ‹Π»ΠΎ ΠΏΠ΅Ρ€Π²ΠΎΠ½Π°Ρ‡Π°Π»ΡŒΠ½ΠΎ?

    Π£Ρ€ΠΎΠΊ 123

    ΠšΠžΠΠ’Π ΠžΠ›Π¬ΠΠΠ― Π ΠΠ‘ΠžΠ’Π

    Π¦Π΅Π»ΠΈ: ΠΏΡ€ΠΎΠ²Π΅Ρ€ΠΈΡ‚ΡŒ усвоСниС Π½ΡƒΠΌΠ΅Ρ€Π°Ρ†ΠΈΠΈ Ρ‚Ρ€Ρ‘Ρ…Π·Π½Π°Ρ‡Π½Ρ‹Ρ… чисСл; ΠΏΡ€ΠΎΒ­Π²Π΅Ρ€ΠΈΡ‚ΡŒ усвоСниС Π²Ρ‹Ρ‡ΠΈΡΠ»ΠΈΡ‚Π΅Π»ΡŒΠ½Ρ‹Ρ… ΠΏΡ€ΠΈΡ‘ΠΌΠΎΠ²; Ρ€Π΅ΡˆΠ΅Π½ΠΈΠ΅ Π·Π°Π΄Π°Ρ‡; по­строСниС ΠΎΡ‚Ρ€Π΅Π·ΠΊΠΎΠ².

    Π’Π°Ρ€ΠΈΠ°Π½Ρ‚ I

    1. Π°) Π’ΡΡ‚Π°Π²ΡŒ Π² окошки ΠΏΡ€ΠΎΠΏΡƒΡ‰Π΅Π½Π½Ρ‹Π΅ числа. 387,388,βͺ,βͺ,βͺ,392,βͺ,βͺ.

    Π±) Π—Π°ΠΏΠΈΡˆΠΈ Ρ†ΠΈΡ„Ρ€Π°ΠΌΠΈ:

    9 сот. 2 дСс. 8 сот.

    4 сот. 3 Π΅Π΄. 5 сот. 1 дСс. 7 Π΅Π΄.

    Π²) ΠŸΡ€Π΅Π΄ΡΡ‚Π°Π²ΡŒ числа Π² Π²ΠΈΠ΄Π΅ суммы разрядных слагаСмых. 510,742.

    2. РСши Π·Π°Π΄Π°Ρ‡Ρƒ.

    3 Π½Π°Π±ΠΎΡ€Π° красок стоят 72 рубля. Бколько стоят 4 Ρ‚Π°ΠΊΠΈΡ… Π½Π°Π±ΠΎΡ€Π° красок?

    3. РСши ΠΏΡ€ΠΈΠΌΠ΅Ρ€Ρ‹.

    549 + 1 702 – 700 60 : (26 + 4) βˆ™2

    800 – 1 930 – 600 42 + 54 : 3 βˆ™ 2

    600 + 50 320 + 70 71 – (28 – 17) βˆ™ 6

    4. Π—Π°ΠΏΠΎΠ»Π½ΠΈ пропуски Π½ΡƒΠΆΠ½Ρ‹ΠΌΠΈ числами.

    85 : 9 = 9 (ост. βͺ) 73 : βͺ= 9 (ост. 1)

    1. 6 = β–‘ (ост. 2) β–‘ : 8 = 7 (ост. 3)

    5.НачСрти ΠΎΡ‚Ρ€Π΅Π·ΠΎΠΊ АВ = 6 см, Π° ΠΎΡ‚Ρ€Π΅Π·ΠΎΠΊ Π’Π‘ Π² 2 Ρ€Π°Π·Π° Π΄Π»ΠΈΠ½Π½Π΅Π΅. На сколько см Π΄Π»ΠΈΠ½Π° ΠΎΡ‚Ρ€Π΅Π·ΠΊΠ° Π’Π‘ большС Π΄Π»ΠΈΠ½Ρ‹ ΠΎΡ‚Ρ€Π΅Π·ΠΊΠ° АВ?

    Π’Π°Ρ€ΠΈΠ°Π½Ρ‚ II

    1. Π°) Π’ΡΡ‚Π°Π²ΡŒ Π² окошки ΠΏΡ€ΠΎΠΏΡƒΡ‰Π΅Π½Π½Ρ‹Π΅ числа. 588, βͺ,βͺ,βͺ, 592, β–‘ , 594,βͺ

    Π±) Π—Π°ΠΏΠΈΡˆΠΈ Ρ†ΠΈΡ„Ρ€Π°ΠΌΠΈ:

    7 сот. 5 сот 7 дСс.

    4 сот. 2 дСс. 3 Π΅Π΄. 6 сот. 9 Π΅Π΄.

    Π²) ΠŸΡ€Π΅Π΄ΡΡ‚Π°Π²ΡŒ числа Π² Π²ΠΈΠ΄Π΅ суммы разрядных слагаСмых. 490,351.

    2. РСши Π·Π°Π΄Π°Ρ‡Ρƒ.

    72 стула расставили Π² 6 рядов, ΠΏΠΎΡ€ΠΎΠ²Π½Ρƒ Π² ΠΊΠ°ΠΆΠ΄Ρ‹ΠΉ ряд. Бколько ΡƒΠ»ΡŒΠ΅Π² Π² 4 Ρ‚Π°ΠΊΠΈΡ… рядах?

    3. РСши ΠΏΡ€ΠΈΠΌΠ΅Ρ€Ρ‹.

    659 + 1 805 – 800 96 : (41 – 9) βˆ™ 3

    900 – 1 760 – 500 36 + 60 : 4 βˆ™ 2

    400 + 80 980 – 30 72 – (44 +16) : 4

    4. Π—Π°ΠΏΠΎΠ»Π½ΠΈ пропуски Π½ΡƒΠΆΠ½Ρ‹ΠΌΠΈ числами.

    74 : 9 = 8 (ост.βͺ) 59 : β–‘ = 7 (ост. 3)

    68 : 8 = β–‘ (ост. 4) β–‘ : 7 = 6 (ост. 1)

    1. НачСрти ΠΎΡ‚Ρ€Π΅Π·ΠΎΠΊ Π‘Π” = 9 см, Π° ΠΎΡ‚Ρ€Π΅Π·ΠΎΠΊ АК Π² 3 Ρ€Π°Π·Π° ΠΊΠΎΡ€ΠΎΡ‡Π΅. На сколько см Π΄Π»ΠΈΠ½Π° ΠΎΡ‚Ρ€Π΅Π·ΠΊΠ° Π‘Π” большС Π΄Π»ΠΈΠ½Ρ‹ ΠΎΡ‚Ρ€Π΅Π·ΠΊΠ° АК?

    Π£Ρ€ΠΎΠΊ 138

    ΠšΠžΠΠ’Π ΠžΠ›Π¬ΠΠΠ― Π ΠΠ‘ΠžΠ’Π

    Π¦Π΅Π»ΠΈ: ΠΏΡ€ΠΎΠ²Π΅Ρ€ΠΈΡ‚ΡŒ ΡΡ„ΠΎΡ€ΠΌΠΈΡ€ΠΎΠ²Π°Π½Π½ΠΎΡΡ‚ΡŒ ΡƒΠΌΠ΅Π½ΠΈΠΉ Π·Π°ΠΏΠΈΡΡ‹Π²Π°Ρ‚ΡŒ ΠΈ ΡΡ€Π°Π²Π½ΠΈΠ²Π°Ρ‚ΡŒ Ρ‚Ρ€Ρ‘Ρ…Π·Π½Π°Ρ‡Π½Ρ‹Π΅ числа; ΡΠΊΠ»Π°Π΄Ρ‹Π²Π°Ρ‚ΡŒ ΠΈ Π²Ρ‹Ρ‡ΠΈΡ‚Π°Ρ‚ΡŒ Ρ‚Ρ€Ρ‘Ρ…Π·Π½Π°Ρ‡Π½Ρ‹Π΅ числа; Ρ€Π΅ΡˆΠ°Ρ‚ΡŒ Π·Π°Π΄Π°Ρ‡ΠΈ; ΡΡ€Π°Π²Π½ΠΈΠ²Π°Ρ‚ΡŒ Π΅Π΄ΠΈΠ½ΠΈΡ†Ρ‹ Π΄Π»ΠΈΠ½Ρ‹.

    1. Π°) Π—Π°ΠΏΠΈΡˆΠΈ число, Π² ΠΊΠΎΡ‚ΠΎΡ€ΠΎΠΌ:

    5 Π΅Π΄. III разряда ΠΈ 2 Π΅Π΄. I разряда;

    7 Π΅Π΄. II разряда ΠΈ 3 Π΅Π΄. I разряда;

    4 Π΅Π΄. III разряда;

    6 Π΅Π΄. III разряда ΠΈ 9 Π΅Π΄. II разряда.

    Π±) Π‘Ρ€Π°Π²Π½ΠΈ числа:

    567…601; 300…299; 110…101

    2. Π‘Ρ€Π°Π²Π½ΠΈ (>,

    5 ΠΌ 4 Π΄ΠΌ … 540 см 8 ΠΌ 1 см…8ΠΌ 1 Π΄ΠΌ

    9 Π΄ΠΌ 2 см … 1ΠΌ 604 см …4ΠΌ 6 см

    3. РСши ΠΏΡ€ΠΈΠΌΠ΅Ρ€Ρ‹.

    640 : 8 = 240 : 6 =

    150-70= 560:8 =

    440: 20 = 80 βˆ™ 5 =

    80 + 40 = 20 βˆ™ 6 =

    4. РСши Π·Π°Π΄Π°Ρ‡Ρƒ.

    Π’ сорСвнованиях ΡƒΡ‡Π°ΡΡ‚Π²ΡƒΡŽΡ‚ 310 ΠΌΠ°Π»ΡŒΡ‡ΠΈΠΊΠΎΠ² ΠΈ 270 Π΄Π΅Π²ΠΎΡ‡Π΅ΠΊ. Гим­настов срСди Π½ΠΈΡ… Π±Ρ‹Π»ΠΎ 250 Ρ‡Π΅Π»ΠΎΠ²Π΅ΠΊ, Π° ΠΎΡΡ‚Π°Π»ΡŒΠ½Ρ‹Π΅ ΠΏΠ»ΠΎΠ²Ρ†Ρ‹. Бколько ΠΏΠ»ΠΎΠ²Ρ†ΠΎΠ² ΡƒΡ‡Π°ΡΡ‚Π²ΡƒΡŽΡ‚ Π² сорСвнованиях?

    5. РСши ΠΏΡ€ΠΈΠΌΠ΅Ρ€Ρ‹ столбиком.

    535 + 65 756-628

    198 + 672 963-817

    189 + 467 707-629

    Π£Ρ€ΠΎΠΊ 162

    Π˜Π’ΠžΠ“ΠžΠ’ΠΠ― ΠšΠžΠΠ’Π ΠžΠ›Π¬ΠΠΠ― Π ΠΠ‘ΠžΠ’Π

    Π¦Π΅Π»ΠΈ: ΠΏΡ€ΠΎΠ²Π΅Ρ€ΠΈΡ‚ΡŒ усвоСниС Π²Ρ‹Ρ‡ΠΈΡΠ»ΠΈΡ‚Π΅Π»ΡŒΠ½Ρ‹Ρ… ΠΏΡ€ΠΈΡ‘ΠΌΠΎΠ² слоТСния, вычитания, умноТСния ΠΈ дСлСния Π² ΠΏΡ€Π΅Π΄Π΅Π»Π°Ρ… 1000; ΠΏΡ€Π°Π²ΠΈΠ», порядка выполнСния дСйствий Π² выраТСниях; Π΅Π΄ΠΈΠ½ΠΈΡ† Π΄Π»ΠΈΠ½Ρ‹, ΠΏΠ»ΠΎΡ‰Π°Π΄ΠΈ; умС­ния Ρ€Π΅ΡˆΠ°Ρ‚ΡŒ Π·Π°Π΄Π°Ρ‡ΠΈ.

    1. Π Π΅ΡˆΠΈΡ‚Π΅ Π·Π°Π΄Π°Ρ‡Ρƒ.

    Π‘ ΠΎΠ΄Π½ΠΎΠ³ΠΎ участка школьники собрали 160 ΠΊΠ³ ΠΌΠΎΡ€ΠΊΠΎΠ²ΠΈ, Π° с Π΄Ρ€ΡƒΒ­Π³ΠΎΠ³ΠΎ – Π² 2 Ρ€Π°Π·Π° большС. Π§Π΅Ρ‚Π²Ρ‘Ρ€Ρ‚ΡƒΡŽ Ρ‡Π°ΡΡ‚ΡŒ всСй ΠΌΠΎΡ€ΠΊΠΎΠ²ΠΈ ΠΎΠ½ΠΈ израсхо­довали Π½Π° ΠΊΠΎΡ€ΠΌ ΠΊΡ€ΠΎΠ»ΠΈΠΊΠ°ΠΌ. Бколько ΠΊΠ³ ΠΌΠΎΡ€ΠΊΠΎΠ²ΠΈ израсходовали Π½Π° ΠΊΠΎΡ€ΠΌ ΠΊΡ€ΠΎΠ»ΠΈΠΊΠ°ΠΌ?

    2. НайдитС Π·Π½Π°Ρ‡Π΅Π½ΠΈΠ΅ Π²Ρ‹Ρ€Π°ΠΆΠ΅Π½ΠΈΠΉ.

    18 + 36:9 + 6-8-50

    400 -(80 +180:3) + 60

    3. Π Π΅ΡˆΠΈΡ‚Π΅ ΠΏΡ€ΠΈΠΌΠ΅Ρ€Ρ‹ столбиком.

    138+567

    447–189

    152 βˆ™ 6

    867 : 3

    4. ΠŸΠ΅Ρ€Π΅Π²Π΅Π΄ΠΈΡ‚Π΅.

    125 см = … ΠΌ … Π΄ΠΌ … см

    847 Π΄ΠΌ = … ΠΌ … Π΄ΠΌ

    7ΠΌ 3см = …м…дм

    700 см2 = … Π΄ΠΌ2

    5. НачСртитС ΠΏΡ€ΡΠΌΠΎΡƒΠ³ΠΎΠ»ΡŒΠ½ΠΈΠΊ со сторонами 5 см ΠΈ 3 см. НайдитС Π΅Π³ΠΎ ΠΏΠ»ΠΎΡ‰Π°Π΄ΡŒ ΠΈ ΠΏΠ΅Ρ€ΠΈΠΌΠ΅Ρ‚Ρ€.

    ПНШ 3 класс. ΠœΠ°Ρ‚Π΅ΠΌΠ°Ρ‚ΠΈΠΊΠ°. Π£Ρ‡Π΅Π±Π½ΠΈΠΊ β„– 1, с. 142

    БоставныС Π·Π°Π΄Π°Ρ‡ΠΈ Π½Π° всС дСйствия

    ΠžΡ‚Π²Π΅Ρ‚Ρ‹ ΠΊ с. 142

    470.Β Π‘ΠΎΡΡ‚Π°Π²ΡŒ Π·Π°Π΄Π°Ρ‡Ρƒ Π½Π° ΠΊΡ€Π°Ρ‚Π½ΠΎΠ΅ сравнСниС, ΠΏΠ΅Ρ€Π²Ρ‹ΠΌ дСйствиСм Ρ€Π΅ΡˆΠ΅Π½ΠΈΡ ΠΊΠΎΡ‚ΠΎΡ€ΠΎΠΉ Π±ΡƒΠ΄Π΅Ρ‚ дСйствиС: Π°) слоТСния; Π±) вычитания.

    Π°) Π’ ΠΌΠ°Π³Π°Π·ΠΈΠ½ ΠΏΡ€ΠΈΠ²Π΅Π·Π»ΠΈ 10 Π±Π°Ρ‚ΠΎΠ½ΠΎΠ², Π° Ρ…Π»Π΅Π±Π° Π½Π° 40 Π±ΡƒΡ…Π°Π½ΠΎΠΊ большС. Π’ΠΎ сколько Ρ€Π°Π· большС ΠΏΡ€ΠΈΠ²Π΅Π·Π»ΠΈ Ρ…Π»Π΅Π±Π°, Ρ‡Π΅ΠΌ Π±Π°Ρ‚ΠΎΠ½ΠΎΠ²?
    1. 10 + 40 = 50 (Π±.) – ΠΏΡ€ΠΈΠ²Π΅Π·Π»ΠΈ Ρ…Π»Π΅Π±Π°
    2. 50 : 10 = 5
    О Ρ‚ Π² Π΅ Ρ‚: Ρ…Π»Π΅Π±Π° ΠΏΡ€ΠΈΠ²Π΅Π·Π»ΠΈ Π² 5 Ρ€Π°Π· большС.

    Π±) Π’ ΠΌΠ°Π³Π°Π·ΠΈΠ½ ΠΏΡ€ΠΈΠ²Π΅Π·Π»ΠΈ 50 Π±ΡƒΡ…Π°Π½ΠΎΠΊ Ρ…Π»Π΅Π±Π°, Π° Π±Π°Ρ‚ΠΎΠ½ΠΎΠ² Π½Π° 40 ΡˆΡ‚ΡƒΠΊ мСньшС. Π’ΠΎ сколько Ρ€Π°Π· мСньшС ΠΏΡ€ΠΈΠ²Π΅Π·Π»ΠΈ Π±Π°Ρ‚ΠΎΠ½ΠΎΠ², Ρ‡Π΅ΠΌ Ρ…Π»Π΅Π±Π°?
    1. 50 – 40 = 10 (Π±.) – ΠΏΡ€ΠΈΠ²Π΅Π·Π»ΠΈ Π±Π°Ρ‚ΠΎΠ½ΠΎΠ²
    2. 50 : 10 = 5
    О Ρ‚ Π² Π΅ Ρ‚: Π±Π°Ρ‚ΠΎΠ½ΠΎΠ² ΠΏΡ€ΠΈΠ²Π΅Π·Π»ΠΈ Π² 5 Ρ€Π°Π· мСньшС.

    471. Π‘ΠΎΡΡ‚Π°Π²ΡŒ Π·Π°Π΄Π°Ρ‡Ρƒ Π½Π° разностноС сравнСниС, ΠΏΠ΅Ρ€Π²Ρ‹ΠΌ дСйствиСм Ρ€Π΅ΡˆΠ΅Π½ΠΈΡ ΠΊΠΎΡ‚ΠΎΡ€ΠΎΠΉ Π±ΡƒΠ΄Π΅Ρ‚ дСйствиС: Π°) умноТСния; Π±) дСлСния.

    Π°) Π’ ΠΌΠ°Π³Π°Π·ΠΈΠ½ ΠΏΡ€ΠΈΠ²Π΅Π·Π»ΠΈ 10 Π±Π°Ρ‚ΠΎΠ½ΠΎΠ², Π° Ρ…Π»Π΅Π±Π° Π² 5 Ρ€Π°Π· большС. На сколько большС ΠΏΡ€ΠΈΠ²Π΅Π·Π»ΠΈ Π±ΡƒΡ…Π°Π½ΠΎΠΊ Ρ…Π»Π΅Π±Π°, Ρ‡Π΅ΠΌ Π±Π°Ρ‚ΠΎΠ½ΠΎΠ²?
    1. 10Β β€’ 5 = 50 (Π±.) – ΠΏΡ€ΠΈΠ²Π΅Π·Π»ΠΈ Ρ…Π»Π΅Π±Π°
    2. 50 – 10 = 40 (Π±.)
    О Ρ‚ Π² Π΅ Ρ‚: Ρ…Π»Π΅Π±Π° ΠΏΡ€ΠΈΠ²Π΅Π·Π»ΠΈ Π½Π° 40 Π±ΡƒΡ…Π°Π½ΠΎΠΊ большС.

    Π±) Π’ ΠΌΠ°Π³Π°Π·ΠΈΠ½ ΠΏΡ€ΠΈΠ²Π΅Π·Π»ΠΈ 50 Π±ΡƒΡ…Π°Π½ΠΎΠΊ Ρ…Π»Π΅Π±Π°, Π° Π±Π°Ρ‚ΠΎΠ½ΠΎΠ² Π² 5 Ρ€Π°Π· мСньшС. На сколько большС ΠΏΡ€ΠΈΠ²Π΅Π·Π»ΠΈ Π±ΡƒΡ…Π°Π½ΠΎΠΊ Ρ…Π»Π΅Π±Π°, Ρ‡Π΅ΠΌ Π±Π°Ρ‚ΠΎΠ½ΠΎΠ²?
    1. 50 : 5 = 10 (Π±.) – ΠΏΡ€ΠΈΠ²Π΅Π·Π»ΠΈ Π±Π°Ρ‚ΠΎΠ½ΠΎΠ²
    2. 50 – 10 = 40 (Π±.)
    О Ρ‚ Π² Π΅ Ρ‚: Ρ…Π»Π΅Π±Π° ΠΏΡ€ΠΈΠ²Π΅Π·Π»ΠΈ Π½Π° 40 Π±ΡƒΡ…Π°Π½ΠΎΠΊ большС.

    472. Π˜Π·Π²Π΅ΡΡ‚Π½ΠΎ Ρ‚Ρ€Π΅Π±ΠΎΠ²Π°Π½ΠΈΠ΅ составной Π·Π°Π΄Π°Ρ‡ΠΈ, ΠΊΠΎΡ‚ΠΎΡ€ΠΎΠ΅ формулируСтся ΡΠ»Π΅Π΄ΡƒΡŽΡ‰ΠΈΠΌ ΠΎΠ±Ρ€Π°Π·ΠΎΠΌ: «Бколько саТСнцСв яблонь ΠΈ Π³Ρ€ΡƒΡˆ посадили учащиСся?Β»
    Из Π΄Π°Π½Π½ΠΎΠ³ΠΎ списка Π²Ρ‹Π±Π΅Ρ€ΠΈ ΠΈ Π½Π°Π·ΠΎΠ²ΠΈ трСбования, ΠΊΠΎΡ‚ΠΎΡ€Ρ‹Π΅ ΠΌΠΎΠ³ΡƒΡ‚ ΠΈΠ³Ρ€Π°Ρ‚ΡŒ Ρ€ΠΎΠ»ΡŒ ΠΏΡ€ΠΎΠΌΠ΅ΠΆΡƒΡ‚ΠΎΡ‡Π½Ρ‹Ρ… Π΄ΠΎΠΏΠΎΠ»Π½ΠΈΡ‚Π΅Π»ΡŒΠ½Ρ‹Ρ… Ρ‚Ρ€Π΅Π±ΠΎΠ²Π°Π½ΠΈΠΉ.
    1. На сколько большС посадили учащиСся саТСнцСв яблонь, Ρ‡Π΅ΠΌ Π³Ρ€ΡƒΡˆ?
    2. Бколько саТСнцСв яблонь посадили учащиСся?
    3. Π’ΠΎ сколько Ρ€Π°Π· мСньшС посадили учащиСся саТСнцСв Π³Ρ€ΡƒΡˆ, Ρ‡Π΅ΠΌ яблонь?
    4. Бколько саТСнцСв Π³Ρ€ΡƒΡˆ посадили учащиСся?
    КакоС ΠΈΠ· Π²Ρ‹Π±Ρ€Π°Π½Π½Ρ‹Ρ… Ρ‚ΠΎΠ±ΠΎΠΉ Ρ‚Ρ€Π΅Π±ΠΎΠ²Π°Π½ΠΈΠΉ Π½ΡƒΠΆΠ½ΠΎ ΠΎΡΡ‚Π°Π²ΠΈΡ‚ΡŒ, Ссли условиС Π΄Π°Π½Π½ΠΎΠΉ Π·Π°Π΄Π°Ρ‡ΠΈ формулируСтся ΡΠ»Π΅Π΄ΡƒΡŽΡ‰ΠΈΠΌ ΠΎΠ±Ρ€Π°Π·ΠΎΠΌ: «УчащиСся посадили 15 саТСнцСв Π³Ρ€ΡƒΡˆ, Π° саТСнцСв яблонь – Π² 4 Ρ€Π°Π·Π° большС»?

    ΠŸΡ€ΠΎΠΌΠ΅ΠΆΡƒΡ‚ΠΎΡ‡Π½Ρ‹ΠΌΠΈ трСбованиями ΠΌΠΎΠ³ΡƒΡ‚ Π±Ρ‹Ρ‚ΡŒ трСбования 2 ΠΈ 4.
    2. Бколько саТСнцСв яблонь посадили учащиСся?

    ΠžΡ‚Π²Π΅Ρ‚Ρ‹ ΠΊ заданиям. ΠœΠ°Ρ‚Π΅ΠΌΠ°Ρ‚ΠΈΠΊΠ°. Π£Ρ‡Π΅Π±Π½ΠΈΠΊ. Π§Π°ΡΡ‚ΡŒ 1. Π§Π΅ΠΊΠΈΠ½ А.Π›. 2013 Π³.

    ΠœΠ°Ρ‚Π΅ΠΌΠ°Ρ‚ΠΈΠΊΠ°. 3 класс. Π§Π΅ΠΊΠΈΠ½ А.Π›.

    4.8 / 5 ( 16 голосов )

    бСсплатных ΠΈΠ½Ρ‚Π΅Ρ€Π²Π΅Π½Ρ†ΠΈΠΎΠ½Π½Ρ‹Ρ… Π·Π°Π΄Π°Π½ΠΈΠΉ ΠΏΠΎ ΠΌΠ°Ρ‚Π΅ΠΌΠ°Ρ‚ΠΈΠΊΠ΅ для 3–5 классов

    Иногда Ρƒ Π½Π°ΡˆΠΈΡ… ΡƒΡ‡Π΅Π½ΠΈΠΊΠΎΠ² 4 ΠΈ 5 классов Π²ΠΎΠ·Π½ΠΈΠΊΠ°ΡŽΡ‚ ΠΏΡ€ΠΎΠ±Π΅Π»Ρ‹ Π² ΠΎΠ±ΡƒΡ‡Π΅Π½ΠΈΠΈ ΠΈΠ»ΠΈ ΠΈΠΌ Π½ΡƒΠΆΠ½ΠΎ ΠΏΠΎΡ€Π°Π±ΠΎΡ‚Π°Ρ‚ΡŒ Π½Π°Π΄ своСй Π±Π΅Π³Π»ΠΎΡΡ‚ΡŒΡŽ ΠΌΠ°Ρ‚Π΅ΠΌΠ°Ρ‚ΠΈΠΊΠΈ. Π­Ρ‚ΠΈ ΠΏΡ€ΠΎΠ±Π΅Π»Ρ‹ часто ΠΌΠΎΠ³ΡƒΡ‚ ΡΠ΄Π΅Ρ€ΠΆΠΈΠ²Π°Ρ‚ΡŒ ΠΈΡ… с ΠΏΠΎΠΌΠΎΡ‰ΡŒΡŽ Π½Π°Π²Ρ‹ΠΊΠΎΠ² Π½Π° ΡƒΡ€ΠΎΠ²Π½Π΅ класса. Π’ этом постС Π²Ρ‹ Π½Π°ΠΉΠ΄Π΅Ρ‚Π΅ нСсколько бСсплатных ΡƒΠΏΡ€Π°ΠΆΠ½Π΅Π½ΠΈΠΉ ΠΏΠΎ ΠΌΠ°Ρ‚Π΅ΠΌΠ°Ρ‚ΠΈΠΊΠ΅ (ΠΈ я Π±ΡƒΠ΄Ρƒ Π΄ΠΎΠ±Π°Π²Π»ΡΡ‚ΡŒ большС Π² Ρ‚Π΅Ρ‡Π΅Π½ΠΈΠ΅ Π³ΠΎΠ΄Π°), Ρ‡Ρ‚ΠΎΠ±Ρ‹ ΠΏΡ€Π΅Π΄ΠΎΡΡ‚Π°Π²ΠΈΡ‚ΡŒ Π΄ΠΎΠΏΠΎΠ»Π½ΠΈΡ‚Π΅Π»ΡŒΠ½Ρ‹Π΅ инструкции ΠΈ ΠΏΠΎΠΌΠΎΡ‰ΡŒ вашим ΡƒΡ‡Π΅Π½ΠΈΠΊΠ°ΠΌ с этими ΠΏΡ€ΠΎΠ±Π΅Π»Π°ΠΌΠΈ ΠΈΠ»ΠΈ Ρ‚Π΅ΠΌ, ΠΊΠΎΡ‚ΠΎΡ€Ρ‹ΠΌ просто Π½ΡƒΠΆΠ½ΠΎ большС ΠΏΡ€Π°ΠΊΡ‚ΠΈΠΊΠΈ ΠΈΠ»ΠΈ Π»ΡƒΡ‡ΡˆΠ΅ Π²Π»Π°Π΄Π΅Ρ‚ΡŒ ΠΎΠΏΡ€Π΅Π΄Π΅Π»Π΅Π½Π½Ρ‹ΠΌΠΈ матСматичСскими Π½Π°Π²Ρ‹ΠΊΠ°ΠΌΠΈ.

    ΠŸΡ€ΠΈΠΌΠ΅Ρ‡Π°Π½ΠΈΠ΅: я Π±ΡƒΠ΄Ρƒ Π΄ΠΎΠ±Π°Π²Π»ΡΡ‚ΡŒ Π² этот пост ΠΏΠΎ ΠΌΠ΅Ρ€Π΅ создания Π½ΠΎΠ²Ρ‹Ρ… ΡƒΠΏΡ€Π°ΠΆΠ½Π΅Π½ΠΈΠΉ ΠΏΠΎ ΠΌΠ°Ρ‚Π΅ΠΌΠ°Ρ‚ΠΈΠΊΠ΅ для использования с ΠΌΠΎΠΈΠΌΠΈ ΡƒΡ‡Π΅Π½ΠΈΠΊΠ°ΠΌΠΈ.

    ΠœΠ°Ρ‚Π΅ΠΌΠ°Ρ‚ΠΈΡ‡Π΅ΡΠΊΠΈΠ΅ Π½Π°Π²Ρ‹ΠΊΠΈ, ΠΊΠΎΡ‚ΠΎΡ€Ρ‹Π΅ я добавляю Π² этот пост, скорСС всСго, относятся ΠΊ Π΄ΠΈΠ°ΠΏΠ°Π·ΠΎΠ½Ρƒ 1-3 классов. Однако ΠΎΠ½ΠΈ Π±ΡƒΠ΄ΡƒΡ‚ прСдставлСны Π² Π±ΠΎΠ»Π΅Π΅ ΠΏΡ€ΠΈΠ΅ΠΌΠ»Π΅ΠΌΠΎΠΉ для учащихся 3-5-Ρ… классов Ρ„ΠΎΡ€ΠΌΠ΅. ΠšΡ€ΠΎΠΌΠ΅ Ρ‚ΠΎΠ³ΠΎ, я ΠΏΠΎΡΡ‚Π°Ρ€Π°ΡŽΡΡŒ, Ρ‡Ρ‚ΠΎΠ±Ρ‹ ΠΈΡ… ΠΏΠΎΠ΄Π³ΠΎΡ‚ΠΎΠ²ΠΊΠ° Π±Ρ‹Π»Π° минимальной.

    Π‘Π²ΠΎΠ±ΠΎΠ΄Π½ΠΎΠ΅ Π²Π»Π°Π΄Π΅Π½ΠΈΠ΅ языком

    БСсплатныС упраТнСния ΠΏΠΎ ΠΌΠ°Ρ‚Π΅ΠΌΠ°Ρ‚ΠΈΠΊΠ΅ для бСглости слоТСния ΠΈ вычитания! Π’ΠΎΡΠΏΠΎΠ»ΡŒΠ·ΡƒΠΉΡ‚Π΅ΡΡŒ этими бСсплатными Π±Π°Π·ΠΎΠ²Ρ‹ΠΌΠΈ упраТнСниями Π½Π° слоТСниС ΠΈ Π²Ρ‹Ρ‡ΠΈΡ‚Π°Π½ΠΈΠ΅, Ρ‡Ρ‚ΠΎΠ±Ρ‹ ΠΏΠΎΠΌΠΎΡ‡ΡŒ своим ΡƒΡ‡Π΅Π½ΠΈΠΊΠ°ΠΌ с умствСнной ΠΌΠ°Ρ‚Π΅ΠΌΠ°Ρ‚ΠΈΠΊΠΎΠΉ ΠΈ ΠΏΠΎΠΌΠΎΡ‡ΡŒ со встроСнными Π½Π°Π²Ρ‹ΠΊΠ°ΠΌΠΈ слоТСния ΠΈ вычитания, ΠΊΠΎΡ‚ΠΎΡ€Ρ‹Π΅ ΠΌΠ΅ΡˆΠ°ΡŽΡ‚ ΠΈΠΌ Π²Ρ‹ΠΏΠΎΠ»Π½ΡΡ‚ΡŒ Ρ€Π°Π±ΠΎΡ‚Ρƒ Π½Π° ΡƒΡ€ΠΎΠ²Π½Π΅ своСго класса.

    ВсСго Π½Π° этом рСсурсС 19 ΠΈΠ³Ρ€ Π² крСстики-Π½ΠΎΠ»ΠΈΠΊΠΈ. Π˜ΡΠΏΠΎΠ»ΡŒΠ·ΡƒΠΉΡ‚Π΅ ΠΎΠ³Π»Π°Π²Π»Π΅Π½ΠΈΠ΅, Ρ‡Ρ‚ΠΎΠ±Ρ‹ Π²Ρ‹Π±Ρ€Π°Ρ‚ΡŒ Ρ‚Π΅, ΠΊΠΎΡ‚ΠΎΡ€Ρ‹Π΅ Π»ΡƒΡ‡ΡˆΠ΅ всСго подходят вашим ΡƒΡ‡Π΅Π½ΠΈΠΊΠ°ΠΌ. НСкоторыС ΠΈΠ· ΠΈΠ³Ρ€ Π»ΡƒΡ‡ΡˆΠ΅ всСго подходят для Ρ‚Ρ€Π΅Π½ΠΈΡ€ΠΎΠ²ΠΊΠΈ бСглости мыслСнной ΠΌΠ°Ρ‚Π΅ΠΌΠ°Ρ‚ΠΈΠΊΠΈ, Π° Π½Π΅ΠΊΠΎΡ‚ΠΎΡ€Ρ‹Π΅ идСально подходят для Ρ‚ΠΎΠ³ΠΎ, Ρ‡Ρ‚ΠΎΠ±Ρ‹ ΠΏΠΎΠΌΠΎΡ‡ΡŒ учащимся ΠΎΠ²Π»Π°Π΄Π΅Ρ‚ΡŒ Π½Π°Π²Ρ‹ΠΊΠ°ΠΌΠΈ слоТСния ΠΈ вычитания, ΠΏΠΎΠΊΠ° ΠΎΠ½ΠΈ Ρ€Π°Π±ΠΎΡ‚Π°ΡŽΡ‚ Π½Π°Π΄ матСматичСскими Π½Π°Π²Ρ‹ΠΊΠ°ΠΌΠΈ Π½Π° ΡƒΡ€ΠΎΠ²Π½Π΅ своСго класса (Π½Π°ΠΏΡ€ΠΈΠΌΠ΅Ρ€, с ΠΌΠ½ΠΎΠ³ΠΎΠ·Π½Π°Ρ‡Π½Ρ‹ΠΌ ΡƒΠΌΠ½ΠΎΠΆΠ΅Π½ΠΈΠ΅ΠΌ).

    *** НаТмитС здСсь, Ρ‡Ρ‚ΠΎΠ±Ρ‹ ΡΠΊΠ°Ρ‡Π°Ρ‚ΡŒ ΠΈΠ³Ρ€Ρ‹ Π² крСстики-Π½ΠΎΠ»ΠΈΠΊΠΈ Π½Π° слоТСниС ΠΈ Π²Ρ‹Ρ‡ΠΈΡ‚Π°Π½ΠΈΠ΅. ΠžΠ±ΡΠ·Π°Ρ‚Π΅Π»ΡŒΠ½ΠΎ ΠΎΠ·Π½Π°ΠΊΠΎΠΌΡŒΡ‚Π΅ΡΡŒ с ΠΎΠ³Π»Π°Π²Π»Π΅Π½ΠΈΠ΅ΠΌ, Π² ΠΊΠΎΡ‚ΠΎΡ€ΠΎΠΌ ΡƒΠΊΠ°Π·Π°Π½Ρ‹ ΠΊΠΎΠ½ΠΊΡ€Π΅Ρ‚Π½Ρ‹Π΅ Π½Π°Π²Ρ‹ΠΊΠΈ, ΠΊΠΎΡ‚ΠΎΡ€Ρ‹Π΅ Π½ΡƒΠΆΠ½Ρ‹ вашим ΡƒΡ‡Π΅Π½ΠΈΠΊΠ°ΠΌ, ΠΈ Π·Π°ΠΌΠ΅Ρ‚ΠΊΠΈ ΠΎ Ρ‚ΠΎΠΌ, для Ρ‡Π΅Π³ΠΎ ΠΎΠ½ΠΈ Π»ΡƒΡ‡ΡˆΠ΅ всСго Ρ€Π°Π±ΠΎΡ‚Π°ΡŽΡ‚ (ΠΌΠ°Ρ‚Π΅ΠΌΠ°Ρ‚ΠΈΠΊΠ° Π² ΡƒΠΌΠ΅ ΠΈΠ»ΠΈ ΠΏΠΎΠΌΠΎΡ‰ΡŒ с ΠΌΠ½ΠΎΠ³ΠΎΠ·Π½Π°Ρ‡Π½Ρ‹ΠΌΠΈ опСрациями).

    Π‘Ρ‚Ρ€Π°Ρ‚Π΅Π³ΠΈΠΈ умноТСния ΠΈ дСлСния

    Когда ΡƒΡ‡Π΅Π½ΠΈΠΊΠΈ Ρ€Π°Π±ΠΎΡ‚Π°ΡŽΡ‚ Π½Π°Π΄ Π·Π°ΠΏΠΎΠΌΠΈΠ½Π°Π½ΠΈΠ΅ΠΌ Ρ„Π°ΠΊΡ‚ΠΎΠ² умноТСния ΠΈ дСлСния, ΠΈΠΌ ΠΏΠΎ-ΠΏΡ€Π΅ΠΆΠ½Π΅ΠΌΡƒ ΠΎΡ‡Π΅Π½ΡŒ Π½ΡƒΠΆΠ½Ρ‹ стратСгии, ΠΊΠΎΡ‚ΠΎΡ€Ρ‹Π΅ ΠΏΠΎΠΌΠΎΠ³ΡƒΡ‚ ΠΈΠΌ ΠΏΠΎΠ»ΡƒΡ‡ΠΈΡ‚ΡŒ ΠΎΡ‚Π²Π΅Ρ‚. Π­Ρ‚ΠΎ распСчатанноС мСроприятиС ΠΏΠΎ Π²ΠΌΠ΅ΡˆΠ°Ρ‚Π΅Π»ΡŒΡΡ‚Π²Ρƒ ΠΏΠΎΠ΄Ρ…ΠΎΠ΄ΠΈΡ‚ для этого.

    На этих Ρ†ΠΈΠ½ΠΎΠ²ΠΊΠ°Ρ… ΡƒΡ‡Π΅Π½ΠΈΠΊΠ°ΠΌ прСдлагаСтся Ρ€Π΅ΡˆΠΈΡ‚ΡŒ Π·Π°Π΄Π°Ρ‡Ρƒ умноТСния ΠΈΠ»ΠΈ дСлСния, ΠΈΡΠΏΠΎΠ»ΡŒΠ·ΡƒΡ Ρ‡Π΅Ρ‚Ρ‹Ρ€Π΅ Ρ€Π°Π·Π½Ρ‹Π΅ стратСгии. Π’ зависимости ΠΎΡ‚ потрСбностСй Π²Π°ΡˆΠΈΡ… ΡƒΡ‡Π΅Π½ΠΈΠΊΠΎΠ² Ρƒ вас Π΅ΡΡ‚ΡŒ нСсколько Π²Π°Ρ€ΠΈΠ°Π½Ρ‚ΠΎΠ² Ρ‚ΠΎΠ³ΠΎ, ΠΊΠ°ΠΊ Π²Ρ‹ Ρ…ΠΎΡ‚ΠΈΡ‚Π΅, Ρ‡Ρ‚ΠΎΠ±Ρ‹ ΡƒΡ‡Π΅Π½ΠΈΠΊΠΈ Ρ€Π°Π±ΠΎΡ‚Π°Π»ΠΈ с Ρ†ΠΈΠ½ΠΎΠ²ΠΊΠ°ΠΌΠΈ.

    Π’Π°Ρ€ΠΈΠ°Π½Ρ‚ β„– 1 – ΠŸΠΎΠΏΡ€ΠΎΡΠΈΡ‚Π΅ учащихся ΠΈΡΠΏΠΎΠ»ΡŒΠ·ΠΎΠ²Π°Ρ‚ΡŒ всС Ρ‡Π΅Ρ‚Ρ‹Ρ€Π΅ стратСгии для Ρ€Π΅ΡˆΠ΅Π½ΠΈΡ Π·Π°Π΄Π°Ρ‡ΠΈ.

    Π’Π°Ρ€ΠΈΠ°Π½Ρ‚ β„– 2 – ΠŸΠΎΠΏΡ€ΠΎΡΠΈΡ‚Π΅ учащихся Π²Ρ‹Π±Ρ€Π°Ρ‚ΡŒ Π½Π°ΠΈΠ±ΠΎΠ»Π΅Π΅ эффСктивный (ΠΏΠΎ ΠΈΡ… мнСнию) способ Ρ€Π΅ΡˆΠ΅Π½ΠΈΡ ΠΏΡ€ΠΎΠ±Π»Π΅ΠΌΡ‹. НапримСр, Π½Π΅ΠΊΠΎΡ‚ΠΎΡ€Ρ‹Π΅ ΠΏΡ€ΠΎΠ±Π»Π΅ΠΌΡ‹ ΠΌΠΎΠ³ΡƒΡ‚ Π±Ρ‹Ρ‚ΡŒ эффСктивно Ρ€Π΅ΡˆΠ΅Π½Ρ‹ с ΠΏΠΎΠΌΠΎΡ‰ΡŒΡŽ ΠΏΠΎΠ²Ρ‚ΠΎΡ€Π½ΠΎΠ³ΠΎ слоТСния (4 x 9 = 9 + 9 + 9 + 9), Π° Π½Π΅ΠΊΠΎΡ‚ΠΎΡ€Ρ‹Π΅ Ρ€Π°Π±ΠΎΡ‚Π°ΡŽΡ‚ Π»ΡƒΡ‡ΡˆΠ΅ ΠΏΡ€ΠΈ использовании связанных Ρ„Π°ΠΊΡ‚ΠΎΠ² (7 x 8 = 2 x 8 ΠΈ 5 x 8).

    НС ΡƒΠ²Π΅Ρ€Π΅Π½Ρ‹ Π² Ρ€Π°Π·Π»ΠΈΡ‡Π½Ρ‹Ρ… стратСгиях умноТСния ΠΈ дСлСния? Π― Ρ‚Π°ΠΊΠΆΠ΅ свяТу ΠΊΠ°Ρ€Ρ‚ΠΎΡ‡ΠΊΠΈ стратСгии умноТСния ΠΈ дСлСния, ΠΊΠΎΡ‚ΠΎΡ€Ρ‹Π΅ Π²Ρ‹ ΠΌΠΎΠΆΠ΅Ρ‚Π΅ ΠΈΡΠΏΠΎΠ»ΡŒΠ·ΠΎΠ²Π°Ρ‚ΡŒ для ΠΎΠ±Π·ΠΎΡ€Π° этих стратСгий ΠΈ ΠΊΠΎΡ‚ΠΎΡ€Ρ‹Π΅ Π·Π°Ρ‚Π΅ΠΌ студСнты ΠΌΠΎΠ³ΡƒΡ‚ ΠΈΡΠΏΠΎΠ»ΡŒΠ·ΠΎΠ²Π°Ρ‚ΡŒ Π² качСствС справочных ΠΌΠ°Ρ‚Π΅Ρ€ΠΈΠ°Π»ΠΎΠ².

    *** Π©Π΅Π»ΠΊΠ½ΠΈΡ‚Π΅ здСсь, Ρ‡Ρ‚ΠΎΠ±Ρ‹ ΠΏΠΎΠ»ΡƒΡ‡ΠΈΡ‚ΡŒ бСсплатныС ΠΌΠ°Ρ‚Ρ‹, ΠΊΠΎΡ‚ΠΎΡ€Ρ‹Π΅ ΠΏΠΎΠΌΠΎΠ³ΡƒΡ‚ вашим ΡƒΡ‡Π΅Π½ΠΈΠΊΠ°ΠΌ Ρ€Π°Π±ΠΎΡ‚Π°Ρ‚ΡŒ Π½Π°Π΄ ΠΈΡ… ΠΊΠΎΠ½Ρ†Π΅ΠΏΡ‚ΡƒΠ°Π»ΡŒΠ½Ρ‹ΠΌΠΈ стратСгиями умноТСния ΠΈ дСлСния.

    *** Π©Π΅Π»ΠΊΠ½ΠΈΡ‚Π΅ здСсь, Ρ‡Ρ‚ΠΎΠ±Ρ‹ ΠΏΠΎΠ»ΡƒΡ‡ΠΈΡ‚ΡŒ ΠΊΠ°Ρ€Ρ‚Ρ‹ стратСгии умноТСния ΠΈ дСлСния.

    Π₯ΠΎΡ‚ΠΈΡ‚Π΅ Π‘ΠžΠ›Π¬Π¨Π• бСсплатных матСматичСских Ρ†Π΅Π½Ρ‚Ρ€ΠΎΠ² ΠΈ занятий?

    Π©Π΅Π»ΠΊΠ½ΠΈΡ‚Π΅ здСсь ΠΈΠ»ΠΈ Π½Π° ΠΈΠ·ΠΎΠ±Ρ€Π°ΠΆΠ΅Π½ΠΈΠΈ Π½ΠΈΠΆΠ΅, Ρ‡Ρ‚ΠΎΠ±Ρ‹ ΠΎΠ·Π½Π°ΠΊΠΎΠΌΠΈΡ‚ΡŒΡΡ с ΠΌΠΎΠΈΠΌΠΈ Π»ΡŽΠ±ΠΈΠΌΡ‹ΠΌΠΈ матСматичСскими Ρ†Π΅Π½Ρ‚Ρ€Π°ΠΌΠΈ ΠΈ упраТнСниями для 3–5 классов ΠΈ Π·Π°Π³Ρ€ΡƒΠ·ΠΈΡ‚ΡŒ мноТСство бСсплатных, Ρ‡Ρ‚ΠΎΠ±Ρ‹ ΠΎΠΏΡ€ΠΎΠ±ΠΎΠ²Π°Ρ‚ΡŒ ΠΈΡ… со своими ΡƒΡ‡Π΅Π½ΠΈΠΊΠ°ΠΌΠΈ.

    ** Π­Ρ‚ΠΎ матСматичСскиС Ρ†Π΅Π½Ρ‚Ρ€Ρ‹ ΠΈ занятия Π½Π° ΡƒΡ€ΠΎΠ²Π½Π΅ класса.

    ΠŸΠ»Π°Π½Ρ‹ ΡƒΡ€ΠΎΠΊΠΎΠ² ΠΌΠ°Ρ‚Π΅ΠΌΠ°Ρ‚ΠΈΠΊΠΈ для Ρ‚Ρ€Π΅Ρ‚ΡŒΠ΅Π³ΠΎ класса

    ΠŸΠΎΡΠΌΠΎΡ‚Ρ€Π΅Ρ‚ΡŒ Π΄Π΅ΠΌΠΎ Π½Π°ΡˆΠΈΡ… ΡƒΡ€ΠΎΠΊΠΎΠ²

    УчСбная ΠΏΡ€ΠΎΠ³Ρ€Π°ΠΌΠΌΠ° ΠΏΠΎ ΠΌΠ°Ρ‚Π΅ΠΌΠ°Ρ‚ΠΈΠΊΠ΅ Time4Learning доступна для учащихся ΠΎΡ‚ Π΄ΠΎΡˆΠΊΠΎΠ»ΡŒΠ½Ρ‹Ρ… ΡƒΡ‡Ρ€Π΅ΠΆΠ΄Π΅Π½ΠΈΠΉ Π΄ΠΎ Π΄Π²Π΅Π½Π°Π΄Ρ†Π°Ρ‚Ρ‹Ρ… классов. Π ΠΎΠ΄ΠΈΡ‚Π΅Π»ΠΈ ΠΌΠΎΠ³ΡƒΡ‚ ΠΎΠΆΠΈΠ΄Π°Ρ‚ΡŒ, Ρ‡Ρ‚ΠΎ ΠΎΠ½ΠΈ увидят ΠΎΡ…Π²Π°Ρ‚Ρ‹Π²Π°Π΅ΠΌΡ‹Π΅ Ρ‚Π΅ΠΌΡ‹, Π²ΠΊΠ»ΡŽΡ‡Π°Ρ ΠΎΠΏΡ€Π΅Π΄Π΅Π»Π΅Π½ΠΈΠ΅ трансформаций ΠΈ симмСтрии, Π΄Π΅ΠΌΠΎΠ½ΡΡ‚Ρ€Π°Ρ†ΠΈΡŽ Π΄Ρ€ΠΎΠ±Π΅ΠΉ, Ρ€Π΅ΡˆΠ΅Π½ΠΈΠ΅ ΠΏΡ€ΠΎΠ±Π»Π΅ΠΌ ΠΈ ΠΌΠ½ΠΎΠ³ΠΎΠ΅ Π΄Ρ€ΡƒΠ³ΠΎΠ΅.

    ΠŸΠΎΠ΄Ρ€ΠΎΠ±Π½Ρ‹Π΅ ΠΏΠ»Π°Π½Ρ‹ ΡƒΡ€ΠΎΠΊΠΎΠ², прСдставлСнныС Π½ΠΈΠΆΠ΅, содСрТат ΠΏΠΎΠ΄Ρ€ΠΎΠ±Π½Ρ‹ΠΉ список ΡƒΡ‡Π΅Π±Π½ΠΎΠΉ ΠΏΡ€ΠΎΠ³Ρ€Π°ΠΌΠΌΡ‹ Time4Learning ΠΏΠΎ ΠΌΠ°Ρ‚Π΅ΠΌΠ°Ρ‚ΠΈΠΊΠ΅ для Ρ‚Ρ€Π΅Ρ‚ΡŒΠ΅Π³ΠΎ класса.

    Участники

    часто ΠΈΡΠΏΠΎΠ»ΡŒΠ·ΡƒΡŽΡ‚ эту страницу ΠΊΠ°ΠΊ рСсурс для Π±ΠΎΠ»Π΅Π΅ ΠΏΠΎΠ΄Ρ€ΠΎΠ±Π½ΠΎΠ³ΠΎ планирования, ΠΊΠ°ΠΊ руководство, ΠΏΠΎΠΌΠΎΠ³Π°ΡŽΡ‰Π΅Π΅ Π²Ρ‹Π±Ρ€Π°Ρ‚ΡŒ ΠΊΠΎΠ½ΠΊΡ€Π΅Ρ‚Π½Ρ‹Π΅ занятия с ΠΏΠΎΠΌΠΎΡ‰ΡŒΡŽ срСдства поиска занятий ΠΈΠ»ΠΈ ΡΡ€Π°Π²Π½ΠΈΡ‚ΡŒ Π½Π°ΡˆΡƒ ΡƒΡ‡Π΅Π±Π½ΡƒΡŽ ΠΏΡ€ΠΎΠ³Ρ€Π°ΠΌΠΌΡƒ с государствСнными стандартами ΠΈ Π·Π°ΠΊΠΎΠ½Π°ΠΌΠΈ ΠΎ домашнСм ΠΎΠ±ΡƒΡ‡Π΅Π½ΠΈΠΈ.

    Π§Ρ‚ΠΎ Π²Ρ…ΠΎΠ΄ΠΈΡ‚ Π² ΠΏΠ»Π°Π½ ΡƒΡ€ΠΎΠΊΠ° Time4Learning?

    1. Полная учСбная ΠΏΡ€ΠΎΠ³Ρ€Π°ΠΌΠΌΠ° для Ρ‚Ρ€Π΅Ρ‚ΡŒΠ΅Π³ΠΎ класса ΠΏΠΎ ΠΌΠ°Ρ‚Π΅ΠΌΠ°Ρ‚ΠΈΠΊΠ΅ с 18 Π³Π»Π°Π²Π°ΠΌΠΈ, Π±ΠΎΠ»Π΅Π΅ 285 заданиями, Ρ€Π°Π±ΠΎΡ‡ΠΈΠΌΠΈ листами ΠΈ Π²ΠΈΠΊΡ‚ΠΎΡ€ΠΈΠ½Π°ΠΌΠΈ
    2. Π“Π»Π°Π²Π° ΡƒΡ€ΠΎΠΊΠΈ с ΠΏΠΎΠ΄Ρ€ΠΎΠ±Π½Ρ‹ΠΌ описаниСм ΠΏΡ€ΠΎΠΉΠ΄Π΅Π½Π½ΠΎΠ³ΠΎ содСрТания
    3. НСсколько Ρ‚ΠΈΠΏΠΎΠ² Π·Π°Π΄Π°Π½ΠΈΠΉ для овладСния Π½Π°Π²Ρ‹ΠΊΠ°ΠΌΠΈ, Π²ΠΊΠ»ΡŽΡ‡Π°Ρ задания Π±Π΅Π· Π±Π°Π»Π»ΠΎΠ², Π²ΠΈΠΊΡ‚ΠΎΡ€ΠΈΠ½Ρ‹ ΠΈ распСчатанныС ΠΊΠ»ΡŽΡ‡ΠΈ ΠΎΡ‚Π²Π΅Ρ‚ΠΎΠ² Π² Π²ΠΈΠΊΡ‚ΠΎΡ€ΠΈΠ½Π°Ρ…
    4. Π Π°Π±ΠΎΡ‡ΠΈΠ΅ листы ΡƒΡ€ΠΎΠΊΠΎΠ² ΠΈ ΠΊΠ»ΡŽΡ‡ΠΈ для ΠΎΡ‚Π²Π΅Ρ‚ΠΎΠ² Π½Π° прСдставлСнныС ΠΌΠ°Ρ‚Π΅Ρ€ΠΈΠ°Π»Ρ‹
    5. Π›Π΅Π³ΠΊΠΈΠΉ доступ ΠΊ Π΄ΠΎΠΏΠΎΠ»Π½ΠΈΡ‚Π΅Π»ΡŒΠ½Ρ‹ΠΌ Π³Π»Π°Π²Π°ΠΌ ΠΏΠΎ ΠΊΠ°ΠΆΠ΄ΠΎΠΉ Ρ‚Π΅ΠΌΠ΅
    6. Time4MathFacts, ΠΊΠΎΡ‚ΠΎΡ€Ρ‹ΠΉ ΠΈΡΠΏΠΎΠ»ΡŒΠ·ΡƒΠ΅Ρ‚ Π·Π°Π±Π°Π²Π½Ρ‹Π΅ ΠΈΠ³Ρ€Ρ‹, Ρ‡Ρ‚ΠΎΠ±Ρ‹ Π²ΠΎΠ²Π»Π΅Ρ‡ΡŒ вашСго Ρ€Π΅Π±Π΅Π½ΠΊΠ° Π² ΠΈΠ·ΡƒΡ‡Π΅Π½ΠΈΠ΅ основ ΠΌΠ°Ρ‚Π΅ΠΌΠ°Ρ‚ΠΈΠΊΠΈ.

    УчащиСся, ΠΎΠ±ΡƒΡ‡Π°ΡŽΡ‰ΠΈΠ΅ΡΡ ΠΏΠΎ матСматичСской ΠΏΡ€ΠΎΠ³Ρ€Π°ΠΌΠΌΠ΅ Ρ‚Ρ€Π΅Ρ‚ΡŒΠ΅Π³ΠΎ класса Time4Learning, Π±ΡƒΠ΄ΡƒΡ‚ ΠΈΠΌΠ΅Ρ‚ΡŒ доступ ΠΊ ΡƒΡ€ΠΎΠΊΠ°ΠΌ ΠΊΠ°ΠΊ Π²Ρ‚ΠΎΡ€ΠΎΠ³ΠΎ, Ρ‚Π°ΠΊ ΠΈ Ρ‡Π΅Ρ‚Π²Π΅Ρ€Ρ‚ΠΎΠ³ΠΎ класса Π² Ρ€Π°ΠΌΠΊΠ°Ρ… своСго члСнства, Ρ‚Π°ΠΊ Ρ‡Ρ‚ΠΎ ΠΎΠ½ΠΈ смогут ΠΏΡ€ΠΎΠ΄Π²ΠΈΠ³Π°Ρ‚ΡŒΡΡ Π²ΠΏΠ΅Ρ€Π΅Π΄ ΠΈΠ»ΠΈ Π΄Π΅Π»Π°Ρ‚ΡŒ повторСния Π² своСм собствСнном Ρ‚Π΅ΠΌΠΏΠ΅.

    ОбъСм ΠΈ ΠΏΠΎΡΠ»Π΅Π΄ΠΎΠ²Π°Ρ‚Π΅Π»ΡŒΠ½ΠΎΡΡ‚ΡŒ занятий ΠΏΠΎ ΠΌΠ°Ρ‚Π΅ΠΌΠ°Ρ‚ΠΈΠΊΠ΅ для 3-Π³ΠΎ класса

    ΠŸΡ€Π΅ΠΎΠ±Ρ€Π°Π·ΡƒΠ΅Ρ‚ числа, содСрТащиС ΠΎΡ‚ Π΄Π²ΡƒΡ… Π΄ΠΎ ΡˆΠ΅ΡΡ‚ΠΈ Ρ†ΠΈΡ„Ρ€, ΠΈΠ· стандартной Ρ„ΠΎΡ€ΠΌΡ‹ Π² Ρ€Π°Π·Π²Π΅Ρ€Π½ΡƒΡ‚ΡƒΡŽ ΠΈ Π½Π°ΠΎΠ±ΠΎΡ€ΠΎΡ‚.

    ЗаписывайтС числа Π΄ΠΎ ΡˆΠ΅ΡΡ‚ΠΈ Ρ†ΠΈΡ„Ρ€, ΠΈΡΠΏΠΎΠ»ΡŒΠ·ΡƒΡ устныС ΠΈ ΠΏΠΈΡΡŒΠΌΠ΅Π½Π½Ρ‹Π΅ подсказки.

    Π—Π°ΠΊΠ°Π·Ρ‹Π²Π°ΠΉΡ‚Π΅ Π½ΠΎΠΌΠ΅Ρ€Π° Π΄ΠΎ ΡˆΠ΅ΡΡ‚ΠΈ Ρ†ΠΈΡ„Ρ€ ΠΈ сравнивайтС Π½ΠΎΠΌΠ΅Ρ€Π° с ΠΏΠΎΠΌΠΎΡ‰ΡŒΡŽ символов <,> ΠΈ =.

    ΠžΠΊΡ€ΡƒΠ³Π»ΠΈΡ‚Π΅ числа Π΄ΠΎ дСсятков тысяч Π΄ΠΎ блиТайшСго дСсятка. Π˜ΡΠΏΠΎΠ»ΡŒΠ·ΡƒΠΉΡ‚Π΅ числовыС Π»ΠΈΠ½ΠΈΠΈ ΠΈ Π·Π½Π°Π½ΠΈΠ΅ разрядов.

    ΠžΠΊΡ€ΡƒΠ³Π»ΠΈΡ‚Π΅ числа Π΄ΠΎ дСсятков тысяч Π΄ΠΎ блиТайшСй сотни.Π˜ΡΠΏΠΎΠ»ΡŒΠ·ΡƒΠΉΡ‚Π΅ числовыС Π»ΠΈΠ½ΠΈΠΈ ΠΈ Π·Π½Π°Π½ΠΈΠ΅ разрядов.

    ΠžΠΊΡ€ΡƒΠ³Π»ΠΈΡ‚Π΅ числа Π΄ΠΎ блиТайшСй дСсятки, Π΄ΠΎ блиТайшСй сотни ΠΈ Π΄ΠΎ блиТайшСй тысячи.

    Π”ΠΎΠ±Π°Π²ΡŒΡ‚Π΅ Ρ‚Ρ€ΠΈ ΠΈΠ»ΠΈ Π±ΠΎΠ»Π΅Π΅ ΠΎΠ΄Π½ΠΎΠ·Π½Π°Ρ‡Π½Ρ‹Ρ… слагаСмых. (свойство Π³Ρ€ΡƒΠΏΠΏΠΈΡ€ΠΎΠ²ΠΊΠΈ) Π”ΠΎΠ±Π°Π²ΡŒΡ‚Π΅ 2- ΠΈ 3-Π·Π½Π°Ρ‡Π½Ρ‹Π΅ числа. (с ΠΏΠ΅Ρ€Π΅Π³Ρ€ΡƒΠΏΠΏΠΈΡ€ΠΎΠ²ΠΊΠΎΠΉ ΠΈ Π±Π΅Π·)

    Π’Ρ‹Ρ‡Ρ‚ΠΈΡ‚Π΅ 2- ΠΈ 3-Π·Π½Π°Ρ‡Π½Ρ‹Π΅ числа. (с ΠΏΠ΅Ρ€Π΅Π³Ρ€ΡƒΠΏΠΏΠΈΡ€ΠΎΠ²ΠΊΠΎΠΉ) Π’Ρ‹Ρ‡Ρ‚ΠΈΡ‚Π΅ 2- ΠΈ 3-Π·Π½Π°Ρ‡Π½Ρ‹Π΅ числа, Ссли minuend ΠΈΠΌΠ΅Π΅Ρ‚ нСсколько Π½ΡƒΠ»Π΅ΠΉ. (с ΠΏΠ΅Ρ€Π΅Π³Ρ€ΡƒΠΏΠΏΠΈΡ€ΠΎΠ²ΠΊΠΎΠΉ)

    ΠžΡ†Π΅Π½ΠΈΡ‚Π΅ суммы ΠΈ Ρ€Π°Π·Π½ΠΈΡ†Ρ‹, ΠΈΡΠΏΠΎΠ»ΡŒΠ·ΡƒΡ ΠΎΠΊΡ€ΡƒΠ³Π»Π΅Π½ΠΈΠ΅.

    Π’Π²Π΅Π΄Π΅Π½ΠΈΠ΅ Π² ΡƒΠΌΠ½ΠΎΠΆΠ΅Π½ΠΈΠ΅ (0-12 x 0-12), Π²ΠΊΠ»ΡŽΡ‡Π°Ρ ΡƒΠΌΠ½ΠΎΠΆΠ΅Π½ΠΈΠ΅ Π½Π° 0 ΠΈ 1, с использованиСм массивов ΠΈ Ρ‚Π°Π±Π»ΠΈΡ†.

    ΠžΠΏΡ€Π΅Π΄Π΅Π»ΠΈΡ‚Π΅ ΠΈ пСрСчислитС ΠΊΡ€Π°Ρ‚Π½Ρ‹Π΅ Π΄Π°Π½Π½ΠΎΠ³ΠΎ числа (1-10). Π˜Π·ΡƒΡ‡ΠΈΡ‚Π΅ ΡƒΠΌΠ½ΠΎΠΆΠ΅Π½ΠΈΠ΅ ΠΊΠ°ΠΊ ΠΏΠΎΠ²Ρ‚ΠΎΡ€ΡΡŽΡ‰Π΅Π΅ΡΡ слоТСниС ΠΈ массивы.

    Π£ΠΌΠ½ΠΎΠΆΠ΅Π½ΠΈΠ΅ Π΄Π²ΡƒΡ… Ρ†Π΅Π»Ρ‹Ρ… чисСл с ΠΏΠ΅Ρ€Π΅Π³Ρ€ΡƒΠΏΠΏΠΈΡ€ΠΎΠ²ΠΊΠΎΠΉ ΠΈ Π±Π΅Π· Π½Π΅Π΅, Π² ΠΊΠΎΡ‚ΠΎΡ€ΠΎΠΌ ΠΎΠ΄ΠΈΠ½ ΠΌΠ½ΠΎΠΆΠΈΡ‚Π΅Π»ΡŒ являСтся ΠΎΠ΄Π½ΠΎΠ·Π½Π°Ρ‡Π½Ρ‹ΠΌ числом, Π° Π΄Ρ€ΡƒΠ³ΠΎΠΉ – Π΄Π²ΡƒΠ·Π½Π°Ρ‡Π½Ρ‹ΠΌ числом. Π£ΠΌΠ½ΠΎΠΆΡŒΡ‚Π΅ мыслСнно Π½Π° 10, 100 ΠΈ 1000.

    Π£ΠΌΠ½ΠΎΠΆΠ°ΠΉΡ‚Π΅ ΠΎΠ΄Π½ΠΎΠ·Π½Π°Ρ‡Π½Ρ‹Π΅ Ρ†Π΅Π»Ρ‹Π΅ числа Π½Π° ΠΊΡ€Π°Ρ‚Π½Ρ‹Π΅ 10 Π² Π΄ΠΈΠ°ΠΏΠ°Π·ΠΎΠ½Π΅ 10–90, ΠΈΡΠΏΠΎΠ»ΡŒΠ·ΡƒΡ стратСгии, основанныС Π½Π° разрядах ΠΈ свойствах ΠΎΠΏΠ΅Ρ€Π°Ρ†ΠΈΠΉ.

    Π’Π²Π΅Π΄Π΅Π½ΠΈΠ΅ Π² простыС Π·Π°Π΄Π°Ρ‡ΠΈ дСлСния, Π²ΠΊΠ»ΡŽΡ‡Π°Ρ дСлСния с использованиСм 0 ΠΈ 1 ΠΈ дСлСния с использованиСм остатков с использованиСм Ρ‚Π°Π±Π»ΠΈΡ† ΠΈ Π΄Ρ€ΡƒΠ³ΠΈΡ… манипуляторов.

    РаспознайтС ΠΈ ΠΈΡΠΏΠΎΠ»ΡŒΠ·ΡƒΠΉΡ‚Π΅ основныС Ρ„Π°ΠΊΡ‚Ρ‹ дСлСния Π½Π° 100 Γ· 10, Π° Ρ‚Π°ΠΊΠΆΠ΅ ΠΎΠΏΡ€Π΅Π΄Π΅Π»ΠΈΡ‚Π΅ Π΄ΠΈΠ²ΠΈΠ΄Π΅Π½Π΄, Π΄Π΅Π»ΠΈΡ‚Π΅Π»ΡŒ ΠΈ частноС. ΠžΠΏΠΈΡˆΠΈΡ‚Π΅ эти свойства дСлСния: Π²Ρ‹ Π½Π΅ ΠΌΠΎΠΆΠ΅Ρ‚Π΅ Π΄Π΅Π»ΠΈΡ‚ΡŒ Π½Π° 0, ΠΈ любоС число, Π΄Π΅Π»Π΅Π½Π½ΠΎΠ΅ Π½Π° 1, равняСтся этому числу.

    ΠŸΡ€Π΅Π΄ΡΡ‚Π°Π²Π»ΡΡ‚ΡŒ ΠΈ Ρ€Π΅ΡˆΠ°Ρ‚ΡŒ Π·Π°Π΄Π°Ρ‡ΠΈ, связанныС с Ρ€Π°Π·Π΄Π΅Π»Π΅Π½ΠΈΠ΅ΠΌ. Π˜Π½Ρ‚Π΅Ρ€ΠΏΡ€Π΅Ρ‚ΠΈΡ€ΡƒΠΉΡ‚Π΅ частныС Ρ†Π΅Π»ΠΎΠ³ΠΎ числа Π»ΠΈΠ±ΠΎ ΠΊΠ°ΠΊ количСство ΠΎΠ±ΡŠΠ΅ΠΊΡ‚ΠΎΠ² Π² ΠΊΠ°ΠΆΠ΄ΠΎΠΉ Π΄ΠΎΠ»Π΅, Ссли ΠΎΠ±ΡŠΠ΅ΠΊΡ‚Ρ‹ Ρ€Π°Π·Π΄Π΅Π»Π΅Π½Ρ‹ ΠΏΠΎΡ€ΠΎΠ²Π½Ρƒ, Π»ΠΈΠ±ΠΎ ΠΊΠ°ΠΊ количСство Π΄ΠΎΠ»Π΅ΠΉ.

    ΠŸΡ€Π΅Π΄ΡΡ‚Π°Π²Π»ΡΡ‚ΡŒ ΠΈ Ρ€Π΅ΡˆΠ°Ρ‚ΡŒ Π·Π°Π΄Π°Ρ‡ΠΈ, связанныС с Ρ€Π°Π·Π΄Π΅Π»Π΅Π½ΠΈΠ΅ΠΌ. Π˜ΡΠΏΠΎΠ»ΡŒΠ·ΡƒΠΉΡ‚Π΅ Π΄Π΅Π»Π΅Π½ΠΈΠ΅ Π² ΠΏΡ€Π΅Π΄Π΅Π»Π°Ρ… 100 для Ρ€Π΅ΡˆΠ΅Π½ΠΈΡ словСсных Π·Π°Π΄Π°Ρ‡ Π² ситуациях с участиСм Ρ€Π°Π²Π½Ρ‹Ρ… Π³Ρ€ΡƒΠΏΠΏ, ΠΈΡΠΏΠΎΠ»ΡŒΠ·ΡƒΡ рисунки ΠΈ уравнСния с символом нСизвСстного числа, Ρ‡Ρ‚ΠΎΠ±Ρ‹ ΠΏΡ€Π΅Π΄ΡΡ‚Π°Π²ΠΈΡ‚ΡŒ ΠΏΡ€ΠΎΠ±Π»Π΅ΠΌΡƒ.

    Π Π°Π·Π΄Π΅Π»ΠΈΡ‚Π΅ Π΄Π²ΡƒΠ·Π½Π°Ρ‡Π½Ρ‹Π΅ Π΄ΠΈΠ²ΠΈΠ΄Π΅Π½Π΄Ρ‹ Π½Π° ΠΎΠ΄Π½ΠΎΠ·Π½Π°Ρ‡Π½Ρ‹Π΅ Π΄Π΅Π»ΠΈΡ‚Π΅Π»ΠΈ с остатками ΠΈ Π±Π΅Π· Π½ΠΈΡ….

    ΠžΠΏΡ€Π΅Π΄Π΅Π»ΠΈΡ‚Π΅ арифмСтичСскиС ΡˆΠ°Π±Π»ΠΎΠ½Ρ‹ с ΠΏΠΎΠΌΠΎΡ‰ΡŒΡŽ Ρ‚Π°Π±Π»ΠΈΡ†Ρ‹ слоТСния.

    ΠžΠΏΡ€Π΅Π΄Π΅Π»ΠΈΡ‚Π΅ арифмСтичСскиС ΡˆΠ°Π±Π»ΠΎΠ½Ρ‹ с ΠΏΠΎΠΌΠΎΡ‰ΡŒΡŽ Ρ‚Π°Π±Π»ΠΈΡ†Ρ‹ умноТСния ΠΈ ΠΎΠ±ΡŠΡΡΠ½ΠΈΡ‚Π΅ ΠΈΡ…, ΠΈΡΠΏΠΎΠ»ΡŒΠ·ΡƒΡ свойства ΠΎΠΏΠ΅Ρ€Π°Ρ†ΠΈΠΉ.

    Π Π΅ΡˆΠΈΡ‚Π΅ ΠΌΠ½ΠΎΠ³ΠΎΡΡ‚ΡƒΠΏΠ΅Π½Ρ‡Π°Ρ‚ΡƒΡŽ Π·Π°Π΄Π°Ρ‡Ρƒ со словами, ΠΈΡΠΏΠΎΠ»ΡŒΠ·ΡƒΡ ΡƒΠΌΠ½ΠΎΠΆΠ΅Π½ΠΈΠ΅ ΠΈ Π΄Π΅Π»Π΅Π½ΠΈΠ΅.

    ΠŸΠΎΠΉΠΌΠΈΡ‚Π΅ Ρ€Π°Π·Π΄Π΅Π»Π΅Π½ΠΈΠ΅ ΠΊΠ°ΠΊ ΠΏΡ€ΠΎΠ±Π»Π΅ΠΌΡƒ с нСизвСстным Ρ„Π°ΠΊΡ‚ΠΎΡ€ΠΎΠΌ.

    Π Π°Π·Π±Π΅Ρ€ΠΈΡ‚Π΅ΡΡŒ Π² ΡƒΠΌΠ½ΠΎΠΆΠ΅Π½ΠΈΠΈ ΠΈ ΠΈΡΠΏΠΎΠ»ΡŒΠ·ΡƒΠΉΡ‚Π΅ стратСгии для ΠΏΠ»Π°Π²Π½ΠΎΠ³ΠΎ умноТСния Π² ΠΏΡ€Π΅Π΄Π΅Π»Π°Ρ… 100.

    ПониманиС дСлСния ΠΈ использованиС стратСгий для ΠΏΠ»Π°Π²Π½ΠΎΠ³ΠΎ дСлСния Π² ΠΏΡ€Π΅Π΄Π΅Π»Π°Ρ… 100.

    РаспознавайтС Π΄Ρ€ΠΎΠ±ΠΈ ΠΊΠ°ΠΊ Ρ‡Π°ΡΡ‚ΡŒ Ρ†Π΅Π»ΠΎΠ³ΠΎ ΠΈ ΠΏΠΎΠ½ΠΈΠΌΠ°ΠΉΡ‚Π΅ Π·Π½Π°Ρ‡Π΅Π½ΠΈΠ΅ числитСля ΠΈ знамСнатСля.

    ΠžΠΏΡ€Π΅Π΄Π΅Π»ΠΈΡ‚Π΅ Π΄Ρ€ΠΎΠ±ΡŒ, ΠΎΠ±ΠΎΠ·Π½Π°Ρ‡Π΅Π½Π½ΡƒΡŽ Ρ‚ΠΎΡ‡ΠΊΠΎΠΉ Π½Π° числовой прямой, ΠΈ ΡƒΠ·Π½Π°ΠΉΡ‚Π΅, ΠΊΠ°ΠΊ Π·Π°ΠΏΠΈΡΠ°Ρ‚ΡŒ Π΄Ρ€ΠΎΠ±ΡŒ Π½Π° числовой прямой.

    Π‘Ρ‡ΠΈΡ‚Π°ΠΉΡ‚Π΅ Π΄Π²Π΅ Π΄Ρ€ΠΎΠ±ΠΈ эквивалСнтными, Ссли ΠΎΠ½ΠΈ ΠΎΠ΄Π½ΠΎΠ³ΠΎ Ρ€Π°Π·ΠΌΠ΅Ρ€Π° ΠΈΠ»ΠΈ ΠΈΠΌΠ΅ΡŽΡ‚ ΠΎΠ΄Π½Ρƒ ΠΈ Ρ‚Ρƒ ΠΆΠ΅ Ρ‚ΠΎΡ‡ΠΊΡƒ Π½Π° числовой прямой.

    Π‘Ρ€Π°Π²Π½ΠΈΡ‚Π΅ Π΄Π²Π΅ Π΄Ρ€ΠΎΠ±ΠΈ с ΠΎΠ΄Π½ΠΈΠΌ ΠΈ Ρ‚Π΅ΠΌ ΠΆΠ΅ числитСлСм ΠΈΠ»ΠΈ ΠΎΠ΄Π½ΠΈΠΌ Π·Π½Π°ΠΌΠ΅Π½Π°Ρ‚Π΅Π»Π΅ΠΌ, ΠΈΡΠΏΠΎΠ»ΡŒΠ·ΡƒΡ ΠΌΠΎΠ΄Π΅Π»ΠΈ Π΄Ρ€ΠΎΠ±Π΅ΠΉ.

    ΠžΠ±ΠΎΠ·Π½Π°Ρ‡Π°Π΅Ρ‚ части мноТСства ΠΈ части Ρ†Π΅Π»ΠΎΠ³ΠΎ с эквивалСнтными дробями со Π·Π½Π°ΠΌΠ΅Π½Π°Ρ‚Π΅Π»Π΅ΠΌ Π΄ΠΎ 10.

    Π£ΠΊΠ°ΠΆΠΈΡ‚Π΅ эквивалСнтныС Π΄Ρ€ΠΎΠ±ΠΈ. (1/2 = 2/4)

    ΠŸΠΎΡ€ΡΠ΄ΠΎΠΊ Π΄Ρ€ΠΎΠ±Π΅ΠΉ с ΠΎΠ΄ΠΈΠ½Π°ΠΊΠΎΠ²Ρ‹ΠΌΠΈ знамСнатСлями ΠΈ сравнСниС Π΄Ρ€ΠΎΠ±Π΅ΠΉ с ΠΏΠΎΠΌΠΎΡ‰ΡŒΡŽ символов <,> ΠΈ =.

    Π˜Π·ΡƒΡ‡ΠΈΡ‚Π΅ взаимосвязь ΠΌΠ΅ΠΆΠ΄Ρƒ дробями ΠΈ дСсятичными Π·Π½Π°ΠΊΠ°ΠΌΠΈ. (дСсятыС ΠΈ сотыС)

    Π£ΠΊΠ°ΠΆΠΈΡ‚Π΅ дСсятичныС Π΄Ρ€ΠΎΠ±ΠΈ с Ρ‚ΠΎΡ‡Π½ΠΎΡΡ‚ΡŒΡŽ Π΄ΠΎ сотых. Π‘Ρ‡ΠΈΡ‚Π°ΠΉΡ‚Π΅ ΠΈ Π·Π°ΠΏΠΈΡˆΠΈΡ‚Π΅ дСсятичныС Π΄Ρ€ΠΎΠ±ΠΈ с Ρ‚ΠΎΡ‡Π½ΠΎΡΡ‚ΡŒΡŽ Π΄ΠΎ сотых.

    УпорядочивайтС дСсятичныС Π΄Ρ€ΠΎΠ±ΠΈ Π΄ΠΎ сотых ΠΈ сравнивайтС дСсятичныС Π΄Ρ€ΠΎΠ±ΠΈ, ΠΈΡΠΏΠΎΠ»ΡŒΠ·ΡƒΡ символы <,> ΠΈ =.

    ΠŸΠΎΠ΄ΡΡ‡Π΅Ρ‚ ΠΊΠΎΠ»Π»Π΅ΠΊΡ†ΠΈΠΈ ΠΌΠΎΠ½Π΅Ρ‚ ΠΈ ΠΊΡƒΠΏΡŽΡ€ Π΄ΠΎ 50 Π΄ΠΎΠ»Π»Π°Ρ€ΠΎΠ². Π‘Π»ΠΎΠΆΠΈΡ‚Π΅ ΠΈ Π²Ρ‹Ρ‡Ρ‚ΠΈΡ‚Π΅ суммы Π² Π΄ΠΎΠ»Π»Π°Ρ€Π°Ρ…. (Π΄ΠΎΠ»Π»Π°Ρ€ ΠΈ Ρ†Π΅Π½Ρ‚Ρ‹)

    Π Π΅ΡˆΠ°ΠΉΡ‚Π΅ словСсныС Π·Π°Π΄Π°Ρ‡ΠΈ, связанныС со ΡΡ‚ΠΎΠΈΠΌΠΎΡΡ‚ΡŒΡŽ ΠΌΠΎΠ½Π΅Ρ‚, Π±Π°Π½ΠΊΠ½ΠΎΡ‚ ΠΈ сдачСй.

    РСшСниС ΠΏΡ€ΠΎΠ±Π»Π΅ΠΌ, связанных с Ρ†Π΅Π½ΠΎΠΉ Π·Π° Π΅Π΄ΠΈΠ½ΠΈΡ†Ρƒ Ρ‚ΠΎΠ²Π°Ρ€Π°.

    ВыявлСниС ΠΈ Ρ€Π°ΡΡˆΠΈΡ€Π΅Π½ΠΈΠ΅ ΠΏΠΎΠ²Ρ‚ΠΎΡ€ΡΡŽΡ‰ΠΈΡ…ΡΡ шаблонов ΠΈ ΠΏΡ€ΠΈΠΌΠ΅Π½Π΅Π½ΠΈΠ΅ ΠΏΡ€Π°Π²ΠΈΠ» шаблонов с использованиСм Ρ„ΠΎΡ€ΠΌ, Ρ†Π²Π΅Ρ‚ΠΎΠ² ΠΈ чисСл.

    Π˜Π΄Π΅Π½Ρ‚ΠΈΡ„ΠΈΡ†ΠΈΡ€ΡƒΠΉΡ‚Π΅ ΠΈ Ρ€Π°ΡΡˆΠΈΡ€ΡΠΉΡ‚Π΅ ΡˆΠ°Π±Π»ΠΎΠ½Ρ‹ ΠΈ примСняйтС ΠΏΡ€Π°Π²ΠΈΠ»Π° шаблонов, ΠΈΡΠΏΠΎΠ»ΡŒΠ·ΡƒΡ ΠΏΠΎΡΠ»Π΅Π΄ΠΎΠ²Π°Ρ‚Π΅Π»ΡŒΠ½ΠΎΡΡ‚ΡŒ связанных чисСл.

    ΠŸΡ€ΠΈΠΌΠ΅Π½ΠΈΡ‚Π΅ ΡΠΎΠΎΡ‚Π²Π΅Ρ‚ΡΡ‚Π²ΡƒΡŽΡ‰Π΅Π΅ ΠΏΡ€Π°Π²ΠΈΠ»ΠΎ, Ρ‡Ρ‚ΠΎΠ±Ρ‹ Π·Π°ΠΏΠΎΠ»Π½ΠΈΡ‚ΡŒ Π΄ΠΈΠ°Π³Ρ€Π°ΠΌΠΌΡƒ, Π²ΠΊΠ»ΡŽΡ‡Π°Ρ Ρ‚Π°Π±Π»ΠΈΡ†Ρ‹ Π²Π²ΠΎΠ΄Π° / Π²Ρ‹Π²ΠΎΠ΄Π°.

    ΠŸΡ€Π΅Π΄ΡΡ‚Π°Π²Π»ΡΡ‚ΡŒ ΠΈ ΠΎΡ†Π΅Π½ΠΈΠ²Π°Ρ‚ΡŒ ΠΏΠΈΡΡŒΠΌΠ΅Π½Π½Ρ‹Π΅ ΠΎΡ‚Π½ΠΎΡˆΠ΅Π½ΠΈΡ Π² Π²ΠΈΠ΄Π΅ числовых Π²Ρ‹Ρ€Π°ΠΆΠ΅Π½ΠΈΠΉ.

    ΠžΠΏΡ€Π΅Π΄Π΅Π»ΠΈΡ‚Π΅ нСизвСстноС Ρ†Π΅Π»ΠΎΠ΅ число Π² ΡƒΡ€Π°Π²Π½Π΅Π½ΠΈΠΈ умноТСния, ΡΠ²ΡΠ·Ρ‹Π²Π°ΡŽΡ‰Π΅ΠΌ Ρ‚Ρ€ΠΈ Ρ†Π΅Π»Ρ‹Ρ… числа.

    ΠžΠΏΡ€Π΅Π΄Π΅Π»ΠΈΡ‚Π΅ нСизвСстноС Ρ†Π΅Π»ΠΎΠ΅ число Π² ΡƒΡ€Π°Π²Π½Π΅Π½ΠΈΠΈ дСлСния, ΡΠ²ΡΠ·Ρ‹Π²Π°ΡŽΡ‰Π΅ΠΌ Ρ‚Ρ€ΠΈ Ρ†Π΅Π»Ρ‹Ρ… числа.

    НайдитС Π½Π΅ΠΈΠ·Π²Π΅ΡΡ‚Π½ΡƒΡŽ Π²Π΅Π»ΠΈΡ‡ΠΈΠ½Ρƒ Π² ΡƒΡ€Π°Π²Π½Π΅Π½ΠΈΠΈ. ΠŸΡ€ΠΈΠΌΠ΅Ρ€: 3 + __ = 7. (ΠŸΡ€ΠΈΠΌΠ΅Ρ€: ΠΏΡ€ΠΎΠΏΡƒΡ‰Π΅Π½Π½ΠΎΠ΅ слагаСмоС ΠΈΠ»ΠΈ ΠΎΡ‚ΡΡƒΡ‚ΡΡ‚Π²ΡƒΡŽΡ‰ΠΈΠΉ ΠΌΠ½ΠΎΠΆΠΈΡ‚Π΅Π»ΡŒ)

    ΠŸΠΎΠΉΠΌΠΈΡ‚Π΅ свойства умноТСния ΠΈ ΠΏΡ€ΠΈΠΌΠ΅Π½ΠΈΡ‚Π΅ эти свойства ΠΊΠ°ΠΊ стратСгии умноТСния.

    ΠŸΠΎΠΉΠΌΠΈΡ‚Π΅ свойства дСлСния ΠΈ ΠΏΡ€ΠΈΠΌΠ΅Π½ΠΈΡ‚Π΅ эти свойства ΠΊΠ°ΠΊ стратСгии дСлСния.

    Π˜ΡΠΏΠΎΠ»ΡŒΠ·ΡƒΠΉΡ‚Π΅ свойства порядка (ΠΊΠΎΠΌΠΌΡƒΡ‚Π°Ρ‚ΠΈΠ²Π½ΠΎΡΡ‚ΡŒ) ΠΈ Π³Ρ€ΡƒΠΏΠΏΠΈΡ€ΠΎΠ²ΠΊΠΈ (Π°ΡΡΠΎΡ†ΠΈΠ°Ρ‚ΠΈΠ²Π½ΠΎΡΡ‚ΡŒ) слоТСния ΠΈ умноТСния, Ρ‡Ρ‚ΠΎΠ±Ρ‹ Π½Π°ΠΉΡ‚ΠΈ эквивалСнтныС выраТСния ΠΈΠ»ΠΈ уравнСния, содСрТащиС Π½Π΅ΠΈΠ·Π²Π΅ΡΡ‚Π½ΡƒΡŽ Π²Π΅Π»ΠΈΡ‡ΠΈΠ½Ρƒ.

    ΠžΠΏΠΈΡˆΠΈΡ‚Π΅ Π»ΠΈΠ½Π΅ΠΉΠ½Ρ‹Π΅ сСгмСнты, Π»ΠΈΠ½ΠΈΠΈ ΠΈ ΠΏΠ°Ρ€Ρ‹ Π»ΠΈΠ½ΠΈΠΉ.

    ΠžΠΏΡ€Π΅Π΄Π΅Π»ΡΠ΅Ρ‚ ΠΈ классифицируСт ΡƒΠ³Π»Ρ‹ ΠΊΠ°ΠΊ прямыС, острыС ΠΈΠ»ΠΈ Ρ‚ΡƒΠΏΡ‹Π΅.

    ΠžΠΏΡ€Π΅Π΄Π΅Π»Π΅Π½ΠΈΠ΅ Π°Ρ‚Ρ€ΠΈΠ±ΡƒΡ‚ΠΎΠ² ΠΌΠ½ΠΎΠ³ΠΎΡƒΠ³ΠΎΠ»ΡŒΠ½ΠΈΠΊΠΎΠ² (стороны ΠΈ ΡƒΠ³Π»Ρ‹) ΠΈ сортировка ΠΏΠΎ ΠΊΠΎΠ½ΠΊΡ€Π΅Ρ‚Π½Ρ‹ΠΌ характСристикам плоской Ρ„ΠΈΠ³ΡƒΡ€Ρ‹.

    ΠžΠΏΡ€Π΅Π΄Π΅Π»ΠΈΡ‚Π΅ Π°Ρ‚Ρ€ΠΈΠ±ΡƒΡ‚Ρ‹ Ρ‚Π²Π΅Ρ€Π΄Ρ‹Ρ… Ρ„ΠΈΠ³ΡƒΡ€ (Ρ€Π΅Π±Ρ€Π°, Π²Π΅Ρ€ΡˆΠΈΠ½Ρ‹ ΠΈ Π³Ρ€Π°Π½ΠΈ), Ρ‚Π°ΠΊΠΈΠ΅ ΠΊΠ°ΠΊ ΠΊΡƒΠ±Ρ‹, ΠΏΡ€ΡΠΌΠΎΡƒΠ³ΠΎΠ»ΡŒΠ½Ρ‹Π΅ ΠΏΡ€ΠΈΠ·ΠΌΡ‹, ΠΏΡ€ΡΠΌΠΎΡƒΠ³ΠΎΠ»ΡŒΠ½Ρ‹Π΅ ΠΏΠΈΡ€Π°ΠΌΠΈΠ΄Ρ‹, конусы, Ρ†ΠΈΠ»ΠΈΠ½Π΄Ρ€Ρ‹ ΠΈ сфСры, ΠΈ Π²Ρ‹ΠΏΠΎΠ»Π½ΠΈΡ‚Π΅ сортировку ΠΏΠΎ ΠΎΠΏΡ€Π΅Π΄Π΅Π»Π΅Π½Π½Ρ‹ΠΌ характСристикам.

    ΠžΠΏΡ€Π΅Π΄Π΅Π»ΡΠ΅Ρ‚ ΠΈ создаСт Π΄Π²ΡƒΠΌΠ΅Ρ€Π½ΠΎΠ΅ прСдставлСниС Ρ‚Ρ€Π΅Ρ…ΠΌΠ΅Ρ€Π½ΠΎΠΉ Ρ„ΠΈΠ³ΡƒΡ€Ρ‹.

    НайдитС расстояниС ΠΏΠΎ Π³ΠΎΡ€ΠΈΠ·ΠΎΠ½Ρ‚Π°Π»ΠΈ ΠΈΠ»ΠΈ Π²Π΅Ρ€Ρ‚ΠΈΠΊΠ°Π»ΠΈ ΠΌΠ΅ΠΆΠ΄Ρƒ двумя Ρ‚ΠΎΡ‡ΠΊΠ°ΠΌΠΈ Π½Π° ΠΊΠΎΠΎΡ€Π΄ΠΈΠ½Π°Ρ‚Π½ΠΎΠΉ сСткС.

    НанСситС Ρ‚ΠΎΡ‡ΠΊΡƒ Π½Π° ΠΊΠΎΠΎΡ€Π΄ΠΈΠ½Π°Ρ‚Π½ΡƒΡŽ сСтку с ΡƒΡ‡Π΅Ρ‚ΠΎΠΌ упорядочСнной ΠΏΠ°Ρ€Ρ‹ ΠΈ Π·Π°ΠΏΠΈΡˆΠΈΡ‚Π΅ ΡƒΠΏΠΎΡ€ΡΠ΄ΠΎΡ‡Π΅Π½Π½ΡƒΡŽ ΠΏΠ°Ρ€Ρƒ Ρ‚ΠΎΡ‡Π΅ΠΊ, ΠΏΠΎΠΊΠ°Π·Π°Π½Π½Ρ‹Ρ… Π½Π° ΠΊΠΎΠΎΡ€Π΄ΠΈΠ½Π°Ρ‚Π½ΠΎΠΉ сСткС.

    ПослС получСния Π½Π°Π²ΠΈΠ³Π°Ρ†ΠΈΠΎΠ½Π½Ρ‹Ρ… ΡƒΠΊΠ°Π·Π°Π½ΠΈΠΉ ΠΎΡ‚ Π½Π°Ρ‡Π°Π»ΡŒΠ½ΠΎΠΉ Ρ‚ΠΎΡ‡ΠΊΠΈ ΠΎΠΏΡ€Π΅Π΄Π΅Π»ΠΈΡ‚Π΅ ΡƒΠΏΠΎΡ€ΡΠ΄ΠΎΡ‡Π΅Π½Π½ΡƒΡŽ ΠΏΠ°Ρ€Ρƒ ΠΊΠΎΠ½Π΅Ρ‡Π½ΠΎΠΉ Ρ‚ΠΎΡ‡ΠΊΠΈ.

    Для плоской Ρ„ΠΈΠ³ΡƒΡ€Ρ‹ ΠΎΠΏΡ€Π΅Π΄Π΅Π»ΠΈΡ‚Π΅ ΠΊΠΎΠ½Π³Ρ€ΡƒΡΠ½Ρ‚Π½ΡƒΡŽ Ρ„ΠΎΡ€ΠΌΡƒ ΠΈ создайтС ΠΊΠΎΠ½Π³Ρ€ΡƒΡΠ½Ρ‚Π½ΡƒΡŽ Ρ„ΠΎΡ€ΠΌΡƒ, ΠΈΡΠΏΠΎΠ»ΡŒΠ·ΡƒΡ Π΄Ρ€ΡƒΠ³ΠΈΠ΅ плоскиС Ρ„ΠΈΠ³ΡƒΡ€Ρ‹.

    ΠŸΡ€ΠΈΠΌΠ΅Π½ΠΈΡ‚Π΅ скольТСниС, ΠΏΠ΅Ρ€Π΅Π²ΠΎΡ€ΠΎΡ‚ ΠΈΠ»ΠΈ ΠΏΠΎΠ²ΠΎΡ€ΠΎΡ‚ ΠΊ плоской Ρ„ΠΈΠ³ΡƒΡ€Π΅ ΠΈ спрогнозируйтС Ρ€Π΅Π·ΡƒΠ»ΡŒΡ‚Π°Ρ‚. ΠžΠΏΡ€Π΅Π΄Π΅Π»ΠΈΡ‚Π΅ ΠΈΠ·ΠΎΠ±Ρ€Π°ΠΆΠ΅Π½ΠΈΠ΅ плоской Ρ„ΠΈΠ³ΡƒΡ€Ρ‹ ΠΊΠ°ΠΊ слайд, ΠΏΠ΅Ρ€Π΅Π²ΠΎΡ€ΠΎΡ‚ ΠΈΠ»ΠΈ ΠΏΠΎΠ²ΠΎΡ€ΠΎΡ‚.

    Π˜ΡΠΏΠΎΠ»ΡŒΠ·ΡƒΠΉΡ‚Π΅ Π»ΠΈΠ½Π΅ΠΉΠ½ΡƒΡŽ ΠΈ Ρ‚ΠΎΡ‡Π΅Ρ‡Π½ΡƒΡŽ ΡΠΈΠΌΠΌΠ΅Ρ‚Ρ€ΠΈΡŽ для обозначСния ΠΈ создания симмСтричных Ρ„ΠΈΠ³ΡƒΡ€.

    ΠžΠΏΡ€Π΅Π΄Π΅Π»ΠΈΡ‚Π΅, скаТитС ΠΈ ΠΏΠΎΠΊΠ°ΠΆΠΈΡ‚Π΅ врСмя с Ρ‚ΠΎΡ‡Π½ΠΎΡΡ‚ΡŒΡŽ Π΄ΠΎ часа, получаса ΠΈ Ρ‡Π΅Ρ‚Π²Π΅Ρ€Ρ‚ΠΈ часа. ΠžΠΏΡ€Π΅Π΄Π΅Π»ΠΈΡ‚Π΅, расскаТитС ΠΈ ΠΏΠΎΠΊΠ°ΠΆΠΈΡ‚Π΅ врСмя для 5- ΠΈ 1-ΠΌΠΈΠ½ΡƒΡ‚Π½Ρ‹Ρ… ΠΈΠ½Ρ‚Π΅Ρ€Π²Π°Π»ΠΎΠ².

    НайдитС ΠΏΡ€ΠΎΡˆΠ΅Π΄ΡˆΠ΅Π΅ врСмя Π² ΠΌΠΈΠ½ΡƒΡ‚Π°Ρ…, часах, днях ΠΈ нСдСлях. Π Π°Π·Π²ΠΈΠ²Π°ΠΉΡ‚Π΅ Π½Π°Π²Ρ‹ΠΊΠΈ измСрСния ΠΈ дСмонстрируйтС ΠΏΠΎΠ½ΠΈΠΌΠ°Π½ΠΈΠ΅ ΠΊΠΎΠ½Ρ†Π΅ΠΏΡ†ΠΈΠΉ, связанных с ΠΈΠ·ΠΌΠ΅Ρ€Π΅Π½ΠΈΠ΅ΠΌ Π²Ρ€Π΅ΠΌΠ΅Π½ΠΈ.

    Π Π΅ΡˆΠΈΡ‚Π΅ ΠΏΡ€ΠΎΠ±Π»Π΅ΠΌΡ‹ с ΠΈΡΡ‚Π΅ΠΊΡˆΠΈΠΌ Π²Ρ€Π΅ΠΌΠ΅Π½Π΅ΠΌ, ΠΈΡΠΏΠΎΠ»ΡŒΠ·ΡƒΡ Ρ‡ΠΈΡΠ»ΠΎΠ²ΡƒΡŽ строку.

    Π Π°ΡΡˆΠΈΡ„Ρ€ΠΎΠ²ΠΊΠ° расписания с использованиСм ΠΌΠΈΠ½ΡƒΡ‚, часов, Π΄Π½Π΅ΠΉ ΠΈ нСдСль.

    ΠžΠΏΡ€Π΅Π΄Π΅Π»ΠΈΡ‚Π΅ Π΅Π΄ΠΈΠ½ΠΈΡ†Ρ‹ Π΄Π»ΠΈΠ½Ρ‹. (дюйм, Ρ„ΡƒΡ‚, ярд, миля) ΠžΡ†Π΅Π½ΠΈΡ‚Π΅ ΠΈ сравнитС Π΄Π»ΠΈΠ½Ρƒ. Π˜Π·ΠΌΠ΅Ρ€ΡŒΡ‚Π΅ с Ρ‚ΠΎΡ‡Π½ΠΎΡΡ‚ΡŒΡŽ Π΄ΠΎ ΠΏΠΎΠ»ΡƒΠ΄ΡŽΠΉΠΌΠ°.

    ΠžΠΏΡ€Π΅Π΄Π΅Π»ΠΈΡ‚Π΅ Π΅Π΄ΠΈΠ½ΠΈΡ†Ρ‹ мощности. (Ρ‡Π°ΡˆΠΊΠ°, ΠΏΠΈΠ½Ρ‚Π°, ΠΊΠ²Π°Ρ€Ρ‚Π°, Π³Π°Π»Π»ΠΎΠ½) ΠžΡ†Π΅Π½ΠΈΡ‚Π΅ ΠΈ сравнитС Π²ΠΌΠ΅ΡΡ‚ΠΈΠΌΠΎΡΡ‚ΡŒ.

    ΠžΠΏΡ€Π΅Π΄Π΅Π»ΠΈΡ‚Π΅ Π΅Π΄ΠΈΠ½ΠΈΡ†Ρ‹ вСса. (унция, Ρ„ΡƒΠ½Ρ‚) ΠžΡ†Π΅Π½ΠΈΡ‚Π΅ ΠΈ сравнитС вСс.

    Π‘Ρ‡ΠΈΡ‚Π°ΠΉΡ‚Π΅ показания Ρ‚Π΅Ρ€ΠΌΠΎΠΌΠ΅Ρ‚Ρ€Π° с Ρ‚ΠΎΡ‡Π½ΠΎΡΡ‚ΡŒΡŽ Π΄ΠΎ блиТайшСго 5-градусного ΠΈΠ½Ρ‚Π΅Ρ€Π²Π°Π»Π°.

    ΠžΠΏΡ€Π΅Π΄Π΅Π»ΠΈΡ‚Π΅ Π΅Π΄ΠΈΠ½ΠΈΡ†Ρ‹ Π΄Π»ΠΈΠ½Ρ‹. (сантимСтр, Π΄Π΅Ρ†ΠΈΠΌΠ΅Ρ‚Ρ€, ΠΌΠ΅Ρ‚Ρ€) ΠžΡ†Π΅Π½ΠΈΡ‚Π΅ ΠΈ сравнитС Π΄Π»ΠΈΠ½Ρƒ. Π˜Π·ΠΌΠ΅Ρ€ΡŒΡ‚Π΅ с Ρ‚ΠΎΡ‡Π½ΠΎΡΡ‚ΡŒΡŽ Π΄ΠΎ сантимСтра.

    ΠžΠΏΡ€Π΅Π΄Π΅Π»ΠΈΡ‚Π΅ Π΅Π΄ΠΈΠ½ΠΈΡ†Ρ‹ мощности. (ΠΌΠΈΠ»Π»ΠΈΠ»ΠΈΡ‚Ρ€Ρ‹, Π»ΠΈΡ‚Ρ€Ρ‹) ΠžΡ†Π΅Π½ΠΈΡ‚Π΅ ΠΈ сравнитС Π΅ΠΌΠΊΠΎΡΡ‚ΡŒ.

    ΠžΠΏΡ€Π΅Π΄Π΅Π»ΠΈΡ‚Π΅ Π΅Π΄ΠΈΠ½ΠΈΡ†Ρ‹ массы. (Π³Ρ€Π°ΠΌΠΌΡ‹, ΠΊΠΈΠ»ΠΎΠ³Ρ€Π°ΠΌΠΌΡ‹) ΠžΡ†Π΅Π½ΠΈΡ‚Π΅ ΠΈ сравнитС массу.

    Π‘Ρ‡ΠΈΡ‚Π°ΠΉΡ‚Π΅ показания Ρ‚Π΅Ρ€ΠΌΠΎΠΌΠ΅Ρ‚Ρ€Π° с Ρ‚ΠΎΡ‡Π½ΠΎΡΡ‚ΡŒΡŽ Π΄ΠΎ блиТайшСго 5-градусного ΠΈΠ½Ρ‚Π΅Ρ€Π²Π°Π»Π°.

    ΠžΡ†Π΅Π½ΠΈΡ‚Π΅ ΠΎΠ±ΡŠΠ΅ΠΌΡ‹ ΠΎΠ±ΡŠΠ΅ΠΊΡ‚ΠΎΠ² Π² Π»ΠΈΡ‚Ρ€Π°Ρ… ΠΈ ΠΌΠΈΠ»Π»ΠΈΠ»ΠΈΡ‚Ρ€Π°Ρ… ΠΏΡƒΡ‚Π΅ΠΌ сравнСния с эталонными ΠΎΠ±ΡŠΠ΅ΠΊΡ‚Π°ΠΌΠΈ.

    РСшСниС Ρ€Π΅Π°Π»ΡŒΠ½Ρ‹Ρ… Π·Π°Π΄Π°Ρ‡, связанных с массой Π² ΠΊΠΈΠ»ΠΎΠ³Ρ€Π°ΠΌΠΌΠ°Ρ… ΠΈ Π³Ρ€Π°ΠΌΠΌΠ°Ρ… ΠΈ объСмом Π² Π»ΠΈΡ‚Ρ€Π°Ρ….

    Π˜Π·ΠΌΠ΅Ρ€ΡŒΡ‚Π΅ ΠΏΠ»ΠΎΡ‰Π°Π΄ΡŒ ΠΏΡ€ΡΠΌΠΎΡƒΠ³ΠΎΠ»ΡŒΠ½ΠΈΠΊΠ° с ΠΏΠΎΠΌΠΎΡ‰ΡŒΡŽ Π΅Π΄ΠΈΠ½ΠΈΡ‡Π½Ρ‹Ρ… ΠΊΠ²Π°Π΄Ρ€Π°Ρ‚ΠΎΠ².

    НайдитС ΠΏΠ»ΠΎΡ‰Π°Π΄ΡŒ Ρ„ΠΈΠ³ΡƒΡ€Ρ‹, считая Π΅Π΄ΠΈΠ½ΠΈΡ‡Π½Ρ‹Π΅ ΠΊΠ²Π°Π΄Ρ€Π°Ρ‚Ρ‹.

    НайдитС ΠΏΠ»ΠΎΡ‰Π°Π΄ΡŒ ΠΏΡ€ΡΠΌΠΎΡƒΠ³ΠΎΠ»ΡŒΠ½ΠΈΠΊΠ°, укладывая ΠΏΠ»ΠΈΡ‚ΠΊΡƒ ΠΈ умноТая Π΄Π»ΠΈΠ½Ρ‹ сторон.

    Π˜Π½Ρ‚Π΅Ρ€ΠΏΡ€Π΅Ρ‚ΠΈΡ€ΠΎΠ²Π°Ρ‚ΡŒ y = mx + b ΠΊΠ°ΠΊ Π»ΠΈΠ½Π΅ΠΉΠ½ΡƒΡŽ Ρ„ΡƒΠ½ΠΊΡ†ΠΈΡŽ.

    НайдитС ΠΏΠ»ΠΎΡ‰Π°Π΄ΡŒ ΠΏΡ€ΡΠΌΠΎΡƒΠ³ΠΎΠ»ΡŒΠ½ΠΈΠΊΠ°, ΡƒΠΌΠ½ΠΎΠΆΠΈΠ² Π΄Π»ΠΈΠ½Ρƒ ΠΈ ΡˆΠΈΡ€ΠΈΠ½Ρƒ.

    НайдитС ΠΏΠ»ΠΎΡ‰Π°Π΄ΡŒ ΠΏΡ€ΡΠΌΠΎΡƒΠ³ΠΎΠ»ΡŒΠ½ΠΈΠΊΠ°, Ρ€Π°Π·Π΄Π΅Π»ΠΈΠ² Π΅Π³ΠΎ Π½Π° Π΄Π²Π° ΠΌΠ΅Π½ΡŒΡˆΠΈΡ… ΠΏΡ€ΡΠΌΠΎΡƒΠ³ΠΎΠ»ΡŒΠ½ΠΈΠΊΠ°.

    НайдитС ΠΎΠ±Π»Π°ΡΡ‚ΡŒ, Ρ€Π°Π·Π»ΠΎΠΆΠΈΠ² составныС Ρ„ΠΎΡ€ΠΌΡ‹ Π½Π° ΠΏΡ€ΡΠΌΠΎΡƒΠ³ΠΎΠ»ΡŒΠ½ΠΈΠΊΠΈ ΠΈ Π΄ΠΎΠ±Π°Π²ΠΈΠ² области.

    НайдитС ΠΏΠ΅Ρ€ΠΈΠΌΠ΅Ρ‚Ρ€, считая Π΅Π΄ΠΈΠ½ΠΈΡ†Ρ‹ измСрСния ΠΈ складывая Π΄Π»ΠΈΠ½Ρ‹. Π˜Π·ΠΌΠ΅Ρ€ΡŒΡ‚Π΅, Ρ‡Ρ‚ΠΎΠ±Ρ‹ Π½Π°ΠΉΡ‚ΠΈ ΠΏΠ΅Ρ€ΠΈΠΌΠ΅Ρ‚Ρ€. Π’Ρ‹Π±Π΅Ρ€ΠΈΡ‚Π΅ ΡΠΎΠΎΡ‚Π²Π΅Ρ‚ΡΡ‚Π²ΡƒΡŽΡ‰ΡƒΡŽ ΠΌΠ΅Ρ‚ΠΊΡƒ для измСрСния.

    НайдитС ΠΎΠ±Π»Π°ΡΡ‚ΡŒ, считая Π΅Π΄ΠΈΠ½ΠΈΡ†Ρ‹. Π£ΠΌΠ½ΠΎΠΆΡŒΡ‚Π΅, Ρ‡Ρ‚ΠΎΠ±Ρ‹ Π½Π°ΠΉΡ‚ΠΈ ΠΏΠ»ΠΎΡ‰Π°Π΄ΡŒ. Π’Ρ‹Π±Π΅Ρ€ΠΈΡ‚Π΅ ΡΠΎΠΎΡ‚Π²Π΅Ρ‚ΡΡ‚Π²ΡƒΡŽΡ‰ΠΈΠ΅ ΠΌΠ΅Ρ‚ΠΊΠΈ измСрСния.

    Π‘Ρ€Π°Π²Π½ΠΈΡ‚Π΅ ΠΏΠ΅Ρ€ΠΈΠΌΠ΅Ρ‚Ρ€ ΠΈ ΠΏΠ»ΠΎΡ‰Π°Π΄ΡŒ.

    ΠžΡ‚ΠΎΠ±Ρ€Π°ΠΆΠ΅Π½ΠΈΠ΅ ΠΈ интСрпрСтация Π΄Π°Π½Π½Ρ‹Ρ… Π² Π²ΠΈΠ΄Π΅ ΠΏΠΈΠΊΡ‚ΠΎΠ³Ρ€Π°ΠΌΠΌ.

    ΠžΡ‚ΠΎΠ±Ρ€Π°ΠΆΠ΅Π½ΠΈΠ΅ ΠΈ интСрпрСтация Π΄Π°Π½Π½Ρ‹Ρ… Π² Π²ΠΈΠ΄Π΅ Π²Π΅Ρ€Ρ‚ΠΈΠΊΠ°Π»ΡŒΠ½Ρ‹Ρ… ΠΈ Π³ΠΎΡ€ΠΈΠ·ΠΎΠ½Ρ‚Π°Π»ΡŒΠ½Ρ‹Ρ… гистограмм.

    ΠžΡ‚ΠΎΠ±Ρ€Π°ΠΆΠ΅Π½ΠΈΠ΅ ΠΈ интСрпрСтация Π΄Π°Π½Π½Ρ‹Ρ… Π² Ρ‚Π°Π±Π»ΠΈΡ†Π°Ρ…, Π²ΠΊΠ»ΡŽΡ‡Π°Ρ Ρ‚Π°Π±Π»ΠΈΡ†Ρ‹ подсчСтов, Π΄Π°Π½Π½Ρ‹Ρ… ΠΈ частот.

    ΠžΡ‚ΠΎΠ±Ρ€Π°ΠΆΠ΅Π½ΠΈΠ΅ ΠΈ интСрпрСтация Π΄Π°Π½Π½Ρ‹Ρ… Π² частотных Ρ‚Π°Π±Π»ΠΈΡ†Π°Ρ… с использованиСм Π΄Π²ΡƒΡ… Π°Ρ‚Ρ€ΠΈΠ±ΡƒΡ‚ΠΎΠ².

    ΠžΠΏΡ€Π΅Π΄Π΅Π»ΠΈΡ‚Π΅ Π΄ΠΎΡΡ‚ΠΎΠ²Π΅Ρ€Π½ΠΎΡΡ‚ΡŒ, Π²Π΅Ρ€ΠΎΡΡ‚Π½ΠΎΡΡ‚ΡŒ ΠΈ ΡΠΏΡ€Π°Π²Π΅Π΄Π»ΠΈΠ²ΠΎΡΡ‚ΡŒ событий.

    ΠžΠΏΡ€Π΅Π΄Π΅Π»ΠΈΡ‚Π΅ ΠΈ пСрСчислитС всС Π²ΠΎΠ·ΠΌΠΎΠΆΠ½Ρ‹Π΅ исходы события.

    Π˜ΡΠΏΠΎΠ»ΡŒΠ·ΡƒΠΉΡ‚Π΅ чСтырСхэтапный ΠΌΠ΅Ρ‚ΠΎΠ΄ Polya для Ρ€Π΅ΡˆΠ΅Π½ΠΈΡ двухэтапных Π·Π°Π΄Π°Ρ‡ со словами с использованиСм Ρ‡Π΅Ρ‚Ρ‹Ρ€Π΅Ρ… ΠΎΠΏΠ΅Ρ€Π°Ρ†ΠΈΠΉ. ΠŸΡ€Π΅Π΄ΡΡ‚Π°Π²ΡŒΡ‚Π΅ эти ΠΏΡ€ΠΎΠ±Π»Π΅ΠΌΡ‹, ΠΈΡΠΏΠΎΠ»ΡŒΠ·ΡƒΡ уравнСния с Π±ΡƒΠΊΠ²ΠΎΠΉ, ΠΎΠ±ΠΎΠ·Π½Π°Ρ‡Π°ΡŽΡ‰Π΅ΠΉ нСизвСстноС.

    Π Π΅ΡˆΠΈΡ‚Π΅ двухэтапныС Π·Π°Π΄Π°Ρ‡ΠΈ со словами, ΠΈΡΠΏΠΎΠ»ΡŒΠ·ΡƒΡ Ρ‡Π΅Ρ‚Ρ‹Ρ€Π΅ ΠΎΠΏΠ΅Ρ€Π°Ρ†ΠΈΠΈ. ΠžΡ†Π΅Π½ΠΈΡ‚Π΅ Ρ€Π°Π·ΡƒΠΌΠ½ΠΎΡΡ‚ΡŒ ΠΎΡ‚Π²Π΅Ρ‚ΠΎΠ² с ΠΏΠΎΠΌΠΎΡ‰ΡŒΡŽ мыслСнных вычислСний ΠΈ стратСгий ΠΎΡ†Π΅Π½ΠΊΠΈ, Π²ΠΊΠ»ΡŽΡ‡Π°Ρ ΠΎΠΊΡ€ΡƒΠ³Π»Π΅Π½ΠΈΠ΅.

    ОбъСм ΠΈ ΠΏΠΎΡΠ»Π΅Π΄ΠΎΠ²Π°Ρ‚Π΅Π»ΡŒΠ½ΠΎΡΡ‚ΡŒ АвторскиС ΠΏΡ€Π°Π²Π°. Β© 2017 Edgenuity, Inc. ВсС ΠΏΡ€Π°Π²Π° Π·Π°Ρ‰ΠΈΡ‰Π΅Π½Ρ‹.

    Π˜Π½ΡΡ‚Ρ€ΡƒΠΌΠ΅Π½Ρ‚ поиска ΡƒΡ‡Π΅Π±Π½Ρ‹Ρ… занятий

    Поиск занятий Π² ΡƒΡ€ΠΎΠΊΠ΅ – ΠΎΠ΄ΠΈΠ½ ΠΈΠ· ΠΌΠ½ΠΎΠ³ΠΈΡ… ΠΏΠΎΠ»Π΅Π·Π½Ρ‹Ρ… инструмСнтов, ΠΊΠΎΡ‚ΠΎΡ€Ρ‹Π΅ Time4Learning ΠΏΡ€Π΅Π΄Π»Π°Π³Π°Π΅Ρ‚ своим участникам. БрСдство поиска занятий – это ярлык, с ΠΏΠΎΠΌΠΎΡ‰ΡŒΡŽ ΠΊΠΎΡ‚ΠΎΡ€ΠΎΠ³ΠΎ Ρ€ΠΎΠ΄ΠΈΡ‚Π΅Π»ΠΈ ΠΌΠΎΠ³ΡƒΡ‚ Π»Π΅Π³ΠΊΠΎ ΠΏΡ€ΠΎΡΠΌΠΎΡ‚Ρ€Π΅Ρ‚ΡŒ ΡƒΡ€ΠΎΠΊΠΈ ΠΈΠ»ΠΈ Π½Π°ΠΉΡ‚ΠΈ Π΄ΠΎΠΏΠΎΠ»Π½ΠΈΡ‚Π΅Π»ΡŒΠ½ΡƒΡŽ ΠΏΡ€Π°ΠΊΡ‚ΠΈΠΊΡƒ для своСго Ρ€Π΅Π±Π΅Π½ΠΊΠ°.

    ΠšΠ°ΠΆΠ΄ΠΎΠΌΡƒ ΡƒΡ€ΠΎΠΊΡƒ Π² ΡƒΡ‡Π΅Π±Π½ΠΎΠΉ ΠΏΡ€ΠΎΠ³Ρ€Π°ΠΌΠΌΠ΅ присвоСн ΡƒΠ½ΠΈΠΊΠ°Π»ΡŒΠ½Ρ‹ΠΉ Π½ΠΎΠΌΠ΅Ρ€ занятия, ΠΊΠΎΡ‚ΠΎΡ€Ρ‹ΠΉ Π² ΠΏΠ»Π°Π½Π°Ρ… ΡƒΡ€ΠΎΠΊΠΎΠ² называСтся Β«Π½ΠΎΠΌΠ΅Ρ€ΠΎΠΌ LAΒ». Π­Ρ‚ΠΈ Π½ΠΎΠΌΠ΅Ρ€Π° ΠΌΠΎΠΆΠ½ΠΎ Π½Π°ΠΉΡ‚ΠΈ Π»ΠΈΠ±ΠΎ Π½Π° страницах содСрТания ΠΈ ΠΏΠΎΡΠ»Π΅Π΄ΠΎΠ²Π°Ρ‚Π΅Π»ΡŒΠ½ΠΎΡΡ‚ΠΈ, Π»ΠΈΠ±ΠΎ Π² ΠΏΠ»Π°Π½Π°Ρ… ΡƒΡ€ΠΎΠΊΠΎΠ² Π½Π° Ρ€ΠΎΠ΄ΠΈΡ‚Π΅Π»ΡŒΡΠΊΠΎΠΉ ΠΈΠ½Ρ„ΠΎΡ€ΠΌΠ°Ρ†ΠΈΠΎΠ½Π½ΠΎΠΉ ΠΏΠ°Π½Π΅Π»ΠΈ.

    Для получСния Π΄ΠΎΠΏΠΎΠ»Π½ΠΈΡ‚Π΅Π»ΡŒΠ½ΠΎΠΉ ΠΈΠ½Ρ„ΠΎΡ€ΠΌΠ°Ρ†ΠΈΠΈ посСтитС наш Ρ€Π°Π·Π΄Π΅Π» подсказок ΠΈ ΠΏΠΎΠΌΠΎΡ‰ΠΈ, Π² ΠΊΠΎΡ‚ΠΎΡ€ΠΎΠΌ Π΅ΡΡ‚ΡŒ Π±ΠΎΠ»Π΅Π΅ подробная информация ΠΎ поисковикС дСйствий.

    Π”ΠΎΠΏΠΎΠ»Π½ΠΈΡ‚Π΅Π»ΡŒΠ½Ρ‹Π΅ рСсурсы ΠΏΠΎ ΠΌΠ°Ρ‚Π΅ΠΌΠ°Ρ‚ΠΈΠΊΠ΅ для Ρ‚Ρ€Π΅Ρ‚ΡŒΠ΅Π³ΠΎ класса

    Если вас ΠΈΠ½Ρ‚Π΅Ρ€Π΅ΡΡƒΡŽΡ‚ ΠΏΠ»Π°Π½Ρ‹ ΡƒΡ€ΠΎΠΊΠΎΠ² ΠΌΠ°Ρ‚Π΅ΠΌΠ°Ρ‚ΠΈΠΊΠΈ для Ρ‚Ρ€Π΅Ρ‚ΡŒΠ΅Π³ΠΎ класса, вас Ρ‚Π°ΠΊΠΆΠ΅ ΠΌΠΎΠ³ΡƒΡ‚ Π·Π°ΠΈΠ½Ρ‚Π΅Ρ€Π΅ΡΠΎΠ²Π°Ρ‚ΡŒ:

    Онлайн-учСбная ΠΏΡ€ΠΎΠ³Ρ€Π°ΠΌΠΌΠ° для домашнСго обучСния, послСшкольного ΠΈ Π»Π΅Ρ‚Π½Π΅Π³ΠΎ использования

    Если Π²Ρ‹ Ρ‚ΠΎΠ»ΡŒΠΊΠΎ ΠΈΠ·ΡƒΡ‡Π°Π΅Ρ‚Π΅ Time4Learning, ΠΌΡ‹ Ρ€Π΅ΠΊΠΎΠΌΠ΅Π½Π΄ΡƒΠ΅ΠΌ сначала ΠΏΠΎΡΠΌΠΎΡ‚Ρ€Π΅Ρ‚ΡŒ наши ΠΈΠ½Ρ‚Π΅Ρ€Π°ΠΊΡ‚ΠΈΠ²Π½Ρ‹Π΅ дСмонстрации ΡƒΡ€ΠΎΠΊΠΎΠ².

    Π—Π°Ρ€Π΅Π³ΠΈΡΡ‚Ρ€ΠΈΡ€ΡƒΠΉΡ‚Π΅ΡΡŒ Π½Π° Time4Learning ΠΈ ΠΏΠΎΠ»ΡƒΡ‡ΠΈΡ‚Π΅ доступ ΠΊ Ρ€Π°Π·Π½ΠΎΠΎΠ±Ρ€Π°Π·Π½Ρ‹ΠΌ ΠΎΠ±Ρ€Π°Π·ΠΎΠ²Π°Ρ‚Π΅Π»ΡŒΠ½Ρ‹ΠΌ ΠΌΠ°Ρ‚Π΅Ρ€ΠΈΠ°Π»Π°ΠΌ, ΠΊΠΎΡ‚ΠΎΡ€Ρ‹Π΅ ΡƒΠ²Π»Π΅ΠΊΡƒΡ‚ ΠΈ побудят вашСго Ρ€Π΅Π±Π΅Π½ΠΊΠ° Π΄ΠΎΠ±ΠΈΡ‚ΡŒΡΡ успСха. Π‘Π΄Π΅Π»Π°ΠΉΡ‚Π΅ Time4Learning Ρ‡Π°ΡΡ‚ΡŒΡŽ рСсурсов для домашнСго обучСния Π²Π°ΡˆΠΈΡ… Π΄Π΅Ρ‚Π΅ΠΉ.

    Π£Ρ‡Π΅Π±Π½Ρ‹ΠΉ ΠΏΠ»Π°Π½ Π½Π°Ρ‡Π°Π»ΡŒΠ½ΠΎΠΉ ΠΌΠ°Ρ‚Π΅ΠΌΠ°Ρ‚ΠΈΠΊΠΈ | УчСбная ΠΏΡ€ΠΎΠ³Ρ€Π°ΠΌΠΌΠ° ΠΏΠΎ ΠΌΠ°Ρ‚Π΅ΠΌΠ°Ρ‚ΠΈΠΊΠ΅ для 1-Π³ΠΎ класса | ΠŸΡ€ΠΎΠ³Ρ€Π°ΠΌΠΌΠ° 2-Π³ΠΎ класса ΠΏΠΎ ΠΌΠ°Ρ‚Π΅ΠΌΠ°Ρ‚ΠΈΠΊΠ΅

    2-й класс

    ΠžΠ±Π·ΠΎΡ€

    УчащиСся ΡƒΠ΄Π΅Π»ΡΡŽΡ‚ особоС Π²Π½ΠΈΠΌΠ°Π½ΠΈΠ΅ Ρ‡Π΅Ρ‚Ρ‹Ρ€Π΅ΠΌ критичСским областям, ΡƒΠΊΠ°Π·Π°Π½Π½Ρ‹ΠΌ Π² ΠžΠ±Ρ‰ΠΈΡ… государствСнных стандартах ΠΌΠ°Ρ‚Π΅ΠΌΠ°Ρ‚ΠΈΠΊΠΈ для Π²Ρ‚ΠΎΡ€ΠΎΠ³ΠΎ класса:

    • Π Π°ΡΡˆΠΈΡ€Π΅Π½ΠΈΠ΅ понимания дСсятичной систСмы счислСния
    • Π Π°Π·Π²ΠΈΡ‚ΠΈΠ΅ Π½Π°Π²Ρ‹ΠΊΠΎΠ² слоТСния ΠΈ вычитания
    • ИспользованиС стандартных Π΅Π΄ΠΈΠ½ΠΈΡ† Π»ΠΈΠ½Π΅ΠΉΠ½ΠΎΠ³ΠΎ измСрСния
    • ОписаниС ΠΈ Π°Π½Π°Π»ΠΈΠ· Ρ„ΠΎΡ€ΠΌ

    ΠŸΠ΅Ρ€Π²Ρ‹ΠΉ Π±Π»ΠΎΠΊ повторяСт ΠΈ Ρ€Π°ΡΡˆΠΈΡ€ΡΠ΅Ρ‚ слоТСниС ΠΈ Π²Ρ‹Ρ‡ΠΈΡ‚Π°Π½ΠΈΠ΅ Π² ΠΏΡ€Π΅Π΄Π΅Π»Π°Ρ… 20, помогая Π³Π°Ρ€Π°Π½Ρ‚ΠΈΡ€ΠΎΠ²Π°Ρ‚ΡŒ, Ρ‡Ρ‚ΠΎ второклассники Ρ€Π°Π±ΠΎΡ‚Π°ΡŽΡ‚ с ΠΏΠΎΠ½ΠΈΠΌΠ°Π½ΠΈΠ΅ΠΌ ΠΈ Π±Π΅Π³Π»ΠΎΡΡ‚ΡŒΡŽ Ρ„Π°ΠΊΡ‚ΠΎΠ² с Π½Π°Ρ‡Π°Π»Π° ΡƒΡ‡Π΅Π±Π½ΠΎΠ³ΠΎ Π³ΠΎΠ΄Π°.

    Π‘Π»ΠΎΠΊΠΈ 2, 3, 5 ΠΈ части Π±Π»ΠΎΠΊΠ° 7 посвящСны разряду ΠΈ слоТСнию ΠΈ Π²Ρ‹Ρ‡ΠΈΡ‚Π°Π½ΠΈΡŽ ΠΌΠ½ΠΎΠ³ΠΎΠ·Π½Π°Ρ‡Π½Ρ‹Ρ… чисСл. Π’ΠΎ врСмя этих Π±Π»ΠΎΠΊΠΎΠ² учащиСся учатся ΡΡ‡ΠΈΡ‚Π°Ρ‚ΡŒ ΠΏΠΎ пяти, дСсяткам ΠΈ сотням, дСсяткам ΠΈ Π΅Π΄ΠΈΠ½ΠΈΡ†Π°ΠΌ; Ρ‡ΠΈΡ‚Π°Ρ‚ΡŒ, ΠΏΠΈΡΠ°Ρ‚ΡŒ ΠΈ ΡΡ€Π°Π²Π½ΠΈΠ²Π°Ρ‚ΡŒ числа с 1000; ΠΈ Ρ€Π°Π·Π²ΠΈΡ‚ΡŒ свободноС Π²Π»Π°Π΄Π΅Π½ΠΈΠ΅ слоТСниСм ΠΈ Π²Ρ‹Ρ‡ΠΈΡ‚Π°Π½ΠΈΠ΅ΠΌ Π΄ΠΎ 100, Ρ€Π΅ΡˆΠ°Ρ ΠΈ ставя самыС Ρ€Π°Π·Π½Ρ‹Π΅ Π·Π°Π΄Π°Ρ‡ΠΈ со словами. ПозТС Π² этом Π³ΠΎΠ΄Ρƒ Π΄Π΅Ρ‚ΠΈ ΠΈΡΠΏΠΎΠ»ΡŒΠ·ΡƒΡŽΡ‚ ΠΊΠΎΠ½ΠΊΡ€Π΅Ρ‚Π½Ρ‹Π΅ ΠΌΠΎΠ΄Π΅Π»ΠΈ ΠΈ наброски, Π° Ρ‚Π°ΠΊΠΆΠ΅ стратСгии, основанныС Π½Π° разрядС, свойствах ΠΎΠΏΠ΅Ρ€Π°Ρ†ΠΈΠΉ ΠΈ ΡΠΎΠΎΡ‚Π½ΠΎΡˆΠ΅Π½ΠΈΠΈ ΠΌΠ΅ΠΆΠ΄Ρƒ слоТСниСм ΠΈ Π²Ρ‹Ρ‡ΠΈΡ‚Π°Π½ΠΈΠ΅ΠΌ, Ρ‡Ρ‚ΠΎΠ±Ρ‹ ΡΠΊΠ»Π°Π΄Ρ‹Π²Π°Ρ‚ΡŒ ΠΈ Π²Ρ‹Ρ‡ΠΈΡ‚Π°Ρ‚ΡŒ Π΄ΠΎ 1000.

    Π Π°Π·Π΄Π΅Π» 6 вращаСтся Π²ΠΎΠΊΡ€ΡƒΠ³ Π³Π΅ΠΎΠΌΠ΅Ρ‚Ρ€ΠΈΠΈ, построСния основ для понимания ΠΏΠ»ΠΎΡ‰Π°Π΄ΠΈ, объСма, конгруэнтности, сходства ΠΈ симмСтрии, ΠΊΠΎΠ³Π΄Π° учащиСся ΠΈΡΡΠ»Π΅Π΄ΡƒΡŽΡ‚, ΠΎΠΏΠΈΡΡ‹Π²Π°ΡŽΡ‚, строят, Ρ€ΠΈΡΡƒΡŽΡ‚, ΠΊΠΎΠΌΠ±ΠΈΠ½ΠΈΡ€ΡƒΡŽΡ‚, Ρ€Π°Π·Π»Π°Π³Π°ΡŽΡ‚ ΠΈ Π°Π½Π°Π»ΠΈΠ·ΠΈΡ€ΡƒΡŽΡ‚ Π΄Π²ΡƒΡ…- ΠΈ Ρ‚Ρ€Π΅Ρ…ΠΌΠ΅Ρ€Π½Ρ‹Π΅ Ρ„ΠΎΡ€ΠΌΡ‹.

    Π Π°Π·Π΄Π΅Π» 4 ΠΈ пСрвая Ρ‡Π°ΡΡ‚ΡŒ Π Π°Π·Π΄Π΅Π»Π° 7 посвящСны Π»ΠΈΠ½Π΅ΠΉΠ½Ρ‹ΠΌ измСрСниям, ΠΏΠΎΡΠΊΠΎΠ»ΡŒΠΊΡƒ учащиСся ΠΊΠΎΠ½ΡΡ‚Ρ€ΡƒΠΈΡ€ΡƒΡŽΡ‚ свои собствСнныС Π»ΠΈΠ½Π΅ΠΉΠΊΠΈ; ΠΎΡ†Π΅Π½ΠΈΠ²Π°Ρ‚ΡŒ ΠΈ ΠΈΠ·ΠΌΠ΅Ρ€ΡΡ‚ΡŒ Π² Π΄ΡŽΠΉΠΌΠ°Ρ…, Ρ„ΡƒΡ‚Π°Ρ…, ярдах, сантимСтрах ΠΈ ΠΌΠ΅Ρ‚Ρ€Π°Ρ…; ΠΈ Ρ€Π΅ΡˆΠ°Ρ‚ΡŒ Π·Π°Π΄Π°Ρ‡ΠΈ, связанныС с слоТСниСм, Π²Ρ‹Ρ‡ΠΈΡ‚Π°Π½ΠΈΠ΅ΠΌ ΠΈ сравнСниСм Π΄Π»ΠΈΠ½.

    ΠœΠΎΠ΄ΡƒΠ»ΡŒ 8 пСрСсматриваСт Π»ΠΈΠ½Π΅ΠΉΠ½Ρ‹Π΅ измСрСния Π² контСкстС Π½Π°ΡƒΠΊΠΈ ΠΈ Ρ‚Π΅Ρ…Π½ΠΈΠΊΠΈ, ΠΏΠΎΡΠΊΠΎΠ»ΡŒΠΊΡƒ студСнты ΠΈΠ·Π³ΠΎΡ‚Π°Π²Π»ΠΈΠ²Π°ΡŽΡ‚ ΠΈ Ρ‚Π΅ΡΡ‚ΠΈΡ€ΡƒΡŽΡ‚ ΠΊΠ°Ρ€Ρ‚ΠΎΠ½Π½Ρ‹Π΅ пандусы Ρ€Π°Π·Π»ΠΈΡ‡Π½Ρ‹Ρ… Ρ‚ΠΈΠΏΠΎΠ², Ρ‡Ρ‚ΠΎΠ±Ρ‹ ΠΈΡΡΠ»Π΅Π΄ΠΎΠ²Π°Ρ‚ΡŒ Π½Π΅ΠΊΠΎΡ‚ΠΎΡ€Ρ‹Π΅ ΠΈΠ· Ρ„Π°ΠΊΡ‚ΠΎΡ€ΠΎΠ², ΠΊΠΎΡ‚ΠΎΡ€Ρ‹Π΅ Π·Π°ΡΡ‚Π°Π²Π»ΡΡŽΡ‚ ΡˆΠ°Ρ€ΠΈΠΊΠΈ ΠΊΠ°Ρ‚ΠΈΡ‚ΡŒΡΡ всС дальшС ΠΈ быстрСС. Π’ процСссС ΠΎΠ½ΠΈ Π³Π΅Π½Π΅Ρ€ΠΈΡ€ΡƒΡŽΡ‚ Π΄Π°Π½Π½Ρ‹Π΅, ΠΌΠ½ΠΎΠ³ΠΎΠΊΡ€Π°Ρ‚Π½ΠΎ измСряя расстояния ΠΌΠ΅ΠΆΠ΄Ρƒ ΠΌΡ€Π°ΠΌΠΎΡ€Π½Ρ‹ΠΌΠΈ Ρ€ΠΎΠ»ΠΈΠΊΠ°ΠΌΠΈ, ΠΎΠ±ΡŠΠ΅Π΄ΠΈΠ½ΡΡŽΡ‚ свои Π΄Π°Π½Π½Ρ‹Π΅ ΠΈ вводят ΠΈΡ… Π½Π° Π»ΠΈΠ½Π΅ΠΉΠ½Ρ‹Ρ… Π³Ρ€Π°Ρ„ΠΈΠΊΠ°Ρ…, Ρ‡Ρ‚ΠΎΠ±Ρ‹ Π»ΡƒΡ‡ΡˆΠ΅ Π²ΠΈΠ΄Π΅Ρ‚ΡŒ, ΠΏΠΎΠ½ΠΈΠΌΠ°Ρ‚ΡŒ ΠΈ Π°Π½Π°Π»ΠΈΠ·ΠΈΡ€ΠΎΠ²Π°Ρ‚ΡŒ, ΠΊΠ°ΠΊ ΠΌΠ°Π½ΠΈΠΏΡƒΠ»ΠΈΡ€ΠΎΠ²Π°Π½ΠΈΠ΅ Ρ€Π°Π·Π»ΠΈΡ‡Π½Ρ‹ΠΌΠΈ ΠΏΠ΅Ρ€Π΅ΠΌΠ΅Π½Π½Ρ‹ΠΌΠΈ влияСт Π½Π° Ρ€Π΅Π·ΡƒΠ»ΡŒΡ‚Π°Ρ‚Ρ‹.

    ΠšΠΎΠ½Ρ‚Π΅Π½Ρ‚ структуры

    Π Π°Π·Π΄Π΅Π» 1 Π¦ΠΈΡ„Ρ€Ρ‹ Π² Ρ„Π°ΠΊΡ‚Π°Ρ…
    • Π‘ΠΎΡ€Ρ‚ΠΈΡ€ΠΎΠ²ΠΊΠ° ΠΈ построСниС Π³Ρ€Π°Ρ„ΠΈΠΊΠΎΠ²
    • ЧисловыС Ρ„Π°ΠΊΡ‚Ρ‹ с ΠΏΠΎΠΌΠΎΡ‰ΡŒΡŽ Number Rack
    • Π’Π²Π΅Π΄Π΅Π½ΠΈΠ΅ Π² стратСгии слоТСния ΠΈ вычитания
    • Π‘Π²ΠΎΠ±ΠΎΠ΄Π½ΠΎΠ΅ Π²Π»Π°Π΄Π΅Π½ΠΈΠ΅ с Π΄ΠΎΠ±Π°Π²Π»Π΅Π½ΠΈΠ΅ΠΌ Ρ„Π°ΠΊΡ‚ΠΎΠ² ΠΊ Π΄Π²Π°Π΄Ρ†Π°Ρ‚ΠΈ
    , Π±Π»ΠΎΠΊ 2, Π·Π½Π°Ρ‡Π΅Π½ΠΈΠ΅ ΠΈ ΠΈΠ·ΠΌΠ΅Ρ€Π΅Π½ΠΈΠ΅ с Π±ΠΎΠ±ΠΎΠ²Ρ‹ΠΌΠΈ стСблями Π”ΠΆΠ΅ΠΊΠ°
    • ΠŸΠΎΠ΄ΡΡ‡Π΅Ρ‚ ΠΈ ΠΌΠΎΠ΄Π΅Π»ΠΈΡ€ΠΎΠ²Π°Π½ΠΈΠ΅ Π΄Π²ΡƒΡ…- ΠΈ Ρ‚Ρ€Π΅Ρ…Π·Π½Π°Ρ‡Π½Ρ‹Ρ… чисСл
    • Π˜Π·ΠΌΠ΅Ρ€Π΅Π½ΠΈΠ΅ гигантских Π±ΠΎΠ±ΠΎΠ² Π”ΠΆΠ΅ΠΊΠ° с ΠΏΠΎΠΌΠΎΡ‰ΡŒΡŽ дСсятков
    • Π”ΠΎΠ±Π°Π²Π»Π΅Π½ΠΈΠ΅ Π² ΠΎΡ‚ΠΊΡ€Ρ‹Ρ‚ΡƒΡŽ Ρ‡ΠΈΡΠ»ΠΎΠ²ΡƒΡŽ строку
    • ΠœΡ‹ΡΠ»ΠΈΡ‚ΡŒ Π΄Π²ΠΎΠΈΠΌΠΈ
    Π‘Π»ΠΎΠΊ 3 Π‘Π»ΠΎΠΆΠ΅Π½ΠΈΠ΅ ΠΈ Π²Ρ‹Ρ‡ΠΈΡ‚Π°Π½ΠΈΠ΅ Π² ΠΏΡ€Π΅Π΄Π΅Π»Π°Ρ… ста
    Π‘Π»ΠΎΠΊ 4 Π˜Π·ΠΌΠ΅Ρ€Π΅Π½ΠΈΠ΅
    • дюймов ΠΈ Ρ„ΡƒΡ‚ΠΎΠ²
    • Π”ΡŽΠΉΠΌΡ‹, Ρ„ΡƒΡ‚Ρ‹ ΠΈ ярды
    • ΠŸΡ€ΠΎΠΏΠΎΡ€Ρ†ΠΈΠΈ ΠΈ Π΄ΠΎΠ»ΠΈ с Π³ΠΈΠ³Π°Π½Ρ‚ΠΎΠΌ
    • ΠœΡ‹ΡΠ»ΠΈΡ‚ΡŒ Ρ‚Ρ€ΠΎΠΉΠΊΠ°ΠΌΠΈ
    Разрядная ΡΡ‚ΠΎΠΈΠΌΠΎΡΡ‚ΡŒ Π΅Π΄ΠΈΠ½ΠΈΡ†Ρ‹ 5 Π΄ΠΎ ΠΎΠ΄Π½ΠΎΠΉ тысячи ΠΈ
    Π‘Π»ΠΎΠΊ 6 ГСомСтрия
    • Атрибуты Π΄Π²ΡƒΠΌΠ΅Ρ€Π½Ρ‹Ρ… Ρ„ΠΎΡ€ΠΌ
    • ИсслСдованиС области ΠΈ массивов
    • БоставлСниС ΠΈ Ρ€Π°Π·Π»ΠΎΠΆΠ΅Π½ΠΈΠ΅ Ρ„ΠΈΠ³ΡƒΡ€ для пэчворка
    • Π€Ρ€Π°ΠΊΡ†ΠΈΠΈ пэчворк
    Π‘Π»ΠΎΠΊ 7 Π˜Π·ΠΌΠ΅Ρ€Π΅Π½ΠΈΠ΅, Π΄Ρ€ΠΎΠ±ΠΈ ΠΈ ΠΌΠ½ΠΎΠ³ΠΎΠ·Π½Π°Ρ‡Π½Ρ‹Π΅ вычислСния с Π³ΠΎΠ»ΠΎΠ΄Π½Ρ‹ΠΌΠΈ ΠΌΡƒΡ€Π°Π²ΡŒΡΠΌΠΈ
    Π‘Π»ΠΎΠΊ 8 Π˜Π·ΠΌΠ΅Ρ€Π΅Π½ΠΈΠ΅, Π΄Π°Π½Π½Ρ‹Π΅ ΠΈ ΠΌΠ½ΠΎΠ³ΠΎΠ·Π½Π°Ρ‡Π½Ρ‹Π΅ вычислСния с ΠΌΡ€Π°ΠΌΠΎΡ€Π½Ρ‹ΠΌΠΈ Π²Π°Π»ΠΊΠ°ΠΌΠΈ
    НомСр Π£Π³ΠΎΠ»
    • Π‘Π΅Ρ‚ΠΊΠ° калСндаря
    • ΠšΠΎΠ»Π»Π΅ΠΊΡ†ΠΈΠΎΠ½Π΅Ρ€ калСндаря
    • Π•ΠΆΠ΅Π΄Π½Π΅Π²Π½Ρ‹ΠΉ ΠΏΡ€ΡΠΌΠΎΡƒΠ³ΠΎΠ»ΡŒΠ½ΠΈΠΊ
    • Π‘Π΅Π³Π»ΠΎΡΡ‚ΡŒ вычислСний
    • НомСрная строка
    ΠŸΠΎΠ΄Ρ€ΠΎΠ±Π½Π΅Π΅ ΠΎ Number Corner

    CUBES ΠœΠ°Ρ‚Π΅ΠΌΠ°Ρ‚ΠΈΡ‡Π΅ΡΠΊΠ°Ρ стратСгия – Lucky Little Learners

    Π‘Ρ‚ΡƒΠ΄Π΅Π½Ρ‚Ρ‹, каТСтся, видят ΠΏΡ€ΠΎΠ±Π»Π΅ΠΌΡƒ Π² рассказС ΠΈ Π·Π°ΠΌΠΈΡ€Π°ΡŽΡ‚.Π—Π°Π΄Π°Ρ‡ΠΈ с рассказом ΠΎΠΊΠ°Π·Ρ‹Π²Π°ΡŽΡ‚ΡΡ слоТной Π·Π°Π΄Π°Ρ‡Π΅ΠΉ для понимания учащимися. ΠœΠ°Ρ‚Π΅ΠΌΠ°Ρ‚ΠΈΡ‡Π΅ΡΠΊΠ°Ρ стратСгия CUBES – ΠΎΡ‚Π»ΠΈΡ‡Π½Ρ‹ΠΉ инструмСнт для учащихся, ΠΏΠΎΠΌΠΎΠ³Π°ΡŽΡ‰ΠΈΠΉ ΡƒΡΠΏΠ΅ΡˆΠ½ΠΎ Ρ€Π΅ΡˆΠ°Ρ‚ΡŒ ΡΡŽΠΆΠ΅Ρ‚Π½Ρ‹Π΅ Π·Π°Π΄Π°Ρ‡ΠΈ.

    Π§Ρ‚ΠΎ Ρ‚Π°ΠΊΠΎΠ΅ матСматичСская стратСгия CUBES

    ΠœΠ°Ρ‚Π΅ΠΌΠ°Ρ‚ΠΈΡ‡Π΅ΡΠΊΠ°Ρ стратСгия CUBES – это простой инструмСнт, с ΠΏΠΎΠΌΠΎΡ‰ΡŒΡŽ ΠΊΠΎΡ‚ΠΎΡ€ΠΎΠ³ΠΎ учитСля ΠΌΠΎΠ³ΡƒΡ‚ Π½Π°ΡƒΡ‡ΠΈΡ‚ΡŒ своих ΡƒΡ‡Π΅Π½ΠΈΠΊΠΎΠ², прСдлагая ΠΈΠΌ ΠΏΠΎΡˆΠ°Π³ΠΎΠ²Ρ‹Π΅ практичСскиС шаги, Ρ‡Ρ‚ΠΎΠ±Ρ‹ Ρ€Π°Π·ΠΎΠ±Ρ€Π°Ρ‚ΡŒ ΠΈ ΠΏΠΎΠ½ΡΡ‚ΡŒ, ΠΎ Ρ‡Π΅ΠΌ ΠΈΠ΄Π΅Ρ‚ Ρ€Π΅Ρ‡ΡŒ Π² Π·Π°Π΄Π°Ρ‡Π΅-рассказС. КаТдая Π±ΡƒΠΊΠ²Π° CUBES ΠΎΠ±ΠΎΠ·Π½Π°Ρ‡Π°Π΅Ρ‚ элСмСнт дСйствия, ΠΊΠΎΡ‚ΠΎΡ€Ρ‹ΠΉ учащиСся выполнят со своими Π·Π°Π΄Π°Ρ‡Π°ΠΌΠΈ со словами.

    ΠžΡ‚ΠΊΠ°Π· ΠΎΡ‚ отвСтствСнности: я Π½Π΅ сторонник использования стратСгии CUBES Π² качСствС Π•Π”Π˜ΠΠ‘Π’Π’Π•ΠΠΠžΠ™ стратСгии для Ρ€Π΅ΡˆΠ΅Π½ΠΈΡ ΡΡŽΠΆΠ΅Ρ‚Π½Ρ‹Ρ… Π·Π°Π΄Π°Ρ‡. Π― Π΄Π΅ΠΉΡΡ‚Π²ΠΈΡ‚Π΅Π»ΡŒΠ½ΠΎ ΡΡ‡ΠΈΡ‚Π°ΡŽ, Ρ‡Ρ‚ΠΎ Π΄Ρ€ΡƒΠ³ΠΈΠ΅ стратСгии, Ρ‚Π°ΠΊΠΈΠ΅ ΠΊΠ°ΠΊ составлСниС списка, рисованиС ΠΊΠ°Ρ€Ρ‚ΠΈΠ½ΠΊΠΈ, ΡƒΠ³Π°Π΄Ρ‹Π²Π°Π½ΠΈΠ΅ ΠΈ ΠΏΡ€ΠΎΠ²Π΅Ρ€ΠΊΠ°, Ρ€Π°Π·Ρ‹Π³Ρ€Ρ‹Π²Π°Π½ΠΈΠ΅, составлСниС Ρ‚Π°Π±Π»ΠΈΡ†Ρ‹, использованиС ΠΏΡ€Π΅Π΄ΠΌΠ΅Ρ‚ΠΎΠ² ΠΈ составлСниС числового прСдлоТСния, – это лишь Π½Π΅ΠΊΠΎΡ‚ΠΎΡ€Ρ‹Π΅ ΠΈΠ· Ρ‚Π΅Ρ…, ΠΊΠΎΡ‚ΠΎΡ€Ρ‹ΠΌ Π½ΡƒΠΆΠ½ΠΎ Π½Π°ΡƒΡ‡ΠΈΡ‚ΡŒ Π² ΠΏΠ΅Ρ€Π²ΡƒΡŽ ΠΎΡ‡Π΅Ρ€Π΅Π΄ΡŒ.

    C – ΠΎΠ±Π²Π΅Π΄ΠΈΡ‚Π΅ числа

    ПослС Ρ‚ΠΎΠ³ΠΎ, ΠΊΠ°ΠΊ студСнты ΠΏΡ€ΠΎΡ‡ΠΈΡ‚Π°ΡŽΡ‚ Π·Π°Π΄Π°Ρ‡Ρƒ со словами Π² ΠΏΠ΅Ρ€Π²Ρ‹ΠΉ Ρ€Π°Π·, попроситС ΠΈΡ… Π²Π΅Ρ€Π½ΡƒΡ‚ΡŒΡΡ ΠΈ обвСсти всС числа ΠΈΠ»ΠΈ числовыС слова Π² рассказС.

    U – ΠΏΠΎΠ΄Ρ‡Π΅Ρ€ΠΊΠ½ΠΈΡ‚Π΅ вопрос

    На ΡΠ»Π΅Π΄ΡƒΡŽΡ‰Π΅ΠΌ этапС учащиСся ΠΏΠΎΠ΄Ρ‡Π΅Ρ€ΠΊΠΈΠ²Π°ΡŽΡ‚ вопрос, ΠΊΠΎΡ‚ΠΎΡ€Ρ‹ΠΉ содСрТится Π² Π·Π°Π΄Π°Ρ‡Π΅-рассказС. НаучитС студСнтов Π΄ΡƒΠΌΠ°Ρ‚ΡŒ Π½Π°Π΄ вопросом ΠΈ Ρ€Π΅ΡˆΠΈΡ‚ΡŒ, Ρ‡Ρ‚ΠΎ ΠΈΠΌΠ΅Π½Π½ΠΎ ΠΈΠΌ задаСтся Π² вопросС с числами.

    Π—Π°Π³Ρ€ΡƒΠ·ΠΈΡ‚Π΅ эти ΠΊΠ°Ρ€Ρ‚ΠΎΡ‡ΠΊΠΈ Π·Π°Π΄Π°Ρ‡-рассказов здСсь

    B – это ΠΏΠΎΠ»Π΅ для ΠΊΠ»ΡŽΡ‡Π΅Π²Ρ‹Ρ… слов

    Когда студСнты обводят ΠΊΠ»ΡŽΡ‡Π΅Π²Ρ‹Π΅ слова Ρ€Π°ΠΌΠΊΠΎΠΉ, ΠΎΠ½ΠΈ часто Π²ΡΡ‚Ρ€Π΅Ρ‡Π°ΡŽΡ‚ΡΡ Π² вопросС, ΠΊΠΎΡ‚ΠΎΡ€Ρ‹ΠΉ Π±Ρ‹Π» Ρ‚ΠΎΠ»ΡŒΠΊΠΎ Ρ‡Ρ‚ΠΎ ΠΏΠΎΠ΄Ρ‡Π΅Ρ€ΠΊΠ½ΡƒΡ‚.Π­Ρ‚ΠΎΡ‚ шаг ΠΌΠΎΠΆΠ΅Ρ‚ Π±Ρ‹Ρ‚ΡŒ Π½Π΅ΠΌΠ½ΠΎΠ³ΠΎ слоТным Π² ΠΎΠΏΡ€Π΅Π΄Π΅Π»Π΅Π½Π½Ρ‹Ρ… ситуациях. НапримСр: Β«Π£ Π”ΠΆΠΎΠ½Π° 16 Π·Π΅Π»Π΅Π½Ρ‹Ρ… яблок. Π—Π΅Π»Π΅Π½Ρ‹Ρ… яблок Ρƒ Π½Π΅Π³ΠΎ Π½Π° 4 большС, Ρ‡Π΅ΠΌ красных. Бколько красных яблок Ρƒ Π”ΠΆΠΎΠ½Π°? Β» Π’ этом случаС ΡƒΡ‡Π΅Π½ΠΈΠΊΠ°ΠΌ Π½ΡƒΠΆΠ½ΠΎ ΠΎΡΡ‚Π°Π½ΠΎΠ²ΠΈΡ‚ΡŒΡΡ ΠΈ ΠΏΠΎΠ΄ΡƒΠΌΠ°Ρ‚ΡŒβ€¦ Β«Π’ Ρ‡Π΅ΠΌ вопрос?Β» ΠΈΠ»ΠΈ “Π§Ρ‚ΠΎ Π½ΡƒΠΆΠ½ΠΎ Π·Π½Π°Ρ‚ΡŒ Π² вопросС?”

    E для удалСния лишнСй ΠΈΠ½Ρ„ΠΎΡ€ΠΌΠ°Ρ†ΠΈΠΈ

    НСкоторыС учитСля K-2 ΠΏΡ€Π΅Π΄ΠΏΠΎΡ‡ΠΈΡ‚Π°ΡŽΡ‚ ΠΏΡ€ΠΎΠΏΡƒΡΠΊΠ°Ρ‚ΡŒ шаг E, ΠΈ Π½Π΅ΠΊΠΎΡ‚ΠΎΡ€Ρ‹Π΅ ΡΡ‡ΠΈΡ‚Π°ΡŽΡ‚ Π΅Π³ΠΎ ΠΏΠΎΠ»Π΅Π·Π½Ρ‹ΠΌ. Π­Ρ‚ΠΎΡ‚ шаг Ρ‚Ρ€Π΅Π±ΡƒΠ΅Ρ‚, Ρ‡Ρ‚ΠΎΠ±Ρ‹ студСнты Π²Π΅Ρ€Π½ΡƒΠ»ΠΈΡΡŒ ΠΊ ΠΏΡ€ΠΎΠ±Π»Π΅ΠΌΠ΅ рассказа, Ρ‡Ρ‚ΠΎΠ±Ρ‹ Ρ€Π΅ΡˆΠΈΡ‚ΡŒ, Π΅ΡΡ‚ΡŒ Π»ΠΈ какая-Π»ΠΈΠ±ΠΎ информация, ΠΊΠΎΡ‚ΠΎΡ€ΡƒΡŽ ΠΌΠΎΠΆΠ½ΠΎ ΠΏΡ€ΠΎΠΏΡƒΡΡ‚ΠΈΡ‚ΡŒ ΠΈΠ»ΠΈ ΠΏΡ€ΠΎΠΈΠ³Π½ΠΎΡ€ΠΈΡ€ΠΎΠ²Π°Ρ‚ΡŒ.

    S ΠΏΡ€Π΅Π΄Π½Π°Π·Π½Π°Ρ‡Π΅Π½ для Ρ€Π΅ΡˆΠ΅Π½ΠΈΡ ΠΈ ΠΏΡ€ΠΎΠ²Π΅Ρ€ΠΊΠΈ ΠΏΡ€ΠΎΠ±Π»Π΅ΠΌΡ‹

    ПослС Ρ‚ΠΎΠ³ΠΎ, ΠΊΠ°ΠΊ учащиСся ΠΏΡ€ΠΎΡˆΠ»ΠΈ этапы матСматичСской стратСгии CUBES, ΠΎΠ½ΠΈ Π½Π΅ΠΌΠ½ΠΎΠ³ΠΎ ΠΏΠΎΡ€Π°Π±ΠΎΡ‚Π°Π»ΠΈ с ΠΏΡ€ΠΎΠ±Π»Π΅ΠΌΠΎΠΉ рассказа. Они ΠΏΠΎΠ»ΡƒΡ‡ΠΈΠ»ΠΈ Ρ‡Π΅Ρ‚ΠΊΠΎΠ΅ прСдставлСниС ΠΎ Ρ‚ΠΎΠΌ, Π² Ρ‡Π΅ΠΌ ΡΡƒΡ‚ΡŒ ΠΏΡ€ΠΎΠ±Π»Π΅ΠΌΡ‹ ΠΈ ΠΎ Ρ‡Π΅ΠΌ ΡΠΏΡ€Π°ΡˆΠΈΠ²Π°ΡŽΡ‚. ПослСдний шаг – Π²Π·ΡΡ‚ΡŒ числа, Ρ€Π΅ΡˆΠΈΡ‚ΡŒ, Ρ‡Ρ‚ΠΎ с Π½ΠΈΠΌΠΈ Π΄Π΅Π»Π°Ρ‚ΡŒ, Ρ€Π΅ΡˆΠΈΡ‚ΡŒ, Π° Π·Π°Ρ‚Π΅ΠΌ ΠΏΡ€ΠΎΠ²Π΅Ρ€ΠΈΡ‚ΡŒ. НаучитС студСнтов ΡΠΏΡ€Π°ΡˆΠΈΠ²Π°Ρ‚ΡŒ сСбя: Β«Π˜ΠΌΠ΅Π΅Ρ‚ Π»ΠΈ ΠΎΡ‚Π²Π΅Ρ‚ смысл?Β»

    Π—Π°Π³Ρ€ΡƒΠ·ΠΈΡ‚Π΅ эти ΠΊΠ°Ρ€Ρ‚ΠΎΡ‡ΠΊΠΈ Π·Π°Π΄Π°Ρ‡-рассказов здСсь.

    Якорная Π΄ΠΈΠ°Π³Ρ€Π°ΠΌΠΌΠ° ΠΈ бСсплатный студСнчСский шаблон

    Якорная Π΄ΠΈΠ°Π³Ρ€Π°ΠΌΠΌΠ° – ΠΎΡ‚Π»ΠΈΡ‡Π½Ρ‹ΠΉ способ ΠΎΠ±ΡƒΡ‡ΠΈΡ‚ΡŒ ΠΈ ΠΏΡ€ΠΎΠ΄Π΅ΠΌΠΎΠ½ΡΡ‚Ρ€ΠΈΡ€ΠΎΠ²Π°Ρ‚ΡŒ ΠΌΠ°Ρ‚Π΅ΠΌΠ°Ρ‚ΠΈΡ‡Π΅ΡΠΊΡƒΡŽ ΡΡ‚Ρ€Π°Ρ‚Π΅Π³ΠΈΡŽ CUBES Π² классС. Π’ΠΎΡ‚ бСсплатный шаблон Π΄ΠΈΠ°Π³Ρ€Π°ΠΌΠΌΡ‹ привязки учащихся, ΠΊΠΎΡ‚ΠΎΡ€Ρ‹ΠΉ Π²Ρ‹ ΠΌΠΎΠΆΠ΅Ρ‚Π΅ Π΄Π°Ρ‚ΡŒ своим ΡƒΡ‡Π΅Π½ΠΈΠΊΠ°ΠΌ Π·Π°ΠΏΠΎΠ»Π½ΠΈΡ‚ΡŒ, ΠΊΠΎΠ³Π΄Π° Π²Ρ‹ ΠΏΡ€ΠΎΡ…ΠΎΠ΄ΠΈΡ‚Π΅ Ρ‡Π΅Ρ€Π΅Π· Π΄ΠΈΠ°Π³Ρ€Π°ΠΌΠΌΡƒ привязки вмСстС с классом. Он доступСн ΠΊΠ°ΠΊ для стратСгии CUBS, Ρ‚Π°ΠΊ ΠΈ для CUBES.

    ΠšΡƒΠΏΠΈΡ‚ΡŒ ΠΊΠ°Ρ€Ρ‚ΠΎΡ‡ΠΊΠΈ с заданиями ΠΏΠΎ ΠΌΠ°Ρ‚Π΅ΠΌΠ°Ρ‚ΠΈΠΊΠ΅

    ΠŸΠ΅Ρ‡Π°Ρ‚Π½Ρ‹Π΅ ΠΊΠ°Ρ€Ρ‚ΠΎΡ‡ΠΊΠΈ с заданиями ΠΏΠΎ ΠΌΠ°Ρ‚Π΅ΠΌΠ°Ρ‚ΠΈΠΊΠ΅ для 1-Π³ΠΎ ΠΈ 2-Π³ΠΎ классов

    Π¦ΠΈΡ„Ρ€ΠΎΠ²Ρ‹Π΅ задания ΠΏΠΎ ΠΌΠ°Ρ‚Π΅ΠΌΠ°Ρ‚ΠΈΠΊΠ΅ для 1-Π³ΠΎ ΠΈ 2-Π³ΠΎ классов

    Π‘ΠΎΡ…Ρ€Π°Π½ΠΈΡ‚ΡŒ эту идСю

    НС совсСм Π³ΠΎΡ‚ΠΎΠ²Ρ‹ Ρ€Π΅Π°Π»ΠΈΠ·ΠΎΠ²Π°Ρ‚ΡŒ эту идСю Π² своСм классС? Π©Π΅Π»ΠΊΠ½ΠΈΡ‚Π΅ здСсь, Ρ‡Ρ‚ΠΎΠ±Ρ‹ Π·Π°ΠΊΡ€Π΅ΠΏΠΈΡ‚ΡŒ эту идСю Π½Π° своСй доскС Pinterest.

    БСрия

    Β«ΠœΠ°Ρ‚Π΅ΠΌΠ°Ρ‚ΠΈΠΊΠ° Π½Π° ΠΏΡ€Π°ΠΊΡ‚ΠΈΠΊΠ΅Β» для Ρ‚Ρ€Π΅Π½Π΅Ρ€ΠΎΠ² ΠΈ ΡƒΡ‡ΠΈΡ‚Π΅Π»Π΅ΠΉ ΠΌΠ°Ρ‚Π΅ΠΌΠ°Ρ‚ΠΈΠΊΠΈ классов K-5

    КаТдая ΡƒΡ‡Π΅Π±Π½ΠΈΠΊ для уровня класса ΠΎΡ€Π³Π°Π½ΠΈΠ·ΠΎΠ²Π°Π½ Π² ΠΌΠΎΠ΄ΡƒΠ»ΠΈ, ΠΊΠΎΡ‚ΠΎΡ€Ρ‹Π΅ Ρ‚Ρ‰Π°Ρ‚Π΅Π»ΡŒΠ½ΠΎ Ρ€Π°ΡΠΊΡ€Ρ‹Π²Π°ΡŽΡ‚ ΠΊΠΎΠ½ΠΊΡ€Π΅Ρ‚Π½Ρ‹ΠΉ ΠΌΠ°Ρ‚Π΅Ρ€ΠΈΠ°Π» ΠΏΠΎ ΠΌΠ°Ρ‚Π΅ΠΌΠ°Ρ‚ΠΈΠΊΠ΅, ΠΏΡ€Π΅ΠΏΠΎΠ΄Π°Π²Π°Π΅ΠΌΡ‹ΠΉ Π² ΠΊΠ°ΠΆΠ΄ΠΎΠΌ классС, K – 5. ВмСсто Ρ‚ΠΎΠ³ΠΎ, Ρ‡Ρ‚ΠΎΠ±Ρ‹ ΠΈΡΠΏΠΎΠ»ΡŒΠ·ΠΎΠ²Π°Ρ‚ΡŒ ΠΈΡ… ΠΏΠΎ порядку, учитСля ΠΌΠΎΠ³ΡƒΡ‚ Π²Ρ‹Π±ΠΈΡ€Π°Ρ‚ΡŒ ΠΌΠΎΠ΄ΡƒΠ»ΠΈ ΠΏΠΎ ΠΌΠ΅Ρ€Π΅ нСобходимости, исходя ΠΈΠ· своСй ΠΊΠ°Ρ€Ρ‚Ρ‹ ΡƒΡ‡Π΅Π±Π½ΠΎΠ³ΠΎ ΠΏΠ»Π°Π½Π°, областСй, Π² ΠΊΠΎΡ‚ΠΎΡ€Ρ‹Ρ… проводится ΠΎΠ±ΡƒΡ‡Π΅Π½ΠΈΠ΅, ΠΈΠ»ΠΈ потрСбностСй Π² ΠΏΠΎΠ²Ρ‚ΠΎΡ€Π½ΠΎΠΌ ΠΎΠ±ΡƒΡ‡Π΅Π½ΠΈΠΈ.

    ΠšΠ°ΠΆΠ΄Ρ‹ΠΉ ΠΌΠΎΠ΄ΡƒΠ»ΡŒ содСрТит:

    • ΠΈΠ΄Π΅ΠΈ ΠΎ ΠΊΠ»ΡŽΡ‡Π΅Π²Ρ‹Ρ… матСматичСских идСях
    • ΡƒΡ‡ΠΈΡ‚Π΅Π»ΡŒ ΠΎΡ‚ΠΌΠ΅Ρ‡Π°Π΅Ρ‚ ΠΏΠΎΠ΄Ρ…ΠΎΠ΄Ρ‹ ΠΊ пониманию ΡƒΡ‡Π΅Π½ΠΈΠΊΠΎΠ²
    • мноТСство классных Π·Π°Π΄Π°Π½ΠΈΠΉ ΠΈ мСроприятий, ΠΈΠ»Π»ΡŽΡΡ‚Ρ€ΠΈΡ€ΡƒΡŽΡ‰ΠΈΡ… ΠΊΠΎΠ½ΠΊΡ€Π΅Ρ‚Π½Ρ‹Π΅ стратСгии
    • ΠΎΠ±ΡˆΠΈΡ€Π½Π°Ρ коллСкция ΡΠΎΠΎΡ‚Π²Π΅Ρ‚ΡΡ‚Π²ΡƒΡŽΡ‰ΠΈΡ… ΠΈΠ½Ρ‚Π΅Ρ€Π½Π΅Ρ‚-рСсурсов

    ΠŸΡ€Π΅Π΄Π²Π°Ρ€ΠΈΡ‚Π΅Π»ΡŒΠ½Ρ‹ΠΉ просмотр ΡˆΠ΅ΡΡ‚ΠΈ ΠΌΠΎΠ΄ΡƒΠ»Π΅ΠΉ K-5


    Π©Π΅Π»ΠΊΠ½ΠΈΡ‚Π΅ Π½ΠΈΠΆΠ΅, Ρ‡Ρ‚ΠΎΠ±Ρ‹ ΡƒΠ²ΠΈΠ΄Π΅Ρ‚ΡŒ ΠΏΠΎΠ»Π½Ρ‹ΠΉ список ΠΌΠΎΠ΄ΡƒΠ»Π΅ΠΉ Π² ΠΊΠ°ΠΆΠ΄ΠΎΠΉ ΠΊΠ½ΠΈΠ³Π΅ ΠΎΠ± ΡƒΡ€ΠΎΠ²Π½Π΅ обучСния:

    ΠœΠΎΠ΄ΡƒΠ»ΠΈ дСтского сада

    ΠŸΠΎΠ΄ΡΡ‡Π΅Ρ‚ ΠΈ количСство элСмСнтов: числа 1–5
    ΠŸΠΎΠ΄ΡΡ‡Π΅Ρ‚ ΠΈ количСство элСмСнтов: числа 0–10
    ΠŸΠΎΠ΄ΡΡ‡Π΅Ρ‚, количСство элСмСнтов ΠΈ разряд: числа 0–20
    ΠŸΠΎΠ΄ΡΡ‡Π΅Ρ‚ чисСл
    Π‘Ρ€Π°Π²Π½Π΅Π½ΠΈΠ΅ чисСл 1–10
    Π Π°Π·Π»ΠΎΠΆΠ΅Π½ΠΈΠ΅ чисСл
    ПониманиС слоТСния
    ПониманиС вычитания
    ПониманиС Ρ„Π°ΠΊΡ‚ΠΎΠ²
    ИсслСдованиС ΠΈΠ·ΠΌΠ΅Ρ€Π΅Π½ΠΈΠΉ
    Π‘ΠΎΡ€Ρ‚ΠΈΡ€ΠΎΠ²ΠΊΠ° ΠΈ классификация ΠΎΠ±ΡŠΠ΅ΠΊΡ‚ΠΎΠ²
    Знакомство с Π³Π΅ΠΎΠΌΠ΅Ρ‚Ρ€ΠΈΠ΅ΠΉ
    Π˜Π·ΡƒΡ‡Π΅Π½ΠΈΠ΅ Π³Π΅ΠΎΠΌΠ΅Ρ‚Ρ€ΠΈΠΈ

    ΠœΠΎΠ΄ΡƒΠ»ΠΈ 1-Π³ΠΎ класса

    Π‘Π»ΠΎΠΆΠΈΡ‚ΡŒ Π·Π°Π΄Π°Ρ‡ΠΈ со словами с суммой Π΄ΠΎ 20
    Π‘ΠΎΠ΅Π΄ΠΈΠ½Π΅Π½ΠΈΠ΅ вычитания ΠΈ слоТСния для Ρ€Π΅ΡˆΠ΅Π½ΠΈΡ Π·Π°Π΄Π°Ρ‡ со словами
    ПониманиС матСматичСских Ρ„Π°ΠΊΡ‚ΠΎΠ² ΠΈ свободноС Π²Π»Π°Π΄Π΅Π½ΠΈΠ΅ языком: Ρ€Π°ΡΡˆΠΈΡ€Π΅Π½ΠΈΠ΅ Π½Π° +/- 1, +/- 0
    ПониманиС матСматичСских Ρ„Π°ΠΊΡ‚ΠΎΠ² ΠΈ свободноС Π²Π»Π°Π΄Π΅Π½ΠΈΠ΅ языком: +/- 2
    Π‘Π»ΠΎΠΆΠΈΡ‚ΡŒ ΠΈ Π²Ρ‹Ρ‡Π΅ΡΡ‚ΡŒ +/- 10
    Π‘Π»ΠΎΠΆΠΈΡ‚ΡŒ ΠΈ Π²Ρ‹Ρ‡Π΅ΡΡ‚ΡŒ Π΄Π²ΠΎΠΉΠ½Ρ‹Π΅
    Π”Π΅Π»Π°Π΅ΠΌ Π΄Π΅ΡΡΡ‚ΡŒ
    ΠŸΠΎΠ΄ΡΡ‡Π΅Ρ‚ ΠΈ ΠΏΠΎΠ½ΠΈΠΌΠ°Π½ΠΈΠ΅ Ρ€Π°Π·ΠΌΠ΅Ρ‚ΠΊΠΈ
    ИсслСдованиС слоТСния ΠΈ вычитания с ΠΏΠΎΠΌΠΎΡ‰ΡŒΡŽ Π΄Π²ΡƒΠ·Π½Π°Ρ‡Π½ΠΎΠ³ΠΎ числа
    Π˜Π·ΠΌΠ΅Ρ€Π΅Π½ΠΈΠ΅ Π΄Π»ΠΈΠ½Ρ‹ с косвСнным сравнСниСм
    ΠžΠΏΡ€Π΅Π΄Π΅Π»Π΅Π½ΠΈΠ΅ Π²Ρ€Π΅ΠΌΠ΅Π½ΠΈ Π΄ΠΎ часа ΠΈ полчаса
    Π Π°Π±ΠΎΡ‚Π° с дСньгами
    ΠŸΡ€Π΅Π΄ΡΡ‚Π°Π²Π»Π΅Π½ΠΈΠ΅ ΠΈ интСрпрСтация Π΄Π°Π½Π½Ρ‹Ρ…
    ПониманиС ΠΈ описаниС Ρ„ΠΎΡ€ΠΌ ΠΈ ΠΎΠΏΡ€Π΅Π΄Π΅Π»Π΅Π½ΠΈΠ΅ Π°Ρ‚Ρ€ΠΈΠ±ΡƒΡ‚ΠΎΠ²
    Π Π°Π·Π΄Π΅Π»Π΅Π½ΠΈΠ΅ Ρ„ΠΈΠ³ΡƒΡ€ Π½Π° ΠΏΠΎΠ»ΠΎΠ²ΠΈΠ½ΠΊΠΈ ΠΈ Ρ‡Π΅Ρ‚Π²Π΅Ρ€Ρ‚ΠΈ

    ΠœΠΎΠ΄ΡƒΠ»ΠΈ 2-Π³ΠΎ класса

    РСшСниС ΠΏΡ€ΠΎΠ±Π»Π΅ΠΌ
    ПониманиС матСматичСских Ρ„Π°ΠΊΡ‚ΠΎΠ² ΠΈ свободноС Π²Π»Π°Π΄Π΅Π½ΠΈΠ΅ языком
    ΠžΡΠ½ΠΎΠ²Ρ‹ умноТСния: Ρ€Π°Π²Π½Ρ‹Π΅ Π³Ρ€ΡƒΠΏΠΏΡ‹
    Π—Π½Π°Ρ‡Π΅Π½ΠΈΠ΅ мСста
    Π‘Ρ€Π°Π²Π½ΠΈΡ‚Π΅ Π΄Π²Π° Ρ‚Ρ€Π΅Ρ…Π·Π½Π°Ρ‡Π½Ρ‹Ρ… числа
    ΠžΠ±Ρ‰ΠΈΠ΅ свСдСния ΠΎ слоТСнии ΠΌΠ½ΠΎΠ³ΠΎΠ·Π½Π°Ρ‡Π½Ρ‹Ρ… чисСл
    ПониманиС ΠΌΠ½ΠΎΠ³ΠΎΠ·Π½Π°Ρ‡Π½ΠΎΠ³ΠΎ вычитания
    Π Π°ΡΡˆΠΈΡ€ΡŒΡ‚Π΅ ΠΏΠΎΠ½ΠΈΠΌΠ°Π½ΠΈΠ΅ слоТСния ΠΌΠ½ΠΎΠ³ΠΎΠ·Π½Π°Ρ‡Π½Ρ‹Ρ… чисСл
    Π Π°ΡΡˆΠΈΡ€ΡŒΡ‚Π΅ ΠΏΠΎΠ½ΠΈΠΌΠ°Π½ΠΈΠ΅ ΠΌΠ½ΠΎΠ³ΠΎΠ·Π½Π°Ρ‡Π½ΠΎΠ³ΠΎ вычитания
    Π˜Π·ΠΌΠ΅Ρ€Π΅Π½ΠΈΠ΅ Π΄Π»ΠΈΠ½Ρ‹
    ВрСмя
    Π”Π΅Π½ΡŒΠ³ΠΈ
    ΠŸΡ€Π΅Π΄ΡΡ‚Π°Π²Π»Π΅Π½ΠΈΠ΅ ΠΈ интСрпрСтация Π΄Π°Π½Π½Ρ‹Ρ…
    ОписаниС гСомСтричСских Ρ„ΠΈΠ³ΡƒΡ€
    Π€ΠΎΡ€ΠΌΡ‹ ΠΏΠ΅Ρ€Π΅Π³ΠΎΡ€ΠΎΠ΄ΠΎΠΊ

    ΠœΠΎΠ΄ΡƒΠ»ΠΈ 3-Π³ΠΎ класса

    ПониманиС умноТСния ΠΈ дСлСния
    ПониманиС свойств умноТСния ΠΈ дСлСния
    Π‘Π²ΠΎΠ±ΠΎΠ΄Π½ΠΎ ΡƒΠΌΠ½ΠΎΠΆΠ°Ρ‚ΡŒ ΠΈ Π΄Π΅Π»ΠΈΡ‚ΡŒ
    РСшСниС ΠΎΠ΄Π½ΠΎ- ΠΈ двухэтапных Π·Π°Π΄Π°Ρ‡ со всСми Ρ‡Π΅Ρ‚Ρ‹Ρ€ΡŒΠΌΡ опСрациями
    ΠžΠΊΡ€ΡƒΠ³Π»Π΅Π½ΠΈΠ΅ чисСл Π΄ΠΎ дСсяти ΠΈΠ»ΠΈ сотни
    Π‘Π²ΠΎΠ±ΠΎΠ΄Π½ΠΎ Π΄ΠΎΠ±Π°Π²Π»ΡΡ‚ΡŒ Π² ΠΏΡ€Π΅Π΄Π΅Π»Π°Ρ… 1000
    Π‘Π²ΠΎΠ±ΠΎΠ΄Π½ΠΎ Π²Ρ‹Ρ‡Π΅ΡΡ‚ΡŒ Π² ΠΏΡ€Π΅Π΄Π΅Π»Π°Ρ… 1000
    ПониманиС Π΄Ρ€ΠΎΠ±Π΅ΠΉ ΠΈ ΠΎΠ±ΠΎΠ·Π½Π°Ρ‡Π΅Π½ΠΈΠΉ Π΄Ρ€ΠΎΠ±Π΅ΠΉ
    Дробная ΡΠΊΠ²ΠΈΠ²Π°Π»Π΅Π½Ρ‚Π½ΠΎΡΡ‚ΡŒ
    Π‘Ρ€Π°Π²Π½Π΅Π½ΠΈΠ΅ Π΄Ρ€ΠΎΠ±Π΅ΠΉ
    ВрСмя
    Масса и объСм
    ΠŸΡ€Π΅Π΄ΡΡ‚Π°Π²Π»Π΅Π½ΠΈΠ΅ ΠΈ интСрпрСтация Π΄Π°Π½Π½Ρ‹Ρ…
    ΠŸΠΎΠ½ΡΡ‚ΡŒ понятиС ΠΏΠ»ΠΎΡ‰Π°Π΄ΠΈ
    ΠŸΠ΅Ρ€ΠΈΠΌΠ΅Ρ‚Ρ€
    ПониманиС ΠΈ описаниС Ρ„ΠΈΠ³ΡƒΡ€

    ΠœΠΎΠ΄ΡƒΠ»ΠΈ 4-Π³ΠΎ класса

    ПониманиС ΠΈ Ρ€Π΅ΡˆΠ΅Π½ΠΈΠ΅ ΠΏΡ€ΠΎΠ±Π»Π΅ΠΌ с ΠΌΡƒΠ»ΡŒΡ‚ΠΈΠΏΠ»ΠΈΠΊΠ°Ρ‚ΠΈΠ²Π½Ρ‹ΠΌ сравнСниСм
    ΠœΠ½ΠΎΠΆΠΈΡ‚Π΅Π»ΠΈ, ΠΊΡ€Π°Ρ‚Π½Ρ‹Π΅, простыС ΠΈ составныС числа
    ПониманиС систСмы цСнностСй
    Π‘Π²ΠΎΠ±ΠΎΠ΄Π½ΠΎ складывайтС ΠΈ Π²Ρ‹Ρ‡ΠΈΡ‚Π°ΠΉΡ‚Π΅ ΠΌΠ½ΠΎΠ³ΠΎΠ·Π½Π°Ρ‡Π½Ρ‹Π΅ числа, ΠΈΡΠΏΠΎΠ»ΡŒΠ·ΡƒΡ стандартный Π°Π»Π³ΠΎΡ€ΠΈΡ‚ΠΌ
    Π˜ΡΠΏΠΎΠ»ΡŒΠ·ΡƒΠΉΡ‚Π΅ разрядноС Π·Π½Π°Ρ‡Π΅Π½ΠΈΠ΅ для выполнСния ΠΌΠ½ΠΎΠ³ΠΎΠ·Π½Π°Ρ‡Π½ΠΎΠ³ΠΎ умноТСния
    ИспользованиС разряда для выполнСния ΠΌΠ½ΠΎΠ³ΠΎΠ·Π½Π°Ρ‡Π½ΠΎΠ³ΠΎ дСлСния
    Π­ΠΊΠ²ΠΈΠ²Π°Π»Π΅Π½Ρ‚Π½ΠΎΡΡ‚ΡŒ ΠΈ порядок Π΄Ρ€ΠΎΠ±Π΅ΠΉ
    Π‘Π»ΠΎΠΆΠ΅Π½ΠΈΠ΅ ΠΈ Π²Ρ‹Ρ‡ΠΈΡ‚Π°Π½ΠΈΠ΅ Π΄Ρ€ΠΎΠ±Π΅ΠΉ с ΠΎΠ΄ΠΈΠ½Π°ΠΊΠΎΠ²Ρ‹ΠΌΠΈ знамСнатСлями
    Π£ΠΌΠ½ΠΎΠΆΠ΅Π½ΠΈΠ΅ Π΄Ρ€ΠΎΠ±Π΅ΠΉ Π½Π° Ρ†Π΅Π»Ρ‹Π΅ числа
    ДСсятичноС прСдставлСниС Π΄Ρ€ΠΎΠ±Π΅ΠΉ
    Π˜Π·ΠΌΠ΅Ρ€Π΅Π½ΠΈΠ΅ ΠΈ ΠΏΡ€Π΅ΠΎΠ±Ρ€Π°Π·ΠΎΠ²Π°Π½ΠΈΠ΅ ΠΈΠ·ΠΌΠ΅Ρ€Π΅Π½ΠΈΠΉ
    ΠŸΠ»ΠΎΡ‰Π°Π΄ΡŒ ΠΈ ΠΏΠ΅Ρ€ΠΈΠΌΠ΅Ρ‚Ρ€
    ΠŸΡ€Π΅Π΄ΡΡ‚Π°Π²Π»Π΅Π½ΠΈΠ΅ ΠΈ интСрпрСтация Π΄Π°Π½Π½Ρ‹Ρ…
    ГСомСтрия ΠΈ гСомСтричСскиС измСрСния
    РСшСниС слоТных ΠΏΡ€ΠΎΠ±Π»Π΅ΠΌ

    ΠœΠΎΠ΄ΡƒΠ»ΠΈ 5-Π³ΠΎ класса

    ΠœΠ΅ΡΡ‚ΠΎ Π·Π½Π°Ρ‡Π΅Π½ΠΈΠ΅
    Π—Π°ΠΏΠΈΡΡŒ ΠΈ интСрпрСтация числовых Π²Ρ‹Ρ€Π°ΠΆΠ΅Π½ΠΈΠΉ
    Π£ΠΌΠ½ΠΎΠΆΠ΅Π½ΠΈΠ΅ Π½Π° ΠΌΠ½ΠΎΠ³ΠΎΠ·Π½Π°Ρ‡Π½Ρ‹Π΅ числа Π”Π΅Π»Π΅Π½ΠΈΠ΅
    с ΠΌΠ½ΠΎΠ³ΠΎΠ·Π½Π°Ρ‡Π½Ρ‹ΠΌΠΈ Ρ†Π΅Π»Ρ‹ΠΌΠΈ числами
    Π‘Π»ΠΎΠΆΠ΅Π½ΠΈΠ΅ ΠΈ Π²Ρ‹Ρ‡ΠΈΡ‚Π°Π½ΠΈΠ΅ дСсятичных Π·Π½Π°ΠΊΠΎΠ²
    Π£ΠΌΠ½ΠΎΠΆΠ΅Π½ΠΈΠ΅ ΠΈ Π΄Π΅Π»Π΅Π½ΠΈΠ΅ с дСсятичными Π·Π½Π°ΠΊΠ°ΠΌΠΈ
    Π‘Π»ΠΎΠΆΠ΅Π½ΠΈΠ΅ ΠΈ Π²Ρ‹Ρ‡ΠΈΡ‚Π°Π½ΠΈΠ΅ Π΄Ρ€ΠΎΠ±Π΅ΠΉ с Ρ€Π°Π·Π½Ρ‹ΠΌΠΈ знамСнатСлями
    Π”Ρ€ΠΎΠ±ΠΈ ΠΊΠ°ΠΊ Π΄Π΅Π»Π΅Π½ΠΈΠ΅
    Π£ΠΌΠ½ΠΎΠΆΠ΅Π½ΠΈΠ΅ Π΄Ρ€ΠΎΠ±Π΅ΠΉ Π½Π° Π΄Ρ€ΠΎΠ±ΠΈ ΠΈ Ρ†Π΅Π»Ρ‹Π΅ числа
    Π”Π΅Π»Π΅Π½ΠΈΠ΅ Ρ†Π΅Π»Ρ‹Ρ… чисСл ΠΈ Π΄Ρ€ΠΎΠ±Π΅ΠΉ
    ΠŸΡ€Π΅ΠΎΠ±Ρ€Π°Π·ΠΎΠ²Π°Π½ΠΈΠ΅ ΠΎΠ΄ΠΈΠ½Π°ΠΊΠΎΠ²Ρ‹Ρ… Π΅Π΄ΠΈΠ½ΠΈΡ† измСрСния
    ΠŸΡ€Π΅Π΄ΡΡ‚Π°Π²Π»Π΅Π½ΠΈΠ΅ ΠΈ интСрпрСтация Π΄Π°Π½Π½Ρ‹Ρ…
    ГСомСтричСскиС измСрСния: исслСдованиС объСма
    БистСма ΠΊΠΎΠΎΡ€Π΄ΠΈΠ½Π°Ρ‚
    ΠšΠ»Π°ΡΡΠΈΡ„ΠΈΠΊΠ°Ρ†ΠΈΡ Π΄Π²ΡƒΡ…ΠΌΠ΅Ρ€Π½Ρ‹Ρ… Ρ„ΠΈΠ³ΡƒΡ€

    Для Ρ€ΠΎΠ΄ΠΈΡ‚Π΅Π»Π΅ΠΉ – повсСднСвная ΠΌΠ°Ρ‚Π΅ΠΌΠ°Ρ‚ΠΈΠΊΠ°

    РодитСлям

    ΠœΠ°Ρ‚Π΅ΠΌΠ°Ρ‚ΠΈΠΊΠ° Π½Π° ΠΊΠ°ΠΆΠ΄Ρ‹ΠΉ дСнь Π΄ΠΎΠΌΠ°

    Everyday Mathematics at Home прСдоставляСт ΠΏΠΎΠΌΠΎΡ‰ΡŒ ΠΏΠΎ Π·Π°Π΄Π°Ρ‡Π°ΠΌ Home ΠΈ Study Link, ΠΈΠ·Π±Ρ€Π°Π½Π½Ρ‹ΠΌ ΠΎΡ‚Π²Π΅Ρ‚Π°ΠΌ, опрСдСлСниям словаря, ΡƒΡ‡Π΅Π±Π½Ρ‹ΠΌ пособиям ΠΏΠΎ Π°Π»Π³ΠΎΡ€ΠΈΡ‚ΠΌΠ°ΠΌ ΠΈ ΠΈΠ³Ρ€Π°ΠΌ.

    EM4 Edition

    ΠŸΠΎΠ΄Ρ€ΠΎΠ±Π½Π΅Π΅ Π² Ρ€Π°Π·Π΄Π΅Π»Π΅ для Ρ€ΠΎΠ΄ΠΈΡ‚Π΅Π»Π΅ΠΉ

    рСсурсов Π² зависимости ΠΎΡ‚ уровня обучСния

    Π Π°Π·Π΄Π΅Π» «Для Ρ€ΠΎΠ΄ΠΈΡ‚Π΅Π»Π΅ΠΉΒ» ΠΎΡ€Π³Π°Π½ΠΈΠ·ΠΎΠ²Π°Π½ ΠΏΠΎ классам, Ρ‡Ρ‚ΠΎΠ±Ρ‹ Ρ€ΠΎΠ΄ΠΈΡ‚Π΅Π»ΠΈ ΠΌΠΎΠ³Π»ΠΈ Π»Π΅Π³ΠΊΠΎ Π½Π°ΠΉΡ‚ΠΈ рСсурсы ΠΈ ΠΈΠ½Ρ„ΠΎΡ€ΠΌΠ°Ρ†ΠΈΡŽ, Π½Π°ΠΈΠ±ΠΎΠ»Π΅Π΅ подходящиС для ΠΈΡ… Ρ€Π΅Π±Π΅Π½ΠΊΠ°. Под ΠΊΠ°ΠΆΠ΄ΠΎΠΉ ΠΎΡ†Π΅Π½ΠΊΠΎΠΉ Π²Ρ‹ Π½Π°ΠΉΠ΄Π΅Ρ‚Π΅ ΠΏΠΎΠ΄Ρ€ΠΎΠ±Π½Ρ‹ΠΉ ΠΎΠ±Π·ΠΎΡ€ Ρ†Π΅Π»Π΅ΠΉ, Ρ…Π°Ρ€Π°ΠΊΡ‚Π΅Ρ€Π½Ρ‹Ρ… для этого класса, Ρ‚Π°Π±Π»ΠΈΡ†Ρ‹, ΠΏΠΎΠΌΠΎΠ³Π°ΡŽΡ‰ΠΈΠ΅ с домашними заданиями ΠΈ концСпциями для ΠΊΠ°ΠΆΠ΄ΠΎΠ³ΠΎ Ρ€Π°Π·Π΄Π΅Π»Π°, ГалСрСя студСнчСских ΠΏΡ€ΠΎΠ΅ΠΊΡ‚ΠΎΠ² ΠΈ занятий, ЕТСднСвная ΠΌΠ°Ρ‚Π΅ΠΌΠ°Ρ‚ΠΈΠΊΠ° Π‘Π΅ΠΌΠ΅ΠΉΠ½Ρ‹Π΅ письма Π½Π° английском ΠΈ испанском языках, Π° Ρ‚Π°ΠΊΠΆΠ΅ сборник практичСских занятий, ΠΊΠΎΡ‚ΠΎΡ€Ρ‹Π΅ ΠΏΠΎΠΌΠΎΠ³ΡƒΡ‚ Π΄ΠΎΠΏΠΎΠ»Π½ΠΈΡ‚ΡŒ занятия Π² классС.

    ПониманиС

    ПовсСднСвная ΠΌΠ°Ρ‚Π΅ΠΌΠ°Ρ‚ΠΈΠΊΠ°

    Π’ этом Ρ€Π°Π·Π΄Π΅Π»Π΅ ΠΏΡ€ΠΈΠ²Π΅Π΄Π΅Π½Ρ‹ ΠΎΡ‚Π²Π΅Ρ‚Ρ‹ Π½Π° ΠΎΠ±Ρ‰ΠΈΠ΅ вопросы Ρ€ΠΎΠ΄ΠΈΡ‚Π΅Π»Π΅ΠΉ ΠΎ ΠΏΡ€ΠΎΠ³Ρ€Π°ΠΌΠΌΠ΅ ПовсСднСвная ΠΌΠ°Ρ‚Π΅ΠΌΠ°Ρ‚ΠΈΠΊΠ° . Π—Π΄Π΅ΡΡŒ Π²Ρ‹ ΠΌΠΎΠΆΠ΅Ρ‚Π΅ ΡƒΠ·Π½Π°Ρ‚ΡŒ большС ΠΎΠ± обосновании Π·Π° ΠΏΠΎΠ·ΠΈΡ†ΠΈΡŽ Everyday Mathematics ΠΏΠΎ Ρ‚Π°ΠΊΠΈΠΌ Ρ‚Π΅ΠΌΠ°ΠΌ, ΠΊΠ°ΠΊ основныС матСматичСскиС Ρ„Π°ΠΊΡ‚Ρ‹ ΠΈ использованиС ΠΊΠ°Π»ΡŒΠΊΡƒΠ»ΡΡ‚ΠΎΡ€Π°, Π° Ρ‚Π°ΠΊΠΆΠ΅ совСты ΠΎ Ρ‚ΠΎΠΌ, ΠΊΠ°ΠΊ ΠΏΠΎΠΌΠΎΡ‡ΡŒ Π²Π°ΡˆΠ΅ΠΌΡƒ Ρ€Π΅Π±Π΅Π½ΠΊΡƒ.

    ΠŸΠΎΠ»Π΅Π·Π½Ρ‹Π΅ ссылки для Ρ€ΠΎΠ΄ΠΈΡ‚Π΅Π»Π΅ΠΉ

    Π‘Ρ‚Ρ€Π°Π½ΠΈΡ†Π° рСсурсов для Ρ€ΠΎΠ΄ΠΈΡ‚Π΅Π»Π΅ΠΉ содСрТит ссылки Π½Π° дСсятки ΠΎΠ½Π»Π°ΠΉΠ½-рСсурсов, ΠΊΠΎΡ‚ΠΎΡ€Ρ‹Π΅ ΠΏΠΎΠΌΠΎΠ³Π°ΡŽΡ‚ родитСлям ΠΏΠΎΠΌΠΎΠ³Π°Ρ‚ΡŒ своим дСтям.Π­Ρ‚ΠΈ ссылки Π²ΠΊΠ»ΡŽΡ‡Π°ΡŽΡ‚ Π² сСбя ΠΎΠ½Π»Π°ΠΉΠ½-ΠΈΠ³Ρ€Ρ‹, ΡƒΡ‡Π΅Π±Π½Ρ‹Π΅ пособия, ΠΌΠΎΠ΄Π΅Π»ΠΈΡ€ΠΎΠ²Π°Π½ΠΈΠ΅ ΠΈ совСты ΠΏΠΎ ΠΎΠ±ΡƒΡ‡Π΅Π½ΠΈΡŽ.

    ΠŸΠΎΠ»Π΅Π·Π½Ρ‹Π΅ ссылки для студСнтов

    На страницС “РСсурсы для учащихся” имССтся мноТСство ΠΏΠΎΠ»Π΅Π·Π½Ρ‹Ρ… рСсурсов для учащихся, Π²ΠΊΠ»ΡŽΡ‡Π°Ρ ссылки Π½Π° ΠΈΠ½Ρ‚Π΅Ρ€Π°ΠΊΡ‚ΠΈΠ²Π½Ρ‹Π΅ ΠΈΠ³Ρ€Ρ‹, Π²ΠΈΡ€Ρ‚ΡƒΠ°Π»ΡŒΠ½Ρ‹Π΅ Π²Π΅Π±-инструмСнты ΠΈ ΡƒΡ‡Π΅Π±Π½Ρ‹Π΅ пособия, Π²ΠΈΠΊΡ‚ΠΎΡ€ΠΈΠ½Ρ‹, Ρ„ΠΎΡ€ΡƒΠΌΡ‹ ΠΈ ΠΌΠ½ΠΎΠ³ΠΎΠ΅ Π΄Ρ€ΡƒΠ³ΠΎΠ΅.

    ПовсСднСвная ΠΌΠ°Ρ‚Π΅ΠΌΠ°Ρ‚ΠΈΠΊΠ° для Ρ€ΠΎΠ΄ΠΈΡ‚Π΅Π»Π΅ΠΉ: Π§Ρ‚ΠΎ Π½ΡƒΠΆΠ½ΠΎ Π·Π½Π°Ρ‚ΡŒ, Ρ‡Ρ‚ΠΎΠ±Ρ‹ ΠΏΠΎΠΌΠΎΡ‡ΡŒ своСму Ρ€Π΅Π±Π΅Π½ΠΊΡƒ Π΄ΠΎΠ±ΠΈΡ‚ΡŒΡΡ успСха

    ΠŸΡ€ΠΎΠ΅ΠΊΡ‚ ΠΌΠ°Ρ‚Π΅ΠΌΠ°Ρ‚ΠΈΠΊΠΈ ΡˆΠΊΠΎΠ»Ρ‹ Чикагского унивСрситСта

    Π˜Π·Π΄Π°Ρ‚Π΅Π»ΡŒΡΡ‚Π²ΠΎ Чикагского унивСрситСта

    ΠŸΠΎΠ΄Ρ€ΠΎΠ±Π½Π΅Π΅ >>

    Бсылки ΠΏΠΎ Ρ‚Π΅ΠΌΠ΅

    ПовсСднСвная ΠΌΠ°Ρ‚Π΅ΠΌΠ°Ρ‚ΠΈΠΊΠ° Онлайн

    Π˜ΡΠΏΠΎΠ»ΡŒΠ·ΡƒΡ Π»ΠΎΠ³ΠΈΠ½, прСдоставлСнный ΡƒΡ‡ΠΈΡ‚Π΅Π»Π΅ΠΌ вашСго Ρ€Π΅Π±Π΅Π½ΠΊΠ°, Π²Ρ‹ ΠΏΠΎΠ»ΡƒΡ‡ΠΈΡ‚Π΅ доступ ΠΊ рСсурсам, ΠΊΠΎΡ‚ΠΎΡ€Ρ‹Π΅ ΠΏΠΎΠΌΠΎΠ³ΡƒΡ‚ Π²Π°ΡˆΠ΅ΠΌΡƒ Ρ€Π΅Π±Π΅Π½ΠΊΡƒ с домашним Π·Π°Π΄Π°Π½ΠΈΠ΅ΠΌ ΠΈΠ»ΠΈ ΠΎΡΠ²Π΅ΠΆΠΈΡ‚ΡŒ свои матСматичСскиС Π½Π°Π²Ρ‹ΠΊΠΈ.

    ПониманиС

    ПовсСднСвная ΠΌΠ°Ρ‚Π΅ΠΌΠ°Ρ‚ΠΈΠΊΠ° для Ρ€ΠΎΠ΄ΠΈΡ‚Π΅Π»Π΅ΠΉ

    Π£Π·Π½Π°ΠΉΡ‚Π΅ большС ΠΎΠ± ΡƒΡ‡Π΅Π±Π½ΠΎΠΉ ΠΏΡ€ΠΎΠ³Ρ€Π°ΠΌΠΌΠ΅ EM ΠΈ ΠΎ Ρ‚ΠΎΠΌ, ΠΊΠ°ΠΊ ΠΏΠΎΠΌΠΎΡ‡ΡŒ своСму Ρ€Π΅Π±Π΅Π½ΠΊΡƒ.

    50 ΡƒΠ²Π»Π΅ΠΊΠ°Ρ‚Π΅Π»ΡŒΠ½Ρ‹Ρ… ΠΈ ΡƒΠ²Π»Π΅ΠΊΠ°Ρ‚Π΅Π»ΡŒΠ½Ρ‹Ρ… Π·Π°Π΄Π°Π½ΠΈΠΉ ΠΏΠΎ ΠΌΠ°Ρ‚Π΅ΠΌΠ°Ρ‚ΠΈΠΊΠ΅ для учащихся 3, 4 ΠΈ 5 классов

    Если Π²Ρ‹ ΠΈΡ‰Π΅Ρ‚Π΅ ΡƒΠ²Π»Π΅ΠΊΠ°Ρ‚Π΅Π»ΡŒΠ½Ρ‹Π΅ задания ΠΏΠΎ ΠΌΠ°Ρ‚Π΅ΠΌΠ°Ρ‚ΠΈΠΊΠ΅, Π½Π΅ ΠΈΡ‰ΠΈΡ‚Π΅ дальшС. Π­Ρ‚ΠΈ занятия, ΠΏΡ€Π΅Π΄Π½Π°Π·Π½Π°Ρ‡Π΅Π½Π½Ρ‹Π΅ для учащихся 3, 4 ΠΈ 5 классов, Π·Π°Π½ΠΈΠΌΠ°Ρ‚Π΅Π»ΡŒΠ½Ρ‹ ΠΈ бСсплатны.

    Π― люблю ΡƒΡ‡ΠΈΡ‚ΡŒ Π΄Π΅Ρ‚Π΅ΠΉ ΡΡ‚Π°Ρ€ΡˆΠΈΡ… классов.Π£Π΄ΠΈΠ²ΠΈΡ‚Π΅Π»ΡŒΠ½ΠΎ, Π½ΠΎ ΠΌΠ½Π΅ нравится Π·Π°Π΄Π°Ρ‡Π° умноТСния Π΄Ρ€ΠΎΠ±Π΅ΠΉ, дСлСния Π² столбик ΠΈ дСсятичных Π΄Ρ€ΠΎΠ±Π΅ΠΉ. Π‘ ΡƒΡ‡Π΅Ρ‚ΠΎΠΌ сказанного, ΠΌΠ½Π΅ нравится, ΠΊΠ°ΠΊ устроСны классы Π² Π½Π°Ρ‡Π°Π»ΡŒΠ½ΠΎΠΉ школС. Π¦Π΅Π½Ρ‚Ρ€Ρ‹, ΠΈΠ³Ρ€Ρ‹ ΠΈ практичСскиС занятия Π²Ρ‹Π·Ρ‹Π²Π°ΡŽΡ‚ Π²ΠΎ ΠΌΠ½Π΅ Ρ‚Π°ΠΊΠΎΠ΅ ΠΆΠ΅ Π²ΠΎΠ»Π½Π΅Π½ΠΈΠ΅, ΠΊΠ°ΠΊ Π² дСтствС Π½Π° РоТдСство.

    Π― Ρ…ΠΎΡ‚Π΅Π», Ρ‡Ρ‚ΠΎΠ±Ρ‹ Π² ΠΌΠΎΠ΅ΠΌ классС ΠΌΠ°Ρ‚Π΅ΠΌΠ°Ρ‚ΠΈΠΊΠΈ Π² ΡΡ‚Π°Ρ€ΡˆΠΈΡ… классах Π½Π°Ρ‡Π°Π»ΡŒΠ½ΠΎΠΉ ΡˆΠΊΠΎΠ»Ρ‹ Π±Ρ‹Π»ΠΈ Ρ‚Π°ΠΊΠΈΠ΅ ΠΆΠ΅ вСсСлыС ΠΈ ΡƒΠ²Π»Π΅ΠΊΠ°Ρ‚Π΅Π»ΡŒΠ½Ρ‹Π΅ занятия ΠΏΠΎ ΠΌΠ°Ρ‚Π΅ΠΌΠ°Ρ‚ΠΈΠΊΠ΅, Ρ‡Ρ‚ΠΎ ΠΈ Ρƒ Π½Π°ΡˆΠΈΡ… ΠΌΠ»Π°Π΄ΡˆΠΈΡ… школьников.

    Моя Тизнь измСнилась, Ρ‚Π°ΠΊ ΠΊΠ°ΠΊ Ρ‚Π΅ΠΏΠ΅Ρ€ΡŒ я имСю Ρ‡Π΅ΡΡ‚ΡŒ ΡƒΡ‡ΠΈΡ‚ΡŒ своих Π΄Π΅Ρ‚Π΅ΠΉ, ΠΊΠΎΡ‚ΠΎΡ€Ρ‹Π΅ Π·Π°ΠΏΠΎΠΌΠΈΠ½Π°ΡŽΡ‚ слоТСниС ΠΈ Π²Ρ‹Ρ‡ΠΈΡ‚Π°Π½ΠΈΠ΅ Ρ„Π°ΠΊΡ‚ΠΎΠ², ΡΡ‡ΠΈΡ‚Π°ΡŽΡ‚ врСмя ΠΈ ΡΡ‡ΠΈΡ‚Π°ΡŽΡ‚ дСньги.Π₯отя я Π΄ΠΎΡ€ΠΎΠΆΡƒ этими ΠΌΠΎΠΌΠ΅Π½Ρ‚Π°ΠΌΠΈ, я Π½Π΅ ΠΌΠΎΠ³Ρƒ ΠΏΠΎΠ»Π½ΠΎΡΡ‚ΡŒΡŽ Π·Π°Π±Ρ‹Ρ‚ΡŒ, ΠΊΠ°ΠΊ ΠΌΠ½Π΅ Π½Ρ€Π°Π²ΠΈΠ»ΠΈΡΡŒ эти Π΄Ρ€ΠΎΠ±ΠΈ.

    Π‘ Ρ‚ΠΎΠΉ ΡΡ‚Ρ€Π°ΡΡ‚ΡŒΡŽ, которая всС Π΅Ρ‰Π΅ ΠΏΡ‹Π»Π°Π΅Ρ‚ Π²ΠΎ ΠΌΠ½Π΅, Ρ‡Π°ΡΡ‚ΡŒ ΠΌΠΎΠ΅Π³ΠΎ Π±Π»ΠΎΠ³Π° посвящСна учитСлям 3-Π³ΠΎ, 4-Π³ΠΎ ΠΈ 5-Π³ΠΎ классов. Π’ΠΎΡ‚ нСсколько практичСских, бСсплатных ΠΈ ΡƒΠ²Π»Π΅ΠΊΠ°Ρ‚Π΅Π»ΡŒΠ½Ρ‹Ρ… Π·Π°Π΄Π°Π½ΠΈΠΉ ΠΏΠΎ ΠΌΠ°Ρ‚Π΅ΠΌΠ°Ρ‚ΠΈΠΊΠ΅, ΠΊΠΎΡ‚ΠΎΡ€Ρ‹Π΅ Π²Ρ‹ ΠΌΠΎΠΆΠ΅Ρ‚Π΅ Π½Π°ΠΉΡ‚ΠΈ Π² ΠΌΠΎΠ΅ΠΌ Π±Π»ΠΎΠ³Π΅!

    Π’Ρ‹ ΡƒΠΆΠ΅ подписались? Если Π½Π΅Ρ‚, ΠΎΡΡ‚Π°Π²ΡŒΡ‚Π΅ свой адрСс, Ρ‡Ρ‚ΠΎΠ±Ρ‹ ΠΏΠΎΠ»ΡƒΡ‡Π°Ρ‚ΡŒ самыС свСТиС ΠΈ новСйшиС бСсплатныС услуги, ΠΊΠΎΡ‚ΠΎΡ€Ρ‹Π΅ ΠΌΡ‹ ΠΌΠΎΠΆΠ΅ΠΌ ΠΏΡ€Π΅Π΄Π»ΠΎΠΆΠΈΡ‚ΡŒ.

    ΠŸΡ€Π°ΠΊΡ‚ΠΈΡ‡Π΅ΡΠΊΠΈΠ΅, бСсплатныС ΠΈ ΡƒΠ²Π»Π΅ΠΊΠ°Ρ‚Π΅Π»ΡŒΠ½Ρ‹Π΅ занятия ΠΏΠΎ ΠΌΠ°Ρ‚Π΅ΠΌΠ°Ρ‚ΠΈΠΊΠ΅

    Π’Ρ‹Ρ‡ΠΈΡ‚Π°Π½ΠΈΠ΅ ΠΏΡ€ΠΈ ΠΏΠ΅Ρ€Π΅Π³Ρ€ΡƒΠΏΠΏΠΈΡ€ΠΎΠ²ΠΊΠ΅ ΠΌΠΎΠΆΠ΅Ρ‚ ΡΡ‚Π°Ρ‚ΡŒ ΠΏΡ€ΠΎΠ±Π»Π΅ΠΌΠΎΠΉ Π΄Π°ΠΆΠ΅ для ΡƒΡ‡Π΅Π½ΠΈΠΊΠΎΠ² Ρ‚Ρ€Π΅Ρ‚ΡŒΠ΅Π³ΠΎ ΠΈ Ρ‡Π΅Ρ‚Π²Π΅Ρ€Ρ‚ΠΎΠ³ΠΎ классов.ΠŸΡ€Π΅Π΄ΠΎΡΡ‚Π°Π²Π»Π΅Π½ΠΈΠ΅ этим дСтям ΡƒΠ²Π»Π΅ΠΊΠ°Ρ‚Π΅Π»ΡŒΠ½Ρ‹Ρ… занятий Π² классС Π²Π°ΠΆΠ½ΠΎ ΠΈ ΠΎΡ‚Π»ΠΈΡ‡Π½Ρ‹ΠΉ способ ΠΏΠΎΠΌΠΎΡ‡ΡŒ ΠΈΠΌ Ρ€Π°Π·Π²ΠΈΡ‚ΡŒ ΠΏΠΎΠ½ΠΈΠΌΠ°Π½ΠΈΠ΅ цСнностСй ΠΈ Π±ΠΎΠ»Π΅Π΅ Π³Π»ΡƒΠ±ΠΎΠΊΠΎΠ΅ ΠΏΠΎΠ½ΠΈΠΌΠ°Π½ΠΈΠ΅ вычитания. Π’ΠΎΡ‚ нСсколько ΠΈΠ΄Π΅ΠΉ, ΠΊΠΎΡ‚ΠΎΡ€Ρ‹Π΅ ΠΏΠΎΠΌΠΎΠ³ΡƒΡ‚ вашим дСтям ΠΎΡΠ²ΠΎΠΈΡ‚ΡŒ Π²Ρ‹Ρ‡ΠΈΡ‚Π°Π½ΠΈΠ΅.

    Π”Π°Π»Π΅Π΅ Ρƒ нас Π΅ΡΡ‚ΡŒ ΡƒΠΌΠ½ΠΎΠΆΠ΅Π½ΠΈΠ΅. Π­Ρ‚ΠΈ практичСскиС распСчатки ΠΏΠΎΠΌΠΎΠ³ΡƒΡ‚ дСтям Π½Π°ΡƒΡ‡ΠΈΡ‚ΡŒΡΡ ΡƒΠΌΠ½ΠΎΠΆΠ°Ρ‚ΡŒ большиС Ρ†ΠΈΡ„Ρ€Ρ‹, Π° Ρ‚Π°ΠΊΠΆΠ΅ ΡƒΠ·Π½Π°Ρ‚ΡŒ свои Ρ„Π°ΠΊΡ‚Ρ‹!

    ΠœΡ‹ ΠΏΠ΅Ρ€Π΅Ρ…ΠΎΠ΄ΠΈΠΌ ΠΊ дСлСнию с Π΅Ρ‰Π΅ Π±ΠΎΠ»Π΅Π΅ ΡƒΠ²Π»Π΅ΠΊΠ°Ρ‚Π΅Π»ΡŒΠ½Ρ‹ΠΌΠΈ матСматичСскими распСчатками для Π²Π°ΡˆΠΈΡ… Π΄Π΅Ρ‚Π΅ΠΉ.

    Π”ΠΎΠ±Π°Π²ΠΈΡ‚ΡŒ ΠΊΠΎΠΌΠΌΠ΅Π½Ρ‚Π°Ρ€ΠΈΠΉ

    Π’Π°Ρˆ адрСс email Π½Π΅ Π±ΡƒΠ΄Π΅Ρ‚ ΠΎΠΏΡƒΠ±Π»ΠΈΠΊΠΎΠ²Π°Π½. ΠžΠ±ΡΠ·Π°Ρ‚Π΅Π»ΡŒΠ½Ρ‹Π΅ поля ΠΏΠΎΠΌΠ΅Ρ‡Π΅Π½Ρ‹ *